Wednesday, December 30, 2009

Localization

It is tough to create MTV bakra (on the lines of'Candid Camera') for Viacom-the owner of MTV-in India. Likewise, if one genuinely wishes to go for rural management one has to learn the ropes really fast. It seems there are many fakes like me who do not have an inkling of how a rural life can be but still take a fancy for such a setup. Absorptive capacity in Indian rural market also requires one to be at least abreast of what is hapening in this other INDIA.
I was flabbergasted at my own inefficiency in this fold despite having spent so many years in a small town practically doing nothing. How local Haats are mini-marts and can realize scale economies was evident when I visited one in a village! Even the cattle are sold and they come at a premium. A 'bhains' can cost anywhere from 10k to 100k thrice the cost of an average 'Gay'. A bakra can be bought near 2-3k price. I bought six small sized tea cups for Rs. 20, a scrubber literally free, and a quilt for Rs. 250. The morning dawn squatting was intitally embarassing but the dew-laden long pointed grasses piercing my tiny hips after a while , while I was washing it shit-clean, was enthralling and I had to do frogjumping to few places to perform the entire act. I am talking only about the grass and do not have a Socrates' dilemma on this issue. My sexual preferences are straight.:)
Waise to, the NGOs are responsive in villages but the problem is personalisation and empathy with the plights of the rural folks. One cannot just get in the skin of the character but one needs to be the character itself- one of their own brethren sharing all their physical and mental woes.
The morning musings are initially not that great and it requires cigarette puffs to overcome the time warp. The sinus congestion hits me thereafter but showers in a nearby brook washes away all the negativities for few hours at least. The Harvard case studies in my hand are redundant when I sit to read something. The novels have unforutunately been sold off due to mental recession to a neighborhood 'kabadi' in kilos a dime transaction.
As the evening sets in especially in the winters, a city folk can get nostalgic or rather wobbly over the basic amenities so much lacking in villages. It is tough to imagine how quite a few families survive on less than 2000 rs per month, which can be the monthly expenditure of a moderate alcoholic. Pulse consumption is rare as they are now priced near 100 rs per kg and vegetables are like a glutton's culinary adventures. The foodie is a quixotic hero. Automobile sector seems to be a laggard in the rural market. Jaywalkers exist in differnet avatars. Tarmac can only be a dream and express train travelling is akin to commuting in the latest Chinese 392 km/hr plus speed wali train, forget flying one's private jet. On a brighter side, lifestyle disorders are also rare becuase of a distinctive lack of the lifestyle.
Perhaps this is also because agriculture sector is the poor cousin of other sectors and has been given a short shrift with reagrds to human capital. The intellectuals feel ashamed in owning up any agri-related tasks and fakes like me would simply buckle under the pressures of tilling our own farm. One of the reasons why agri-exports are still at an abysmal low. There were early showers this morning boding well for wheat crops at least. But the optimal rainfall required was not met.
The disparities are stark. Life seems to be gently leading on affectionately but the trials and tribulations are testing for someone like me who is not used to wading in different if not troubled waters. But I am trying to be one with the vilagers and I will love to do something genuine so that my consciousness does not trouble me calling me names as a 'fake'.

Saturday, December 19, 2009

candid love confessions

What is there to hide in this beautiful feeling? So what if age and lethargy of purpose are catching up fast!These were never the 'neuntam aharta' or eligibility to dare to think beyond love!
After all these ego-shattering experiences the egotist in me lies crushed but lo there comes from the heaven someone who still makes me believe that I am worth a lot! She panders to my ego in a gentle and caring fashion but never loses track of the purpose which might bind our life in a common thread. The 'girl-next-door' image can be deceptive and you are caught unawares when she rings you in the morning asking a rather intricate Math, DI or LR puzzle(she is good at these but is much better in linguistics!) You wonder if she is checking whether you are really up early in the morning or not! On occasions she is morphed into a traditional Indian beauty with a 'bindi' and kewl-eyed gentleness mesmerizing your softer longings.
She is otherwise reticent but is very good in catching up with the 'shana' type lingo as and when required. 'Kyon a gaye ... ke...se' types hahaha.
Perhaps an underrated intellectual, she has immense possibilities but little craving for those as she has a bit of inherent detachment from the mundane, which is enviable for someone who is trying to attain that by dint of reading lots of scriptures, trying to practice meditation, aplomb, karma et al.
Moreover, she is equally at ease and the same unchanged persona wherever you find her. 'Manhattan' is just another place for her as it is with 'Mankapur'. That is indeed stunning. The egolessness in her is an in-built uncomplicated code in her and she does not need to rush to Himalayas for that. But Binomial Theorem can distract her. Math is more applied in remembering numbers and in calculating all the schemes that the cellphone services providers proffer. And she is rather good in that, which puts a theory in practice. 'Tohun marde' is a favorite line for her. You miss her when she is not around and sometimes end up with your eyes welling in tears. The misty winter mornings remind a lot about her and any session seems empty without her. When questioned even a bit in numerical, there is a coy face hiding herself in her dupatta and you feel the universe smiling in bliss at this innocent acceptance of a seemingly innocuous travesty. She resonates within my heart and only God knows how much I crave to get her back. How much do I miss my evening cafeteria coffee with her ordering noodles from the counter! And this evening I am proud of everything that includes my ad-hoc lethargy too. I know I will remove that asap. Such is the power of love!

Monday, December 14, 2009

my musings

Our sweetest songs are those that tell us of the saddest thoughts. Fantastic! However, it is better to recall that every flower enjoys the air it breathes. It has been years since a few blokes last felt the soulful bliss. What followed thereafter was ennui?
Levin in Anna Karenina pondered whether we just live for the sake of belly. His belly was full by dint of legacy. So is it with most of us, which gives us time and space to think of the Godhead. The literature is endless. But it rarely provides the divine insight that is so naturally and obliviously present in a simpleton. Are we caught in a time warp?
There have been persistent efforts by all to get closer to the divine. It mostly takes us away from it. Few say love is worship. Few say work is worship and so on and so forth. To an idle adult this exploration is not essentially a quest but an escape route. Does the confession of this crime bring levity to soul? Few hope it does!
The luxurious quest for the divine is an anomaly. And the idleness has so viciously established its foothold in the prodigal but indolent spirit that a sabbatical is only a farce. The mental age is a deterrent. The fascination of a Yogic state holds true but the path is extremely difficult. The seasons immediately revolt and uproot the determined psyche. It is not only the ephemeral flower that wilts away awaiting or even completing the blossoming phase. The spirit fades into oblivion even when the body is living its sad course. To worsen the idleness there is a conflict between optimism and pessimism in which the latter rules the roost. Osho says that man is basically optimistic by nature. The belief that there will be the next breath is optimism per se. Life goes on with optimism. But it is true for the real life. What about the ‘plastic’ life?
What enormous strength is required to remain dispassionate amidst the whirlpool of plasticity all set to devour the human soul! Plasticity of this kind detests the freedom of human soul. And it is a monster few can escape.
Work keeps the soul away from many overwhelming confusions, which afflict the idle ones more. The confession has assuaged the injured consciousness a bit.
When one thinks of work, myriad images get conjured up in mind and one gets lost in the labyrinth of hierarchy. Someone else holds the reign and we are made a puppet of certain antiquated norms. They are antiquated because they can seldom rise above the lethargy of status quo. What gets lost in the process is the ‘I’ in one. It sounds contradictory when one says that there are so many layers of ‘I’ that even that is the genesis of labyrinth of ego. However, the latter one is destructive in nature as is based on the dead pattern of conformity. The former is constructive as it evolves from creativity. Both reside within the same soul and therefore create a confusion of likeness whereas they are poles apart.
Within a soul the latter exists parasitically at the expense of the former and the soul is led to believe that both are indispensable to each other. The difference is more palpable when there are moments of love and it seems that latter has vanished and the former has acquired even the better but morphed aspect of the latter-that of creative submission. This creative submission is different from servile conformity.
Love is the mother of all emotions. Even the painful longings have a ‘yogic’ detachment from the materialistic desires. This is the microcosm of the pinnacle of bliss- that of getting united with the divine. The divine flashes to give a glimpse. Thereafter the soul forgets the flash and complains that there is no sign from the divine despite persistent efforts to seek union with Him.

a patchwork of few cases

I found a floating patchwork of cases through which I am trying to find out the nature of the cases.


Traditional market allocates the available resources in the form of legacy. In this only the individual needful is produced and the surplus is exchanged with valuation and in comparison with some other product. This was earlier known as the barter system. Traditional economy had its loopholes they needed to plug-in like lack of currency exchange system, lack of quality products etc…
In this Asterix story, Gaulish village followed traditional economy paradigm.
There was the lack of common currency for them and the quality of fish sold in market was pathetic. As these both-market and trade- were replete with oligopoly, quality was a trade-off because of fewer dependencies.
On the other hand in a competitive economy there is no oligopoly and it encourages competition between different people to sell a particular product. With the emergence of competitive economy there came a paradigm shift in the ways the traditional market operated.
The comic story of Asterix and Obelix makes us understand the way capitalistic markets behave before and after they came into practice. Obelix and other folk members of Gaul used to hunt boars, drink magic potion, and lived life with gay abandon. The monarch of Rome planned to invade Gaul but Obelix thwarted his ambition by defeating Roman warriors. Julius Caesar then craftily employs Preposterous who taught at Latin School of Economics to teach the Gauls a lesson by his marketing strategies.
Preposterous falsely engages Obelix in manufacturing menhirs by doling 200 sesterts (Gaulish currency) initially and goes on doling increasing sesterts to engage him in always producing the otherwise dirt-cheap menhir. He gets the funding from Caesar. Ergo, there was an articial creation of demand by Preposterous and this made Obelix wealthy soon. Seeing his wealth others in Gaul started making menhirs and aborted their convention line of business (LOB). Very few were now hunting boars or catching fish in Gaul.
This infectious enthusiasm caught Rome and it also started producing relatively worthless mehirs. This spread to other countries too. There came a point of time when valuation for menhir got back to its realistic rate and then it led to an economic downturn. Menhirs were no longer in demand and those engaged in producing menhirs found themselves jobless. The economy of Rome and Gaul collapsed too.
The lesson from this story is that any demand, which has been artificially created or inflated, is bound to find itself plummeting in the natural course.
This is what is happening in the recessionary eras, especially in the recent one, perhaps the largest recession experienced after the Great depression in 1930.
Rome can be found analogous to America, which is reeling under a sub-prime crisis, created due to artificial demand for the housing bubble, which got bust. Gaul can be analogous to India, which has been dependent on American economy, but has managed to remain insulated and back to the original and happy ways very soon. Preposterous can be analogous to the economists at Wall Street who speculated greedily and took all sorts of wrong decisions leading to the entire world order thrown in the state of economic downturn.
This can be an interesting paradigm on how capitalistic system, with little checks and balances, can go awry. It also signifies how greed and incorrect understanding or manipulation of demand and supply can detrimentally damage economy on such baleful scales. Last but not the least, life of content and happiness is also a gift, which we fail to realize in our urban machinations.
Q1. How has Natureview succeeded in the natural foods channel?
Unlike the major players in the US yogurt selling brands-Dannon, Yoplait, and Beyers, Natureview used natural ingredients in its recipe for yogurt manufacturing, which became its USP in the natural yogurt foods market. As a best practice, it prepared yogurt from milk of cows untreated with rGBH , an artificial growth harmone misused by the major US brands talked above to increase milk production unnaturally. This best practice used by Natureview increased its shelf life to 50 days vis-à-vis 30 days of the top brands, enabling it to cut significantly on building multiple production plants. However, Natureview did not go for a trade-off with high quality and wow taste.
Therefore, Natureview’s yogurt became a big hit in the natural foods segment and facilitated it to liaise with big revenue churning natural food retailers like Whole Foods and Wild Oats, who also found its high quality and tasty natural yogurt immensely good as it was also based on creative, low-cost ‘guerilla marketing’ strategy. It offered higher margins over other dairy products. Over a period of time the natural food retailers became the niche and loyal customers of Natureview. The consumers were also loyal to Natureview’s yogurt as it was more health-supplementary. Besides, for more than a whopping 58% of US households, organic foods if lesser priced were in the highest priority. Natureview’s yogurt consumers rose in numbers majorly by dint of its excellent relationship and marketing activities synergistically with natural food retailers. By the year 1999 Natureview Farm captured 24% in the Natural Foods Channel and became the market leader in this segment.
Q2. What are the two primary types of growth strategies under consideration by Natureview?
Natureview is mulling over entering in supermarket channel and also over retaining its niche leadership presence in natural food stores who majorly helped it growing as big as it is now since its inception. The strategy was also to reflect and thereafter execute differences in business models, consumer audiences, and distribution systems per se in both channels.
Q3. How do the three options compare financially in terms of yearly revenue, gross margin, required investment and profit potential?
Option 1. The calculations using Exhibits 3 and 6 reveal that estimated profit using this option was approx $89100000 for Natureview not taking into slotting expenses in consideration. It leveraged a good shelf presence, besides incurring lesser slotting expense. It also had significant revenue potential as 8 oz cups represented the largest dollar and unit share of the refrigerated yogurt market. This segment was growing at a breakneck speed and Natureview was correctly positioned to capitalize effectively on this. Last but not the least, Natureview wanted to tap the first-mover advantage as there were rumors of its leading competitor in the natural foods segment was planning to make a foray in the supermarket channel. They currently generated a gross profit margin of 36%.

Option 2. The calculations using Exhibits 3 and 6 reveal that estimated profit using this option was approx $37500000 for Natureview not taking into slotting expenses in consideration. Despite 32 oz cups representing the smaller dollar and unit share of the refrigerated yogurt market vis-à-vis 8 oz cups, they currently generated an above-average gross profit margin of 43% vis-à-vis 36% of 8 oz cups. Besides, owing to this products longer shelf life Natureview had been able to capture 45% chunk in the pie of this segment and could easily have extended this leadership in 64 supermarket retail stores in supermarket channels. With only higher slotting expenses being the flip side, it was compensated by much lower marketing expenses pegged at $120000, which was only 10% of that projected for the 8 oz cups.
Option 3. The calculations using Exhibits 3 and 6 reveal that estimated profit using this option was approx $5770000for Natureview not taking into slotting expenses in consideration. This option seems to have its demerits superficially as it does not proffer higher profit and due to a host of other planning reasons. However, gross profitability of the line was 37.6% and the future growth potential was immense with 15% projected CAGR over the next 5-year and it could have lead to be yielding the strongest profit consideration of all the considered strategies.
Q4. What are the strategic advantages and risks of each option? What channel management and conflict issues are involved?
Option 1. Like mentioned before, option 1 leveraged a good shelf presence, besides incurring lesser slotting expense. It also had significant revenue potential as 8 oz cups represented the largest dollar and unit share of the refrigerated yogurt market. This segment was growing at a breakneck speed and Natureview was correctly positioned to capitalize effectively on this. Last but not the least, Natureview wanted to tap the first-mover advantage as there were rumors of its leading competitor in the natural foods segment was planning to make a foray in the supermarket channel. All this indicate that upside potential of Option 1 was immense.
The caveats were that the requisite quarterly trade promotions and meaty marketing budget incurred a lot of expenditure. In addition to this, sales, general, and administrative expenses(SG&A) were also a rip-off in this option 1. This could delay the break-even and could also result in lesser market share.
Option 2. Like mentioned in the earlier question, despite 32 oz cups representing the smaller dollar and unit share of the refrigerated yogurt market vis-à-vis 8 oz cups, they currently generated an above-average gross profit margin of 43% vis-à-vis 36% of 8 oz cups. Besides, owing to this, products longer shelf life, Natureview had been able to capture 45% chunk in the pie of this segment and could easily have extended this leadership in 64 supermarket retail stores in supermarket channels. With only higher slotting expenses being the flip side, it was compensated by much too lower marketing expenses. Launching a 32 oz cup in the supermarkets could make them stand in the competition of bigger brands and increase visibility and could also create awareness in the customers in the supermarket channels.
The risks as perceived by Natureview were that this multi-use size may be warily considered by new users. There was also skepticism over the credentials of the sales team’s competence in achieving full distribution nationwide within a year.
Option 3. Like mentioned in the earlier question, gross profitability of the line was 37.6% and the future growth potential was immense with 15% projected CAGR over the next 5-year and it could have lead to be yielding the strongest profit consideration of all the considered strategies. This option had perfect positioning in the core sales channel and had an easier distribution facility.
The risks were manifold. This option seems to have its demerits superficially as it does not proffer higher profit and due to a host of other planning reasons. Its foray in the supermarket channel could have risked the ire of natural food retailers more, who had enabled Natureview to stand to a position where it was now. It also lacked the requisite skill-set to enter the supermarket channel and was underprepared from marketing, sales, branding, and channel-partner arrangement perspectives.


Q5. What action plan should the company pursue? What changes in the current marketing mix, sales, brand, and channel partner arrangements do you recommend in order to implement the action plan?
Apart from mulling over which option to choose to enter in the Supermarket channel, Christine Walker, Vice President of marketing for Natureview Farm Inc., should start negotiating with Natural Food channels, their niche clientele and the ones who enabled to scale such heights from the company inception onwards as to how venturing in the supermarket channel with similar products priced 15% lower vis-à-vis natural retail channels will not dent the latter’s profit margin but will help them to understand themselves too how to effectively make a foray in a competitive market. The strategic cues derived from Natureview experience will also help them get a better hang of the shape of things to come, despite losing the first-mover advantage in certain variants.
That done, they can also offer the natural retail channels to enter in the supermarket channel in collaboration. For that both will need to do a detailed SWOT analysis of the feasibility of it and analyze whether it will lead to a definitive synergy or not.
Going for all the options in a piece-meal approach can be done by Natureview. However, it needs to raise the bar first wherever they find the Achilles Heel in marketing, sales, distribution, networking or any other underperforming department.
Skunk works can be done to forecast effectively with cautious optimism and staffing can be done afresh for it, if need be arises.

1. Why do so many countries want to get back on gold standard after World War I?

World War 1 turmoil brought economic upheavals across the entire world leading to swing in capital inflows. Gold became the international currency even in the backdrop of World War 1 turmoil. To fill in the money supply countries resorted to gold standards whereby they were able to go for ‘convertibility’ of notes into gold, which had become necessary.

2. Why did Sweden give up the gold standard in Sept. 1931?

Britain had a dominant influence on many other countries, which were recoupled. On September 21, Britain abandoned gold standard. Moreover, because of drastic fall in Swedish reserves gold continued to flow out of the bank and out of the jittery public. This lead to Sweden too abandoning the gold standard in order to preserve the domestic purchasing power of Krona using all available means.

3. Was Sweden helped by its statement that it would use “all available means to support the domestic purchasing power of krona “?

Sweden intended to impart the public definitive stand points for estimated future developments in prices and also to establish superiority of the new consumption index vis-à-vis other index series. The domestic purchasing power of the Krona was prone to fluctuations in the existing wholesale indexes. The development of new index was an important touchstone in the implementation of the price stability policy.

4. Why the Riksbank starting 1931 does, appear to be foreign on export rate stability?

Stable exchange rate was imperative at the monetary policy in Sweden post-1931 as it facilitated planning in both the planning and labor markets. Recommendations were made to increase rates when prices were rising and vice-versa. If it was not done it could have ramifications on real wages and on meeting immediate or pending liabilities.

5. Why was Sweden a pioneer in marketing and fiscal policies?

Sweden had a good pack of economists at the helm after the change in Social Democrats government. The emphasis was on the bigger picture by ensuring stable price for consumer goods. In lieu of it, the monetary policy makers pioneered to redeem beyond depression damages and thereafter ‘improve the internal price level and the needs of Swedish life.’ The stakes of the country were deemed higher than anything else. In this way, Riksband became the first bank to declare price stabilization as its policy norm with a floating exchanging rate.



Prima facie Tanishq, the jewelry division of Titan, the watch selling division of the famous Tata corpus, does not seem much too bogged down with any problem in dipping of sales even by comparative standards. It is uniquely poised as the numero uno in the niche upworldly mobile Indian with a market capture of 70% in the India’s total jewelry market, especially in urban market as it sells high-end gold and gem studded jewelry that is branded akin to western standards.
However, Tanishq launched an out-and-out plain god wedding jewelry GoldPlus in India’s smaller cities and rural locations by dint of research and development team’s strategy of implementing this as pilot in two locations initially for nine months and thereafter deciding further based on the performance metrics of the same within nine months, GoldPlus showed healthy signs of progress.
Tanishq, positioned by design as a western brand, has morphed latterly as a strategic rejig to cater to the conservative but modern Indian woman. Even as an oxymoron this segment exists in huge economies of scale. The brand still carries some baggage from its past. GoldPlus, on the other hand, is a new brand that is positioned to serve the plain gold wedding jewelry market. A variety of strategic, economic, organizational and brand investment reasons make the decision an important one.
REFERENCE: http://harvardbusiness.org/product/tanishq-positioning-to-capture-the-indian-woman-s-/an/507025-HCC-ENG?N=0&Ntt=Product+positioning
Tanishq or GoldPlus has ambitiously target a billion dollar sales by the year 2010. In India the consumer psyche is different in the jewelry segment as women in India are more enamored with gold, jocularly called the ‘behanji types’ and they value gold as ‘Streedhan’. Ergo, their capture of psyche was imperative for Tanishq to aggressively increase sales. In mofussil areas gold is an integral component and is wearable in many forms such as necklace, nose or ear rings, rings generally, nose pins et al. Women love to wear ornate gold designs as a force of habit. The audience analysis of GoldPlus should have been majorly focused on smaller town women. Tanishq was able to tap its niche market segment by dint of aggressive marketing like co-sponsoring gala fashionistas’ events like Femina Miss India contest, Milan Fashion Week et al and it also roped in Kolkata-based famous designer sabyasachi Mukherji as a brand endorser once. All these paid dividends. Nonetheless, not much was done on the girl-next-door front. A celebrity like Gracy Singh of ‘Lagaan’ fame or possibly Bollywood actor Vidya Balan with whom smaller town girls are better able to relate to vis-à-vis any sultry siren like Sheetal Malhar who appeared in a D’damas ad.
Buoyed by successes of watches in Titan, Tanishq was launched to tap similar potential in jewelry segment. Despite few failures initially in generating sales for Tanishq, it geared itself to meet the challenges by deciding to market precious studded jewelry under the Tanishq brand. As it happened in Titan’s watch segment, initially awareness had to be created by establishing service centers and by other means that electronic watches were actually better than obsolete mechanical ones and that indigenized Titan watches were the best solution. This strategy worked wonders and Titan became the numero uno within a decade beating public-sector HMT, its arch rival in this segment.
The Indian jewelry value chain largely remained initially an unorganized industry with more than 30 lakh ‘karigars’ or workers laboring hard to produce gold artifacts. The entire business paradigm was more focused on paying the karigars on a per gram basis as monitored by a head karigar. Less focus was given on innovating newer designs or styles. The skunk works in this case were not in place in Tanishq.
The development phase followed in Tanishq more so in urban areas because of other reasons besides quality standards and aggressive advertising. The consumers were willing to dole enough money in those areas for a fashionable cause like gold purchase. The years 2000 onwards witnessed a surge in profit booking for Tanishq along expected lines because of a slew of diversified products within the gold segment that Tanishq proffered. Tanishq rebranded itself from the year 2003 onwards that it was not just for the help English speaking fashion savvy modern women of India but also for women who had the eclectic mix of modernity and tradition. The perceived snob quotient of a typical Tanishq consumer was strategically alleviated and the other half of the society was imparted a new self-evolved transformed but grounded image of the Indian women in Tanishq ads.
Tanishq designed jewelry for the not-so-hit but critically acclaimed Oscar nominated fairytale bollywood movie ‘Paheli’ shot in the backdrop of rural Rajasthan, thereby envisaging again its changed focus on the market segmentation. The movie captured the lure of intrinsic psychological shifts in proclivities of a typical Indian woman, a motif that was supposed to lure the typical market towards GoldPlus. It tried to tap the majority of the jewelry market (well nigh near 85% and 42 million odd plus crowd). The positive effects shone as Tanishq stores had 80 stores spread across 60 towns across the length and breadth of the country.
Then came the paradigm shift from branding GoldPlus or Tanishq as ‘jewelry as adornment’ to ‘jewelry as investment’ considering that urban jewelry market was worth 24000 crore rupees vs 36000 crore rupees in the rural market. Moreover, ‘jewelry as investment’ market was alone worth 48000 crore rupees. Karatmeter, a machine that, in three minutes, measured the karatage of gold without destroying it, was introduced by Tanishq to authenticate the customer the genuine quality of gold and leverage the already famous Tata brand. It took care of the 18 carat gold segment alongside the more popular 22 carat segment and tried to spread the message that without a trusted brand one can be duped into buying a 14 carat gold considering it to be a 22 carat one. In the year 2000 itself more than 5000 women tested karatmeter and found themselves duped under under-karatage. The sales that year hit Rs. 100 crore mark and it inflated genuinely in the years to come.
Besides Bhaskar Bhat. Titan’s managing director and Venkat, head of Tanishq, marketing heads realized that the mark-up of 15% to 25% in Tanishq was not compensating for their stringent stands on the carat front in certain local markets. This also led to markup losses and to cut cost they put a moratorium on television ad costs by putting them on the hold.
Then ensued pilot project plans by Bhat in mid-2005 to launch GoldPlus outlets at two smaller towns- Erode and Ratlam. Supply chain costs were amply taken care of and multiple locations like these were shortlisted and the message was loud and clear for any average gold customer in these areas, “The tata name is like gold in India” and that “There is no company more trusted.”
Then again incentivization process by means of quizzes rewards, giveaways like car gifting on Diwali eve et al ensured rural market penetration of GoldPlus especially in Erode and not that much in Ratlam comparatively speaking. However, Ratlam experience provided Bhat with an insight into the failure of a model with no under-karatage problems.
So far it had been a smooth sail. Resentment started brewing hereafter as Tanishq staff got euphoric and started feeling that GoldPlus’ success in Erode was a flash in the pan and could not easily be replicated in 20 plus locations. Then again GoldPlus’ success anywhere was due to the principles that Tanishq imbued in it and it did not deserve further expansion as it could cannibalize the operations of Tanishq-its parent company. There was an inadvertent strain that developed between the two companies, which became the dilemma for the top brass of Tanishq.
Bhat and Venkat were already perplexed over the complexities that could arise with the foray of other major players in this segment like DeBeers. Then again they had to penetrate other target potential locations. The stand-off, even if it was a minor one, between Tanishq and GoldPlus was totally uncalled for. Bhat and Venkat need to call a centralized meeting of all the employees of Tanishq and GoldPlus staffers to resolve any notional issues, if any. All must understand that they are not disparate entities but a part of Tata fraternity, a legacy which rare staffers inherit.
It was very complacent of employees of Tanishq to believe that they had captured the psyche of Indian woman, something which even Hindu gods dreaded of doing- in a lighter vein so to say. The behavioral attributes of Indian woman- rural or urban- was far more complicated than could have been dreamt of in heaven and earth, Ergo, skunkwork needed to be done more on this niche segment for Tanishq.
Bhat and Venkat ought to realize that India is way more diverse and geographically agrarian to justify two pilot projects and then draw a hazy conclusion, which most of us tend to do while we hit the black boxes. The franchisee model was 90% and should have been exploited to perchance over 100 kindred locations on a pan-India level. For a brand like Tata and with huge undisclosed money flowing in such kindred locations finding the franchisees would not have been a bottleneck. However, it is mandatory to have checks and balances by implementing standardizations of systems, which is a cakewalk for a company of Tata flagship.
Besides, companies like Mcdonalds have already proved that apart from imparting quality products revenues can be created by ways of renting franchising models as the legendary Ray Krock managed to do even after the age of 50 plus.
By any chance, in a country so diverse as India consumer psyche what to say of just a woman psyche becomes so different that a wee bit of punter attitude will only be tantamount to be a calculated risk, if practiced.
Innovation is still the prime mover for any company to succeed. GoldPlus is in the process of improvising on it. Here is a brief lowdown on how they are trying to do that:
GoldPlus, a mass-market jewellery brand from the house of Tata (Titan Industries) was launched at Bheemavaram and at 15 places across Tamil Nadu, Madhya Pradesh, Andhra Pradesh and Maharashtra and has received resounding acceptance from the consumers. GoldPlus is targeted at semi-urban and rural areas and aims to herald standardisation in a largely undifferentiated market by establishing quality benchmarks.

The Bheemavaram GoldPlus showroom was inaugurated by Meera Harish, head, strategy planning, Titan Industries.
Promise of purity
Speaking at the inauguration of the GoldPlus showroom, LR Natarajan said, "Gold has tremendous historical, religious, cultural, social and economic significance across our country. However, the absence of standardised practices in markets often leads to a latent fear among consumers about the purity of gold. GoldPlus from Tata aims to create awareness about gold purity and thereby set new jewellery standards in Bheemavaram."
Harvest, marriages and festivals are the key drivers of jewellery purchases in Bheemavaram. GoldPlus aims to establish an emotional connect by participating in every gold purchase need of the consumers with the promise of purity.
Pure gold, sheer variety
An extensive range of over 5,000 designs in 22 kt plain gold and diamond jewellery will be offered in the showroom to cater to the needs of quality-conscious consumers of Bheemawaram. GoldPlus stands for purity and fulfilling promises, and in line with this, purity certificates would be provided for all GoldPlus jewelry products, which guarantees the purity mentioned with the ornaments.
Backed by Titan's experience
GoldPlus will leverage the design and retail expertise of Titan Industries to present market relevant offerings catering to local tastes and preferences. The product range, showroom ambience and marketing efforts will showcase the company's superior understanding of local consumers. Titan's investment in IT and retail will further enable GoldPlus to achieve cost-effective supply chain logistics, which will deliver greater value for our customers.
Innovative golden schemes for all
GoldPlus runs a monthly saving scheme called "Golden Future Savings Scheme", which offers buyers the benefit of getting more jewelry than what they have paid for. The scheme allows consumers to plan future purchases in advance and pay for them in easy instalments. An attractive gift would be available for all the enrollers.
REFERENCE: http://www.tata.com/company/releases/inside.aspx?artid=iEryTTxfKjE=
Tanishq needs to do legworks to revamp its marketing strategy in order to brace up to the challenge of changing business dynamics. It can be perceived as a double whammy or a synergy but will inevitably benefit the Tata corpus in the jewelry segment. Bhat and Venkat need to keep on reminding Tanishq staffers the good work that will simply propel their sales growth without doing anything extraordinary but by simply following the basics as they used to do earlier. Sometimes, out-of-the-box thinking is not required. In lieu of it, conventional tested model of success needs to be practiced further. Here is an example how Tanishq did it once. They can do it again and indefatigably so. Yes, they can!
High-end jewellery retail chain, Tanishq, is likely to see a 15-20 per cent rise in gold jewellery sales in the current financial year even if prices remain volatile, a senior company official said on Wednesday.
The jewellery division of Titan Industries Ltd. would be able to increase sales as the clients it targets are not very averse to high prices, C.K. Venkatraman, chief operating officer of Tanishq, said in an interview.
"Tanishq's customers are slightly above average in terms of income and quality consciousness, therefore less price-sensitive," said Venkatraman. "My sense is that volume growth for jewellery alone would rise by 15-20 per cent (in the full year)."
Sales of gold coins and bars could see an even bigger growth as investment-driven buying of the yellow metal catches on in the country, he said.
Venkatraman said consumers are more brand conscious from 10 years ago, when Tanishq began operations - a trend that could help boost its appeal.
"We have typically been getting stronger in the wedding segment," he said. "This has driven our sales significantly."
Also, the aggressive advertising and promotional activity undertaken by the company during April to June, could push gold sales further, he said.
In April-June, Tanishq's net profit fell to 55.8 million rupees from 92.4 million rupees a year ago.
Venkatraman said the decline was largely due to the increase in expenditure on brand building.
"We have made a lot of investment in advertising and marketing in the first quarter, which will pay off in subsequent quarters," he said.
Across India's bullion industry, high and volatile gold prices have affected trade with many dealers speaking of a sales drop ranging from 5 to 40 per cent last month.
Gold prices have risen significantly since the beginning of the year and have been quite volatile, touching a 26-year peak of $730 an ounce in May, before tumbling to $543 in mid-June.
On Wednesday, spot gold prices were at $648.1, a rise of 50 per cent on year.
Tanishq plans to open several new retail outlets by end-2007 taking their number to 100 from 82 now, Venkatraman said.

Tanishq contributed 32 per cent to the net profit of Titan Industries in the last fiscal year that ended on March 31.
REFERENCE: http://www.financialexpress.com/old/latest_full_story.php?content_id=136052

Violation of rules is consequential of lack of ownership. If a person is made to feel that he owns a particular task it instills a trust and he does the task better as he is the master of that task and is not enslaved in it. Once work at Wixom was over the workers were deemed redundant and small favors like mailing paychecks were not considered necessary.
The roles assigned at Wixom were extremely confined and there was no scope of power enhancement that could have triggered independent decision-making that majorly results in extra and positive contribution. The workers were doing the run-of-the-mill work leading to ennui. Fatigue in the speed of the line had majorly triggered half a dozen local strikes of United Auto Workers Unions members at other auto assembly plants in the past few years.
Job empowerment often motivates the worker to walk the extra mile for the organization and there was simply no time for the Wixom management to mull over it as the only focus was on churning out cars at a breakneck speed unmindful of the future prospects (or lack thereof).


Hiring process at Wixom was not meticulous as it should have been. It was done just because there was demand for manpower due to rapid turnover and a major expansion of Wixom. However, foresight was missing as unskilled or incompetent workers did enough mistakes that were detrimental to Wixom’s success in the long run. For example, a few workers messed up with installation of color vent plates because they were colour-blind and there was no check-up for colour-blindness during the recruitment process. Most of the workers were dissatisfied with the wages they were receiving and the management at Wixom was blissfully oblivious of the resentment that was brewing within the workers. There were fair chances of it imploding, which was very harmful for Wixom. Leave aside poor wages, there was no incentive to perk up the motivation of the workers, which was already at an abysmal low. There was always the risk of the worker switching over to other company because of a pay-hike.

Besides, the work was mind deadening in the sense that same work was done day in and day out leading to no value addition. The unintelligible work killed the human being inside the workers and made them a ‘Robot’ aptly summed up in the command, “When you are at job, leave your mind at home.” Eventually, the efficiency in the worker was lost.
Handling diversified portfolios and exchanging duties can be the possible solutions of this malaise. Instead of a single job being done by the worker all throughout, different jobs in different time slots can be allotted to insure that the worker breaks free of the monotony. This can lead to higher efficiency and productivity of the worker.

Sunday, December 6, 2009

Numero uno in test wow India

Good to claim the top slot in test cricket for team India. However, their reign is shaky because of fewer matches under their belt. A fantastic team it is and it will not be lonely for them to stay at the top as the team is a big one with a batting order that can bat uptil number 7. The inclusion of Zak has bolstered the bowling attack with the spiners Bhazzi and Ojha looking good if not ominous. Sehwag is redefining test criket batting pecking order and Sachin is already high in praise for the current team touting it as the best India team side he has ever played with. As a die-hard Indian cricket afficando I am wasting my time blogging on it, which I am sure few wil read.
Par confidence to dekho boss!!!
Aisa hi confidence tha when I sat idle after B.Tech. to write a prosaic fairytale long story of a love affair and few of my friends hailed it as a 'national comedy'. I am planning to rewrite it but after preserving the original manuscript just to showcase how a starry-eyed youth succumbs to reality in his early thirties and tweaks the entire script in a realistic fashion. Is is the right thing to do? It is debatable and an open-ended discussion can follow suit. Practically, this was destined. Philosophically, this is perhaps not right. Let me see if I can manage muster to do an encore!

Saturday, November 28, 2009

CAT online exam goes bust at places

Technical glitches in CAT 09 left a few candidates feel like marroned in a cast away island of intellectual imprisonment. This a blot on their image and CAT-Prometric double whammy are in damage mitigation mode. It is indeed conventional but time tested to work in pencil-and-paper format. Why fiddle with the conventional toeing the lines of the Western counterparts? They should have taken a cue from XLRI's abortive attempt few years earlier.
Sherni manti nahin hai na sher ki mook samarpan ki baton ka aur kutta jeet jata hai:)
I read a case study of University of Phoenix wherein a lot of effort was done on 'flexible' learning instead of 'engaged, experiential, interactive ya lauda lahsun' kinds of pedagogies...
It goes without saying that that majorly the efforts were for-profit ones but what else can one expect in this ratrace! A rat has such a small asshole but is literally fucking human asses globally.
However, hazar galiyon bhi prashansa lagen to samjho ki jeeven ke kaante bhi phool ban rahe hain-Osho

Wednesday, November 25, 2009

A crown of flowers

Feeling worked up with substantial reasons... Watched the documentary 'Cannibal Holoucast' and felt like puking at places... Amazon is so wild and we mostly read Amazon.com...Seeing it I gave hearty thanks to my parents for making a human out of me, albiet a worthless one. Then I thought that it is better to be a sensible cannibal than to be a nonsensical human. The Zen effect was so much pronounced in their lives...It is tough due to varied frames of references...How to make the transition as smooth as ever?

Tuesday, November 10, 2009

lost in jargons

God knows where exists a jargon-free world...
Again watched cast away and became ekdum outcast thereafter...
Even rural management is so urban nowadays...
missing rajuapur-my village...
i live in a dingy congested room and have never even stepped inside a villa but village is all where i want to be...
God gimme back my rustic lifestyle...
abhi to hippie wali ban gayi hai kuch salon se...
God save us...
give me my daily bread of my own effort...

Friday, October 30, 2009

Musing in the times of Swine flu!

Heard somewhere that love is blind. I truly wish it were! If it were not see to t that a million imaginations would not get lost. The dreams are like scintillating star that can also power the reality at times. It indeed comes easily to half-aroused intellgence and becomes hopelessly shattering at the final awakening. Decades have changed and so have our ways of loving. However, in each of the ages it has been as intense if one does not look through the eyes of a cynic. Life is not despondent and is an expression of love that engenders happiness. Opposite values are complimentary to each other and similar values are symmetric. And tis our faith too Mr. Wordsworth that every flower enjoys the air it breathes.
The darling winter is back again. It can be hypothermic but so is love. Chills our heart to extreme severity. Ah,you are no longer in physical periphery and you almost vanish in the mudane anxities of earning our daily bread. But you come frequently to remind of a world beyond a parallel self wthout the need of pelf. And more things are wrought by prayers than this world dreams of!!!

Thursday, October 22, 2009

Bangalored in India

In Bangalore on a meditative trip, which is not apparently all that meditative...
In a campus, the beginning itself began not with a wlecome but with a warning of no alchohol, cigarettes, drugs, tea, coffee etc etc, which prompted me to sip a cup of coffee after a long, long time. God knows when they will stop prohibiting Eve from eating a particular fruit from a particular tree!!!
But on the health front it was good and can be reckoned as a good 'prana' management. The homilies delivered were mundane. Nonetheless, it helped me in abstinence from a few good substances for few days!!!
Looking forward to earn a pittance very soon...

Thursday, October 8, 2009

gift of magi

Life is beautiful ended with a fictitious(for a child) tank gifted by a father to his son who sacrificed his life to save his Jewish son from the Germans during Hitler's regime...
Superb!!!

Monday, September 21, 2009

The muse appeared

It was a flash after so many years but kept me satisfied till now!!! A serendipity indeed!

Wednesday, September 9, 2009

Fortunate accident

Serendepity, serendipidiocity, serendipitous, and serendipaciousness are all good and related words nealy meaning fortunate accident, especially if one is looking for something else. A good movie in 2001 featuring John Cusack and Kate Beckinsale. A good watch! One can repeat watching it or writing on it!

Something about the word 'Serendipity'from the wikipedia:(I hope it is not plagiarism):(

Etymology
The word derives from Swarnadip, the Sanskrit language name for Sri Lanka, and was coined by Horace Walpole on 28 January 1754 in a letter he wrote to his friend Horace Mann (not the same man as the famed American educator), an Englishman then living in Florence. The letter read,

"It was once when I read a silly fairy tale, called The Three Princes of Serendip: as their highnesses traveled, they were always making discoveries, by accidents and sagacity, of things which they were not in quest of: for instance, one of them discovered that a camel blind of the right eye had traveled the same road lately, because the grass was eaten only on the left side, where it was worse than on the right—now do you understand serendipity? One of the most remarkable instances of this accidental sagacity (for you must observe that no discovery of a thing you are looking for, comes under this description) was of my Lord Shaftsbury, who happening to dine at Lord Chancellor Clarendon's, found out the marriage of the Duke of York and Mrs. Hyde, by the respect with which her mother treated her at table."

Role in science and technology
One aspect of Walpole's original definition of serendipity that is often missed in modern discussions of the word is the "sagacity" of being able to link together apparently innocuous facts to come to a valuable conclusion. Thus, while some scientists and inventors are reluctant about reporting accidental discoveries, others openly admit its role; in fact serendipity is a major component of scientific discoveries and inventions. According to M.K. Stoskopf "it should be recognized that serendipitous discoveries are of significant value in the advancement of science and often present the foundation for important intellectual leaps of understanding".

The amount of contribution of serendipitous discoveries varies extensively among the several scientific disciplines. Pharmacology and chemistry are probably the fields where serendipity is more common.

Most authors who have studied scientific serendipity both in a historical, as well as in an epistemological point of view, agree that a prepared and open mind is required on the part of the scientist or inventor to detect the importance of information revealed accidentally. This is the reason why most of the related accidental discoveries occur in the field of specialization of the scientist. About this, Albert Hofmann, the Swiss chemist who discovered LSD properties by unintentionally ingesting it at his lab, wrote

It is true that my discovery of LSD was a chance discovery, but it was the outcome of planned experiments and these experiments took place in the framework of systematic pharmaceutical, chemical research. It could better be described as serendipity.

The French scientist Louis Pasteur also famously said: "In the fields of observation chance favors only the prepared mind." This is often rendered as "Chance favors the prepared mind." William Shakespeare expressed the same sentiment 250 years earlier in act 4 of his play Henry V: "All things are ready if our minds be so."

History, of course, does not record accidental exposures of information which could have resulted in a new discovery, and we are justified in suspecting that they are many. There are several examples of this, however, and prejudice of preformed concepts are probably the largest obstacle. See for example [1] for a case where this happened (the rejection of an accidental discovery in the field of self-stimulation of the brain in humans).

Monday, August 31, 2009

happy bday

Time flies and life has come full circle! :((satire)
Forgive the lethargy and forget the excess baggage from the past. Marge has grown a year older. Her birthay is on 2nd Sep. i.e. today (the blog date is incorrect).
Happy bday Marge!

Sunday, August 30, 2009

Monk with a glass of wine

A monk exists who meditates after getting a kick with a peg or two.
"There are lies, there are damned lies, and there is Statistics."
Jaisi hai waise hi bacha lo re duniya ori duniya ri duniya...
Which state is real: the sober or the inebriated?
The insane laughs at the sane and vice-versa. Pagalon ki mehfil me bhagwan bhi hans pade ki chalo koi to alag mila...
Ah! how blissful it is to lose mind at times!
The arrogance of youth, smile of child, beauty of teenage girl, the orgasm in art, the greenery of farm, the evolution of a flower...
The natural happiness lost but not found since so many years...
The absence of eternal love once held dear to the heart...
The shame of failing to live up to expectations of family...
The farce of management in everything...
The desperation of a schizophrenic to socialise...
The severe homesickness of the dislocated...
The failure to conform to the norms...
The loss of motivation...
The dread of accepting the so-called reality of a loser...
The inability to generate own capital and the ability to destroy other's capital...
The ignorance of death approaching slowly but faster...
The desire to feel the bliss of the Supreme...
Yeh duniya agar mil bhi jaye to kya hai?
pan duniya hai to hai...
Great expectations gone awry but nonetheless they are expectations...
A moment of artificially created conviction that gets shattered by reality...
pan jaisa hai waisa hai aur shayad accha hi hai.:)
Rock on!

happy bday in advance m...point someone

mrs ergo virgo,
tumhe to pata bhi nahin hai ki koi haapy bday itne salon ke bad advance me bhi kah sakta hai...
anyways,
happy bday in advance...:)

Sunday, August 23, 2009

Accha hai sawere-2 gussa karna

Torn out of context again with the familiar disconnect, it was fun-filled evening again at a marketplace. Doing window shopping was like meditating in the marketplace.
Having rural upbringing, it was tough to enjoy in a mall but somehow it was manageable...

Saturday, August 15, 2009

how dare they detain srk in us?

it is a national shame...
our brand icon srk being detained in us for some trivial reasons..
aur talua chato america ka...
nicole kidman ati hai jodhpur me to pura desh pagal ho jata hai bhai...
usko detain karo delhi airport pe swine flu ya aids quarantine ke liye...
kahanse karoge...
mann mein hi gulami aur nakal bhari hai...
despite all, srk does not need america or anyone else to rock...
he is the best...
yeah, ashok kamte got a road named after him in mumbai...
thatz gr8...

Tuesday, August 11, 2009

Tower of Hanoi

This problem is interesting and can be solved by recursive algorithm.
The link is:
http://en.wikipedia.org/wiki/Tower_of_hanoi
The number of ways for n weights is 2^n-1.
The logic is yet to be found by me on net or elsewhere...

Monday, August 10, 2009

Zen or the art of mismanagement

The Zen approach in which one is ‘in the moment’ and rationality can be enjoyed together as per Pirsig. Quality of life can be improved by a synergy of rationality and romanticism. In some cases it becomes a trade-off, which can be different from compromise. Few wonder how mismanagement can be viewed in a better perspective and does any benefit accrue from it even if it does rarely so. Do non-intellectualizing and non-conceptualizing have their charm at times? Hitting the black boxes most of the times takes the zing out of things and that is what one is made to believe mostly. Zombies and dumbos are better survivors. The human frailties carry baggage from the past and stigma suffocates sometimes. Is it not okay to be frail somewhere? If not, then one is in the wrong planet or maybe in the wrong era. Call it quit dude before it is again too late. Do not be so unabashed to shamelessly try again living in a civilized society and do not look up to anyone anymore now. You have to take care of everything irrespective of what comes of it.

Monday, August 3, 2009

Bhatija's birthday night was fun-filled

my bhatija is a year old. I enjoyed serving drinks to tipplers and was not too keen on serving food. In a pepsi bottle I concocted blender's pride with beer and then we relatives got nostalgic over our rural and family connections. Brother was his sober self and acted like a perfect host.
Anyways, what is the remainder when 25! is divided by 10^7 and what is the first non-zero digit of 96!?
Spot the connect!:)
Now in NCR for sometime...

Tuesday, July 28, 2009

5th day of rehab

What kind of rehab is this? Desperately waiting for it to get over! Today the nurse asked me to open my nimbu hips for injection. I opened it fully and she reprimanded me saying that, 'Itna nahi re baba bas thoda sa dikhane ka'.I was apologetic and covered the sundry space. It was a mixed emotion-a lady nurse giving me an injection on my nimbu hips and the pain of the injection. Nonetheless, the 5th day is over and the last the booze day is today:). I will at least satisfy myself that I successfully finished a 5 day rehab hahaha...Well nigh it...
Soch hi choti ho gayi hai sali:)

Mom, dad, family and me

 
Posted by Picasa

Monday, July 27, 2009

drying out is very dry but have to try for it

I bought the medicines and was injected strength twice within 3 days. It was difficult to find a muscular area in my framework as even my hips do not lie and are as starved as ever.
However,the going has been okayish so far...
it is a 5 days rehab and 3 have already passed by. Lost quite a lot on the academic front but more confindent on the personal front now!

Tuesday, July 21, 2009

Nath sampradaya

it was interesting to read about the basic tenets of Nath Sampradaya following Nath cult originally propounded by Guru Gorakhnath Ji. He has a temple named after him in Gorakhpur which was his seat of divinity.
The kanphatiyas have their ears pierced and then wear a ring just to prepare themselves for the rigours of yogic processes ahead.
Guru Gorakhnathji propunded Hath Yoga, which has several processes which if one follows truly one attains the divine bliss and becomes a Yogi. A wandering Yogi is untouched by any materialistic desires and has transcended any temporal limitations that ordinary people have. He attains Samrasya i.e. spiritual unity of all existences in the nature of the Supreme Spirit, he then transcends the difference between matter(jada) and spirit(chit), body(deha, pinda) and soul(atma, pada), finite(santa) and infinite(ananta), the temporary(anitya) and the Eternal(nitya), the changing(parinami) and the changeless(aparinami), plurality(vahutwa) and unity(ekatwa), duality(dwaita) and non-duality(adwaita).
Great thing it is to become a Yogi but a very-2 difficult one.

Friday, July 3, 2009

birthday paradox

I was going throught the question that what is the minimum batch size required in a system to ensure that two people at least have the same birthdays and did a simulation of that in excel.
However, I thought that if we can have a maxima or minima funda in P(n)=1-365!/(365-n)!*365^n we can find the value of n.
The answer is 50 approx.
Let us see.

Thursday, June 11, 2009

Outbound kept me bound out

It was fun and interactive learning at an outbound programme at Nainital. It started from Delhi and culminated in situ. The case study and adventure activities made me feel a tad younger and I now love even more the caveman from the days of yore.

Wednesday, May 13, 2009

interpret the concept of gulaal

I think it exhorts us to shed real blood when the need arises instead of masquerading with a gulaal on face. 'Jack and Jill went up the hill hi hoga to krantikari kahan se layenge bhonsri ke' was a good dialogue by Dukey Bana. Do we need to outsource revolutionaries?
The flip side is that you cannot trap a sun's light in a box try as you may. It will escape nonetheless. Negativity is real but is again so because we have forgot positivism.
A fisherman fainting in a street full of perfume vendors! A perfume vendor fainting in a fishmarket! Which one is a better example? Both are real.
Phir bhi badi-2 baten karna asan hai par ek normal human being wants reality irrespective of the cognition of it being really real! Let us face it and own it up!

Sunday, May 10, 2009

Gulaal rocks at orkut too

Great movie with amazing lyrics, cast, music, theme etc etc.
सरफ़रोशी की तमन्ना अब हमारे दिल में है
देखना है ज़ोर कितना बाज़ुए कातिल में है
वक्त आने दे बता देंगे तुझे ए आसमान,
हम अभी से क्या बतायें क्या हमारे दिल में है

ओ रे बिस्मिल काश आते आज तुम हिन्दोस्तां
देखते कि मुल्क सारा यूँ टशन में थ्रिल में है

आज का लौंडा तो कहता हम तो बिस्मिल थक गए
अपनी आज़ादी तो भैया लौंडिया के दिल में है

आज के जलसों में बिस्मिल एक गूंगा गा रहा
और बहरों का वो रेला नाचता महफ़िल में है

हाथ की खादी बनाने का ज़माना लद गया
आज तो चड्डी भी सिलती इंग्लिसों की मिल में है

वक्त आने दे बता देंगे तुझे ए आसमान,
हम अभी से क्या बतायें क्या हमारे दिल में है
सरफ़रोशी की तमन्ना अब हमारे दिल में है
देखना है ज़ोर कितना बाज़ुए कातिल में है

मुल्क़ ने हर शख़्स को जो काम था सौंपा
उस शख़्स ने उस काम की माचिस जला के छोड़ दी

बनता था तैरने में उस्ताद शहर का
कर गोमती को पार तेरी माँ का ___
kya mast lyrics hain...
chaddhi pehan ke phool khil gaya aur majak majak me picture bhi badiya ban gayi bhai!
mera bhi chashma gol hai isiliye meri understanding bhi gol hai aur meri koi sunta nahin aur main bhi sapnon me sochta hoon tashan me kabhi-2 ki i m a senior cambridge passout!
meri bhi ajadi kuch saal pehle laundiye ke bill me thi!
kabhi-2 main bhi sapnon me chillata hoon ki i am a senior hsc passout.:)
mera bhi chashma gol hai isiliye meri bhi koi sunta nahin hai kyonki understanding bhi gol hai...
jokes apart, amazing lyics and a different genre of creativity altogether! Anurag Kashyap, Piyush Mishra, Amit Trivedi, Kay Kay Menon, Masculine Ransa and all of them rock for sure!

Friday, May 8, 2009

Steve Waugh cleans muck in Kolkata

He heaped encomiums on the Little Master-Sachin-comparing him with the Don Bradman; I love any legend praising Sachin(most of them do that!)Inspiring legend he is! His muck-cleaning act instead of muck-raising that one is wont to is a tad different for a celebrity of his stature.I will try to do an encore in my gali/muhalla as I feel that at this point of time and as an individual this is the best I can offer to the society as a token of gratitude for everything...
God bless Udayan- his NGO that works as a rehabilitation home for children from leprosy colonies near Barrackpore in Kolkata!

Monday, May 4, 2009

Big B in the 80s-a genius actor

What a man he used to be! I was again watching NAMAKHALAL!
Amitabh Bacchan is a complete package insofar as acting is concerned...
'tum mere daddu nahin ho sakte' and that cricket commentary...
Oh my God!
He along with Sachin and the young SRK are the biggest rolemodels for me after my mother and God!

Sunday, May 3, 2009

Raju ban gaya gentleman!

Nice walima it was yesterday and Raju is a gentleman now...
Hashim Sir worked extremely hard to mange the event and so did the members of his retinue...
Yesterday again I was tempted by the mutton and held back but had a bit of gravy instead. Ergo, I am an imperfect vegetarian(gosh, perfectly imperfect always!)
Shahi Tukda was as usual extremely sumptuous and tickled my soft palate...
Due to extremely hot conditions, I had to take the help of one-rupee pouch of digestive Shodhi-harre...
The breeze in the night was gentle and very soothing though and it helped in feeling better after braving the onslaught of scorching heat waves in the afternoon.

Thursday, April 30, 2009

a good nikah today

May God bless his marriage forever! It was of Raju- who is in the car driving business. He is not single but double from today. We went to Gola bazar near Gorakhpur and it was a good one. Only that the scorching heat played spoilsport to the joyride that it was supposed to be! Depite that I saw the farmers toiling to grow tamatar, bel, dhaniya, neneua and many other good things that are dirt-cheap over there but are lmost a rip-off in cities. I was tempted but stayed away from non-veg(mutton) in the food as I have been doing since past two years. I must say that these khansasamas are different and better than the rest. Somewhat like what Karim offers you in old Delhi.
Came back and had a chat with friends in the evening; had a dosa at a hang-out and then watched Jerry Maguire- a good movie, featuring the trials and tribulations of the sports fraternity! Like a no-brainer, I enjoyed the spectacle and despite that it is a cynical world especially in tough compettive businesss world showcased in the movie, I enjoyed the finer emotions in a human being.

Tuesday, April 28, 2009

Kya pata- maya memsaab!

Yeh kya pata bahut khatarnak bimari hai...
yeh kya pata hota kyon ho?
kya pata?
kya pata man me koi paap ho
kya pata kisijogi ka shrap ho
kya pata dimag me koi keeda ho
kya pata bachpan ki koi peeda ho
kya pata khoon mde koi bhed ho
kya pata asman me ched ho
kya pata koi dukhti hui rat ho
kya pata man me koi thug ho
kya pata sitaron me kharabi ho
kya pata pardada koi sharabi ho
kya pata bhagwab ki koi bhool ho
kya pata ankhon ki dhool ho
kya pata manufacturing defect ho
kya pata special effect ho
kya pata raat me kuch khaya ho
kya pata kuch khoya ho, kuch paya ho sab maya ho
it means ki tumko sab pata hai:)
Awesome movie, music, cast, cinematography etc etc and last but not the least and the novel madame bovary!
Gulzar comes up with brilliant scripting!

Friday, April 24, 2009

Sachin 36 not out!

Happy birthday to my greatest role model after my mother- Sachin Ramesh Tendulkar! He is now 36 not out and hopefully will continue regaling the sports fraternity for years to come. Cricket-wise he is 19 not out. He rocks!!!

Sunday, April 19, 2009

KKR taken for a ride

Buchanan's multiple captaincy theory has failed at least on the batting front or else KKR would not have been bolwed out for a paltry 101. Dada was very slow today even by test standards scoring 1 out of 12 balll. Was it deliberate as he did not anymore believe in letting the bat do the talk to slam the detractors like Dravid did yesterday? Whatever it was. it now only calls for immediate axing. Firstly, the performance even at critical stages in T20 cannot be so lax. Secondly, it only adds fuel to the fire raging in the KKR bastion regarding the captaincy conflict. SRK cannot afford this much at this point of time.
It has been strange but all the teams mired in controversies of any kind prior to the beginning of the IPL-2 gala event have struggled especially Rajastah Royals who were skittled to an abysmal low score of 58 all out in reply to a modest total posted by Royal Challangers. This perhaps is an indication of the shape of things to come.
T20 weds cricket with entertainment. Purists are perhaps right in declaring that this slam bang format is not the real cricket. Then let the teams spare themselves from any controversy that can turn into a spoilsport and might also hang like the damocles sword in the matches to be played. Let us take it sportingly and do not make a masala conflict movie out of it.

Friday, April 17, 2009

mushy-mushy honey-honey

Sometimes it is delightful to watch those mushy-2 4 layered naam wali movies...
A huge let-out against the routine idleness of mine!
Though there is a feeling that it is illusory and will not last long, the heart is so absorbed in that very moment that it does not care of anything else!
Good if I had more of such moments!
The time enjoyed wasting is not wasted time!:)

Friday, April 10, 2009

IIM results are out

Nothing to do with the results but got a lil bit curious about the current crop which is in and will be embarking on a new journey with the entry in the hallowed portals. After all, they are now in the reckoning for the best positions in the society! IIM call-getters are rare and IIM-BLACKI converts are rarer. Kudos to all of them and may the best be rewarded for their successes!

Tuesday, April 7, 2009

Is Amir taller in malls?

I had a photo shot by friend at a mall in Noida in which I was resting my shoulders on a mannequin of Amir's Ghajini caricature. I am nearly six feet tall and Amir is just five point someone in height. But it was not the same in that model. I was shorter there, which can mean that it is rare to have a larger-than-life image.
Gulal is interesting though a rather dark movie. How realistic can we actually portray? For example, can we show on the screens the relief when pressure is off the morning shit? I am at loss to understand the extent to which we can stretch the realism factor in cinema. Sometimes it makes me feel that the real world is really a sham but do we have the balls to accept all?
i do not know if there is a split personality which pines for a simple sweet notalgic song from the movie Rajnigandha and also is taken aback when I encounter the vulgar realities in certain movies like Gulal. Will try to figure out the paradoxes in future if I do not technically grow insane in that process.

Thursday, April 2, 2009

CAT verbal raw content dump

VERBAL SECTION

CAT 08 carried the maximum weightage on Verbal section, which left quite a few flummoxed as to why this injustice to other sections. Let us face it. CAT is not much of a speed game nowadays and globalization requires a wannabe mager to be comfortable in the most standardized medium of communication by dint of English language. We cannot start without that. CAT has evolved over the years. It has now become a test which has more meaning than the earlier so-called intelligence per second test. With the time limit going up by 30 minutes, that is, a good 25 percent, CAT is now focusing more on the reasoning and managerial abilities of the candidate.


Coming to English, students can be categorised as under:
1. Those with strong English
2. Those with okay English
3. Those with weak English


Each category approaches the CAT differently.

The first category of students with strong English, will attempt the English section first, allot more time to it and attempt maximum questions to increase the percentile. I have seen candidates devoting 60 minutes and getting a net score of 40 which easily propels them to 98 percentile and above (based on the difficulty level of CAT 2007), without compromising on other sectional cut-offs.

The second category of students is likely to attempt the English section last, devoting adequate time and attempting sufficient questions to clear cut-offs and get good marks.

The third category of students will focus only on cut-offs. Once they attempt a reasonable number of questions to clear cut-offs, they move to their stronger areas. Here, students tend to focus more on English usage rather than reading comprehension (RC) questions.

CAT English strategy

RC should not be attempted first or last. In the beginning there is usually a lot of disturbance as candidates settle down and invigilators check the admit slips. Towards the end you are under tremendous time pressure and the likelihood of errors goes up. It’s better to attempt English usage at this time.

Students who feel RC or EU are their forte should attempt more of those questions. Don’t guess wildly because the penalty is high – one-third plus the question you got wrong.
The English section of CAT tests your reasoning abilities. In the RC section, questions tend to be tricky and the options are very close. If you have not been able to reason out the passage and the author’s point of view, you will not be able to identify the right answer. In CAT, reading speed is not that crucial but your capacity to read diverse topics and comprehend them well is more important. If you have not developed the skill of reading and understanding diverse topics then it is of no use. So, it is important that you read a lot.

One should read editorials very carefully, make a summary and note it in a register. Include new words you have come across with their meaning and usage. One should also develop the knack of finding the meaning of words from its usage in the article.

Students who are aggressive readers, who have gulped information from all the books in the library, are likely to score more in English. Their voracious reading habit will help them not only in RCs but also in paragraph jumbles, critical reasoning, fill in the blanks, parentheses etc. In paragraph jumbles, finding the right link is crucial for the right answer. In fill-in-the-blanks and parentheses, understanding the sentence or paragraph well is crucial to pick upon the right option.

Students who are late risers should practise and revise the material given to them by their respective training institutes. They should definitely allot 2-2.5 hours daily for reading newspapers and business magazines. Please develop the habit of arriving at the themes of whatever you read.

Earlier the emphasis used to be on vocabulary, but now the paper is moving more towards deductive and inferential reasoning in language. So, it is a question of knowing not only the meanings but also the correct usage.

Vocabulary is a concern area for many. If you read consistently, vocabulary should not be a problem. Otherwise, you can master vocabulary through mnemonics and associations.

Grammar guidelines

Grammar is basically the set of rules governing a language. Is it necessary to know the rules to master grammar?

Literal comprehension of the rule is necessary. Only repeated exposure (written and oral) to the rule will make the rule stick in your mind. Therefore, practice exercises are a must. Follow the rules even when you speak.

Exceptions to the rule can be a pain. Again, repeated exposure is the key. Remember, no amount of exposure to the language is enough. English is not our native language and secondly it is dynamic.

Read the sentences carefully. Do not get bogged down by difficult words. Focus on the error.

Try working backwards from the options and eliminate the error-ridden sentences

A regular reader will develop a “feel” for the language and will immediately “sense” an error. Verify it from the alternatives and pick the right option.

Cultivate the skill for intelligent guesswork to pick the most likely answer from the options.

Remember, there is NO shortcut to grammar. So start by completing the basics of grammar and continue practising by solving as many questions as possible.

You will need to put in consistent effort every day. All it takes is about 150 minutes of focused English preparation every day to crack this section.

Go ahead and conquer CAT Verbal section!




SENTENCE CORRECTIONS


1.A “calendar stick” carved centuries ago by the Winnebago tribe may provide the first evidence that the North American Indians have developed advanced full-year calendars basing them on systematic astronomical observation.
(A) that the North American Indians have developed advanced full-year calendars basing them
(B) of the North American Indians who have developed advanced full-year calendars and based them
(C) of the development of advanced full-year calendars by North American Indians, basing them
(D) of the North American Indians and their development of advanced full-year calendars based
(E) that the North American Indians developed advanced full-year calendars based
2.A 1972 agreement between Canada and the United States reduced the amount of phosphates that municipalities had been allowed to dump into the Great Lakes.
(A) reduced the amount of phosphates that municipalities had been allowed to dump
(B) reduced the phosphate amount that municipalities had been dumping
(C) reduces the phosphate amount municipalities have been allowed to dump
(D) reduced the amount of phosphates that municipalities are allowed to dump
(E) reduces the amount of phosphates allowed for dumping by municipalities
3.A collection of 38 poems by Phillis Wheatley, a slave, was published in the 1770’s, the first book by a Black woman and it was only the second published by an American woman.
(A) it was only the second published by an American woman
(B) it was only the second that an American woman published
(C) the second one only published by an American woman
(D) the second one only that an American woman published
(E) only the second published by an American woman
4.A common disability in test pilots is hearing impairment, a consequence of sitting too close to large jet engines for long periods of .
(A) a consequence of sitting too close to large jet engines for long periods of
(B) a consequence from sitting for long periods of too near to large jet engines
(C) a consequence which resulted from sitting too close to large jet engines for long periods of
(D) damaged from sitting too near to large jet engines for long periods of
(E) damaged because they sat too close to large jet engines for long periods of
5.A controversial figure throughout most of his public life, the Black leader Marcus Garvey advocated that some Blacks return to Africa, the land that, to him, symbolized the possibility of freedom.
(A) that some Blacks return to Africa, the land that, to him, symbolized the possibility of freedom
(B) that some Blacks return to the African land symbolizing the possibility of freedom to him
(C) that some Blacks return to Africa which was the land which symbolized the possibility of freedom to him
(D) some Black’s returning to Africa which was the land that to him symbolized the
possibility of freedom
(E) some Black’s return to the land symbolizing the possibility of freedom to him, Africa
6.A fire in an enclosed space burns with the aid of reflected radiation that preheats the fuel, making ignition much easier and flames spreading more quickly.
(A) flames spreading
(B) flame spreads
(C) flames are caused to spread
(D) causing flames to spread
(E) causing spreading of the flames
7.A firm that specializes in the analysis of handwriting claims from a one-page writing sample that it can assess more than three hundred personality traits, including enthusiasm,imagination, and ambition.
(A) from a one-page writing sample that it can assess
(B) from a one-page writing sample it has the ability of assessing
(C) the ability, from a one-page writing sample, of assessing
(D) to be able, from a one-page writing sample, to assess
(E) being able to assess, from a one-page writing sample,
8.A huge flying reptile that died out with the dinosaurs some 65 million years ago, the Quetzalcoatlus had a wingspan of 36 feet, believed to be the largest flying creature the world has ever seen.
(A) believed to be
(B) and that is believed to be
(C) and it is believed to have been
(D) which was, it is believed,
(E) which is believed to be
9.A Labor Department study states that the numbers of women employed outside the home grew by more than a thirty-five percent increase in the past decade and accounted for more than sixty-two percent of the total growth in the civilian work force.
(A) numbers of women employed outside the home grew by more than a thirty-five percent increase
(B) numbers of women employed outside the home grew more than thirty-five percent
(C) numbers of women employed outside the home were raised by more than thirty-five percent
(D) number of women employed outside the home increased by more than thirty-five percent
(E) number of women employed outside the home was raised by more than a thirty-five percent increase
10.A large rise in the number of housing starts in the coming year should boost new
construction dollars by several billion dollars, making the construction industry’s economic health much more robust than five years ago.
(A) making the construction industry’s economic health much more robust than five years ago
(B) and make the construction industry’s economic health much more robust than five years ago
(C) making the construction industry’s economic health much more robust than it was five years ago
(D) to make the construction industry’s economic health much more robust than five years ago
(E) in making the construction industry’s economic health much more robust than it as five years ago
11.A letter by Mark Twain, written in the same year as The Adventures of Huckleberry Finn were published, reveals that Twain provided financial assistance to one of the first Black students at Yale Law School.
(A) A letter by Mark Twain, written in the same year as The Adventures of Huckleberry Finn were published,
(B) A letter by Mark Twain, written in the same year of publication as The Adventures of Huckleberry Finn,
(C) A letter by Mark Twain, written in the same year that The Adventures of Huckleberry Finn was published,
(D) Mark Twain wrote a letter in the same year as he published The Adventures of Huckleberry Finn that
(E) Mark Twain wrote a letter in the same year of publication as The Adventures of
Huckleberry Finn that
12.A little under a million years ago, the briny waters of the Baltic Sea began flooding into the cold North Atlantic: geologists are still debating whether the flood was gradual or created a cataclysm.
(A) whether the flood was gradual or created a cataclysm
(B) if the flood was gradual or created a cataclysm
(C) about whether the flood was gradual or cataclysmic
(D) whether the flood was gradual or cataclysmic
(E) whether the flood was gradual or it created a cataclysm
13.A majority of the international journalists surveyed view nuclear power stations as unsafe at present but that they will, or could, be made sufficiently safe in the future.
(A) that they will, or could,
(B) that they would, or could,
(C) they will be or could
(D) think that they will be or could
(E) think the power stations would or could
14.A migraine typically afflicts one side of the head, lasts for hours or days, and may recur as infrequently as once every other month or often, as daily.
(A) as infrequently as once every other month or often, as
(B) as infrequently as once every other month or as often as
(C) infrequently, as often as once every other month, or often, like
(D) infrequently, like once every other month, or often, like
(E) infrequently, like once every other month, or as often as
15.A new phenomena, which is visible at Managua’s major inters, are waves of vendors and beggars, which include many children and mob cars at the stoplights.
(A) A new phenomena, which is visible at Managua’s major inters, are waves of vendors and beggars, which include many children and
(B) Visible at Managua’s major inters are waves of vendors and beggars with many children, new phenomena that
(C) A new phenomenon visible at Managua’s major inters is waves of vendors and beggars, many of them children, who
(D) Phenomenally new waves of vendors, beggars, and many children are visible at Managua’s major inters, which
(E) A wave of vendors and beggars, many of whom are children, are visible at Managua’s major inters, where they are a new phenomenon and
16.A number of linguists contend that all of the thousands of languages spoken by the world’s five billion people can be traced back to a common root language.
(A) that all of the thousands of languages spoken by the world’s five billion people can be traced
(B) that the world’s five billion people speak thousands of languages of which all can be traced
(C) the world’s five billion people speak thousands of languages which are all traceable
(D) all of the thousands of languages spoken by the world’s five billion people to be traceable
(E) the ability to trace all of the thousands of languages that are spoken by the world’s five billion people
17.A patient accusing a doctor of malpractice will find it difficult to prove damage if there is a lack of some other doctor to testify about proper medical procedures.
(A) if there is a lack of some other doctor to testify
(B) unless there will be another doctor to testify
(C) without another doctor’s testimony
(D) should there be no testimony from some other doctor
(E) lacking another doctor to testify
18.A peculiar feature of the embryonic mammalian circulatory system is that in the area of the heart the cells adhere to one another, beating in unison and adopting specialized orientations exclusive of one another.
(A) beating in unison and adopting
(B) they beat in unison while adopting
(C) beat in unison, and adopt
(D) beating in unison yet adopting
(E) even though they beat in unison and adopt
19.A President entering the final two years of a second term is likely to be at a severe disadvantage and is often unable to carry out a legislative program.
(A) likely to be at a severe disadvantage and is often unable to
(B) likely severely disadvantaged and often unable to
(C) liable to be severely disadvantaged and cannot often
(D) liable that he or she is at a severe disadvantage and cannot often
(E) at a severe disadvantage, often likely to be unable that he or she can
20.A prolific architect who worked from the turn of the century until the late 1950’s, Julia Morgan designed nearly 800 buildings in California, perhaps most notably William Randolph Hearst’s monumental estate at San Simeon.
(A) Julia Morgan designed nearly 800 buildings in California, perhaps most notably William Randolph Hearst’s monumental estate at San Simeon
(B) perhaps the most notable of the nearly 800 buildings in California designed by Julia Morgan was William Randolph Hearst’s monumental estate at San Simeon
(C) of the nearly 800 buildings in California designed by Julia Morgan, perhaps the most notable was William Randolph Hearst’s monumental estate at San Simeon
(D) nearly 800 buildings in California were designed by Julia Morgan, of which William Randolph Hearst’s monumental estate at San Simeon is perhaps the most notable
(E) William Randolph Hearst’s monumental estate at San Simeon is perhaps the most notable of the nearly 800 buildings in California designed by Julia Morgan
21.A proposal has been made to trim the horns from rhinoceroses to discourage poachers; the question is whether tourists will continue to visit game parks and see rhinoceroses after their horns are trimmed.
(A) whether tourists will continue to visit game parks and see rhinoceroses after their horns are
(B) whether tourists will continue to visit game parks to see one once their horns are
(C) whether tourists will continue to visit game parks to see rhinoceroses once the animals’ horns have been
(D) if tourists will continue to visit game parks and see rhinoceroses once the animals’ horns are
(E) if tourists will continue to visit game parks to see one after the animals’ horns have been
22.A recent national study of the public schools shows that there are now one microcomputer for every thirty-two pupils, four s as many than there were four years ago.
(A) there are now one microcomputer for every thirty-two pupils, four s as many than there were
(B) there is now one microcomputer for every thirty-two pupils, four s as many than there were
(C) there is now one microcomputer for every thirty-two pupils, four s as many as there were
(D) every thirty-two pupils now have one microcomputer, four s as many as there were
(E) every thirty-two pupils now has one microcomputer, four s as many as
23.A recent New York s editorial criticized the city’s election board for, first of all, failing to replace outmoded voting machines prone to breakdowns, and secondarily, for their failure to investigate allegations of corruption involving board members.
(A) secondarily, for their failure to
(B) secondly, for their failure to
(C) secondly, that they failed and did not
(D) second, which they failed to
(E) second, for failing to
24.A recent study has found that within the past few years, many doctors had elected early retirement rather than face the threats of lawsuits and the rising costs of malpractice insurance.
(A) had elected early retirement rather than face
(B) had elected early retirement instead of facing
(C) have elected retiring early instead of facing
(D) have elected to retire early rather than facing
(E) have elected to retire early rather than face
25.A recent study of ancient clay deposits has provided new evidence supporting the theory of global forest fires ignited by a meteorite impact that contributed to the extinction of the dinosaurs and many other creatures some 65 million years ago.
(A) supporting the theory of global forest fires ignited by a meteorite impact that
(B) supporting the theory that global forest fires ignited by a meteorite impact
(C) that supports the theory of global forest fires that were ignited by a meteorite impact and that
(D) in support of the theory that global forest fires were ignited by a meteorite impact and that
(E) of support for the theory of a meteorite impact that ignited global forest fires and
26.A recording system was so secretly installed and operated in the Kennedy Oval Office that even Theodore C. Sorensen, the White House counsel, did not know it existed.
(A) A recording system was so secretly installed and operated in the Kennedy Oval Office that
(B) So secret was a recording system installation and operation in the Kennedy Oval Office
(C) It was so secret that a recording system was installed and operated in the Kennedy Oval Office
(D) A recording system that was so secretly installed and operated in the Kennedy Oval Office
(E) Installed and operated so secretly in the Kennedy Oval Office was a recording system that
27.A report by the American Academy for the Advancement of Science has concluded that much of the currently uncontrolled dioxins to which North Americans are exposed come from the incineration of wastes.
(A) much of the currently uncontrolled dioxins to which North Americans are exposed comes
(B) much of the currently uncontrolled dioxins that North Americans are exposed to come
(C) much of the dioxins that are currently uncontrolled and that North Americans are exposed to comes
(D) many of the dioxins that are currently uncontrolled and North Americans are exposed to come
(E) many of the currently uncontrolled dioxins to which North Americans are exposed come
28.A representative of the Women’s Bureau of the United States Department of Labor contends that employers who offer benefits which permit that employees can balance home and work responsibilities better, realizing gains in attendance, recruiting, and retention.
(A) which permit that employees can balance home and work responsibilities better, realizing
(B) which permit employees balancing home and work responsibilities better will realize
(C) that permit employees to balance the responsibilities of home and work better will realize
(D) that permit employees a better balance between the responsibilities of home and work, thus realizing
(E) such that employees are permitted a balance between home and work responsibilities, and they will realize
29.A shy, religious-minded publisher who had married a duke’s daughter, Harold Macmillan’s rise to the position of Prime Minister in 1957 surprised many, though Churchill had since the 1930s been extolling Macmillan’s courage.
(A) Harold Macmillan’s rise to the position of Prime Minister in 1957 surprised many
(B) Harold Macmillan’s rise in 1957 to the position of Prime Minister surprised many
(C) Harold Macmillan’s becoming Prime Minister in 1957 surprised many
(D) Harold Macmillan surprised many by rising to the position of Prime Minister in 1957
(E) the position of Prime Minister attained by Harold Macmillan in 1957 surprised many
30.A special Japanese green tea called genmai-cha contains brown rice and is considered as a delicacy fit for a gourmet by most Japanese, though it is virtually unavailable outside Yokohama.
(A) A special Japanese green tea called genmai-cha contains brown rice and is considered as a delicacy fit for a gourmet by most Japanese, though it is virtually unavailable outside Yokohama.
(B) Considered to be a delicacy fit for a gourmet by most Japanese, genmai-cha is a special green tea that contains brown rice, virtually unavailable outside Yokohama.
(C) A special Japanese green tea called genmai-cha contains brown rice and is considered a gourmet delicacy by most Japanese, though it is virtually unavailable outside Yokohama.
(D) Most Japanese consider genmai-cha, a special green tea which contains brown rice, as a delicacy virtually unavailable outside Yokohama.
(E) Though virtually unavailable outside Yokohama, most Japanese consider genmai-cha, a special green tea that contains brown rice, a gourmet delicacy.
31.A star will compress itself into a white dwarf, a neutron star, or a black hole after it passes through a red giant stage, depending on mass.
(A) A star will compress itself into a white dwarf, a neutron star, or a black hole after it passes through a red giant stage, depending on mass.
(B) After passing through a red giant stage, depending on its mass, a star will compress itself into a white dwarf, a neutron star, or a black hole.
(C) After passing through a red giant stage, a star’s mass will determine if it compresses itself into a white dwarf, a neutron star, or a black hole.
(D) Mass determines whether a star, after passing through the red giant stage, will compress itself into a white dwarf, a neutron star, or a black hole.
(E) The mass of a star, after passing through the red giant stage, will determine whether it compresses itself into a white dwarf, a neutron star, or a black hole.
32.A study commissioned by the Department of Agriculture showed that if calves exercise and associated with other calves, they will require less medication and gain weight quicker than do those raised in confinement.
(A) associated with other calves, they will require less medication and gain weight quicker than do
(B) associated with other calves, they require less medication and gain weight quicker than
(C) associate with other calves, they required less medication and will gain weight quicker than do
(D) associate with other calves, they have required less medication and will gain weight more quickly than do
(E) associate with other calves, they require less medication and gain weight more quickly than
33.A substance derived from the Madagascar periwinkle, which has proved useful in decreasing mortality among young leukemia patients, is cultivated in China as part of a program to integrate traditional herbal medicine into a contemporary system of health care.
(A) A substance derived from the Madagascar periwinkle, which has proved useful in
decreasing mortality among young leukemia patients,
(B) A derivative, which has proved useful in decreasing mortality among young leukemia patients, of the Madagascar periwinkle,
(C) A Madagascar periwinkle derivative, which has proved useful in decreasing mortality among young leukemia patients,
(D) The Madagascar periwinkle has a derivative which has proved useful in decreasing
mortality among young leukemia patients, that
(E) The Madagascar periwinkle, a derivative of which has proved useful in decreasing
mortality among young leukemia patients,
34.A wildlife expert predicts that the reintroduction of the caribou into northern Minnesota would fail if the density of the timber wolf population in that region is more numerous than one wolf for every 39 square miles.
(A) would fail if the density of the timber wolf population in that region is more numerous than
(B) would fail provided the density of the timber wolf population in that region is more than
(C) should fail if the timber wolf density in that region was greater than
(D) will fail if the density of the timber wolf population in that region is greater than
(E) will fail if the timber wolf density in that region were more numerous than
35.According to a panel of health officials, there has been a great deal of confusion in the medical profession about whether obesity is a biological disorder posing serious health risks or a condition more related to appearance than to health.
(A) about whether obesity is a biological disorder posing serious health risks or a
condition more related to appearance than to
(B) with respect to obesity being a biological disorder posing serious health risks or if it is related more to appearance than
(C) over whether or not obesity is a biological disorder posing serious health risks or it is a condition more related to appearance than to
(D) about obesity and if it is a biological disorder posing serious health risks or a condition related to appearance more than to
(E) concerning whether obesity is a biological disorder posing serious health risks or it is a condition related to appearance more than
36.According to a recent poll, owning and living in a freestanding house on its own land is still a goal of a majority of young adults, like that of earlier generations.
(A) like that of earlier generations
(B) as that for earlier generations
(C) just as earlier generations did
(D) as have earlier generations
(E) as it was of earlier generations
37.According to a recent study by Rutgers University, the number of women in state
legislatures has grown in every election since 1968.
(A) the number of women in state legislatures has grown
(B) the number of women who are in state legislatures have grown
(C) there has been growth in the number of women in state legislatures
(D) a growing number of women have been in state legislatures
(E) women have been growing in number in state legislatures
38.According to a recent study, the elderly in the United States are four s more likely to give regular financial aid to their children as to receive it from them.
(A) the elderly in the United States are four s more likely to give regular financial aid to their children as
(B) the elderly in the United States are four s as likely to give regular financial aid to their children as it is for them
(C) the elderly in the United States are four s more likely to give regular financial aid to their children than
(D) it is four s more likely for the elderly in the United States to give regular financial aid to their children than they are
(E) it is four s as likely that the elderly in the United States will give their children regular financial aid as they are
39.According to a ruling by the state supreme court, the owner of polluted land is liable for the cleanup of the property even if the owner did not have the responsibility that pollution occurred before the title changed hands.
(A) the owner did not have the responsibility that pollution
(B) the owner is not responsible for pollution that
(C) it was not the owner’s responsibility that pollution would have
(D) the responsibility of the owner is not that pollution
(E) the responsibility was not the owner’s that pollution would have
40.According to a study by the Carnegie Foundation for the Advancement of Teaching,
companies in the United States are providing job training and general education for nearly eight million people, about equivalent to the enrollment of the nation’s four-year colleges and universities.
(A) equivalent to the enrollment of
(B) the equivalent of those enrolled in
(C) equal to those who are enrolled in
(D) as many as the enrollment of
(E) as many as are enrolled in
41.According to a study published by Dr. Myrna Weissman, only one percent of Americans born before 1905 had suffered major depression by the age of seventy-five; of those born since 1955, six percent had become depressed by age twenty-four.
(A) only one percent of Americans born before 1905 had suffered major depression by the age of seventy-five; of those born since 1955, six percent had become depressed by age twenty-four
(B) only one percent of Americans born before 1905 suffer major depression by the age of seventy-five; if they are born since 1955, six percent become depressed by age tweanty-four
(C) of Americans born before 1905, only one percent of them have suffered major depression by age seventy-five, but six percent of those born since 1955 do by the age of twenty-four
(D) major depression is suffered by the age of seventy-five by only one percent of Americans born before 1905, and by age twenty-four by the six percent born since 1955
(E) Americans born before 1905 suffer major depression by the age of seventy-five only one percent of the , but six percent of those born since 1955 did so by age twenty-four
42.According to a survey of graduating medical students conducted by the Association of American Medical Colleges, minority graduates are nearly four s more likely than are other graduates in planning to practice in socioeconomically deprived areas.
(A) minority graduates are nearly four s more likely than are other graduates in planning to practice
(B) minority graduates are nearly four s more likely than other graduates who plan on practicing
(C) minority graduates are nearly four s as likely as other graduates to plan on practicing
(D) it is nearly four s more likely that minority graduates rather than other graduates will plan to practice
(E) it is nearly four s as likely for minority graduates than other graduates to plan to practice
43.According to Booker T. Whatley’s recent analysis, planting the same crops as are planted on large farms will lead to economic disaster for the small farmer, who should plan a succession of high-value crops that will provide a year-round cash flow.
(A) planting the same crops as are planted on large farms will lead to economic disaster for the small farmer, who
(B) it will lead to economic disaster for the small farmer to plant the same crops as on the large farms; they
(C) economic disaster will result from planting the same crops as large farms to the small farmer, who
(D) economic disaster for the small farmer will result from planting the same crops as on the large farms; they
(E) the small farmer planting the same crops as are planted on large farms will lead to economic disaster; they
44.According to Henry David Thoreau, the reason a majority is allowed to rule is not that it is more likely to be right, but because it is stronger.
(A) the reason a majority is allowed to rule is not that it is more likely to be right, but because it is stronger
(B) a majority is allowed to rule not because it is more likely to be right, but because it is stronger
(C) the reason for majority rule is not because they are more likely to be right, they are stronger
(D) the majority is allowed to rule because of its strength, not because it is more likely to be right
(E) the reason why the majority rules is that it is strong, not because it is likely to be right
45.According to his own account, Frederic-Auguste Bartholdi, the sculptor of the Statue of Liberty, modeled the face of the statue like his mother’s and the body like his wife’s.
(A) modeled the face of the statue like his mother’s and the body like his wife’s
(B) modeled the face of the statue after that of his mother and the body after that of his wife
(C) modeled the face of the statue like his mother and the body like his wife
(D) made the face of the statue after his mother and the body after his wife
(E) made the face of the statue look like his mother and the body look like his wife
46.According to Interstudy, a nonprofit organization that studies health maintenance
organizations (HMO’s), they estimate that, in comparison to last year, when only 36 percent of the nation’s 607 HMO’s was profitable, this year 73 percent will be.
(A) they estimate that, in comparison to last year, when only 36 percent of the nation’s 607 HMO’s was profitable, this year 73 percent will be
(B) compared to only 36 percent of the nation’s 607 HMO’s being profitable last year, they estimate 73 percent would be this year
(C) only 36 percent of the nation’s 607 HMO’s were profitable last year; it estimates that this year 73 percent will be
(D) it estimates 73 percent of the nation’s 607 HMO’s would be profitable this year; last year that was only 36 percent
(E) only 36 percent of the nation’s 607 HMO’s last year were profitable, whereas they estimate it this year to be 73 percent
47.According to scientists at the University of California, the pattern of changes that have occurred in human DNA over the millennia indicate the possibility that everyone alive today might be descended from a single female ancestor who lived in Africa some between 140,000 and 280,000 years ago.
(A) indicate the possibility that everyone alive today might be descended from a single female ancestor who
(B) indicate that everyone alive today might possibly be a descendant of a single female ancestor who had
(C) may indicate that everyone alive today has descended from a single female ancestor who had
(D) indicates that everyone alive today may be a descendant of a single female ancestor who
(E) indicates that everyone alive today might be a descendant from a single female ancestor who
48.According to some analysts, whatever its merits, the proposal to tax away all capital gains on short-term investments would, if enacted, have a disastrous effect on Wall Street trading and employment.
(A) its merits, the proposal to tax
(B) its merits may be, the proposal of taxing
(C) its merits as a proposal, taxing
(D) the proposal’s merits, to tax
(E) the proposal’s merits are, taxing
49.According to some economists, Japan is in danger of plunging into a depression that, with double-digit unemployment, could severely strain a society that regards life employment as a virtual right of citizenship.
(A) that, with double-digit unemployment, could severely strain
(B) that, because of double-digit unemployment, could be a severe strain for
(C) with double-digit unemployment, and it could severely strain
(D) with double-digit unemployment and could be a severe strain
(E) with double-digit unemployment and could severely strain
50.According to some economists, the July decrease in unemployment so that it was the lowest in two years suggests that the gradual improvement in the job market is continuing.
(A) so that it was the lowest in two years
(B) so that it was the lowest two-year rate
(C) to what would be the lowest in two years
(D) to a two-year low level
(E) to the lowest level in two years
51.According to surveys by the National Institute on Drug Abuse, about 20 percent of young adults used cocaine in 1979, doubling those reported in the 1977 survey.
(A) doubling those reported in the 1977 survey
(B) to double the number the 1977 survey reported
(C) twice those the 1977 survey reported
(D) twice as much as those reported in the 1977 survey
(E) twice the number reported in the 1977 survey
52.According to the Better Business Bureau, if you fail to advertise the highest price in a range of prices for a service or product as prominently as that of the lowest, it violates the New York Consumer Protection Law.
(A) if you fail to advertise the highest price in a range of prices for a service or product as prominently as that of the lowest, it
(B) if one fails to advertise the highest price in a range of prices for a service or product as prominently as the lowest price, it
(C) failure to advertise the highest price in a range of prices for a service or product as prominently as the lowest
(D) failure to advertise as prominently the highest price in a range of prices for a service or product as the lowest
(E) failing to advertise as prominently the highest price in a range of prices for a service or products as that of the lowest
53.According to the National Pasta Association, per-capita consumption of pasta in the United States, which has already been approaching 19 pounds a year, will achieve 30 pounds a year by the twenty-first century.
(A) According to the National Pasta Association, per-capita consumption of pasta in the United States, which has already been approaching 19 pounds a year, will achieve 30 pounds a year by the twenty-first century.
(B) Already approaching 19 pounds a year in the United States, the National Pasta
Association predicts that per-capita consumption of pasta will reach 30 pounds a year by the twenty-first century.
(C) The National Pasta association predicts by the twenty-first century that per-capita consumption of pasta in the United States, which is already approaching 19 pounds a year, will reach 30 pounds a year.
(D) By the twenty-first century, the National Pasta Association predicts that per-capita consumption of pasta in the United States, having already approached 19 pounds a year, will reach 30 pounds a year.
(E) According to the National Pasta Association, per-capita consumption of pasta in the United States is already approaching 19 pounds a year and will reach 30 pounds a year by the twenty-first century.
54.According to the professor’s philosophy, the antidote to envy is one’s own work, always one’s own work: not thinking about it, not assessing it, but simply doing it.
(A) one’s own work, always one’s own work: not thinking about it, not assessing it, but simply doing it
(B) always work; because you don’t think about it or assess it, you just do it
(C) always one’s own work: not thinking about or assessing it, but simply to do it
(D) not to think or assess, but doing one’s own work
(E) neither to think about one’s own work nor to assess it, it is always simply doing it
55.According to United States Air Force officials, a cannon shooting dead chickens at airplanes has proved helpful to demonstrate what kind of damage can result when jets fly into a flock of large birds.
(A) shooting dead chickens at airplanes has proved helpful to demonstrate
(B) shooting dead chickens at airplanes has proved itself helpful as a demonstration of
(C) shooting dead chickens at airplanes proves itself helpful as demonstrating
(D) that shoots dead chickens at airplanes proves itself helpful to demonstrate
(E) that shoots dead chickens at airplanes has proved helpful in demonstrating
56.Acid rain and snow result from the chemical reactions between industrial emissions of sulfur dioxide and nitrogen oxides with atmospheric water vapor to produce highly corrosive sulfuric and nitric acids.
(A) with atmospheric water vapor to produce highly corrosive sulfuric and nitric acids
(B) with atmospheric water vapor producing highly corrosive sulfuric and nitric acids
(C) and atmospheric water vapor which has produced highly corrosive sulfuric and nitric acids
(D) and atmospheric water vapor which have produced sulfuric and nitric acids which are highly corrosive
(E) and atmospheric water vapor to produce highly corrosive sulfuric and nitric acids
57.Added to the increase in hourly wages requested last July, the railroad employees are now seeking an expanded program of retirement benefits.
(A) Added to the increase in hourly wages requested last July, the railroad employees are now seeking an expanded program of retirement benefits.
(B) Added to the increase in hourly wages which had been requested last July, the employees of the railroad are now seeking an expanded program of retirement benefits.
(C) The railroad employees are now seeking an expanded program of retirement benefits added to the increase in hourly wages that were requested last July.
(D) In addition to the increase in hourly wages that were requested last July, the railroad employees are now seeking an expanded program of retirement benefits.
(E) In addition to the increase in hourly wages requested last July, the employees of the railroad are now seeking an expanded program of retirement benefits.
58.Adult survivors of child abuse traditionally have had little or no chance that they could get their symptoms recognized and treated.
(A) that they could get their symptoms recognized and treated
(B) to recognize and treat their symptoms
(C) of getting their symptoms recognized and treated
(D) of recognizing and treating symptoms
(E) of getting his or her symptoms recognized and treated
59.Affording strategic proximity to the Strait of Gibraltar, Morocco was also of interest to the French throughout the first half of the twentieth century because they assumed that if they did not hold it, their grip on Algeria was always insecure.
(A) if they did not hold it, their grip on Algeria was always insecure
(B) without it their grip on Algeria would never be secure
(C) their grip on Algeria was not ever secure if they did not hold it
(D) without that, they could never be secure about their grip on Algeria
(E) never would their grip on Algeria be secure if they did not hold it
60.After a few weeks’ experience, apprentice jewelers can usually begin to discriminate, though not with absolute certainty, genuine diamonds from imitation diamonds.
(A) genuine diamonds from imitation diamonds
(B) genuine diamonds apart from imitations
(C) between genuine diamonds and imitation diamonds
(D) among genuine diamonds and imitation diamonds
(E) whether diamonds are imitation or genuine
61.After crude oil, natural gas is the United States second biggest fuel source and supplied almost exclusively from reserves in North America.
(A) After crude oil, natural gas is the United States second biggest fuel source and
supplied almost exclusively from reserves in North America.
(B) Natural gas, after crude oil the United States second biggest fuel source, supplied almost exclusively from reserves in North America.
(C) Being supplied almost exclusively from reserves in North America, natural gas, the United States second biggest fuel source after crude oil.
(D) Natural gas, the United States’ second biggest fuel source after crude oil, is supplied almost exclusively from reserves in North America.
(E) Natural gas is supplied almost exclusively from reserves in North America, being the United States’ second biggest fuel source after crude oil.
62.After gradual declension down to about 39 hours in 1970, the workweek in the United States has steadily increased to the point that the average worker now puts in an estimated
164 extra hours of paid labor a year.
(A) After gradual declension down
(B) Following a gradual declension down
(C) After gradual declining down
(D) After gradually declining
(E) Following gradually declining
63.After July, anyone disposing of or servicing refrigerators must capture the
chlorofluorocarbons in the refrigerant chemicals.
(A) anyone disposing of or servicing
(B) those who dispose or service
(C) anyone disposing of or who services
(D) the disposal or repair of
(E) someone who disposes or repairs
64.After Queen Isabella asked Admiral Columbus to describe the island of Hispaniola (now Haiti), which was newly discovered, he had reached for a sheet of paper, crumpled it, and said, “It looks like that—beyond the mountains, more mountains.”
(A) After Queen Isabella asked Admiral Columbus to describe the island of Hispaniola (now Haiti), which was newly discovered, he had reached
(B) On being asked to describe the new discovery of the island of Hispaniola (now Haiti) by Queen Isabella, Admiral Columbus, reaching
(C) Queen Isabella asked Admiral Columbus to describe the newly discovered island of
Hispaniola (now Haiti), then he reached
(D) When asked by Queen Isabella to describe the newly discovered island of Hispaniola (now Haiti), Admiral Columbus reached
(E) After Queen Isabella had asked Admiral Columbus to describe the discovery of the island of Hispaniola (now Haiti), he had reached
65.After suffering $2 billion in losses and 25,000 layoffs, the nation’s semiconductor industry, which makes chips that run everything from computers and spy satellites to dishwashers, appears to have made a long-awaited recovery.
(A) computers and spy satellites to dishwashers, appears to have
(B) computers, spy satellites, and dishwashers, appears having
(C) computers, spy satellites, and dishwashers, appears that it has
(D) computers and spy satellites to dishwashers, appears that it has
(E) computers and spy satellites as well as dishwashers, appears to have
66.After the Arab conquest of Egypt in A.D. 640, Arabic became the dominant language of the Egyptians, replacing older languages and writing systems.
(A) became the dominant language of the Egyptians, replacing older languages
(B) became the dominant language of the Egyptians, replacing languages that were older
(C) becomes the dominant language of the Egyptians and it replaced older languages
(D) becomes the dominant language of the Egyptians and it replaced languages that were older
(E) becomes the dominant language of the Egyptians, having replaced languages that were older
67.After the Civil War, contemporaries of Harriet Tubman’s maintained that she has all of the qualities of a great leader, coolness in the face of danger, an excellent sense of strategy, and an ability to plan in minute detail.
(A) Tubman’s maintained that she has
(B) Tubman’s maintain that she had
(C) Tubman’s have maintained that she had
(D) Tubman maintained that she had
(E) Tubman had maintained that she has
68.After the Colonial period’s 50 percent mortality rate, life expectancy improved for children, but as late as the nineteenth century about one child in three died before reaching the age of six.
(A) After the Colonial period’s 50 percent mortality rate, life expectancy improved for children, but
(B) Even though children’s life expectancy, which improved over the Colonial period’s 50 percent mortality rate,
(C) Although life expectancy for children improved after the Colonial period, during which the mortality rate was 50 percent,
(D) While there was an improvement in life expectancy for children after the 50 percent mortality rate of the Colonial period, still
(E) Despite children’s life expectancy improvement from the Colonial period’s 50 percent mortality rate,
69.After the Vietnam war Bettye Granther, a U.S. Army nurse, continued her efforts on behalf of injured Vietnamese children, providing medical care, helping to reunite estranged families, and the establishment of a fund for the children’s future education.
(A) the establishment of a fund for the children’s future education
(B) the establishing of a fund for the future education of children
(C) establishing a fund for the children’s future education
(D) establishing a fund for the childrens’ future education
(E) the establishment of a fund for the childrens’ future education
70.After this year’s record-shattering January performance in Madison Square Garden, the ensemble were touted as the country’s best new group in decades; no critic or reviewer had anything but praise for the young musicians.
(A) the ensemble were touted as the country’s
(B) the ensemble was touted as the country’s
(C) the country touted the ensemble like the
(D) touting the ensemble as the country’s
(E) they were touting the ensemble as the country’s
71.Aging is a property of all animals that reach a fixed size at maturity, and the
variations in life spans among different species are far greater as that among individuals from the same species: a fruit fly is ancient at 40 days, a mouse at 3 years, a horse at 30, a man at 100, and some tortoises at 150.
(A) among different species are far greater as that among individuals from
(B) among different species are far greater than that among individuals from
(C) among different species are far greater than those among individuals of
(D) between different species are far more than that between individuals of
(E) between different species are greater by far than is that between individuals from
72.Aho, a Kiowa matriarch, held festivals in her home, they featured the preparation of great quantities of ceremonial food, the wearing of many layers of colorful clothing adorned with silver, and the recounting of traditional tribal jokes and stories.
(A) Aho, a Kiowa matriarch, held festivals in her home, they featured
(B) Festivals were held in Aho, a Kiowa matriarch’s home, which featured
(C) Aho, who was a Kiowa matriarch in her home, held festivals featuring
(D) In her home, Aho, a Kiowa matriarch, held festivals that featured
(E) Aho, a Kiowa matriarch, held festivals in her home that featured
73.Alaska regularly deposits some of its profits from the sale of oil into a special fund, with the intention to sustain the state’s economy after the exhaustion of its oil reserves.
(A) fund, with the intention to sustain the state’s economy after the exhaustion of its oil reserves0
(B) fund, the intention of which is to sustain the state’s economy after they have exhausted their oil reserves
(C) fund intended to sustain the state’s economy after oil reserves are exhausted
(D) fund intended to sustain the state’s economy after exhausting its oil reserves
(E) fund that they intend to sustain the state’s economy after oil reserves are exhausted
74.All-terrain vehicles have allowed vacationers to reach many previously inaccessible areas, but they have also been blamed for causing hundreds of deaths, injury to thousands, and seriously damaging the nation’s recreational areas.
(A) deaths, injury to thousands, and seriously damaging
(B) deaths and injuring thousands, and serious damage to
(C) deaths, thousands who are injured, as well as seriously damaging
(D) deaths and thousands of injuries, as well as doing serious damage to
(E) deaths, thousands are injured, and they do serious damage to
75.Along with the drop in producer prices announced yesterday, the strong retail sales figures released today seem like it is indicative that the economy, although growing slowly, is not nearing a recession.
(A) like it is indicative that
(B) as if to indicate
(C) to indicate that
(D) indicative of
(E) like an indication of
76.Although all the proceedings of the Communist party conference held in Moscow were not carried live, Soviet audiences have seen a great deal of coverage.
(A) all the proceedings of the Communist party conference held in Moscow were not carried live
(B) all the Communist party conference’s Moscow proceedings were not carried live
(C) all the Communist party conference Moscow proceedings have not been carried alive
(D) not all the Communist party conference Moscow proceedings have been carried alive
(E) not all the proceedings of the Communist party conference held in Moscow were carried live
77.Although aspirin irritates the stomach, it can be avoided if the aspirin tablet is given a coating that will not dissolve until the tablet reaches the intestine.
(A) Although aspirin irritates the stomach, it
(B) The irritation of the stomach caused by aspirin
(C) The fact that aspirin causes irritation of the stomach
(D) Aspirin causes stomach irritation, although it
(E) Aspirin irritates the stomach, which
78.Although dozens of New York’s small museums are either devoted to local history or various ethnic groups, there are many one-of-a-kind museums from Manhattan to the Bronx that are open for exploration on summer weekends.
(A) Although dozens of New York’s small museums are either devoted to local history or various ethnic groups, there are
(B) Although dozens of New York’s small museums are devoted to local history or various ethnic groups,
(C) Dozens of New York’s small museums are devoted to local history or various ethnic groups, but there are
(D) Dozens of New York’s small museums are devoted to local history or various ethnic groups, and there are also
(E) Devoted to local history or various ethnic groups, dozens of New York’s small museums and also
79.Although early soap operas were first aired on evening radio in the 1920’s, they had moved to the day hours of the 1930’s when the evening schedule became crowded with comedians and variety shows.
(A) were first aired on evening radio in the 1920’s, they had moved to the day hours of the 1930’s
(B) were first aired on evening radio in the 1920’s, they were moved to the day hours in the 1930’s
(C) were aired first on evening radio in the 1920’s, moving to the day hours in the 1930’s
(D) were aired first in the evening on 1920’s radio, they moved to the day hours of the 1930’s
(E) aired on evening radio first in the 1920’s, they were moved to the 1930’s in the day hours
80.Although films about the American West depict coyotes as solitary animals howling
mournfully on the tops of distant hills, in reality these gregarious creatures live in stable groups that occupy the same territory for long periods.
(A) films about the American West depict coyotes as solitary animals howling mournfully on the tops of distant hills
(B) in films about the American West coyotes are depicted to be solitary animals that howl mournfully on the tops of distant hills
(C) coyotes are depicted as solitary animals howling mournfully on the tops of distant hills in films about the American West
(D) films about the American West depict coyotes as if they were solitary, mournfully howling animals on the tops of distant hills
(E) films about the American West depict coyotes to be solitary and mournfully howling animals on the tops of distant hills
81.Although fruit can no longer grow once it is picked, it continues for some to respire, taking in oxygen and giving off carbon dioxide, similar to the way human beings breathe.
(A) similar to the way human beings breathe
(B) similarly to human beings who are breathing
(C) just like the breathing of human beings
(D) as human beings when breathing
(E) just as human beings do when they breathe
82.Although he is as gifted as, if not more gifted than, many of his colleagues, he is extremely modest and his poetry is unpublished.
(A) Although he is as gifted as, if not more gifted than, many of his colleagues, he is extremely modest and his poetry is unpublished.
(B) Although he is as gifted, if not more gifted, than many of his colleagues, he is
extremely modest and with his poetry remaining unpublished.
(C) Although he is as gifted as, if not more gifted than, many of his colleagues, he is extremely modest and will not publish his poetry.
(D) Despite his being gifted, if not more gifted than his colleagues, he is extremely modest and will not publish his poetry.
(E) Being a gifted as, or more gifted than, many of his colleagues, he is extremely modest and his poetry is unpublished.
83.Although it claims to delve into political issues, television can be superficial such as when each of the three major networks broadcast exactly the same statement from a political candidate.
(A) superficial such as when each of the three major networks
(B) superficial, as can sometimes occur if all of the three major networks
(C) superficial if the three major networks all
(D) superficial whenever each of the three major networks
(E) superficial, as when the three major networks each
84.Although it was expected that workers under forty would show hostility to the plan, the research report indicates that both younger and the older people approve of governmental appropriations for Social Security.
(A) younger and the older people
(B) younger people and the older
(C) the younger and the older people
(D) younger and older people
(E) people who are younger and those who are older
85.Although it was once funded entirely by the government, the Victoria and Albert Museum was one of the first of Britain’s national museums seeking support from corporations and private donors and to increase income by increasing attendance.
(A) one of the first of Britain’s national museums seeking support from
(B) one of Britain’s first national museums seeking support of
(C) among Britain’s first national museums to seek support of
(D) among the first of Britain’s national museums to seek support from
(E) among Britain’s first national museums that have sought the support of
86.Although just inside the orbit of Jupiter, amateur astronomers with good telescopes should be able to see the comet within the next few weeks.
(A) Although just inside the orbit of
(B) Although it is just inside the orbit of
(C) Just inside the orbit of
(D) Orbiting just inside
(E) Having orbited just inside
87.Although many art patrons can readily differentiate a good debenture from an undesirable one, they are much less expert in distinguishing good paintings and poor ones, authentic art and fakes.
(A) much less expert in distinguishing good paintings and poor ones, authentic art and
(B) far less expert in distinguishing good paintings from poor ones, authentic art from
(C) much less expert when it comes to distinguishing good paintings and poor ones, authentic art from
(D) far less expert in distinguishing good paintings and poor ones, authentic art and
(E) far less the expert when it comes to distinguishing between good painting, poor ones, authentic art, and
88.Although many Whites, noting the presence of some Blacks in the middle class, think that the for enforcing civil rights measures is past, Blacks generally are aware that the figures for average income and unemployment show as wide of a radical discrepancy as ever.
(A) that the figures for average income and unemployment show as wide of
(B) that average-income and unemployment figures show as wide
(C) that the average-income and unemployment figures are showing as wide of
(D) of average-income and unemployment figures that show as wide of
(E) of figures for average income and unemployment showing as wide
89.Although Ms. Bakara had previously emphasized that she could not speak for other Black people, she ventured to do so on this one occasion because she firmly believed that many minority people, likely most, would agree with her.
(A) do so on this one occasion because she firmly believed that many minority people, likely most, would agree
(B) speak on this one occasion since she firmly believed that many minority people, likely most, would have agreed
(C) so speak on this one occasion due to her firmly believing that many minority people, even most, would likely agree
(D) do so on this one occasion because she firmly believed that many minority people, if not most, would agree
(E) do so on this one occasion since she firmly believed many minority people, and even most, would likely agree
90.Although Napoleon’s army entered Russia with far more supplies than they had in their previous campaigns, it had provisions for only twenty-four days.
(A) they had in their previous campaigns
(B) their previous campaigns had had
(C) they had for any previous campaign
(D) in their previous campaigns
(E) for any previous campaign
91.Although one link in the chain was demonstrated to be weak, but not sufficiently so to require the recall of the automobile.
(A) demonstrated to be weak, but not sufficiently so to require
(B) demonstrated as weak, but it was not sufficiently so that it required
(C) demonstrably weak, but not sufficiently so to require
(D) demonstrably weak, it was not so weak as to require
(E) demonstrably weak, it was not weak enough that it required
92.Although partially destroyed, the archaeologists were able to infer from what remained of the inscription that the priest Zonainos was buried in the crypt.
(A) Although partially destroyed, the archaeologists were able to infer
(B) Although partially destroyed, the archaeologists had inferred
(C) Although it had been partially destroyed, the archaeologists were able to infer
(D) Partially destroyed though it had been, the archaeologists had been able to infer
(E) Destroyed partially, the archaeologists were able to infer
93.Although schistosomiasis is not often fatal, it is so debilitating that it has become an economic drain on many developing countries.
(A) it is so debilitating that it has become an economic
(B) it is of such debilitation, it has become an economical
(C) so debilitating is it as to become an economic
(D) such is its debilitation, it becomes an economical
(E) there is so much debilitation that it has become an economical
94.Although she had signed a pledge of abstinence while being an adolescent, Frances Willard was 35 years old before she chose to become a temperance activist.
(A) while being an adolescent
(B) while in adolescence
(C) at the of her being adolescent
(D) as being in adolescence
(E) as an adolescent
95.Although some officials noted that using machines for farming in China costs more than traditional hand cultivation, the mechanization of farming in the village of Long Bow doubled the corn yield while the previous year’s costs were cut in half.
(A) mechanization of farming in the village of Long Bow doubled the corn yield while the previous year’s costs were cut in half
(B) mechanization of farming in the village of Long Bow doubled the corn yield while cutting costs to half those of the previous year
(C) mechanization of farming in the village of Long Bow doubled the corn yield as cost were cut to half of the previous year’s
(D) farming mechanization in the village of Long Bow doubled the corn yield as it cut in half the previous year’s costs
(E) farming mechanization in the village of Long Bow doubled the corn yield while costs were cut to half that of the previous year
96.Although the bite of brown recluse spiders is rarely fatal, they cause chronic flesh wounds, posing the greatest danger to the infant and elderly, who are particularly vulnerable to its poison.
(A) brown recluse spiders are rarely fatal, they cause chronic flesh wounds, posing the greatest danger to the infant and elderly, who are particularly vulnerable to its
(B) brown recluse spiders are rarely fatal, they cause chronic flesh wounds and pose the greatest danger to the infant and elderly, who are particularly vulnerable to their
(C) the brown recluse spider is rarely fatal, it causes chronic flesh wounds, posing the greatest danger to the infant and elderly, who are particularly vulnerable to their
(D) the brown recluse spider is rarely fatal, it causes chronic flesh wounds and poses the greatest danger to infants and the elderly, who are particularly vulnerable to its
(E) the brown recluse spider is rarely fatal, they cause chronic flesh wounds, posing the greatest danger to the infant and elderly, who are particularly vulnerable to its
97.Although the coordination of monetary policy can help facilitate the orderly financing of existing imbalances, it is unlikely that its effect on their size is significant in the absence of an appropriate fiscal adjustment.
(A) it is unlikely that its effect on their size is significant
(B) it is unlikely that the size of their effect would be significant
(C) affecting their sizes are not likely to be significant
(D) the significance of their effect on its size is unlikely
(E) its effect on their size is not likely to be significant
98.Although the lesser cornstalk borer is widely distributed, control of them is necessary only in the South.
(A) the lesser cornstalk borer is widely distributed, control of them is
(B) widely distributed, measures to control the lesser cornstalk borer are
(C) widely distributed, lesser cornstalk borer control is
(D) the lesser cornstalk borer is widely distributed, measures to control it are
(E) it is widely distributed, control of the lesser cornstalk borer is
99.Although the manager agreed to a more flexible work schedule, he said that it must be posted on the bulletin board so that both management and labor will know what everyone is assigned to do.
(A) he said that it must be posted on the bulletin board so that both management and labor will know what everyone is
(B) he said it had to be posted on the bulletin board so that both management and labor knows what everyone is
(C) he said that they would have to post the assignments on the bulletin board so that management and labor knew what everyone was
(D) he said that the schedule would have to be posted on the bulletin board so that both management and labor would know what everyone was
(E) saying that the schedule had to be posted on the bulletin board so that both management and labor would know what everyone had been
100.Although the phenomenon of withdrawal has always been the crucial physiological Leng for distinguishing addictive from nonaddictive drugs, it has become increasingly evident that not all regular heroin users experience withdrawal symptoms.
(A) addictive from
(B) addictive and
(C) addictive or
(D) between addictive or
(E) among addictive or
101.Although the Supreme Court ruled as long ago as 1880 that Blacks could not be excluded outright from jury service, nearly a century of case-by-case adjudication has been necessary to develop and enforce the principle that all juries must be drawn from “a fair cross of the community.”
(A) has been necessary to develop and enforce the principle that all juries must be
(B) was necessary for developing and enforcing the principle of all juries being
(C) was to be necessary in developing and enforcing the principle of all juries to be
(D) is necessary to develop and enforce the principle that all juries must be
(E) will be necessary for developing and enforcing the principle of all juries being
102.Although the term “psychopath” is popularly applied to an especially brutal criminal, in psychology it is someone who is apparently incapable of feeling compassion or the pangs of conscience.
(A) it is someone who is
(B) it is a person
(C) they are people who are
(D) it refers to someone who is
(E) it is in reference to people
103.American productivity is declining in relation to Europe’s; the energy expended per unit of production in the United States is as much as twice that expended in West Germany.
(A) as much as twice that expended in West Germany
(B) as much as twice that of West Germany’s expenditure
(C) up to two s of West Germany’s expenditure
(D) up to two s what West Germans expended
(E) up to double the West German expenditure
104.Among the cossacks, vegetable farming was once so despised that it was forbidden on pain of death.
(A) so despised that it was
(B) so despised to be
(C) so despised it had been
(D) despised enough that it was
(E) despised enough as to be
105.Among the emotions on display in the negotiating room were anger for repeatedly raising the issue over and over again and preventing the raw wounds from earlier battles from ever beginning to heal.
(A) were anger for repeatedly raising the issue over and over again and preventing the raw wounds from earlier battles from ever beginning to heal
(B) was anger for repeatedly raising the issue and preventing the raw wounds from earlier battles from ever beginning to heal
(C) were anger over repeatedly raising the issue and preventing the raw wounds from earlier battles to begin healing
(D) was anger about the issue, which was raised over and over, and preventing the wounds from earlier battles, still raw, to begin healing
(E) were anger about the issue, which was raised repeatedly, and preventing the raw wounds from earlier battles to begin to heal
106.Among the objects found in the excavated temple were small terra-cotta effigies left by supplicants who were either asking the goddess Bona Dea’s aid in healing physical and mental ills or thanking her for such help.
(A) in healing physical and mental ills or thanking her for such help
(B) in healing physical and mental ills and to thank her for helping
(C) in healing physical and mental ills, and thanking her for helping
(D) to heal physical and mental ills or to thank her for such help
(E) to heal physical and mental ills or thanking her for such help
107.Among the reasons for the decline of New England agriculture in the last three decades were the high cost of land, the pressure of housing and commercial development, and basing a marketing and distribution system on importing produce from Florida and California.
(A) basing a marketing and distribution system on importing produce from Florida and
California
(B) basing a marketing and distribution system on the imported produce of Florida and California
(C) basing a system of marketing and distribution on the import of produce from Florida and California
(D) a marketing and distribution system based on importing produce from Florida and
California
(E) a marketing and distribution system importing produce from Florida and California as its base
108.An array of tax incentives has led to a boom in the construction of new office
buildings; so abundant has capital been for commercial real estate that investors regularly scour the country for areas in which to build.
(A) so abundant has capital been for commercial real estate that
(B) capital has been so abundant for commercial real estate, so that
(C) the abundance of capital for commercial real estate has been such,
(D) such has the abundance of capital been for commercial real estate that
(E) such has been an abundance of capital for commercial real estate,
109.An artistic presence of the first order, one frequently ranked with Picasso, Stravinsky, and James Joyce, Martha Graham was acclaimed as a great dancer long before her innovative masterworks made her the most honored of American choreographers.
(A) Martha Graham was acclaimed as
(B) Martha Graham was acclaimed to be
(C) Martha Graham’s acclaim is as
(D) Martha Graham’s acclaim to be
(E) Martha Graham’s acclaim was in being
110.An attempt to ratify the Equal Rights Amendment, begun almost two decades ago, has been unsuccessful despite efforts by many important groups, including the National Organization for Women.
(A) to ratify the Equal Rights Amendment, begun almost two decades ago,
(B) begun almost two decades ago, for ratifying the Equal Rights Amendment
(C) begun for ratifying the Equal Rights Amendment almost two decades ago
(D) at ratifying the Equal Rights Amendment, begun almost two decades ago,
(E) that has begun almost two decades ago to ratify the Equal Rights Amendment
111.An inventory equal to 90 days sales is as much as even the strongest businesses carry, and then only as a way to anticipate higher prices or ensure against shortages.
(A) as much as even
(B) so much as even
(C) even so much as
(D) even as much that
(E) even so much that
112.An unusually strong cyclist can, it is hoped, provide enough power to set a new distance record for human-powered aircraft in MIT’s diaphanous construction of graphite fiber and plastic.
(A) can, it is hoped, provide enough power to set
(B) it is hoped, can provide enough power that will set
(C) hopefully can provide enough power, this will set
(D) is hopeful to set
(E) hopes setting
113.Analysts blamed May’s sluggish retail sales on unexciting merchandise as well as the weather, colder and wetter than was usual in some regions, which slowed sales of barbecue grills and lawn furniture.
(A) colder and wetter than was usual in some regions, which slowed
(B) which was colder and wetter than usual in some regions, slowing
(C) since it was colder and wetter than usually in some regions, which slowed
(D) being colder and wetter than usually in some regions, slowing
(E) having been colder and wetter than was usual in some regions and slowed
114.Ancient Romans found it therapeutic to bathe in cold milk, in strawberries that had been crushed, or in bathtubs filled with black caviar.
(A) to bathe in cold milk, in strawberries that had been crushed, or in bathtubs filled with black caviar
(B) that they bathe in cold milk, in strawberries that had been crushed, or in caviar that was black
(C) to bathe in cold milk, crushed strawberries, or black caviar
(D) that they bathe in cold milk, crushed strawberries, or black caviar
(E) to bathe in milk , strawberries, or caviar
115.Any medical test will sometimes fail to detect a condition when it is present and indicate that there is one when it is not.
(A) a condition when it is present and indicate that there is one
(B) when a condition is present and indicate that there is one
(C) a condition when it is present and indicate that it is present
(D) when a condition is present and indicate its presence
(E) the presence of a condition when it is there and indicate its presence
116.Archaeologists in Ireland believe that a recently discovered chalice, which dates from the eighth century, was probably buried to keep from being stolen by invaders.
(A) to keep from
(B) to keep it from
(C) to avoid
(D) in order that it would avoid
(E) in order to keep from
117.Architects and stonemasons, huge palace and temple clusters were built by the Maya without benefit of the wheel or animal transport.
(A) huge palace and temple clusters were built by the Maya without benefit of the wheel or animal transport
(B) without the benefits of animal transport or the wheel, huge palace and temple clusters were built by the Maya
(C) the Maya built huge palace and temple clusters without the benefit of animal transport or the wheel
(D) there were built, without the benefit of the wheel or animal transport, huge palace and temple clusters by the Maya
(E) were the Maya who, without the benefit of the wheel or animal transport, built huge palace and temple clusters
118.Art museums do not usually think of their collections as capital or consider the
interest income that would be generated if a portion of the capital would have been invested in another form.
(A) be generated if a portion of the capital would have been
(B) have been generated if a portion of the capital would have been
(C) be generated if a portion of the capital were
(D) be generated if a portion of the capital was
(E) be generated if a portion of the capital had been
119.Artificial intelligence emerged during the late 1950’s as an academic discipline based on the assumption that computers are able to be programmed to think like people.
(A) are able to be programmed to think like people
(B) were able to be programmed to think as people
(C) can be programmed to think as people can
(D) could be programmed to think like people
(E) are capable of being programmed to think like people do
120.As a baby emerges from the darkness of the womb with a rudimentary sense of vision, it would be rated about 20/500, or legally blind if it were an adult with such vision.
(A) As a baby emerges from the darkness of the womb with a rudimentary sense of vision, it would be rated about 20/500, or legally blind if it were an adult with such vision.
(B) A baby emerges from the darkness of the womb with a rudimentary sense of vision that would be rated about 20/500, or legally blind as an adult.
(C) As a baby emerges from the darkness of the womb, its rudimentary sense of vision would be rated about 20/500; qualifying it to be legally blind if an adult.
(D) A baby emerges from the darkness of the womb with a rudimentary sense of vision that would be rated about 20/500; an adult with such vision would be deemed legally blind.
(E) As a baby emerges from the darkness of the womb, its rudimentary sense of vision, which would be deemed legally blind for an adult, would be rated about 20/500.
121.As a result of medical advances, many people that might at one have died as children of such infections as diphtheria, pneumonia, or rheumatic fever now live well into old age.
(A) that might at one have died as children
(B) who might once have died in childhood
(C) that as children might once have died
(D) who in childhood might have at one died
(E) who, when they were children, might at one have died
122.As a result of the continuing decline in the birth rate, less people will enter the labor force in the 1980’s than did in the 1960’s and 1970’s, a twenty year period during which people born after the war swelled the ranks of workers.
(A) less people will enter the labor force in the 1980’s than
(B) less people will be entering the labor force in the 1980’s as
(C) fewer people will enter the labor force in the 1980’s as
(D) fewer people will be entering the labor force in the 1980’s as
(E) fewer people will enter the labor force in the 1980’s than
123.As a result of the ground-breaking work of Barbara McClintock, many scientists now believe that all of the information encoded in 50,000 to 100,000 of the different genes found in a human cell are contained in merely three percent of the cell’s DNA.
(A) 50,000 to 100,000 of the different genes found in a human cell are contained in merely
(B) 50,000 to 100,000 of the human cell’s different genes are contained in a mere
(C) the 50,000 to 100,000 different genes found in human cells are contained in merely
(D) 50,000 to 100,000 of human cell’s different genes is contained in merely
(E) the 50,000 to 100,000 different genes found in a human cell is contained in a mere
124.As business grows more complex, students majoring in specialized areas like those of finance and marketing have been becoming increasingly successful in the job market.
(A) majoring in specialized areas like those of finance and marketing have been becoming increasingly
(B) who major in such specialized areas as finance and marketing are becoming more and more
(C) who majored in specialized areas such as those of finance and marketing are being increasingly
(D) who major in specialized areas like those of finance and marketing have been becoming more and more
(E) having majored in such specialized areas as finance and marketing are being increasingly
125.As contrasted with the honeybee, the yellow jacket can sting repeatedly without dying and carries a potent venom that can cause intense pain.
(A) As contrasted with the honeybee
(B) In contrast to the honeybee’s
(C) Unlike the sting of the honeybee
(D) Unlike that of the honeybee
(E) Unlike the honeybee
126.As envisioned by researchers, commercial farming of lobsters will enable fisheries to sell the shellfish year-round, taking advantage of off-season demand, standardize its sizes and colors, and to predict sales volume in advance.
(A) taking advantage of off-season demand, standardize
(B) taking advantage of off-season demand, to standardize
(C) taking advantage of off-season demand, standardizing
(D) take advantage of off-season demand, standardizing
(E) take advantage of off-season demand, to standardize
127.As Hurricane Hugo approached the Atlantic coast, it increased dramatically in strength, becoming the tenth most intense hurricane to hit the United States mainland in the twentieth century and most intense since Camille in 1969.
(A) most intense since Camille in 1969
(B) most intense after Camille in 1969
(C) the most intense since Camille in 1969
(D) the most intense after 1969, which had Camille
(E) since 1969 and Camille, the most intense
128.As it becomes more frequent to have spouses who both work outside the home, companies are beginning to help in finding new employment for the spouses of transferred employees.
(A) it becomes more frequent to have spouses who both work outside the home
(B) it becomes more frequent to have couples both working outside the home
(C) it becomes more common that both husband and wife should be working outside the home
(D) it becomes more common for both husband and wife to work outside the home
(E) couples in which both of the spouses working outside the home become more common
129.As litigation grows more complex, the need that experts explain technical issues becomes more apparent.
(A) that experts explain technical issues becomes
(B) for experts to explain technical issues became
(C) for experts to explain technical issues becomes
(D) that technical issues be explained by experts became
(E) that there be explanations of technical issues by experts has become
130.As many as 300 of the 720 paintings attributed to Rembrandt may actually be the works of his students or other admirers.
(A) the 720 paintings attributed to Rembrandt may
(B) the 720 paintings attributed to be Rembrandt’s might
(C) the 720 paintings that were attributed to be by Rembrandt may
(D) the 720 Rembrandt paintings that were once attributed to him might
(E) Rembrandt’s paintings, although 720 were once attributed to him, may
131.As measured by the Commerce Department, corporate profits peaked in the fourth quarter of 1988 and have slipped since then, as many companies have been unable to pass on higher costs.
(A) and have slipped since then, as many companies have been unable to pass on higher costs
(B) and have slipped since then, the reason being because many companies have been unable to pass on higher costs
(C) and slipped since then, many companies being unable to pass on higher costs
(D) but, many companies unable to pass on higher costs, they have slipped since then
(E) yet are slipping since then, because many companies were unable to pass on higher costs
132.As more and more people invest their money in savings certificates or money-market funds

in order to earn higher interest, they are abandoning traditional low-interest investment

havens such as passbook accounts and life insurance policies.
(A) As more and more people invest their money
(B) While people have more and more been investing their money
(C) As money is more and more invested by people
(D) More and more, when investors put their money
(E) While, more and more, investors have been putting their money
133.As part of their therapy, young polio victims learning to live with their disabilities were helped to practice falling during the 1950s, so that they could learn to fall without being hurt.
(A) As part of their therapy, young polio victims learning to live with their disabilities were helped to practice falling during the 1950s
(B) As part of their therapy, young polio victims learning to live during the 1950s with their disabilities were helped to practice falling
(C) Young polio victims learning to live during the 1950s with their disabilities were helped to practice falling as part of their therapy
(D) Young polio victims learning to live with their disabilities were helped to practice falling during the 1950s as part of their therapy
(E) During the 1950s, as part of their therapy, young polio victims learning to live with their disabilities were helped to practice falling
134.As rare as something becomes, be it a baseball card or a musical recording or a postage stamp, the more avidly it is sought by collectors.
(A) As rare as something becomes, be it
(B) As rare as something becomes, whether it is
(C) As something becomes rarer and rarer, like
(D) The rarer something becomes, like
(E) The rarer something becomes, whether it is
135.As recently as 1950, tuberculosis was never curable unless sequestered in sanitariums; today, the drug Isoniazid has made such treatment obsolete.
(A) unless sequestered
(B) without sequestering
(C) without being sequestered
(D) unless it was sequestered
(E) unless patients were sequestered
136.As researchers continue to probe the highly expressive vocal and postural language of wolves, their close resemblance to dogs has become ever more striking.
(A) their close resemblance to dogs has become
(B) the closeness of their resemblance to dogs has become
(C) the close resemblance between them and dogs has become
(D) the close resemblance between wolves and dogs becomes
(E) the close resemblance of wolves with dogs becomes
137.As Russell Banks suggests, it is a lack of grand ideas that have only left writers with semiotics, hermeneutics, and deconstruction.
(A) As Russell Banks suggests, it is a lack of grand ideas that have only left writers with
(B) Writers, Russell Banks suggests, who lack of grand ideas and leave only
(C) Russell Banks suggests that a lack of grand ideas has left writers with only
(D) It is Russell Banks’ suggestion that a lack of grand ideas have left writers only with
(E) It is only a lack of grand ideas, Russell Banks suggests, which leave writers with
138.As sales of cars and light trucks made in North America were declining 13.6 percent in late February, many analysts conclude that evidence of a recovering automotive market remains slight.
(A) As sales of cars and light trucks made in North America were declining 13.6 percent in late February, many analysts conclude
(B) Since sales of cars and light trucks made in North America declined 13.6 percent in late February, and many analysts conclude
(C) With sales of cars and light trucks made in North America declining 13.6 percent in late February, with many analysts concluding
(D) Because sales of cars and light trucks made in North America declined 13.6 percent in late February, many analysts concluded
(E) Because of sales of cars and light trucks made in North America declining 13.6 percent in late February, therefore, many analysts concluded
139.As the etched lines on computer memory chips have become thinner and the chips’ circuits more complex, both the power of the chips and the electronic devices they drive have vastly increased.
(A) the chips’ circuits more complex, both the power of the chips and the electronic devices they drive have
(B) the chips’ circuits more complex, the power of both the chips and the electronic devices they drive has
(C) the chips’ circuits are more complex, both the power of the chips and the electronic devices they drive has
(D) their circuits are more complex, the power of both the chips and the electronic devices they drive have
(E) their circuits more complex, both the power of the chips and the electronic devices they drive have
140.As the housing affordability gap widens, middle-income families are especially hard-hit, and these families can no longer qualify to buy homes, and rising rental rates force them to use far more than the standard 25 percent of their incomes for housing, leaving them with no equity or tax write-offs to offset the expenditures.
(A) and these families can no longer qualify to buy homes, and
(B) since these families can no longer afford to buy homes, furthermore
(C) for these families can no longer afford to buy homes, yet
(D) and these families can no longer afford to buy homes; however,
(E) and these families can no longer afford to buy homes, for
141.As the price of gasoline rises, which makes substituting alcohol distilled from cereal grain attractive, the prices of bread and livestock feed are sure to increase.
(A) which makes substituting alcohol distilled from cereal grain attractive
(B) which makes substituting the distillation of alcohol from cereal grain attractive
(C) which makes distilling alcohol from cereal grain an attractive substitute
(D) making an attractive substitution of alcohol distilled from cereal grain
(E) making alcohol distilled from cereal grain an attractive substitute
142.As U.S. nuclear attack submarines prowl their familiar haunts deep within the oceans of the world these days, they increasingly are engaged in missions far different from the tasks for which they were built and for which their crews were trained over the last forty years.
(A) they increasingly are engaged in missions far different from the tasks for which they were built
(B) the missions they increasingly engage in are far different than the tasks they were built for
(C) they engage increasingly in missions that differ significantly from the tasks they were built for
(D) the missions they engage in are increasingly different than the tasks for which they were built
(E) they increasingly engage in missions far different than the tasks for which they were built
143.As virtually all the nation’s 50 busiest airports are, New York’s were built for an age of propellers, before jet planes weighing 800,000 pounds needed over two miles of runway.
(A) As virtually all the nation’s 50 busiest airports are
(B) As with virtually all of the nation’s 50 busiest airports
(C) Like virtually all of the nation’s 50 busiest airports
(D) Like the cities where virtually all the nation’s 50 busiest airports are
(E) Like other cities where virtually all the nation’s 50 busiest airports are
144.As well as heat and light, the Sun is the source of a continuous stream of atomic particles known as the solar wind.
(A) As well as heat and light, the Sun is the source of a continuous stream
(B) Besides heat and light, also the Sun is the source of a continuous stream
(C) Besides heat and light, the Sun is also the source of a continuous streaming
(D) The Sun is the source not only of heat and light, but also of a continuous stream
(E) The Sun is the source of not only heat and light but, as well, of a continuous streaming
145.Astronomers at the Palomar Observatory have discovered a distant supernova explosion, one that they believe is a type previously unknown to science.
(A) that they believe is
(B) that they believe it to be
(C) they believe that it is of
(D) they believe that is
(E) they believe to be of
146.Astronomers studying the newly discovered star say that it provides clues about our galaxy’s origin, that it may supply data about how fast our galaxy is expanding, and moreover it is perhaps the most distant star in the entire Milky Way.
(A) about how fast our galaxy is expanding, and moreover it is perhaps
(B) regarding the speed our galaxy expands at; moreover, they think it may be
(C) about how fast our galaxy expands and perhaps be
(D) on the galaxy’s expansion rate and perhaps
(E) about how fast our galaxy is expanding, and that it is perhaps
147.At a recent session, the French government has decided that Paris needs a second, larger opera house to complement the famous Paris Opera.
(A) has decided that Paris needs
(B) decided that Paris needs
(C) has decided that Paris will need
(D) decided that Paris has a need of
(E) has decided that Paris needed
148.At a when it was unusual to do it, Dorothy Sterling wrote about such major figures of Black history as Harriet Tubman and W. E. B. Du Bois.
(A) it was unusual to do it
(B) it was unusual to do so
(C) doing that was unusual
(D) that was an unusual thing to be doing
(E) it was not usual to do
149.At ground level, ozone is a harmful pollutant, but in the stratosphere it shields the Earth from the most biologically harmful radiation emitted by the Sun, radiation in the ultraviolet band of the spectrum.
(A) in the stratosphere
(B) in the stratosphere, in which
(C) it is in the stratosphere in which
(D) in the stratosphere where
(E) it is in the stratosphere and
150.At Shiprock, New Mexico, a perennially powerful girls’ high school basketball team has become a path to college for some and a source of pride for a community where the household incomes of 49 percent of them are below the poverty level.
(A) where the household incomes of 49 percent of them are
(B) where they have 49 percent of the household incomes
(C) where 49 percent of the household incomes are
(D) which has 49 percent of the household incomes
(E) in which 49 percent of them have household incomes
151.At the beginning of the year, the city allocated $150 million to cover the increase in wages that it expected to approve as a result of negotiations with the municipal labor unions.
(A) increase in wages that it expected to approve as a result of negotiations
(B) increased wages it expected to approve from negotiations
(C) increasing wages expected to be approved as a result of negotiating
(D) negotiated increases in wages it expected to approve
(E) increases expected to be approved in wages from negotiating
152.At the invitation of Kwame Nkrumah, W. E. B. Du Bois settled in Gharla and became the editor of the Encyclopedia Africana.
(A) At the invitation of
(B) When he was invited by
(C) An invitation coming from
(D) An invitation that came from
(E) After having an invitation by
153.At the of the Mexican agrarian revolution, the most radical faction, that of Zapata and his followers, proposed a return to communal ownership of land, to what had been a pre-Columbian form of ownership respected by the Spaniards.
(A) land, to what had been a pre-Columbian form of ownership respected by the Spaniards
(B) land, a form of ownership of the pre-Columbians and respected by the Spaniards
(C) land, respected by the Spaniards and a pre-Columbian form of ownership
(D) land in which a pre-Columbian form of ownership was respected by the Spaniards
(E) land that had been a pre-Columbian form of ownership respected by the Spaniards
154.Automation has undermined the traditional position of labor as much by robbing workers of the special Skills that were once their most important strength than by the elimination of jobs.
(A) than by the elimination of jobs
(B) rather than by the elimination of a job
(C) than by eliminating jobs
(D) as by the elimination of a job
(E) as by eliminating jobs
155.Aviation officials have not only failed to determine the cause of the crash, but they have also ignored demands by the pilots union that the investigation should be expedited.
(A) crash, but they have also ignored demands by the pilots union that the investigation should be expedited
(B) crash but also ignored demands by the pilots union that the investigation be expedited
(C) crash but have ignored demands for expediting the investigation by the pilots’ union
(D) crash, but they are also ignoring the pilots’ union’s demands to expedite the
investigation
(E) crash: the demands by the pilots’ union that the investigation should be expedited have also been ignored
156.Balding is much more common among White males than males of other races.
(A) than
(B) than among
(C) than is so of
(D) compared to
(E) in comparison with
157.Balzac drank more than fifty cups of coffee a day and died of caffeine poisoning; furthermore, caffeine did not seem to bother Samuel Johnson, the great writer and lexicographer, who was reported to have drunk twenty-five cups of tea at one sitting.
(A) furthermore, caffeine did not seem to bother
(B) however, caffeine did not seem to bother
(C) however, caffeine did not seem to have bothered
(D) furthermore, caffeine did not seem to have bothered
(E) in addition, caffeine did not seem to bother
158.Bankers require that the financial information presented to them by mortgage applicants be complete and follow a prescribed format.
(A) be complete and follow a prescribed format
(B) is complete and it follows a prescribed format
(C) be complete and a prescribed format is followed
(D) to be complete and a prescribed format be following
(E) be completed, and it followed a prescribed format
159.Based on accounts of various ancient writers, scholars have painted a sketchy picture of the activities of an all-female cult that, perhaps as early as the sixth century B.C., worshipped a goddess known in Latin as Bona Dea, “the good goddess.”
(A) Based on accounts of various ancient writers
(B) Basing it on various ancient writers’ accounts
(C) With accounts of various ancient writers used for a basis
(D) By the accounts of various ancient writers they used
(E) Using accounts of various ancient writers
160.Based on pinto beans and corn tortillas, the Pima Indians have a diet that derives 70 percent of its protein from vegetable sources and only 30 percent from animal foods, the reverse of the typical North American diet.
(A) Pima Indians have a diet that derives
(B) Pima Indians in their diet derive
(C) diet of the Pima Indians derives
(D) diets of the Pima Indians have derived
(E) diet of the Pima Indians, deriving
161.Based on the growth rates of large modern reptiles such as the Galapagos tortoise and examinations of fossils of juvenile dinosaurs, scientists estimate that the largest dinosaurs probably lived to be between 100 and 200 years old.
(A) Based on the growth rates of large modern reptiles such as
(B) On the basis of growth rates of large modern reptiles such as
(C) Based on the growth rates of large modern reptiles like
(D) On the basis of growth rates of large modern reptiles, like those of
(E) Based on such growth rates as those of large modern reptiles like
162.Beatrix Potter, in her book illustrations, carefully coordinating them with her
narratives, capitalized on her keen observation and love of the natural world.
(A) Beatrix Potter, in her book illustrations, carefully coordinating them with her
narratives,
(B) In her book illustrations, carefully coordinating them with her narratives, Beatrix Potter
(C) In her book illustrations, which she carefully coordinated with her narratives, Beatrix Potter
(D) Carefully coordinated with her narratives, Beatrix Potter, in her book illustrations
(E) Beatrix Potter, in her book illustrations, carefully coordinated them with her
narratives and
163.Because Halley’s comet changes orbit slightly during the seventy-six-year interval between passing close to Earth, it may veer onto a collision course with a planet some in the distant future.
(A) between passing
(B) of passing
(C) between its passes
(D) of its passes
(E) as it passes
164.Because natural gas is composed mostly of methane, a simple hydrocarbon, vehicles powered by natural gas emit less of certain pollutants than the burning of gasoline or diesel fuel.
(A) less of certain pollutants than the burning of gasoline or diesel fuel
(B) fewer of certain pollutants than burning gasoline or diesel fuel do
(C) less of certain pollutants than gasoline or diesel fuel
(D) fewer of certain pollutants than does burning gasoline or diesel fuel
(E) less of certain pollutants than those burning gasoline or diesel fuel
165.Because of the business community’s uncertainty about the President’s position in regard to the issue of the budget deficit, an unanticipated rise in interest rates has occurred.
(A) in regard to the issue of the budget deficit, an unanticipated rise in interest rates has
(B) on the deficit, an unanticipated rise in interest rates has
(C) regarding the budgetary deficit, an unanticipated rise in interest rates have
(D) on the deficit, an unanticipated rise in interest rates have
(E) regarding the deficit, an unanticipated rise in interest rates have
166.Because of the enormous research and development expenditures required to survive in the electronics industry, an industry marked by rapid innovation and volatile demand, such firms tend to be very large.
(A) to survive
(B) of firms to survive
(C) for surviving
(D) for survival
(E) for firms’ survival
167.Because paper of all kinds is the biggest single component of municipal trash, many municipalities have tried recycling to reduce the cost of trash disposal.
(A) Because paper of all kinds is the biggest single component of municipal trash, many municipalities have tried recycling to reduce the cost of trash disposal.
(B) Because paper of all kinds is the biggest single component in municipal trash, many municipalities tried to recycle so that the cost of trash disposal is reduced.
(C) Because paper of all kinds are the biggest single components in municipal trash, many municipalities have tried to recycle to reduce the cost of trash disposal.
(D) All kinds of paper are the biggest single components of municipal trash, and so many municipalities have tried recycling to reduce the cost of trash disposal.
(E) All kinds of paper is the biggest single component of municipal trash, so many
municipalities have tried to recycle so that the cost of trash disposal could be reduced.
168.Because the Earth’s crust is more solid there and thus better able to transmit shock waves, an earthquake of a given magnitude typically devastates an area 100 s greater in the eastern United States than it does in the West.
(A) of a given magnitude typically devastates an area 100 s greater in the eastern United States than it does in the West
(B) of a given magnitude will typically devastate 100 s the area if it occurs in the eastern United States instead of the West
(C) will typically devastate 100 s the area in the eastern United States than one of the comparable magnitude occurring in the West
(D) in the eastern United States will typically devastate an area 100 s greater than will a quake of comparable magnitude occurring in the West
(E) that occurs in the eastern United States will typically devastate 100 s more area than if it occurred with comparable magnitude in the West
169.Because the enemy’s new ship is the quietest and it is therefore the most elusive submarine, it is being increasingly viewed by the military as a threat to security.
(A) and it is therefore the most elusive submarine, it is being increasingly viewed
(B) it is therefore the most elusive of submarines, and it has increased the view
(C) and therefore the most elusive submarine, it is being increasingly viewed
(D) and therefore it is the most elusive of submarines, there is an increasing view
(E) therefore being the most elusive of submarines, it is increasingly viewed
170.Because the financial review covered only the last few fiscal years, and therefore the investigators were unable to determine the extent to possible earlier overpayments.
(A) and therefore the investigators were unable to determine the extent to possible
(B) so therefore the investigators were not capable of determining the possible extent of
(C) therefore the investigators were unable to determine the possible extent of
(D) the investigators were not capable of determining the possible extent of
(E) the investigators were unable to determine the extent of possible
171.Because young children do not organize their attention or perceptions systematically, like adults, they may notice and remember details that their elders ignore.
(A) like adults
(B) unlike an adult
(C) as adults
(D) as adults do
(E) as an adult
172.Before the Civil War, Harriet Tubman, herself an escaped slave, returned again and again to Maryland to guide other slaves along the Underground Railroad to freedom.
(A) herself an escaped slave, returned again and again to Maryland to guide
(B) being an escaped slave herself, returned again and again to Maryland so as to guide
(C) an escaped slave herself, returned again and again to Maryland for guiding
(D) herself as an escaped slave, returned again and again to Maryland so as to be the guide of
(E) who had been herself as an escaped slave, returned again and again to Maryland for the guiding of
173.Being a United States citizen since 1988 and born in Calcutta in 1940, author Bharati Mukherjee has lived in England and Canada, and first came to the United States in 1961 to study at the Iowa Writers’ Workshop.
(A) Being a United States citizen since 1988 and born in Calcutta in 1940, author Bharati Mukherjee has
(B) Having been a United States citizen since 1988, she was born in Calcutta in 1940; author Bharati Mukherjee
(C) Born in Calcutta in 1940, author Bharati Mukherjee became a United States citizen in 1988; she has
(D) Being born in Calcutta in 1940 and having been a United States citizen since 1988, author Bharati Mukherjee
(E) Having been born in Calcutta in 1940 and being a United States citizen since 1988, author Bharati Mukherjee
174.Believed to originate from a small area on their foreheads, elephants emit low-frequency sounds that may be used as a secret language to communicate with other members of the herd.
(A) Believed to originate from a small area on their foreheads, elephants emit low-frequency sounds that may be used
(B) Elephants emit low-frequency sounds that are believed to originate from a small area on their foreheads, and they may use this
(C) Elephants emit low-frequency sounds, believed to originate from a small area on their foreheads, which they may use
(D) Originating, it is believed, from a small area on their foreheads; elephants emit low-frequency sounds they may use
(E) Originating, it is believed, from a small area on their foreheads, low-frequency sounds are emitted by elephants that may be used
175.Besides yielding such psychological rewards as relief from stress, deep relaxation, if practiced regularly, can strengthen the immune system and produce a host of other physiological benefits.
(A) deep relaxation, if practiced regularly, can strengthen the immune system
(B) one can strengthen the immune system through deep relaxation, if it is practiced
regularly
(C) the immune system can be strengthened as a result of deep relaxation if practiced regularly
(D) when deep relaxation is practiced regularly, the immune system can be strengthened
(E) when practiced regularly, the results of deep relaxation can be to strengthen the immune system
176.Beyond the immediate cash flow crisis that the museum faces, its survival depends on if it can broaden its membership and leave its cramped quarters for a site where it can store and exhibit its more than 12,000 artifacts.
(A) if it can broaden its membership and leave
(B) whether it can broaden its membership and leave
(C) whether or not it has the capability to broaden its membership and can leave
(D) its ability for broadening its membership and leaving
(E) the ability for it to broaden its membership and leave
177.Bihar is India’s poorest state, with an annual per capita income of $111, lower than in the most impoverished countries of the world.
(A) lower than in
(B) lower than that of
(C) and lower than that of
(D) which is lower than in
(E) which is lower than it is in
178.Bill Walton continued on playing, even though he had injuries that recurred over and over again, always hoping to return back to his position as a regular starter in the game he loved.
(A) on playing, even though he had injuries that recurred over and over again, always hoping to return back
(B) playing, in spite of recurrent injuries, always hoping to return
(C) playing, though injured over and over, and he was always hoping to return back
(D) on playing, even with injuries that recurred, and always hoped to return
(E) to play, despite recurring injuries, hoping that the return
179.Biologists believe that they have found one of the substances that tell individual genes both when to become active and when to remain quiescent in the earliest phases of an embryo’s development.
(A) tell individual genes both when to become active and when to remain
(B) tell individual genes both at which they should become active or should remain
(C) tells individual genes both when to become active or remain
(D) tells individual genes both when to become active or when to remain
(E) will tell an individual gene both about when it should become active and remain
180.Bluegrass musician Bill Monroe, whose repertory, views on musical collaboration, and vocal style were influential on generations of bluegrass artists, was also an inspiration to many musicians that included Elvis Presley and Jerry Garcia, whose music differed significantly from his own.
(A) were influential on generations of bluegrass artists, was also an inspiration to many musicians, that included Elvis Presley and Jerry Garcia, whose music differed significantly from
(B) influenced generations of bluegrass artists, also inspired many musicians, including Elvis Presley and Jerry Garcia, whose music differed significantly from
(C) was influential to generations of bluegrass artists, was also inspirational to many musicians, that included Elvis Presley and Jerry Garcia, whose music was different significantly in comparison to
(D) was influential to generations of bluegrass artists, also inspired many musicians, who included Elvis Presley and Jerry Garcia, the music of whom differed significantly when compared to
(E) were an influence on generations of bluegrass artists, was also an inspiration to many musicians, including Elvis Presley and Jerry Garcia, whose music was significantly different from that of
181.Bob Wilber became Sidney Bechet’s student and protégé when he was nineteen and, for a few years in the 1940’s, came as close to being a carbon copy of the jazz virtuoso in performance as anyone has ever come.
(A) as anyone has ever come
(B) as anyone ever had been
(C) as anyone ever had done
(D) that anyone ever did
(E) that anyone ever came
182.Bringing the Ford Motor Company back from the verge of bankruptcy shortly after the Second World War was a special governmentally sanctioned price increase during a period of wage and price controls.
(A) Bringing the Ford Motor Company back from the verge of bankruptcy shortly after the Second World War was a special governmentally sanctioned price increase during a period of wage and price controls.
(B) What brought the Ford Motor Company back from the verge of bankruptcy shortly after the Second World War was a special price increase that the government sanctioned during a period of wage and price controls.
(C) That which brought the ford Motor Company back from the verge of bankruptcy shortly after the Second World War was a special governmentally sanctioned price increase during a period of wage and price controls.
(D) What has brought the Ford Motor Company back from the verge of bankruptcy shortly after the Second World War was a special price increase that the government sanctioned during a period of wages and price controls.
(E) To bring the Ford Motor Company back from the verge of bankruptcy shortly after the Second World War, there was a special price increase during a period of wages and price controls that government sanctioned.
183.Bufo marinus toads, fierce predators that will eat frogs, lizards, and even small birds, are native to South America but were introduced into Florida during the 1930’s in an attempt to control pests in the state’s vast sugarcane fields.
(A) are native to South America but were introduced into Florida during the 1930’s in an attempt to control
(B) are native in South America but were introduced into Florida during the 1930’s as attempts to control
(C) are natives of South America but were introduced into Florida during the 1930’s in an attempt at controlling
(D) had been native to South America but were introduced to Florida during the 1930’s as an attempt at controlling
(E) had been natives of South America but were introduced to Florida during the 1930’s as attempts at controlling
184.Building large new hospitals in the bistate area would constitute a wasteful use of resources, on the basis of avoidance of duplicated facilities alone.
(A) on the basis of avoidance of duplicated facilities alone
(B) on the grounds of avoiding duplicated facilities alone
(C) solely in that duplicated facilities should be avoided
(D) while the duplication of facilities should be avoided
(E) if only because the duplication of facilities should be avoided
185.By 1914, ten of the western states had granted women the right to vote, but only one in the East.
(A) only one in the East
(B) only one eastern state
(C) in the East there was only one state
(D) in the East only one state did
(E) only one in the East had
186.By a vote of 9 to 0, the Supreme Court awarded the Central Intelligence Agency broad discretionary powers enabling it to withhold from the public the identities of its sources of intelligence information.
(A) enabling it to withhold from the public
(B) for it to withhold from the public
(C) for withholding disclosure to the public of
(D) that enable them to withhold from public disclosure
(E) that they can withhold public disclosure of
187.By analyzing the advanced olfactory apparatus of Pleistocene chordates, paleozoologists have discovered a link between the brain’s regions of scent discrimination and its regions of long-term memory storage, a link that could prove invaluable in the treatment of amnesia victims.
(A) paleozoologists have discovered a link between the brain’s regions of scent
discrimination and its regions of long-term memory storage
(B) a link between the brain’s regions of scent discrimination and its regions of long-term memory storage has been discovered by paleozoologists
(C) there is a link that paleozoologists have discovered between the brain’s regions of scent discrimination and its regions of long-term memory storage
(D) the discovery of a link between the brain’s regions of scent discrimination and its regions of long-term memory storage was made by paleozoologists
(E) the brain’s regions of scent discrimination and long-term memory storage have a link that was discovered by paleozoologists
188.By installing special electric pumps, farmers’ houses could be heated by the warmth from cows’ milk, according to one agricultural engineer.
(A) farmers’ houses could be heated by the warmth from cows’ milk, according to one

agricultural engineer
(B) the warmth from cows’ milk could be used by farmers to heat their houses, according to one agricultural engineer
(C) one agricultural engineer reports that farmers could use the warmth from cows’ milk to heat their houses
(D) farmers, according to one agricultural engineer, could use the warmth from cows’ milk to heat their houses
(E) one agricultural engineer reports that farmers’ houses could be heated by the warmth from cows’ milk
189.By offering lower prices and a menu of personal communications options, such as caller identification and voice mail, the new telecommunications company has not only captured customers from other phone companies but also forced them to offer competitive prices.
(A) has not only captured customers from other phone companies but also forced them
(B) has not only captured customers from other phone companies, but it also forced them
(C) has not only captured customers from other phone companies but also forced these
companies
(D) not only has captured customers from other phone companies but also these companies have been forced
(E) not only captured customers from other phone companies, but it also has forced them
190.By showing that South Africa does not have a free market and is in fact a kind of collectivist welfare state for Whites only, Sowell argues that American conservatives have no valid ideological grounds to be in sympathy with the Pretoria regime.
(A) to be in sympathy with
(B) to sympathize with
(C) for sympathizing with
(D) that they should sympathize with
(E) that they should have sympathy for
191.By studying the primitive visual systems of single-cell aquatic organisms, biophysicists have discovered a striking similarity between algae and cows, a similarity that indicates the common evolutionary origin of plants and animals: both algae and cows produce a light-sensitive protein called rhodopsin.
(A) biophysicists have discovered a striking similarity between algae and cows
(B) a striking similarity between algae and cows has been discovered by biophysicists
(C) there is a striking similarity that biophysicists have discovered between algae and cows
(D) the discovery of a striking similarity between algae and cows was made by biophysicists
(E) algae and cows have a striking similarity that was discovered by biophysicists
192.By the mid-seventeenth century, Amsterdam had built a new town hall so large that only St. Peter’s in Rome, the Escorial in Spain, and the Palazza Ducale in Venice could rival it for scale or magnificence.
(A) could rival it for
(B) were the rivals of it in their
(C) were its rival as to
(D) could be its rivals in their
(E) were rivaling its
193.Byron possessed powers of observation that would have made him a great anthropologist and that makes his letters as a group the rival of the best novels of the .
(A) makes his letters as a group the rival of
(B) makes his letters as a group one to rival
(C) makes his letters a group rivaling
(D) make his letters as a group the rival of
(E) make his letters a group which is the rival of
194.Cajuns speak a dialect brought to southern Louisiana by the four thousand Acadians who migrated there in 1755; their language is basically seventeenth-century French to which has been added English, Spanish and Italian words.
(A) to which has been added English, Spanish and Italian words
(B) added to which is English, Spanish, and Italian words
(C) to which English, Spanish, and Italian words have been added
(D) with English, Spanish, and Italian words having been added to it
(E) and, in addition, English, Spanish, and Italian words are added
195.California’s child-support payments are as high or higher than other states.
(A) as high or higher than other states
(B) at least as high as any states
(C) as high as, or higher than, those of any other state
(D) higher than any state’s, or just as high
(E) higher, or as high as, other states
196.Camille Claudet worked continuously through the 1880’s and early 1890’s with the
sculptor Auguste Rodin; since there are very few signed works of hers, the inescapable conclusion seems to be one of Claudet conceiving and executing part of Rodin’s enormous production of that period.
(A) inescapable conclusion seems to be one of Claudet conceiving and executing part of Rodin’s enormous production of that period
(B) conclusion of Claudet conceiving and executing part of Rodin’s enormous production of that period seems inescapable
(C) conclusion seems inescapable that part of Rodin’s enormous production of that period was conceived and executed by Claudet
(D) conclusion of part of Rodin’s enormous production of that period having been conceived and executed by Claudet seems inescapable
(E) seemingly inescapable conclusion is that Claudet would have conceived and executed part of Rodin’s enormous production of that period
197.Camus broke with Sartre in a bitter dispute over the nature of Stalinism.
(A) in a bitter dispute over
(B) over bitterly disputing
(C) after there was a bitter dispute over
(D) after having bitterly disputed about
(E) over a bitter dispute about
198.Canadian scientists have calculated that one human being should be struck every nine years by a meteorite, while each year sixteen buildings can be expected to sustain damage from such objects.
(A) one human being should be struck every nine years by a meteorite
(B) a human being should be struck by a meteorite once in every nine years
(C) a meteorite will strike one human being once in every nine years
(D) every nine years a human being will be struck by a meteorite
(E) every nine years a human being should be struck by a meteorite
199.Car owners who inflate their tires properly can substantially boost their vehicles’ fuel efficiency, since the increase in car-road friction can waste up to five percent of car fuel by underinflation.
(A) Car owners who inflate their tires properly can substantially boost their vehicles’ fuel efficiency, since the increase in car-road friction can waste up to five percent of car fuel by underinflation.
(B) Because the underinflation of tires can waste up to five percent of a car’s fuel by increasing car-road friction, car owners can substantially boost their vehicles’ fuel efficiency by properly inflating the tires.
(C) Their vehicles’ fuel efficiency is substantially boosted by car owners through the proper inflation of tires that, when underinflated, can waste up to five percent of car fuel by an increase in car-road friction.
(D) The proper inflation of tires by car owners, due to the fact that underinflation can waste up to five percent of a car’s fuel by the increase of car-road friction, can substantially boost their fuel efficiency.
(E) Because up to five percent of a car’s fuel are wasted through the increases in car-road friction when the tires are underinflated, car owners properly inflating tires can substantially boost their fuel efficiency.
200.Carbon-14 dating reveals that the megalithic monuments in Brittany are nearly 2,000 years as old as any of their supposed Mediterranean predecessors.
(A) as old as any of their supposed
(B) older than any of their supposed
(C) as old as their supposed
(D) older than any of their supposedly
(E) as old as their supposedly
201.Cardiopulmonary resuscitation should begin between one to four after a cardiac arrest in order to be a success.
(A) Cardiopulmonary resuscitation should begin between one to four after a cardiac arrest in order to be a success.
(B) Between one and four after a cardiac arrest, cardiopulmonary resuscitation should begin to be successful.
(C) Successful cardiopulmonary resuscitation should be begun from one to four after a cardiac arrest occurs.
(D) If it is to be successful, a cardiac arrest should be treated with cardiopulmonary resuscitation one to four afterward.
(E) To be successful, cardiopulmonary resuscitation should begin within one to four after a cardiac arrest.
202.Carnivorous mammals can endure what would otherwise be lethal levels of body heat because they have a heat-exchange network which kept the brain from getting too hot.
(A) which kept
(B) that keeps
(C) which has kept
(D) that has been keeping
(E) having kept
203.Carpenters, dentists, sewing machine operators, needlepointers, piano players, and indeed anyone who works with their hands for long hours can get carpal tunnel syndrome.
(A) anyone who works
(B) anyone working
(C) workers
(D) those for whom work is
(E) any people who work
204.Cartographers have long struggled with the problem of having the spherical Earth to draw on a flat sheet of paper.
(A) having the spherical Earth to draw on a flat sheet of paper
(B) having a flat sheet of paper on which to draw the spherical Earth
(C) how can one draw the spherical Earth on a flat sheet of paper
(D) how they could use a flat sheet of paper to draw the spherical Earth
(E) how to draw the spherical Earth on a flat sheet of paper
205.Certain pesticides can become ineffective if used repeatedly in the same place; one reason is suggested by the finding that there are much larger populations of
pesticide-degrading microbes in soils with a relatively long history of pesticide use than in soils that are free of such chemicals.
(A) Certain pesticides can become ineffective if used repeatedly in the same place; one reason is suggested by the finding that there are much larger populations of
pesticide-degrading microbes in soils with a relatively long history of pesticide use than in soils that are free of such chemicals.
(B) If used repeatedly in the same place, one reason that certain pesticides can become ineffective is suggested by the finding that there are much larger populations of pesticide-degrading microbes in soils with a relatively long history of pesticide use than in soils that are free of such chemicals.
(C) If used repeatedly in the same place, one reason certain pesticides can become
ineffective is suggested by the finding that much larger populations of pesticide-degrading microbes are found in soils with a relatively long history of pesticide use than those that are free of such chemicals.
(D) The finding that there are much larger populations of pesticide-degrading microbes in soils with a relatively long history of pesticide use than in soils that are free of such chemicals is suggestive of one reason, if used repeatedly in the same place, certain pesticides can become ineffective.
(E) The finding of much larger populations of pesticide-degrading microbes in soils with a relatively long history of pesticide use than in those that are free of such chemicals suggests one reason certain pesticides can become ineffective if used repeatedly in the same place.
206.Charles Lindbergh, for his attempt at a solo transatlantic flight, was very reluctant to have any extra weight on his plane, he therefore refused to carry even a pound of mail, despite being offered $1,000 to do so.
(A) Charles Lindbergh, for his attempt at a solo transatlantic flight, was very reluctant to have any extra weight on his plane, he therefore
(B) When Charles Lindbergh was attempting his solo transatlantic flight, being very
reluctant to have any extra weight on his plane, he
(C) Since he was very reluctant to carry any extra weight on his plane when he was
attempting his solo transatlantic flight, so Charles Lindbergh
(D) Being very reluctant to carry any extra weight on his plane when he attempted his solo transatlantic flight was the reason that Charles Lindbergh
(E) Very reluctant to have any extra weight on his plane when he attempted his solo
transatlantic flight, Charles Lindbergh
207.Charlotte Perkins Gilman, a late nineteenth-century feminist, called for urban apartment houses including child-care facilities and clustered suburban houses including communal eating and social facilities.
(A) including child-care facilities and clustered suburban houses including communal eating and social facilities
(B) that included child-care facilities, and for clustered suburban houses to include communal eating and social facilities
(C) with child-care facilities included and for clustered suburban houses to include
communal eating and social facilities
(D) that included child-care facilities and for clustered suburban houses with communal eating and social facilities
(E) to include child-care facilities and for clustered suburban houses with communal eating and social facilities included
208.Chicago, where industrial growth in the nineteenth century was more rapid than any other American city, was plagued by labor troubles like the Pullman Strikes of 1894.
(A) where industrial growth in the nineteenth century was more rapid than any other American city
(B) which had industrial growth in the nineteenth century more rapid than that of other American cities
(C) which had growth industrially more rapid than any other American city in the nineteenth century
(D) whose industrial growth in the nineteenth century was more rapid than any other American city
(E) whose industrial growth in the nineteenth century was more rapid than that of any other American city
209.Child prodigies are marked not so much by their skills but instead by the fact that these skills are fully developed at a very early age.
(A) but instead
(B) rather than
(C) than
(D) as
(E) so much as
210.Chinese, the most ancient of living writing systems, consists of tens of thousands of ideographic characters, each character a miniature calligraphic composition inside its own square frame.
(A) each character a miniature calligraphic composition inside its
(B) all the characters a miniature calligraphic composition inside their
(C) all the characters a miniature calligraphic composition inside its
(D) every character a miniature calligraphic composition inside their
(E) each character a miniature calligraphic composition inside their
211.Chronic low-level leaking and the routine discharge of drilling mud and mineral salts present considerable environmental risk during offshore oil drilling.
(A) Chronic low-level leaking and the routine discharge of drilling mud and mineral salts present considerable environmental risk during offshore oil drilling.
(B) The reason offshore oil drilling presents a considerably environmental risk is because of chronic low-level leaking and the routine discharge of drilling mud and mineral salts.
(C) A considerable risk to the environment is presented during offshore oil drilling, where low-level leaks are chronic and the routine discharge of mud and mineral salts.
(D) Offshore oil drilling presents a considerable risk to the environment due to the fact of chronic low-level leaks, and mud and mineral salts are routinely discharged.
(E) Chronic low-level leaking, along with the routine discharge of drilling mud and mineral salts, are what make offshore oil drilling environmentally risky.
212.Citing evidence that the carbon dioxide content of the atmosphere has increased more than seven percent in the last 30 years, many scientists have expressed a fear of destroying forests and continued use of fossil fuels will cause an irreversible shift in Earth’s climatic pattern.
(A) many scientists have expressed a fear of destroying forests and continued use of
(B) many scientists have expressed a fear that destruction of forests and continued use of
(C) many scientists have expressed a fear that destruction of forests and continually using
(D) a fear that many scientists have expressed is that destroying forests and continually using
(E) a fear that many scientists have expressed is that destruction of forests and continual using of
213.Clark and Florence Wallace, a husband-and-wife medical team, worked steadily and
efficiently through the night, but sipping their coffee the next morning, she noticed that he seemed disoriented.
(A) sipping their coffee the next morning, she noticed that he seemed disoriented
(B) sipping their coffee the next morning, he seemed to be disoriented, she noticed
(C) as they sipped their coffee the next morning, she noticed that he seemed disoriented
(D) as they were sipping their coffee the next morning, he seemed, she noticed, disoriented
(E) he seemed disoriented, she noticed, sipping their coffee the next morning
214.Climatic shifts are so gradual as to be indistinguishable at first from ordinary
fluctuations in the weather.
(A) so gradual as to be indistinguishable
(B) so gradual they can be indistinguishable
(C) so gradual that they are unable to be distinguished
(D) gradual enough not to be distinguishable
(E) gradual enough so that one cannot distinguish them
215.Comparing the Quechans with other Native Americans of the Southwest, the Quechans were singularly uninterested in the accumulation of material wealth or in the crafting of elaborate pottery and basketry.
(A) Comparing the Quechans with other Native Americans of the Southwest, the Quechans
(B) When you compare the Quechans to other Native Americans of the Southwest, they
(C) When other Native Americans of the Southwest are compared to the Quechans, they
(D) Comparison to other Native Americans of the Southwest shows that the Quechans
(E) Compared with other Native Americans of the Southwest, the Quechans
216.Computers are becoming faster, more powerful, and more reliable, and so too are modems, they are the devices to allow two or more computers to share information over regular telephone lines.
(A) so too are modems, they are the devices to allow
(B) so too are modems, the devices that allow
(C) so too modems, the devices allowing
(D) also modems, they are the devices that allow
(E) also modems, which are the devices to allow
217.Concerned at the increase in accident fatalities, Tennessee adopted a child-passenger protection law requiring the parents of children under four years of age to be restrained in a child safety seat.
(A) the parents of children under four years of age to be restrained in a child safety seat
(B) the restraint of parents of children under four years of age in a child safety seat
(C) that parents restrain children under four years of age in a child safety seat
(D) that children be restrained under four years of age in a child safety seat by their parents
(E) children to be restrained under four years of age by their parents in a child safety seat
218.Confused by the many strata and substrata of ancient civilizations overlying one
another, Schliemann’s excavations of the fabled city of Ilium, the ancient Troy, were temporarily called to a halt.
(A) Schliemann’s excavations of the fabled city of Ilium, the ancient Troy, were temporarily called to a halt
(B) Schliemann’s excavations of the fabled city of Ilium, the ancient Troy, temporarily halted
(C) Schliemann temporarily halted his excavations of the fabled city of Ilium, the ancient Troy
(D) excavations of the fabled city of Ilium, the ancient Troy, were temporarily halted by Schliemann
(E) excavations of the fabled city of Ilium, and of the ancient Troy, were temporarily halted by Schliemann
219.Congress is debating a bill requiring certain employers provide workers with unpaid leave so as to care for sick or newborn children.
(A) provide workers with unpaid leave so as to
(B) to provide workers with unpaid leave so as to
(C) provide workers with unpaid leave in order that they
(D) to provide workers with unpaid leave so that they can
(E) provide workers with unpaid leave and
220.Constance Horner, chief of the United States government’s personnel agency, has
recommended that the use of any dangerous or illegal drug in the five years prior to
application for a job be grounds for not hiring an applicant.
(A) the use of any dangerous or illegal drug in the five years prior to application for a job be grounds for not hiring an applicant
(B) any dangerous or illegal drug, if used in the five years prior to applying for a job, should be grounds not to hire the applicant
(C) an applicant’s use of any dangerous or illegal drug in the five years prior to
application for a job be grounds not to hire them
(D) an applicant’s use of any dangerous or illegal drug in the five years prior to applying for a job are grounds that they not be hired
(E) for five years prior to applying for a job, an applicant’s use of any dangerous or illegal drug be grounds for not hiring them
221.Consumers may not think of household cleaning products to be hazardous substances, but many of them can be harmful to health, especially if they are used improperly.
(A) Consumers may not think of household cleaning products to be
(B) Consumers may not think of household cleaning products being
(C) A consumer may not think of their household cleaning products being
(D) A consumer may not think of household cleaning products as
(E) Household cleaning products may not be thought of, by consumers, as
222.Contrary to earlier conjectures, it may be that increased atmospheric carbon dioxide as a result of burning fossil fuels would cool the globe by reducing the amount of solar energy absorbed by snow.
(A) increased atmospheric carbon dioxide as a result of burning
(B) increased atmospheric carbon dioxide resulting from the combustion of
(C) increasing carbon dioxide in the atmosphere resulting from the combustion of
(D) carbon dioxide in the atmosphere that increase from burning
(E) atmospheric carbon dioxide that increased from burning
223.Contrary to popular belief, victors in the ancient Greek Olympic Games received cash prizes in addition to their laurel wreaths.
(A) Contrary to
(B) In contrast with
(C) Opposite of
(D) Unlike
(E) In spite of
224.Contrary to popular opinion, the movement toward a service economy is leading neither to lower standards of living, more of an unequal distribution of income, or displacing the physical production of goods.
(A) leading neither to lower standards of living, more of an unequal distribution of income, or
(B) leading neither to lower standards of living nor a more unequal distribution of income, or
(C) not leading to either lower standards of living nor to more of an unequal distribution of income, and neither is it
(D) not leading to lower standards of living, more of an unequal distribution of income, and it is not
(E) not leading to lower standards of living or to a more unequal distribution of income, nor is it
225.Convinced at last of the soundness of her advice, the villagers tried crop rotation, built crude sanitary facilities, and even the use of goat’s milk for the making of cheese.
(A) the use of goat’s milk for the making of cheese
(B) used goat’s milk to make cheese
(C) the use of goat’s milk in cheese making
(D) making cheese from goat’s milk
(E) goat’s milk to make cheese
226.Cooperative apartment houses have the peculiar distinction of being dwellings that must also operate as businesses.
(A) of being dwellings that must also operate as businesses
(B) of dwellings that must also operate like business
(C) that they are dwellings that must operate like business
(D) that, as dwellings, they must also operate like businesses
(E) to be a dwelling that must also operate as a business
227.Coronary angiography, a sophisticated method for diagnosing coronary disease involving the introduction of a dye into the arteries of the heart, is now administered selectively, because it uses x-rays to observe cardiac function.
(A) for diagnosing coronary disease involving the introduction of a dye into the arteries of the heart, is now administered selectively, because it uses
(B) for diagnosing coronary disease involving the introduction of a dye into the arteries of heart, is now administered selectively, because of using
(C) for diagnosing coronary disease, involves the introduction of dye into the arteries of the heart and is now administered selectively, because it uses
(D) to diagnose coronary disease that involves the introduction of a dye into the arteries of the heart, is now administered selectively, because it uses
(E) to diagnose coronary disease involving the introduction of a dye into the arteries of the heart, which is now administered selectively, uses
228.Corporations will soon be required to report to the government whether they have the necessary reserves to pay the pension benefits earned by their employees and that the information be published in annual reports to shareholders.
(A) earned by their employees and that the information be published
(B) that their employees earned and that the information be published
(C) that was earned by their employees with the information being published
(D) earned by their employees, information that must also be published
(E) earned by their employees and published the information
229.Crises in international diplomacy do not always result from malice; for nations, like individuals, can find themselves locked into difficult positions, unable to back down.
(A) do not always result from malice; for nations, like individuals, can find
(B) do not always results from malice; nations, just as individuals, finding
(C) do not always results from malice; nations, such as individuals, can find
(D) aren’t always the results of malice; nations in the same way that individuals can find
(E) aren’t resulting always from malice; just like individuals who can find
230.Critics of the trend toward privately operated prisons consider corrections facilities to be an integral part of the criminal justice system and question if profits should be made from incarceration.
(A) to be an integral part of the criminal justice system and question if
(B) as an integral part of the criminal justice system and they question if
(C) as being an integral part of the criminal justice system and question whether
(D) an integral part of the criminal justice system and question whether
(E) are an integral part of the criminal justice system, and they question whether
231.Cut-paper design, a popular pas of Colonial women, became an art form in the hands of Abigail Lefferts Lloyd, a Revolutionary War heroine.
(A) design, a popular pas of colonial women, became an art form in the hands of
(B) design, a popular Colonial women’s pas, and it became an art form in the hands of
(C) design was a popular pas for Colonial women, then it became an art form in the hands of
(D) design is a popular pas of Colonial women that has become an art form by
(E) design, the popular Colonial pastime, became an art form for
232.Dance fans knew Tamara Geva as a soloist in several Ballanchine works, a dancer who introduced his choreography to the United States, and as a star in Broadway theater.
(A) a dancer who introduced
(B) as a dancer which introduced
(C) the dancer to introduce
(D) who introduced
(E) as the dancer who introduced
233.Darwin was not the first to advance a theory of evolution; his tremendous originality lay in the fact that he proposed the idea of natural selection as the means by which evolution worked.
(A) lay in the fact that he proposed the idea
(B) lay in the fact of his proposing the idea
(C) laid in the fact of his proposing the idea
(D) laid in his proposal
(E) lay in his proposal
234.Declining values for farm equipment and land, the collateral against which farmers borrow to get through the harvest season, is going to force many lenders to tighten or deny credit this spring.
(A) the collateral against which farmers borrow to get through the harvest season, is
(B) which farmers use as collateral to borrow against to get through the harvest season, is
(C) the collateral which is borrowed against by farmers to get through the harvest season, is
(D) which farmers use as collateral to borrow against to get through the harvest season, are
(E) the collateral against which farmers borrow to get through the harvest season, are
235.Defense attorneys have occasionally argued that their clients’ misconduct stemmed from a reaction to something ingested, but in attributing criminal or delinquent behavior to some food allergy, the perpetrators are in effect told that they are not responsible for their actions.
(A) in attributing criminal or delinquent behavior to some food allergy
(B) if criminal or delinquent behavior is attributed to an allergy to some food
(C) in attributing behavior that is criminal or delinquent to an allergy to some food
(D) if some food allergy is attributed as the cause of criminal or delinquent behavior
(E) in attributing a food allergy as the cause of criminal or delinquent behavior
236.Degler does more than merely summarizing existing research; he constructs a coherent picture of two centuries of studies dealing with the changing roles of women.
(A) Degler does more than merely summarizing
(B) Degler’s study is more than a mere summarizing of
(C) Degler has done more than a mere summarizing of
(D) Degler’s study has done more than summarize merely
(E) Degler does more than merely summarize
237.Delighted by the reported earnings for the first quarter of the fiscal year, it was decided by the company manager to give her staff a raise.
(A) it was decided by the company manager to give her staff a raise
(B) the decision of the company manager was to give her staff a raise
(C) the company manager decided to give her staff a raise
(D) the staff was given a raise by the company manager
(E) a raise was given to the staff by the company manager
238.Dental caries and gingivitis can be exacerbated not only by the foods patients eat but also by when the patients eat them.
(A) not only by the foods patients eat but also by when the patients eat them
(B) by not only the foods patients eat but also by when the patients eat them
(C) not only by the foods patients eat but also by when the foods are eaten
(D) by not only the foods that are eaten by patients but also by the s the foods are eaten
(E) not only by what patients eat but also by when they eat it
239.Despite its attractiveness, investing abroad can still pose big risks, ranging from the potential for political instability in some countries to the shortage of regulations to protect investors and a serious lack of information about investments in others.
(A) to the shortage of regulations to protect investors and a serious lack of information about investments in others
(B) to the shortage of regulations to protect investors and in others a serious lack of information about investments
(C) and the shortage of regulations to protect investors and a serious lack of information about investments in others
(D) and the shortage of regulations to protect investors to a serious lack of information about investments in others
(E) to the shortage of regulations to protect investors in others and a serious lack of information about investments
240.Despite no proof that the consumption of any particular foods reverse hardening of the arteries, studies indicate that refraining from eating certain foods could help reverse blockage of coronary arteries, the blood vessels that feed the heart.
(A) Despite no proof that the consumption of any particular foods reverse hardening of the arteries, studies indicate that refraining from eating certain foods could
(B) Despite no foods having been proved to reverse hardening of the arteries when consumed, studies indicate that refraining from eating certain foods can
(C) Although the consumption of no particular foods have been proved to reverse hardening of the arteries, studies indicate that to refrain from eating certain foods could
(D) Although not proved that the consumption of any foods reverse hardening of the arteries, studies indicate that refraining from eating certain foods can
(E) Although it has not been proved that the consumption of any particular food will reverse hardening of the arteries, studies indicate that refraining from eating certain foods can
241.Despite protests from some waste-disposal companies, state health officials have ordered the levels of bacteria in seawater at popular beaches to be measured and that the results be published.
(A) the levels of bacteria in seawater at popular beaches to be measured and that the results be
(B) that seawater at popular beaches should be measured for their levels of bacteria, with the results being
(C) the measure of levels of bacteria in seawater at popular beaches and the results to be
(D) seawater measured at popular beaches for levels of bacteria, with their results
(E) that the levels of bacteria in seawater at popular beaches be measured and the results
242.Despite the recent election of a woman to the office of prime minister, the status of women in Pakistan is little changed from how it was in the last century.
(A) is little changed from how it was
(B) is a little change from how it was
(C) has changed little
(D) has changed little from how it has been
(E) is little changed from the way it was
243.Developing nations in various parts of the world have amassed $700 billion in debts; at stake, should a significant number of these debts be repudiated, is the solvency of some of the world’s largest multinational banks.
(A) should a significant number of these debts be repudiated, is
(B) should a significant number of these debts be repudiated, are
(C) should they repudiate a significant number of these debts, are
(D) if there is a repudiation of a significant number of these debts, would be
(E) if a significant number of these debts will be repudiated, is
244.Dirt roads may evoke the bucolic simplicity of another century, but financially strained townships point out that dirt roads cost twice as much as maintaining paved roads.
(A) dirt roads cost twice as much as maintaining paved roads
(B) dirt roads cost twice as much to maintain as paved roads do
(C) maintaining dirt roads costs twice as much as paved roads do
(D) maintaining dirt roads costs twice as much as it does for paved roads
(E) to maintain dirt roads costs twice as much as for paved roads
245.Discrimination in wages paid in occupations that are predominantly male over the
predominantly female have given rise to substantial differentials between the wage of housepainters and secretaries and between the wages of parking-lot attendants and library assistants.
(A) paid in occupations that are predominantly male over the predominantly female have
(B) paid in occupations that are predominantly make over those that are predominantly female have
(C) that favors predominantly male occupations over the predominantly female have
(D) that favors predominantly male occupations over those that are predominantly female has
(E) paid in predominantly male occupations over the predominantly female has
246.Displays of the aurora borealis, or “northern lights,” can heat the atmosphere over the arctic enough to affect the trajectories of ballistic missiles, induce electric currents that can cause blackouts in some areas and corrosion in north-south pipelines.
(A) to affect the trajectories of ballistic missiles, induce
(B) that the trajectories of ballistic missiles are affected, induce
(C) that it affects the trajectories of ballistic missiles, induces
(D) that the trajectories of ballistic missiles are affected and induces
(E) to affect the trajectories of ballistic missiles and induce
247.Distinguished architecture requires the expenditure of large sums of money, even if it is by no means certain that the expenditure of large sums of money produce distinguished architecture.
(A) even if it is by no means certain that the expenditure of large sums of money produce
(B) even if it is by no means certain that the expenditure of large sums of money will produce
(C) even though there is no certainty that the expenditure of money in large sums produces
(D) even though it is by no means certain that the expenditure of large sums of money produces
(E) though there is no certainty as to the expenditure of money in large sums producing
248.Doctors generally agree that such factors as cigarette smoking, eating rich foods high in fats, and alcohol consumption not only do damage by themselves but also aggravate genetic predispositions toward certain diseases.
(A) not only do damage by themselves but also aggravate
(B) do damage by themselves but also are aggravating to
(C) are damaging by themselves but also are aggravating
(D) not only do damage by themselves, they are also aggravating to
(E) are doing damage by themselves, and they are also aggravating
249.Doctors still know little about how the Listeria bacterium is spread and why the disease it causes, listeriosis, afflicts some people in a contaminated area though it spares many others.
(A) though it spares
(B) where it spares
(C) despite sparing
(D) when sparing
(E) while sparing
250.Domestic automobile manufacturers have invested millions of dollars into research to develop cars more gasoline-efficient even than presently on the road.
(A) into research to develop cars more gasoline-efficient even than presently on the road
(B) into research for developing even more gasoline-efficient cars on the road than at present
(C) for research for cars to be developed that are more gasoline-efficient even than
presently on the road
(D) in research to develop cars even more gasoline-efficient than those at present on the road
(E) in research for developing cars that are even more gasoline-efficient than presently on the road
251.Downzoning, zoning that typically results in the reduction of housing density, allows for more open space in areas where little water or services exist.
(A) little water or services exist
(B) little water or services exists
(C) few services and little water exists
(D) there is little water or services available
(E) there are few services and little available water
252.Dr. Hakuta’s research among Hispanic children in the United States indicates that the more the children use both Spanish and English, their intellectual advantage is greater in skills underlying reading ability and nonverbal logic.
(A) their intellectual advantage is greater in skills underlying reading ability and
nonverbal logic
(B) their intellectual advantage is the greater in skills underlaying reading ability and nonverbal logic
(C) the greater their intellectual advantage in skills underlying reading ability and nonverbal logic
(D) in skills that underlay reading ability and nonverbal logic, their intellectual
advantage is the greater
(E) in skills underlying reading ability and nonverbal logic, the greater intellectual advantage is theirs
253.Dr. Hegsted argues that just as polio vaccine is given to every person to protect the few who might actually contract polio, mass dietary change is needed to protect the significant number who are susceptible to the life-threatening effects of press eating habits.
(A) just as polio vaccine is given to every person to protect the few who might actually contract polio
(B) like polio vaccine, which is given to every person to protect the few who might contract polio
(C) similar to polio vaccine which is given to every person in order to protect the few who might actually contract polio
(D) while, to protect the few who might actually contract polio, polio vaccine is given to every person
(E) similar to the giving of polio vaccine to every person in order to protect the few who might contract polio actually
254.Dr. Sayre’s lecture recounted several little-known episodes in the relations between nations that illustrates what is wrong with alliances and treaties that do not have popular support.
(A) relations between nations that illustrates
(B) relation of one nation with another that illustrates
(C) relations between nations that illustrate
(D) relation of one nation with another and illustrate
(E) relations of nations that illustrates
255.Dr. Tonegawa won the Nobel Prize for discovering how the body can constantly change its genes to fashion a seeming unlimited number of antibodies, each specifically targeted at an invading microbe or foreign substance.
(A) seeming unlimited number of antibodies, each specifically targeted at
(B) seeming unlimited number of antibodies, each targeted specifically to
(C) seeming unlimited number of antibodies, all specifically targeted at
(D) seemingly unlimited number of antibodies, all of them targeted specifically to
(E) seemingly unlimited number of antibodies, each targeted specifically at
256.During an era when interracial tensions in the United States have run high, Rosa Parks became a quiet, unassuming symbol of the continued struggle for human dignity.
(A) have run high
(B) ran high
(C) had run highly
(D) run high
(E) were running highly
257.During an ice age, the buildup of ice at the poles and the drop in water levels near the equator speed up the Earth’s rotation, like a spinning figure skater whose speed increases when her arms are drawn in.
(A) like a spinning figure skater whose speed increases when her arms are drawn in
(B) like the increased speed of a figure skater when her arms are drawn in
(C) like a figure skater who increases speed while spinning with her arms drawn in
(D) just as a spinning figure skater who increases speed by drawing in her arms
(E) just as a spinning figure skater increases speed by drawing in her arms
258.During her lecture the speaker used map to clarify directional terms, for not everyone in attendance was knowledgeable that winds are designated by the direction from which they come.
(A) or not everyone in attendance was knowledgeable
(B) for everyone in attendance did not know
(C) with everyone in attendance not knowing
(D) with everyone attending not knowledgeable
(E) for not everyone attending knew
259.During Roosevelt’s years in office Black Americans began voting for Democrats rather than Republicans in national elections, but Black support for Democrats at the state and local levels developed only after when civil rights legislation was supported by Harry Truman.
(A) developed only after when civil rights legislation was supported by Harry Truman
(B) developed only after when Harry Truman supported civil rights legislation
(C) developed only after Harry Truman’s support of civil rights legislation
(D) develops only at the after the supporting of civil rights legislation by Harry Truman
(E) developed only after there being Harry Truman’s support of civil rights legislation
260.During the 1980’s approximately $50 billion in private investment capital is estimated to have left Mexico and added to the strain on the country’s debt-ridden economy.
(A) During the 1980’s approximately $50 billion in private investment capital is estimated to have left Mexico and added
(B) During the 1980’s it is estimated that approximately $50 billion in private investment capital left Mexico and added
(C) It is estimated that there was approximately $50 billion in private investment capital that left Mexico during the 1980’s and added
(D) It is estimated that during the 1980’s approximately $50 billion in private investment capital left Mexico, adding
(E) Approximately $50 billion in private investment capital is estimated as having left Mexico during the 1980’s, adding
261.During the 1980s it became clear that soliciting private funds was far more efficient for environmentalists who sought financial aid than to go to state or federal agencies.
(A) that soliciting private funds was far more efficient for environmentalists who sought financial aid
(B) that for environmentalists who sought financial aid, it was far more efficient to solicit private funds
(C) that for environmentalists seeking financial aid, private organizations were far more efficient to go to
(D) for environmentalists seeking financial aid, going to private organizations was far more efficient
(E) for environmentalists who sought financial aid, private organizations were far more efficient
262.During the early years of European settlement on a continent that was viewed as
“wilderness” by the newcomers, Native Americans, intimately knowing the ecology of the land, were a help in the rescuing of many Pilgrims and pioneers from hardship, or even death.
(A) Native Americans, intimately knowing the ecology of the land, were a help in the
rescuing of
(B) Native Americans knew the ecology and the land intimately and this enabled them to help in the rescue of
(C) Native Americans, with their intimate knowledge of the ecology of the land, helped to rescue
(D) having intimate knowledge of the ecology of the land, Native Americans helped the rescue of
(E) knowing intimately the ecology of the land, Native Americans helped to rescue
263.During the first nine months of 1979, textbook publishers incurred substantial costs for creating products that, due to a decline in public funding for instructional material, never were sold.
(A) funding for instructional material, never were
(B) funding for instructional material, never was
(C) funding, the instructional material, was never
(D) funding for instructional material, the products were never
(E) funding, they were never
264.During the first one hundred fifty years of the existence of this republic, no one expected the press was fair; newspapers were mostly shrill, scurrilous, and partisan.
(A) was
(B) to be
(C) of being
(D) should be
(E) had to be
265.During the first year after the corporate reorganization, no one considered the
management was well-organized; managers were largely untrained and directionless.
(A) was well-organized
(B) well-organized
(C) were well-organized
(D) seemed to be well-organized
(E) seemed well-organized
266.During the Great Depression, industrial output fell by nearly fifty percent from its peak in 1929 down to its nadir in 1933.
(A) During the Great Depression, industrial output fell by nearly fifty percent from its peak in 1929 down to its nadir in 1933.
(B) During the Great Depression, industrial output fell by nearly fifty percent from its peak in 1929 to its nadir in 1933.
(C) At the of the Great Depression, industrial output fell by almost fifty percent from its 1929 peak down to its 1933 nadir.
(D) At the of the Great Depression, industrial output fell from its peak in 1929, by nearly fifty percent, to its nadir in 1933.
(E) During the Great Depression, industrial output fell from its peak in 1929 to its nadir in 1933 by nearly fifty percent.
267.During the nineteenth century Emily Eden and Fanny Parks journeyed throughout India, sketching and keeping journals forming the basis of news reports about the princely states where they had visited.
(A) forming the basis of news reports about the princely states where they had
(B) that were forming the basis of news reports about the princely states
(C) to form the basis of news reports about the princely states which they have
(D) which had formed the basis of news reports about the princely states where they had
(E) that formed the basis of news reports about the princely states they
268.During the recession of 1973, home mortgage foreclosures resulted in tens of thousands of Americans being evicted from homes that they can no longer afford monthly payments.
(A) that they can
(B) that they could
(C) on which they can
(D) because they can
(E) for which they could
269.During the Renaissance, scholars were uncertain as to the location of Troy, and by the eighteenth century many historians doubted that Troy had ever existed.
(A) doubted that Troy had ever existed
(B) doubt that Troy has ever existed
(C) were in doubt as to the existence of Troy
(D) were doubtful concerning Troy’s existence
(E) had doubts about Troy’s ever existing
270.Each of Hemingway’s wives—Hadley Richardson, Pauline Pfeiffer, Martha Gelhorn, and Mary Welsh—were strong and interesting women, very different from the often pallid women who populate his novels.
(A) Each of Hemingway’s wives—Hadley Richardson, Pauline Pfeiffer, Martha Gelhorn, and Mary Welsh—were strong and interesting women,
(B) Hadley Richardson, Pauline Pfeiffer, Martha Gelhorn, and Mary Welsh—each of them
Hemingway’s wives—were strong and interesting women,
(C) Hemingway’s wives—Hadley Richardson, Pauline Pfeiffer, Martha Gelhorn, and Mary
Welsh—were all strong and interesting women,
(D) Strong and interesting women—Hadley Richardson, Pauline Pfeiffer, Martha Gelhorn, and Mary Welsh—each a wife of Hemingway, was
(E) Strong and interesting women—Hadley Richardson, Pauline Pfeiffer, Martha Gelhorn, and Mary Welsh—every one of Hemingway’s wives were
271.Eaten in the Mediterranean countries, northern Europeans viewed the tomato with
suspicion, for they assumed it had poisonous properties because of its relationship to deadly nightshade.
(A) northern Europeans viewed the tomato with suspicion, for they
(B) northern Europeans were suspicious of the tomato, and they
(C) the tomato was viewed with suspicion by northern Europeans, who
(D) the tomato was suspicious to northern Europeans, and it was
(E) the tomato was viewed with suspicion by northern Europeans, it being
272.Eating saltwater fish may significantly reduce the risk of heart attacks and also aid for sufferers of rheumatoid arthritis and asthma, according to three research studies published in the New England Journal of Medicine.
(A) significantly reduce the risk of heart attacks and also aid for
(B) be significant in reducing the risk of heart attacks and aid for
(C) significantly reduce the risk of heart attacks and aid
(D) cause a significant reduction in the risk of heart attacks and aid to
(E) significantly reduce the risk of heart attacks as well as aiding
273.Efforts to equalize the funds available to school districts, a major goal of education reformers and many states in the 1970’s, has not significantly reduced the gaps existing between the richest and poorest districts.
(A) has not significantly reduced the gaps existing
(B) has not been significant in reducing the gap that exists
(C) has not made a significant reduction in the gap that exists
(D) have not significantly reduced the gap that exists
(E) have not been significant in a reduction of the gaps existing
274.Egyptians are credited as having pioneered embalming methods as long ago as 2650 B.C.
(A) as having
(B) with having
(C) to have
(D) as the ones who
(E) for being the ones who
275.El Nino, the periodic abnormal warming of the sea surface off Peru, a phenomenon in which changes in the ocean and atmosphere combine allowing the warm water that has accumulated in the western Pacific to flow back to the east.
(A) a phenomenon in which changes in the ocean and atmosphere combine allowing the warm water that has accumulated
(B) a phenomenon where changes in the ocean and atmosphere are combining to allow the warm water that is accumulating
(C) a phenomenon in which ocean and atmosphere changes combine and which allows the warm water that is accumulated
(D) is a phenomenon in which changes in the ocean and atmosphere combine to allow the warm water that has accumulated
(E) is a phenomenon where ocean and atmosphere changes are combining and allow the warm water accumulating
276.Erasmus Montanus, a seventeenth-century farce written by Ludwig Holberg, both predates and resembles Moliere’s Tartuffe and is therefore thought to be one of Moliere’s sources.
(A) both predates and resembles
(B) it both predates and resembles
(C) both predated and resembles
(D) has both predated and resembled
(E) because it both predated and resembled
277.Europe’s travel industry is suffering as a result of a sluggish economy, a stretch of bad weather, as well as the chilling effects of terrorist activity that is persistent.
(A) as well as the chilling effects of terrorist activity that is persistent
(B) and the chilling effect of terrorist activity that is persistent
(C) but persistent terrorist activity has had a chilling effect too
(D) and the chilling effects of persistent terrorist activity
(E) as well as the chilling effects of terrorist activity that persists
278.Europeans have long known that eating quail sometimes makes the eater ill, but only recently has it been established that the illness is caused by a toxin present in the quail’s body only under certain conditions.
(A) Europeans have long known that eating quail sometimes makes
(B) Europeans have long known quail eating is sometimes able to make
(C) Eating quail has long been known to Europeans to sometimes make
(D) It has long been known to Europeans that quail eating will sometimes make
(E) It has long been known to Europeans that quail, when it is eaten, has sometimes made
279.Even as they never forgave the Crusaders who overran their homeland, the Syrians have never absolved the French for taking territory from them.
(A) Even as they never forgave
(B) While they never forgave
(C) Just like they never forgave
(D) Similarly to not forgiving
(E) In spite of their never forgiving
280.Even astronomers were amazed at the success of the Neptune flyby, which produced a photograph of a previously undetected moon; this is likely to result in increased
governmental support for the hitherto neglected U.S. space program.
(A) this is likely to result in
(B) that will cause
(C) and which is likely to result in
(D) this success is likely to result in
(E) it is likely to result with
281.Even their most ardent champions concede that no less than a technical or scientific breakthrough is necessary before solar cells can meet the goal of providing one percent of the nation’s energy needs.
(A) that no less than a technical or scientific breakthrough is necessary
(B) that nothing other than a technical or scientific breakthrough is needed
(C) that a technical or scientific breakthrough is necessary
(D) the necessity for an occurrence of a technical or scientific breakthrough
(E) the necessity for a technical or scientific breakthrough occurring
282.Even though Béla Bartók’s music has proved less popular than Igor Stravinsky’s and less influential than Arnold Schonberg’s, it is no less important.
(A) Stravinsky’s and less influential than Arnold Schonberg’s, it
(B) Stravinsky’s and less influential than Arnold Schonberg’s, he
(C) Stravinsky’s is and less influential than Arnold Schonberg’s is, it
(D) Stravinsky and not as influential as Arnold Schonberg, he
(E) Stravinsky and not as influential as Arnold Schonberg, it
283.Even though its per capita food supply hardly increased during two decades, stringent rationing and planned distribution have allowed the People’s Republic of China to ensure nutritional levels of 2,000 calories per person per day for its population.
(A) Even though its per capita food supply hardly increased during
(B) Even though its per capita food supply has hardly increased in
(C) Despite its per capita food supply hardly increasing over
(D) Despite there being hardly any increase in its per capita food supply during
(E) Although there is hardly any increase in per capita food supply for
284.Even though the direct costs of malpractice disputes amounts to a sum lower than one percent of the $541 billion the nation spent on health care last year, doctors say fear of lawsuits plays major role in health-care inflation.
(A) amounts to a sum lower
(B) amounts to less
(C) amounted to less
(D) amounted to lower
(E) amounted to a lower sum
285.Even though the state has spent ten years and seven million dollars planning a reservoir along the Ubi River, the project will have to be abandoned as a result of the river becoming so heavily polluted.
(A) will have to be abandoned as a result of the river becoming so heavily polluted
(B) is to be abandoned on account of the heavy pollution which the river received
(C) had to be abandoned because the river had received such heavy pollution
(D) has to be abandoned because of the river and its heavy pollution
(E) must be abandoned because the river has become so heavily polluted
286.Even today, a century after Pasteur developed the first vaccine, rabies almost always kills its victims unless inoculated in the earliest stages of the disease.
(A) its victims unless inoculated
(B) its victims unless they are inoculated
(C) its victims unless inoculation is done
(D) the victims unless there is an inoculation
(E) the victims unless inoculated
287.Everyone participating in the early sociological study committed a crucial
methodological error by failing to fully consider alternative ways of classifying their data.
(A) by failing to fully consider alternative ways of classifying their
(B) by failing fully to consider alternative ways of classifying their
(C) they failed to consider alternative ways to classify their
(D) by not fully considering alternative ways to classify the
(E) by not considering fully alternative ways of classifying his or her
288.Except for a concert performance that the composer himself staged in 1911, Scott
Joplin’s rag opera Treemonisha was not produced until 1972, sixty-one years after its completion.
(A) Except for a concert performance that the composer himself staged
(B) Except for a concert performance with the composer himself staging it
(C) Besides a concert performance being staged by the composer himself
(D) Excepting a concert performance that the composer himself staged
(E) With the exception of a concert performance with the staging done by the composer himself
289.Executives and federal officials say that the use of crack and cocaine is growing rapidly among workers, significantly compounding the effects of drug and alcohol abuse, which already are a cost to business of more than $100 billion a year.
(A) significantly compounding the effects of drug and alcohol abuse, which already are a cost to business of
(B) significantly compounding the effects of drug and alcohol abuse, which already cost business
(C) significantly compounding the effects of drug and alcohol abuse, already with business costs of
(D) significant in compounding the effects of drug and alcohol abuse, and already costing business
(E) significant in compounding the effects of drug and alcohol abuse, and already costs business
290.Faced with an estimated $2 billion budget gap, the city’s mayor proposed a nearly 17 percent reduction in the amount allocated the previous year to maintain the city’s major cultural institutions and to subsidize hundreds of local arts groups.
(A) proposed a nearly 17 percent reduction in the amount allocated the previous year to maintain the city’s major cultural institutions and to subsidize
(B) proposed a reduction from the previous year of nearly 17 percent in the amount it was allocating to maintain the city’s major cultural institutions and for subsidizing
(C) proposed to reduce, by nearly 17 percent, the amount from the previous year that was allocated for the maintenance of the city’s major cultural institutions and to subsidize
(D) has proposed a reduction from the previous year of nearly 17 percent of the amount it was allocating for maintaining the city’s major cultural institutions, and to subsidize
(E) was proposing that the amount they were allocating be reduced by nearly 17 percent from the previous year for maintaining the city’s major cultural institutions and for the subsidization
291.Factory outlet stores, operated by manufacturers, are usually located miles from
downtown and regional shopping centers so as not directly to be competitive against
department stores in the same trading area.
(A) so as not directly to be competitive against
(B) in order for them not to have direct competition with
(C) so that they do not compete directly with
(D) in order that they are not directly competitive against
(E) for the purpose of not competing directly with
292.Federal authorities involved in the investigation have found the local witnesses are difficult to locate, reticent, and are suspicious of strangers.
(A) the local witnesses are difficult to locate, reticent, and are
(B) local witnesses to be difficult to locate, reticent, and are
(C) that local witnesses are difficult to locate, reticent, and
(D) local witnesses are difficult to locate and reticent, and they are
(E) that local witnesses are difficult to locate and reticent, and they are
293.Federal incentives now encourage investing capital in commercial office buildings despite vacancy rates in existing structures that are exceptionally high and no demand for new construction.
(A) investing capital in commercial office buildings despite vacancy rates in existing structures that are exceptionally high and
(B) capital investment in commercial office buildings, even though vacancy rates in existing structures are exceptionally high and there is
(C) capital to be invested in commercial office buildings even though there are
exceptionally high vacancy rates in existing structures with
(D) investing capital in commercial office buildings even though the vacancy rates are exceptionally high in existing structures with
(E) capital investment in commercial office buildings despite vacancy rates in existing structures that are exceptionally high, and although there is
294.Federal legislation establishing a fund for the cleanup of sites damaged by toxic chemicals permits compensating state governments for damage to their natural resources but does not allow claims for injury to people.
(A) compensating state governments for damage to
(B) compensating state governments for the damaging of
(C) giving state governments compensation for damaging
(D) giving compensation to state governments for the damage of
(E) the giving of compensation to state governments for damaging
295.Federally imposed restrictions on how much they may pay small savers has made
difficulties for savings banks as they are competing with such unregulated investment vehicles as money market certificates.
(A) has made difficulties for savings banks as they are competing with such
(B) has made difficulties for savings banks competing with such
(C) have made difficulties for savings banks as they are competing with
(D) have made it difficult for savings banks to compete with such
(E) have made it difficult for savings banks as they are competing with such
296.Few people realize that the chance of accidental injury or death may be as great or greater in the “safety” of their own homes than in a plane or on the road.
(A) may be as great or greater in the “safety” of their own homes than
(B) is at least as great or greater in the “safety” of their own homes than
(C) might be so great or greater in the “safety” of their own home as
(D) may be at least as great in the “safety” of their own homes as
(E) can be at least so great in the “safety” of their own home as
297.Fifty-two percent of United States high school graduates go on to college, compared with Canada’s thirty-five percent and Great Britain, Japan, and West Germany’s fifteen percent.
(A) Fifty-two percent of United States high school graduates go on to college, compared with Canada’s thirty-five percent and Great Britain, Japan, and West Germany’s fifteen percent.
(B) Fifty-two percent of United States high school graduates go on to college; in Canada it is thirty-five percent and in Great Britain, Japan, and West Germany it is fifteen percent.
(C) In the United States, Fifty-two percent of high school graduates go on to college, compared with thirty-five percent in Canada and fifteen percent in Great Britain, Japan, and West Germany.
(D) The percentage of high school graduates in the United States who go on to college is fifty-two, compared with Canada’s thirty-five percent, Great Britain’s fifteen, Japan’s fifteen, and West Germany’s fifteen.
(E) The percentage of United States high school graduates going on to college is fifty-two that in Canada is thirty-five, and that in Great Britain, Japan, and West Germany is fifteen.
298.Five fledgling sea eagles left their nests in western Scotland this summer, bringing to 34 the number of wild birds successfully raised since transplants from Norway began in 1975.
(A) bringing
(B) and brings
(C) and it brings
(D) and it brought
(E) and brought
299.Florida will gain another quarter-million jobs this year alone, many of them in
high-paying fields like electronics and banking, making the state’s economy far more
diversified than ten years ago.
(A) high-paying fields like electronics and banking, making the state’s economy far more diversified than
(B) high-paying fields like electronics and banking, and making the state’s economy far more diversified than its economy
(C) high-paying fields such as electronics and banking, to make the state’s economy far more diversified than
(D) such high-paying fields as electronics and banking, making the state’s economy far more diversified than it was
(E) such high-paying fields as electronics and banking, and make the state’s economy far more diversified than it was
300.Following the destruction of the space shuttle Challenger, investigators concluded that many key people employed by the National Aeronautics and Space Administration and its contractors work an excessive amount of over that has the potential of causing errors in judgment.
(A) over that has the potential of causing
(B) over that has the potential to cause
(C) over that potentially can cause
(D) over, a practice that has the potential for causing
(E) over, a practice that can, potentially, cause
301.Following the nutrition board’s advice on salt consumption would mean a virtual end of the use of salt in cooking, an avoidance of obviously salty foods, and reducing the reliance on processed foods that contain significant amounts of often hidden sodium.
(A) reducing the reliance on processed foods that contain significant amounts of often hidden sodium
(B) reducing the reliance on processed foods containing often hidden but significant amounts of sodium
(C) a reduction of the reliance on processed foods, containing as they do often hidden sodium in significant amounts
(D) a reduced reliance on the significant amounts of hidden sodium often contained in processed foods
(E) a reduced reliance on processed foods that contain significant but often hidden amounts of sodium
302.For all his professed disdain of such activities, Auden was an inveterate literary gossip.
(A) For all his professed disdain of such activities
(B) Having always professed disdain for such activities
(C) All such activities were, he professed, disdained, and
(D) Professing that all such activities were disdained
(E) In spite of professions of disdaining all such activities
303.For almost a hundred years after having its beginning in 1788, England exiled some 160,000 criminals to Australia.
(A) For almost a hundred years after having its beginning in 1788
(B) Beginning in 1788 for a period of a hundred years
(C) Beginning a period of almost a hundred years, in 1788
(D) During a hundred years, a period beginning in 1788
(E) Over a period of a hundred years beginning in 1788
304.For many people, household labor remains demanding even if able to afford household appliances their grandparents would find a miracle.
(A) even if able to afford household appliances their grandparents would find a miracle
(B) despite being able to afford household appliances their grandparents would find a miracle
(C) even if they can afford household appliances their grandparents would have found
miraculous
(D) although they could afford household appliances their grandparents would find miraculous
(E) even if they are able to afford household appliances which would have been a miracle to their grandparents
305.For many travelers, charter vacations often turn out to cost considerably more than they originally seemed.
(A) they originally seemed
(B) they originally seem to
(C) they seemingly would cost originally
(D) it seemed originally
(E) it originally seemed they would
306.For members of the seventeenth-century Ashanti nation in Africa, animal-hide shields with wooden frames were essential items of military equipment, a method to protect warriors against enemy arrows and spears.
(A) a method to protect
(B) as a method protecting
(C) protecting
(D) as a protection of
(E) to protect
307.For most consumers, the price of automobile insurance continues to rise annually, even if free of damage claims and moving violations.
(A) even if
(B) despite being
(C) even if they are
(D) although they may be
(E) even if remaining
308.For protection from the summer sun, the Mojave lived in open-sided, flat-topped
dwellings known as shades, each a roof of poles and arrowweed supported by posts set in a rectangle.
(A) each a roof of poles and arrowweed
(B) each a roof of poles and arrowweed that are being
(C) with each being a roof of poles and arrowweed
(D) with roofs of poles and arrowweed to be
(E) with roofs of poles and arrowweed that are
309.For some birds the sense of smell appears to play a role in navigation, since pigeons with surgically removed olfactory nerves were found to have increased difficulties in homing.
(A) were found to have increased difficulties
(B) have been found to have increased difficulty
(C) were found to have increasing difficulty
(D) had been found to have increased difficulties
(E) have been found to have increasing difficulties
310.For some reason the new consultant treats his clients like idiots, talking to them like they were mentally deficient and incapable of understanding more than the simplest ideas.
(A) like idiots, talking to them like they
(B) as if they were idiots, talking to them like they
(C) like idiots, talking to them as if they
(D) as idiots, talking to them like they
(E) like idiots who
311.Foreign investors, because of their growing confidence in their capability for making profitable investments in the United States, have been led to move from passive involvement in commercial real estate partnerships to active development of their own increasingly ambitious projects.
(A) Foreign investors, because of their growing confidence in their capability for making profitable investments in the United States, have been led
(B) Foreign investors, growing confident about their capability for making profitable investments in the United States, has led them
(C) Growing confidence in their ability to make profitable investments in the United States has led foreign investors
(D) Growing confidence in their ability for making profitable investments in the United States have led foreign investors
(E) Growing confident about their capabilities for making profitable investments in the United States, foreign investors have been led
312.Formulas for cash flow and the ratio of debt to equity do not apply to new small
businesses in the same way as they do to established big businesses, because they are growing and are seldom in equilibrium.
(A) Formulas for cash flow and the ratio of debt to equity do not apply to new small
businesses in the same way as they do to established big businesses, because they are growing and are seldom in equilibrium.
(B) Because they are growing and are seldom in equilibrium, formulas for cash flow and the ratio of debt to equity do not apply to new small businesses in the same way as they do to established big businesses.
(C) Because they are growing and are seldom in equilibrium, new small businesses are not subject to the same applicability of formulas for cash flow and the ratio of debt to equity as established big businesses.
(D) Because new small businesses are growing and are seldom in equilibrium, formulas for cash flow and the ratio of debt to equity do not apply to them in the same way as to establish big businesses.
(E) New small businesses are not subject to the applicability of formulas for cash flow and the ratio of debt to equity in the same way as established big businesses, because they are growing and are seldom in equilibrium.
313.Found throughout Central and South America, sloths hang from trees by long rubbery limbs and sleep fifteen hours a day, moving infrequently enough that two species of algae grow on its coat and between its toes.
(A) sloths hang from trees by long rubbery limbs and sleep fifteen hours a day, moving infrequently enough
(B) sloths hang from trees by long rubbery limbs, they sleep fifteen hours a day, and with such infrequent movements
(C) sloths use their long rubbery limbs to hang from trees, sleep fifteen hours a day, and move so infrequently
(D) the sloth hangs from trees by its long rubbery limbs, sleeping fifteen hours a day and moving so infrequently
(E) the sloth hangs from trees by its long rubbery limbs, sleeps fifteen hours a day, and it moves infrequently enough
314.Founded in 1983, the magazine increased its circulation more than double since then, and its advertising.
(A) increased its circulation more than double since then,
(B) has since increased its circulation more than double,
(C) has since more than doubled its circulation
(D) since then more than doubled its circulation
(E) more than doubled its circulation since then
315.Framed by traitorous colleagues, Alfred Dreyfus was imprisoned for twelve years before there was exoneration and his freedom.
(A) there was exoneration and his freedom
(B) he was to be exonerated with freedom
(C) being exonerated and freed
(D) exoneration and his freedom
(E) being freed, having been exonerated
316.Frances Wright’s book on America contrasted the republicanism of the United States with what she saw as the aristocratic and corrupt institutions of England.
(A) with what she saw as
(B) with that which she saw to be
(C) to that she saw being
(D) and that which she saw as
(E) and what she saw to be
317.Freedman’s survey showed that people living in small towns and rural areas consider themselves no happier than do people living in big cities.
(A) no happier than do people living
(B) not any happier than do people living
(C) not any happier than do people who live
(D) no happier than are people who are living
(E) not as happy as are people who live
318.From 1965 on, Yugoslavia’s standard of living has soared, but unemployment and prices too.
(A) but unemployment and prices too
(B) and also unemployment and prices
(C) but so have unemployment and prices
(D) and so also unemployment and prices
(E) but so did unemployment and prices
319.From 1982 to 1987 sales of new small boats increased between five and ten percent annually.
(A) From 1982 to 1987 sales of new small boats increased between five and ten percent annually.
(B) Five to ten percent is the annual increase in sales of new small boats in the years 1982 to 1987.
(C) Sales of new small boats have increased annually five and ten percent in the years 1982 to 1987.
(D) Annually an increase of five to ten percent has occurred between 1982 and 1987 in the sales of new small boats.
(E) Occurring from 1982 to 1987 was an annual increase of five and ten percent in the sales of new small boats.
320.From the bark of the paper birch tree the Menomini crafted a canoe about twenty feet long and two feet wide, with small ribs and rails of cedar, which could carry four persons or eight hundred pounds of baggage so light that a person could easily portage it around impeding rapids.
(A) baggage so light
(B) baggage being so light
(C) baggage, yet being so light
(D) baggage, and so light
(E) baggage yet was so light
321.From the earliest days of the tribe, kinship determined the way in which the Ojibwa society organized its labor, provided access to its resources, and defined rights and obligations involved in the distribution and consumption of those resources.
(A) and defined rights and obligations involved in the distribution and consumption of those resources
(B) defining rights and obligations involved in their distribution and consumption
(C) and defined rights and obligations as they were involved in its distribution and
consumption
(D) whose rights and obligations were defined in their distribution and consumption
(E) the distribution and consumption of them defined by rights and obligations
322.From the of its defeat by the Germans in 1940 until its liberation in 1944, France was a bitter and divided country; a kind of civil war raged in the Vichy government between those who wanted to collaborate with the Nazis with those who opposed them.
(A) between those who wanted to collaborate with the Nazis with those who opposed
(B) between those who wanted to collaborate with the Nazis and those who opposed
(C) between those wanting to collaborate with the Nazis with those opposing
(D) among those who wanted to collaborate with the Nazis and those who opposed
(E) among those wanting to collaborate with the Nazis with those opposing
323.Galileo was convinced that natural phenomena, as manifestations of the laws of physics, would appear the same to someone on the deck of a ship moving smoothly and uniformly through the water as a person standing on land.
(A) water as a
(B) water as to a
(C) water; just as it would to
(D) water, as it would to the
(E) water; just as to the
324.Gall’s hypothesis of there being different mental functions localized in different parts of the brain is widely accepted today.
(A) of there being different mental functions localized in different parts of the brain is widely accepted today
(B) of different mental functions that are localized in different parts of the brain is widely accepted today
(C) that different mental functions are localized in different parts of the brain is widely accepted today
(D) which is that there are different mental functions localized in different parts of the brain is widely accepted today
(E) which is widely accepted today is that there are different mental functions localized in different parts of the brain
325.Geologists believe that the Bering land bridge, over which human beings are thought to have first entered the Americans, disappeared about 14,000 years ago when massive glaciers melted and caused the sea level to rise several hundred feet worldwide.
(A) are thought to have first entered
(B) were thought first to enter
(C) were thought at first to enter
(D) are thought of as first entering
(E) were thought to first enter
326.Geologists believe that the warning signs for a major earthquake may include sudden fluctuations in local seismic activity, tilting and other deformations of the Earth’s crust, changing the measured stain across a fault zone, and varying the electrical properties of underground rocks.
(A) changing the measured strain across a fault zone, and varying
(B) changing measurements of the strain across a fault zone, and varying
(C) changing the strain as measured across a fault zone, and variations of
(D) changes in the measured strain across a fault zone, and variations in
(E) changes in measurements of the strain across a fault zone, and variations among
327.George Sand (Aurore Lucile Dupin) was one of the first European writers to consider the rural poor to be legitimate subjects for literature and portray these with sympathy and respect in her novels.
(A) to be legitimate subjects for literature and portray these
(B) should be legitimate subjects for literature and portray these
(C) as being legitimate subjects for literature and portraying them
(D) as if they were legitimate subjects for literature and portray them
(E) legitimate subjects for literature and to portray them
328.Green anole lizards, familiar to schoolchildren as chameleons, have recently become familiar to biologists as an excellent animal for laboratory studies of the interaction between stimuli with hormones.
(A) an excellent animal for laboratory studies of the interaction between stimuli with
(B) an excellent animal for laboratory studies of interaction of stimuli and
(C) being excellent animals for laboratory studies of the interaction between stimuli with
(D) excellent animals for laboratory studies of the interaction between stimuli with
(E) excellent animals for laboratory studies of the interaction of stimuli and
329.Growing competitive pressures may be encouraging auditors to bend the rules in favor of clients; auditors may, for instance, allow a questionable loan to remain on the books in order to maintain a bank’s profits on paper.
(A) clients; auditors may, for instance, allow
(B) clients, as an instance, to allow
(C) clients, like to allow
(D) clients, such as to be allowing
(E) clients; which might, as an instance, be the allowing of
330.Having the right hand and arm being crippled by a sniper’s bullet during the First World War, Horace Pippin, a Black American painter, worked by holding the brush in his right hand and guiding its movements with his left.
(A) Having the right hand and arm being crippled by a sniper’s bullet during the First World War
(B) In spite of his right hand and arm being crippled by a sniper’s bullet during the First World War
(C) Because there had been a sniper’s bullet during the First World War that crippled his right hand and arm
(D) The right hand and arm being crippled by a sniper’s bullet during the First World War
(E) His right hand and arm crippled by a sniper’s bullet during the First World War
331.Health officials estimate that 35 million Africans are in danger of contracting
trypanosomiasis, or “African sleeping sickness,” a parasitic disease spread by the bites of tsetse flies.
(A) are in danger of contracting
(B) are in danger to contract
(C) have a danger of contracting
(D) are endangered by contraction
(E) have a danger that they will contract
332.His studies of ice-polished rocks in his Alpine homeland, far outside the range of present-day glaciers, led Louis Agassiz in 1837 to propose the concept of an age in which great ice sheets had existed in now currently temperate areas.
(A) in which great ice sheets had existed in now currently temperate areas
(B) in which great ice sheets existed in what are now temperate areas
(C) when great ice sheets existed where there were areas now temperate
(D) when great ice sheets had existed in current temperate areas
(E) when great ice sheets existed in areas now that are temperate
333.Houseflies that hatch in summer live only about three weeks, but those that emerge in the cooler days of fall often live longer than six months.
(A) weeks, but those that emerge in the cooler days of fall often live
(B) weeks, but those that emerge in the cooler days of fall often live as long or
(C) weeks, which is different from those that emerge in the cooler days of fall and often live
(D) weeks; then those that emerge in the cooler days of fall often live as long as or
(E) weeks; this is different from those that emerge in the cooler days of fall, who often live
334.However much United States voters may agree that there is waste in government and that the government as a whole spends beyond its means, it is difficult to find broad support for a movement toward a minimal state.
(A) However much United States voters may agree that
(B) Despite the agreement among United States voters to the fact
(C) Although United States voters agree
(D) Even though United States voters may agree
(E) There is agreement among United States voters that
335.Humans have been damaging the environment for centuries by overcutting trees and farming too intensively, and though some protective measures, like the establishment of national forests and wildlife sanctuaries, having been taken decades ago, great increases in population and in the intensity of industrialization are causing a worldwide ecological crisis.
(A) though some protective measures, like the establishment of national forests and wildlife sanctuaries, having been taken decades ago, great increases in population
(B) though some protective measures, such as the establishment of national forests and wildlife sanctuaries, were taken decades ago, great increases in population
(C) though some protective measures, such as establishing national forests and wildlife sanctuaries, having been taken decades ago, great population increases
(D) with some protective measures, like establishing national forests and wildlife
sanctuaries that were taken decades ago, great increases in population
(E) with some protective measures, such as the establishment of national forests and
wildlife sanctuaries, having been taken decades ago, great population increases
336.Ideally, the professional career diplomat should help in the ongoing maintenance of an effective American foreign policy despite changes in administration.
(A) in the ongoing maintenance of
(B) in the maintaining of
(C) maintain
(D) to maintain and continue
(E) the maintenance of
337.Idioglossia is a phenomenon, incompletely understood at best, where two persons develop a unique and private language with highly original vocabulary and syntax.
(A) where two persons develop a unique and private language with
(B) when two persons develop a unique and private language having
(C) in which two persons have unique and private language development with
(D) having two persons who develop a unique and private language that has
(E) in which two persons develop a unique and private language with
338.If a single strain of plant is used for a given crop over a wide area, a practice fostered by modern seed-marketing methods, it increases the likelihood that the impact of a single crop disease or pest will be disastrous.
(A) If a single strain of plant is used for a given crop over a wide area, a practice fostered by modern seed-marketing methods, it
(B) If a single strain of plant is used for a given crop over a wide area, as is fostered by modern seed-marketing methods, it
(C) A practice fostered by modern seed-marketing methods, a single strain of plant used for a given crop over a wide area
(D) A single strain of plant used for a given crop over a wide area, a practice fostered by modern seed-marketing methods,
(E) The use of single strain of plant for a given crop over a wide area, a practice fostered by modern seed-marketing methods,
339.If additional deposits of oil are found, it will expand the amount that can be used as fuel and reduce the price of oil, even if the deposits are not immediately tapped.
(A) it will expand the amount that can be used as fuel and reduce the price of oil
(B) the amount that is able to be used as fuel will expand and the price of oil be reduced
(C) it will cause an increase in the amount that is able to be used as fuel and a reduction in the price of oil
(D) the amount that can be used as fuel will increase and the price of oil will drop
(E) it will increase the amount of oil that can be used as fuel and cause a drop in the price
340.If anyone at InterCom Financial Advisers would have anticipated, or even suspected, the impending sale of the Koniko kelp processing plant, they would have advised owners of Koniko stock to unload all shares immediately.
(A) If anyone at InterCom Financial Advisers would have anticipated
(B) Had anyone at InterCom Financial Advisers anticipated
(C) If any people at InterCom Financial Advisers would have anticipated
(D) If any people at InterCom Financial Advisers had anticipated
(E) If anybody at InterCom Financial Advisers anticipated
341.If Dr. Wade was right, any apparent connection of the eating of highly processed foods and excelling at sports is purely coincidental.
(A) If Dr. Wade was right, any apparent connection of the eating of
(B) Should Dr. Wade be right, any apparent connection of eating
(C) If Dr. Wade is right, any connection that is apparent between eating of
(D) If Dr. Wade is right, any apparent connection between eating
(E) Should Dr. Wade have been right, any connection apparent between eating
342.If industrial pollution continues to deplete the ozone layer, the resulting increase in ultraviolet radiation will endanger human health, causing a rise in the incidence of skin cancer and eye disease, and perhaps even threatening global ecological systems.
(A) and perhaps even threatening
(B) and may even threaten
(C) and even a possible threat to
(D) as well as possibly threatening
(E) as well as a possible threat to
343.If the new airboat does what it is to be doing—travel at high speeds undeterred by sandbars, crocodile-infested mudflats, or marshy hippo haunts—it could revolutionize transport on the 2,900-mile-long Congo River.
(A) If the new airboat does what it is to be doing
(B) If the new airboat does what it is supposed to do
(C) If it does as the new airboat is supposed to do
(D) Doing what it is the new airboat is supposed to do
(E) Doing what the new airboat is to be doing
344.If the proposed expenditures for gathering information abroad are reduced even further, international news reports have been and will continue to diminish in number and quality.
(A) have been and will continue to diminish
(B) have and will continue to diminish
(C) will continue to diminish, as they already did,
(D) will continue to diminish, as they have already,
(E) will continue to diminish
345.If the reporter would have known the landlord’s side of the story, she would not have written an article so favorable to the 81-year-old tenant.
(A) would have known the landlord’s side of the story, she would not have written
(B) would of known the landlord’s side of the story, she would not of written
(C) had known the landlord’s side of the story, she would not have written
(D) had known the landlord’s side of the story, she would not have wrote
(E) knew the landlord’s side of the story, she would not have written
346.Ignoring the admonitions of his staff, the chief financial officer accepted the advice of the consulting company because he believed that the standardized accounting procedures would prove not only inexpensive but reliable indicators of economic performance.
(A) he believed that the standardized accounting procedures would prove not only inexpensive but
(B) the standardized accounting procedures will prove both inexpensive and also
(C) he believed the standardized accounting procedures would prove themselves to be both inexpensive and
(D) he believed that the standardized accounting procedures would prove to be both
inexpensive and
(E) standardized accounting procedures will prove his belief that they are both inexpensive and
347.Iguanas have been an important food source in Latin America since prehistoric s, and it is still prized as a game animal by the campesinos, who typically cook the meat in a heavily spiced stew.
(A) it is still prized as a game animal
(B) it is still prized as game animals
(C) they are still prized as game animals
(D) they are still prized as being a game animal
(E) being still prized as a game animal
348.In 1527 King Henry VIII sought to have his marriage to Queen Catherine annulled so as to marry Anne Boleyn.
(A) so as so marry
(B) and so could be married to
(C) to be married to
(D) so that he could marry
(E) in order that he would marry
349.In 1791 Robert Carter III, one of the wealthiest plantation owners in Virginia, stunned his family, friends, and neighbors by filing a deed of emancipation, setting free the more than 500 slaves who were legally considered his property.
(A) setting free the more than 500 slaves who were legally considered
(B) setting free more than the 500 slaves legally considered as
(C) and set free more than 500 slaves, who were legally considered as
(D) and set free more than the 500 slaves who were legally considered
(E) and he set free the more than 500 slaves who were legally considered as
350.In 1922, when Truman was almost forty years old, he was living in his mother-in-law’s house, watching the haberdashery store he opened three years earlier go bankrupt, and he faced a future with no visible prospects.
(A) opened three years earlier go bankrupt, and he faced
(B) opened three years earlier go bankrupt and faced
(C) had opened three years earlier go bankrupt, and he was facing
(D) had opened three years earlier go bankrupt, and facing
(E) was opening three years earlier going bankrupt, and facing
351.In 1929 relatively small declines in the market ruined many speculators having bought on margin; they had to sell, and their selling pushed other investors to the brink.
(A) speculators having bought on margin; they had to sell, and
(B) speculators who had bought on margin; having had to sell,
(C) speculators who had bought on margin; they had to sell, and
(D) speculators, those who had bought on margin; these speculators had to sell, and
(E) speculators, who, having bought on margin and having to sell,
352.In 1933 the rubber, clothing, and shipbuilding industries put into effect a six-hour workday, believing it a seeming permanent accommodation rather than a temporary expedient for what many observers thought was an economy made overproductive by advances in technology.
(A) believing it a seeming permanent accommodation rather than a temporary expedient for what many observers thought was
(B) believing it a seeming permanent accommodation instead of a temporary expedient for what many observers thought was
(C) believing that it was not a temporary expedient but a seeming permanent accommodation to what many observers thought of as a
(D) not as a temporary expedient but as a seemingly permanent accommodation to what many observers thought was
(E) not as a temporary expedient but believing it a seemingly permanent accommodation for what many observers thought
353.In 1973 mortgage payments represented twenty-one percent of an average thirty-year-old male’s income; and forty-four percent in 1984.
(A) income; and forty-four percent in 1984
(B) income; in 1984 the figure was forty-four percent
(C) income, and in 1984 forty-four percent
(D) income, forty-four percent in 1984 was the figure
(E) income that rose to forty-four percent in 1984
354.In 1978 a national study found that not only had many contractors licensed by a
self-policing private guild failed to pass qualifying exams, they in addition falsified their references.
(A) they in addition falsified their references
(B) they had their references falsified in addition
(C) but they had also falsified their references
(D) they had also falsified their references
(E) but their references were falsified as well
355.In 1978 only half the women granted child support by a court received the amount
awarded; at least as much as a million and more others had not any support agreements whatsoever.
(A) at least as much as a million and more others had not any
(B) at least as much as more than a million others had no
(C) more than a million others had not any
(D) more than a million others had no
(E) there was at least a million or more others without any
356.In 1980 the United States exported twice as much of its national output of goods as they had in 1970.
(A) twice as much of its national output of goods as they had
(B) double the amount of their national output of goods as they did
(C) twice as much of its national output of goods as it did
(D) double the amount of its national output of goods as it has
(E) twice as much of their national output of goods as they had
357.In 1980, for the first , the number of foreigners touring the United States were in excess of the number of Americans going abroad.
(A) were in excess of the number of Americans
(B) had an excess over the Americans who were
(C) exceeded the Americans who were
(D) numbered more than the Americans
(E) exceeded the number of Americans
358.In 1982 the median income for married-couple families with a wage-earning wife was $9,000 more than a family where the husband only was employed.
(A) a family where the husband only
(B) of a family where only the husband
(C) that for families in which only the husband
(D) a family in which only the husband
(E) those of families in which the husband only
359.In 1990’s, there are more babies born by women over thirty years old than under it.
(A) than under it
(B) than were they under it
(C) than had been under it
(D) than were the babies
(E) than those were under it
360.In a 5-to-4 decision, the Supreme Court ruled that two upstate New York counties owed restitution to three tribes of Oneida Indians for the unlawful seizure of their ancestral lands in the eighteenth century.
(A) that two upstate New York counties owed restitution to three tribes of Oneida Indians for the unlawful seizure of
(B) that two upstate New York counties owed restitution to three tribes of Oneida Indians because of their unlawful seizure of
(C) two upstate New York counties to owe restitution to three tribes of Oneida Indians for their unlawful seizure of
(D) on two upstate New York counties that owed restitution to three tribes of Oneida Indians because they unlawfully seized
(E) on the restitution that two upstate New York counties owed to three tribes of Oneida Indians for the unlawful seizure of
361.In a period of when women typically have had a narrow range of choices, Mary Baker Eddy became a distinguished writer and the founder, architect, and builder of a growing church.
(A) In a period of when women typically have
(B) During a in which typically women have
(C) Typically, during a when women
(D) At a when women typically
(E) Typically in a in which women
362.In a plan to stop the erosion of East Coast beaches, the Army Corps of Engineers
proposed building parallel to shore a breakwater of rocks that would rise six feet above the waterline and act as a buffer, so that it absorbs the energy of crashing waves and protecting the beaches.
(A) act as a buffer, so that it absorbs
(B) act like a buffer so as to absorb
(C) act as a buffer, absorbing
(D) acting as a buffer, absorbing
(E) acting like a buffer, absorb
363.In a recent poll, 86 percent of the public favored a Clean Air Act as strong or stronger than the present act.
(A) a Clean Air Act as strong or stronger than
(B) a Clean Air Act that is stronger, or at least so strong as,
(C) at least as strong a Clean Air Act as is
(D) a Clean Air Act as strong or stronger than is
(E) a Clean Air Act at least as strong as
364.In A.D. 391, resulting from the destruction of the largest library of the ancient world at Alexandria, later generations lost all but the Iliad and Odyssey among Greek epics, most of the poetry of Pindar and Sappho, and dozens of plays by Aeschylus and Euripides.
(A) resulting from the destruction of the largest library of the ancient world at
Alexandria,
(B) the destroying of the largest library of the ancient world at Alexandria resulted and
(C) because of the result of the destruction of the library at Alexandria, the largest of the ancient world,
(D) as a result of the destruction of the library at Alexandria, the largest of the ancient world,
(E) Alexandria’s largest library of the ancient world was destroyed, and the result was
365.In addition to having more protein than wheat does, the protein in rice is higher quality than that in wheat, with more of the amino acids essential to the human diet.
(A) the protein in rice is higher quality than that in
(B) rice has protein of higher quality than that in
(C) the protein in rice is higher in quality than it is in
(D) rice protein is higher in quality than it is in
(E) rice has a protein higher in quality than
366.In an effort to reduce their inventories, Italian vintners have cut prices; their wines have been priced to sell, and they are.
(A) have been priced to sell, and they are
(B) are priced to sell, and they have
(C) are priced to sell, and they do
(D) are being priced to sell, and have
(E) had been priced to sell, and they have
367.In ancient s, Nubia was the principal corridor where there were cultural influences transmitted between Black Africa and the Mediterranean basin.
(A) where there were cultural influences transmitted
(B) through which cultural influences were transmitted
(C) where there was a transmission of cultural influences
(D) for the transmitting of cultural influences
(E) which was transmitting cultural influences
368.In Aristophanes’ Lysistrata women are seen as the means of bringing peace and good sense to a war-torn world.
(A) as
(B) as if they are
(C) that they will be
(D) that they are
(E) for being
369.In assessing the problems faced by rural migrant workers, the question of whether they are better off materially than the urban working poor is irrelevant.
(A) In assessing the problems faced by rural migrant workers, the question of whether they are better off materially than the urban working poor is irrelevant.
(B) The question of whether the rural migrant worker is better off materially than the urban working poor is irrelevant in assessing the problems that they face.
(C) A question that is irrelevant in assessing the problems that rural migrant workers face is whether they are better off materially than the urban working poor.
(D) In an assessment of the problems faced by rural migrant workers, the question of whether they are better off materially than the urban working poor is irrelevant.
(E) The question of whether the rural migrant worker is better off materially than the urban working poor is irrelevant in an assessment of the problems that they face.
370.In astronomy the term “red shift” denotes the extent to which light from a distant galaxy has been shifted toward the red, or long-wave, end of the light spectrum by the rapid motion of the galaxy away from the Earth.
(A) to which light from a distant galaxy has been shifted
(B) to which light from a distant galaxy has shifted
(C) that light from a distant galaxy has been shifted
(D) of light from a distant galaxy shifting
(E) of the shift of light from a distant galaxy
371.In cold-water habitats, certain invertebrates and fish convert starches into complex carbohydrates called glycerols, in effect manufacturing its own antifreeze.
(A) in effect manufacturing its own antifreeze
(B) effectively manufacturing antifreeze of its own
(C) in effect manufacturing their own antifreeze
(D) so that they manufacture their own antifreeze
(E) thus the manufacture of its own antifreeze
372.In contrast to large steel plants that take iron ore through all the steps needed to produce several different kinds of steel, processing steel scrap into a specialized group of products has enabled small mills to put capital into new technology and remain economically viable.
(A) processing steel scrap into a specialized group of products has enabled small mills to put capital into new technology and remain
(B) processing steel scrap into a specialized group of products has enabled small mills to put capital into new technology, remaining
(C) the processing of steel scrap into a specialized group of products has enabled small mills to put capital into new technology, remaining
(D) small mills, by processing steel scrap into a specialized group of products, have been able to put capital into new technology and remain
(E) small mills, by processing steel scrap into a specialized group of products, have been able to put capital into new technology and remained
373.In contrast to true hibernators such as woodchucks and hedgehogs, whose body
temperatures drop close to the freezing point during the winter months, the body temperature of bears remains nearly normal throughout their prolonged sleep.
(A) the body temperature of bears remains nearly normal
(B) a nearly normal body temperature is maintained by bears
(C) a bear’s body temperature remains nearly normal
(D) a bear maintains a body temperature that is nearly normal
(E) bears maintain a nearly normal body temperature
374.In December of 1987 an automobile manufacturer pleaded no contest to criminal charges of odometer tampering and agreed to pay more than $16 million in civil damages for cars that were test-driven with their odometers disconnected.
(A) cars that were test-driven with their odometers disconnected
(B) cars that it had test-driven with their disconnected odometers
(C) its cars having been test-driven with disconnected odometers
(D) having test-driven cars with their odometers disconnected
(E) having cars that were test-driven with disconnected odometers
375.In developing new facilities for the incineration of solid wastes, we must avoid the danger of shifting environmental problems from landfills polluting the water to polluting the air with incinerators.
(A) landfills polluting the water to polluting the air with incinerators
(B) landfills polluting the water to the air being polluted with incinerators
(C) the pollution of water by landfills to the pollution of air by incinerators
(D) pollution of the water by landfills to incinerators that pollute the air
(E) water that is polluted by landfills to incinerators that pollute the air
376.In Egypt in the late Paleolithic period, the climate changed, pastures became deserts, and the inhabitants were forced to withdraw to the land bordering the Nile from their hunting grounds.
(A) inhabitants were forced to withdraw to the land bordering the Nile from their hunting grounds
(B) inhabitants had been forced to withdraw from their hunting grounds to the land that bordered the Nile
(C) inhabitants were forced to withdraw from their hunting grounds to the land bordering the Nile
(D) inhabitants having been forced to, withdrew from their hunting grounds to the land that bordered the Nile
(E) inhabitants withdrew, because they were forced to, from their hunting grounds to the land bordering the Nile
377.In England the well-dressed gentleman of the eighteenth century protected their clothing while having their wig powdered by poking their head through a device that resembled the stocks.
(A) gentleman of the eighteenth century protected their clothing while having their wig powdered by poking their head
(B) gentleman of the eighteenth century protected his clothing while having his wig powdered by poking his head
(C) gentleman of the eighteenth century protected their clothing while having their wigs powdered by poking their heads
(D) gentlemen of the eighteenth century protected his clothing while having his wig powdered by poking his head
(E) gentlemen of the eighteenth century protected their clothing while having his wig powdered by poking his head
378.In feudal Europe, urban areas developed from clusters of houses where peasants lived and commuted to farmlands in the countryside, unlike homesteading policies in the American West that required residency on the land itself in order to obtain eventual ownership.
(A) In feudal Europe, urban areas developed from clusters of houses where peasants lived and commuted to farmlands in the countryside, unlike homesteading policies in the American West that
(B) In feudal Europe, urban areas developed from clusters of houses where peasants lived and from which they commuted to farmlands in the countryside, but in the American West homesteading policies
(C) Unlike feudal Europe where urban areas developed from clusters of houses where peasants lived and commuted to farmlands in the countryside, the American West’s homesteading policies
(D) Unlike feudal Europe where urban areas developed from clusters of houses where peasants lived and commuted to farmlands in the countryside, the homesteading policies of the American West
(E) Urban areas developed from clusters of houses where peasants lived from which they commuted to farmlands in the countryside in feudal Europe, unlike the American West where homesteading policies
379.In good years, the patchwork of green fields that surround the San Joaquin Valley town bustles with farm workers, many of them in the area just for the season.
(A) surround the San Joaquin Valley town bustles with farm workers, many of them
(B) surrounds the San Joaquin Valley town bustles with farm workers, many of whom are
(C) surround the San Joaquin Valley town bustles with farm workers, many of who are
(D) surround the San Joaquin Valley town bustle with farm workers, many of which
(E) surrounds the San Joaquin Valley town bustles with farm workers, many are
380.In her recently published study, Rubin asserts that most women do not suffer from the “empty nest syndrome”; they are, in fact, relieved when their children depart.
(A) they are, in fact, relieved when their children depart
(B) and they are, in fact, relieved when their children departed
(C) they are, in fact, relieved when their children departed
(D) in fact, they are relieved when their children departed
(E) they are relieved at the departure of the children, in fact
381.In his eagerness to find a city worthy of Priam, the German archaeologist Schliemann cut through Troy and uncovered a civilization a thousand years older as was the city Homer’s heroes knew.
(A) older as was the city Homer’s heroes knew
(B) more ancient than the city known to Homer’s heroes
(C) older than was the city known to Homer’s heroes
(D) more ancient of a city than Homer’s heroes knew
(E) older of a city than was the one known to Homer’s heroes
382.In his research paper, Dr. Frosh, medical director of the Payne Whitney Clinic,
distinguishes mood swings, which may be violent without their being grounded in mental disease, from genuine manic-depressive psychosis.
(A) mood swings, which may be violent without their being grounded in mental disease, from genuine manic-depressive psychosis
(B) mood swings, perhaps violent without being grounded in mental disease, and genuine manic-depressive psychosis
(C) between mood swings, which may be violent without being grounded in mental disease, and genuine manic-depressive psychosis
(D) between mood swings, perhaps violent without being grounded in mental disease, from genuine manic-depressive psychosis
(E) genuine manic-depressive psychosis and mood swings, which may be violent without being grounded in mental disease
383.In his work, Leon Forrest is more reminiscent of Henry Miller’s obsessive narratives and Toni Morrison’s mythic languages than James Joyce’s internal explorations.
(A) In his work, Leon Forrest is more reminiscent of
(B) Leon Forrest writes more like
(C) Leon Forrest’s work is more reminiscent of
(D) Leon Forrest reminds one more of
(E) Leon Forrest’s work more resembles that of
384.In Holland, a larger percentage of the gross national product is spent on defense of their coasts from rising seas than is spent on military defense in the United States.
(A) In Holland, a larger percentage of the gross national product is spent on defense of their coasts from rising seas than is spent on military defense in the United States.
(B) In Holland they spend a larger percentage of their gross national product on defending their coasts from rising seas than the United States does on military defense.
(C) A larger percentage of Holland’s gross national product is spent on defending their coasts from rising seas than the United States spends on military defense.
(D) Holland spends a larger percentage of its gross national product defending its coasts from rising seas than the military defense spending of the United States.
(E) Holland spends a larger percentage of its gross national product on defending its coasts from rising seas than the United States does on military defense.
385.In its most recent approach, the comet Crommelin passed the Earth at about the same distance and in about the same position, some 25 degrees above the horizon, that Halley’s comet will pass the next it appears.
(A) that Halley’s comet will pass
(B) that Halley’s comet is to be passing
(C) as Halley’s comet
(D) as will Halley’s comet
(E) as Halley’s comet will do
386.In Japan elderly people are treated with far greater respect than most Western
countries.
(A) most Western countries
(B) most Western countries do
(C) most Western countries are
(D) they do in most Western countries
(E) they are in most Western countries
387.In Japan, a government advisory committee called for the breakup of Nippon Telephone and Telegraph Company, the largest telephone company in the world, so it would be two local phone companies and one long-distance provider.
(A) In Japan, a government advisory committee called for the breakup of Nippon Telephone and Telegraph Company, the largest telephone company in the world, so it would be
(B) The breakup of the world’s largest telephone company, Nippon Telephone and Telegraph Company, was called for by a government advisory committee in Japan, so it would be
(C) A government advisory committee in Japan called for the breakup of Nippon Telephone and Telegraph Company, the world’s largest telephone company, into
(D) The breakup of Nippon Telephone and Telegraph Company, the world’s largest telephone company, was called for by a government advisory committee in Japan, so it would be
(E) Called for by a government advisory committee, the breakup of Nippon Telephone and Telegraph Company in Japan, the world’s largest telephone company, was to be into
388.In June of 1987, The Bridge of Trinquetaille, Vincent van Gogh’s view of an iron bridge over the Rhone sold for $20.2 million and it was the second highest price ever paid for a painting at auction.
(A) Rhone sold for $20.2 million and it was
(B) Rhone, which sold for $20.2 million, was
(C) Rhone, was sold for $20.2 million,
(D) Rhone was sold for $20.2 million, being
(E) Rhone, sold for $20.2 million, and was
389.In June of 1989, Princeton Township approved a developer’s plans to build 300 houses on a large portion of the 210-acre site of the Battle of Princeton, one of only eight Revolutionary War battlefields that had remained undeveloped.
(A) one of only eight Revolutionary War battlefields that had remained undeveloped
(B) one of eight of the only Revolutionary War battlefields that have remained undeveloped
(C) one of the only eight undeveloped Revolutionary War battlefields that remains
(D) only one of eight Revolutionary War battlefields to remain undeveloped
(E) only one of the eight remaining undeveloped Revolutionary War battlefields
390.In June, 1981, six teenagers in the village of Medjugorje, Yugoslavia, claimed to have had visions of the Virgin Mary, who they say has continued to appear to them over the ensuing years.
(A) claimed to have had visions of the Virgin Mary, who
(B) claimed to have visions of the Virgin Mary, whom
(C) claimed to have had visions of the Virgin Mary, whom
(D) claimed to have visions of the Virgin Mary, who
(E) had claimed to have had visions of the Virgin Mary, whom
391.In large doses, analgesics that work in the brain as antagonists to certain chemicals have caused psychological disturbances in patients, which may limit their potential to relieve severe pain.
(A) which may limit their potential to relieve
(B) which may limit their potential for relieving
(C) which may limit such analgesics’ potential to relieve
(D) an effect that may limit their potential to relieve
(E) an effect that may limit the potential of such analgesics for relieving
392.In metalwork one advantage of adhesive-bonding over spot-welding is that the contact, and hence the bonding, is effected continuously over a broad surface instead of a series of regularly spaced points with no bonding in between.
(A) instead of
(B) as opposed to
(C) in contrast with
(D) rather than at
(E) as against being at
393.In one of the bloodiest battles of the Civil War, fought at Sharpsburg, Maryland, on September 17, 1862, four s as many Americans were killed as would later be killed on the beaches of Normandy during D-Day.
(A) Americans were killed as
(B) Americans were killed than
(C) Americans were killed than those who
(D) more Americans were killed as there
(E) more Americans were killed as those who
394.In one of the most stunning reversals in the history of marketing, the Coca-Cola company in July 1985 yielded to thousands of irate consumers demanding that it should bring back the original Coke formula.
(A) demanding that it should
(B) demanding it to
(C) and their demand to
(D) who demanded that it
(E) who demanded it to
395.In presenting his modus vivendi proposal, Lansing implied that the American government accepted the German view that armed merchant vessels were warships: however, when the proposal was dropped by the Wilson administration, it seemed to be reverting to the British view on this question.
(A) when the proposal was dropped by the Wilson administration, it
(B) after it was dropped, the Wilson administration
(C) by dropping the proposal, the Wilson administration
(D) the Wilson administration dropped the proposal when it
(E) when they dropped the proposal, the Wilson administration
396.In recent years cattle breeders have increasingly used crossbreeding, in part that their steers should acquire certain characteristics and partly because crossbreeding is said to provide hybrid vigor.
(A) in part that their steers should acquire certain characteristics
(B) in part for the acquisition of certain characteristics in their steers
(C) partly because of their steers acquiring certain characteristics
(D) partly because certain characteristics should be acquired by their steers
(E) partly to acquire certain characteristics in their steers
397.In reference to the current hostility toward smoking, smokers frequently expressed anxiety that their prospects for being hired and promoted are being stunted by their habit.
(A) In reference to the current hostility toward smoking, smokers frequently expressed anxiety that
(B) Referring to the current hostility toward smoking, smokers frequently expressed anxiety about
(C) When referring to the current hostility toward smoking, smokers frequently express anxiety about
(D) With reference to the current hostility toward smoking, smokers frequently expressed anxiety about
(E) Referring to the current hostility toward smoking, smokers frequently express anxiety that
398.In spite of continuing national trends toward increased consumption of specialty foods, agronomists in the Midwest foresee a gradual reversion to the raising of agricultural staples: feed corn and hard red wheat.
(A) a gradual reversion to the raising of agricultural staples
(B) that a gradual reversion back will feature the raising of agricultural staples
(C) a gradual reversion back to the raising of agricultural staples again
(D) a gradual reversion to raise agricultural staples
(E) a gradual reversion into the raising of agricultural staples
399.In spite of federal subsidizing of public transportation systems massively and major local efforts to persuade the public to use public transportation, mass transit has been steadily losing patronage to the private automobile for the past thirty years.
(A) In spite of federal subsidizing of public transportation systems massively and major local efforts to persuade
(B) In spite of massive federal subsidizing of public transportation systems and major efforts locally at persuading
(C) Despite massive federal subsidies to public transportation systems and the making of major efforts locally to persuade
(D) Despite massive federal subsidies to public transportation systems and major local efforts to persuade
(E) Despite massive federal subsidies to public transportation systems and making major local efforts at persuading
400.In such works as his 1889 masterpiece, A Dash for the Timber, Frederic Remington caught the public’s fancy by portraying those moments of conflict that defined the West’s romantic heroes.
(A) In such works as his 1889 masterpiece, A Dash for the Timber, Frederic Remington caught the public’s fancy
(B) In such works as Frederic Remington’s 1889 masterpiece, A Dash for the Timber, the public’s fancy was caught
(C) Frederic Remington, catching the public’s fancy in such works as the 1889 masterpiece A Dash for the Timber, did it
(D) The fancy of Frederic Remington’s public was caught in his 1889 masterpiece, A Dash for the Timber,
(E) The public’s fancy was caught by Frederic Remington in such works as his 1889
masterpiece, A Dash for the Timber,
401.In terms of physics, the characteristic feature of the roller coaster is that the cars’ potential energy, gained through their being lifted by a chain drive through the Earth’s gravity to the top of the first drop, has been converted to kinetic energy by the ride ends.
(A) cars’ potential energy, gained through their being lifted by a chain drive
(B) cars’ potential energy, a gain achieved as they are lifted by a chain drive
(C) potential energy from the cars’ being lifted by a chain drive
(D) potential energy of the cars, gained as a chain drive lifts them
(E) potential energy gained by the cars, being achieved while a chain drive lifts them
402.In the 1950’s astronomers were divided between those who believed the universe began in a cosmic explosion (the “big bang”) with those who favored the model of an eternal and infinite steady-state universe.
(A) universe began in a cosmic explosion (the “big bang”) with
(B) universe began with a cosmic explosion (the “big bang”) and
(C) universe had a beginning a cosmic explosion (the “big bang”) or
(D) universe’s beginning was a cosmic explosion (the “big bang”) or
(E) universe’s beginning was a cosmic explosion (the “big bang”) against
403.In the 1950s, when the Chrysler Corporation sponsored a live television show about the assassination of Abraham Lincoln, it forbade the actors to mention Lincoln’s name or the name of the Ford Theater because it did not want to plug the competition.
(A) it forbade the actors to mention Lincoln’s name or the name of the Ford Theater because it
(B) they forbade the actors from mentioning Lincoln or the Ford Theater because they
(C) it was forbidden to mention Lincoln or the Ford Theater as they
(D) the actors were forbidden to mention Lincoln’s name or the name of the Ford Theater because they
(E) the actors were forbidden from mentioning Lincoln or the Ford Theater, since they
404.In the 1980’s the rate of increase of the minority population of the United States was nearly twice as fast as the 1970’s.
(A) twice as fast as
(B) twice as fast as it was in
(C) twice what it was in
(D) two s faster than that of
(E) two s greater than
405.In the face of widespread concern about environmental waste, compact disc manufacturers are attempting to find a replacement for the disposable plastic box in which they package their product.
(A) the disposable plastic box in which they
(B) the disposable plastic box where they
(C) a disposable plastic box in which to
(D) disposable plastic boxes inside which they
(E) the disposable plastic boxes in which to
406.In the fall of 1985, only 10 percent of the women entering college planned to major in education, while 28 percent chose business, making it the most popular major for women as well as for men.
(A) as well as for men
(B) as well as the men
(C) and men too
(D) and men as well
(E) and also men
407.In the initial planning stages, the condominium corporation took into account only the concerns of its prospective clients, not those of surrounding homeowners.
(A) the condominium corporation took into account only the concerns of its prospective clients
(B) the condominium corporation has only taken into account the concerns of their
prospective clients
(C) the condominium corporation only took their prospective clients’ concerns into account
(D) the concerns of its prospective clients only were taken into account by the condominium corporation
(E) prospective clients had their concerns only taken into account by the condominium corporation
408.In the last few years, the number of convicted criminals given community service
sentences, which allow the criminals to remain unconfined while they perform specific jobs benefiting the public, have risen dramatically.
(A) sentences, which allow the criminals to remain unconfined while they perform specific jobs benefiting the public, have
(B) sentences, performing specific jobs that benefit the public while being allowed to remain unconfined, have
(C) sentences, performing specific jobs beneficial to the public while they are allowed to remain unconfined, have
(D) sentences which allow them to remain unconfined in their performing of specific jobs beneficial to the public has
(E) sentences allowing them to remain unconfined while performing specific jobs that benefit the public has
409.In the last ten years, the dropout rate among Black high school students fell
substantially over the past decade, while the number of Blacks who attend college is more than twice what it was.
(A) fell substantially over the past decade, while the number of Blacks who attend college is more than twice what it was
(B) fell substantially, while the number of Blacks attending college is more than double what it was at that
(C) has fallen substantially, while the number of Blacks attending college has more than doubled
(D) has fallen substantially over the past decade, while the number of Blacks attending college is more than twice what it was at that
(E) has fallen substantially over the past decade, while the number of Blacks who are attending college are more than double what they were
410.In the last twenty years, despite the chauvinism of European connoisseurs, California wines are respected throughout the world.
(A) are respected
(B) are becoming better respected
(C) which have gained respect
(D) have gained respect
(E) have since become respected
411.In the late seventh century, in a dispute over whether the Prophet Muhammad’s
son-in-law, Ali, should carry on as the fourth caliph, Muhammad’s successor, Islam split into two branches, the Sunnis and the Shiites.
(A) over whether the Prophet Muhammad’s son-in-law, Ali, should carry on as the fourth caliph, Muhammad’s successor
(B) over if Ali, the Prophet Muhammad’s son-in-law, was going to carry on and be the fourth caliph, Muhammad’s successor
(C) over whether Ali, the Prophet Muhammad’s son-in-law, was going to carry on and be the fourth caliph, Muhammad’s successor
(D) as to whether the fourth caliph, Muhammad’s successor, is to be the Prophet Muhammad’s son-in-low, Ali
(E) concerning if the fourth caliph, Muhammad’s successor, was to be the Prophet Muhammad’s son-in-law, Ali
412.In the main, incidents of breakdowns in nuclear reactors have not resulted from lapses of high technology but commonplace inadequacies in plumbing and wiring.
(A) not resulted from lapses of high technology but
(B) resulted not from lapses of high technology but from
(C) resulted from lapses not of high technology but
(D) resulted from lapses not of high technology but have stemmed from
(E) resulted not from lapses of high technology but have stemmed from
413.In the mid-1920’s the Hawthorne Works of the Western Electric Company was the scene of an intensive series of experiments that would investigate changes in working conditions as to their effects on workers’ performance.
(A) that would investigate changes in working conditions as to their effects on workers’ performance
(B) investigating the effects that changes in working conditions would have on workers’ performance
(C) for investigating what are the effects in workers’ performance that changes in working conditions would cause
(D) that investigated changes in working conditions’ effects on workers’ performance
(E) to investigate what the effects changes in working conditions would have on workers’ performance
414.In the mid-1960’s a newly installed radar warning system mistook the rising of the moon as a massive missile attack by the Soviets.
(A) rising of the moon as a massive missile attack by the Soviets
(B) rising of the moon for a massive Soviet missile attack
(C) moon rising to a massive missile attack by the Soviets
(D) moon as it was rising for a massive Soviet missile attack
(E) rise of the moon as a massive Soviet missile attack
415.In the minds of many people living in England, before Australia was Australia, it was the antipodes, the opposite pole to civilization, an obscure and unimaginable place that was considered the end of the world.
(A) before Australia was Australia, it was the antipodes
(B) before there was Australia, it was the antipodes
(C) it was the antipodes that was Australia
(D) Australia was what was the antipodes
(E) Australia was what had been known as the antipodes
416.In the most common procedure for harvesting forage crops such as alfalfa, as much as 20 percent of the leaf and small-stem material, which is the most nutritious of all the parts of the plant, shattered and fell to the ground.
(A) which is the most nutritious of all the parts of the plant, shattered and fell
(B) the most nutritious of all parts of the plant, shatter and fall
(C) the parts of the plant which were most nutritious, will shatter and fall
(D) the most nutritious parts of the plant, shatters and falls
(E) parts of the plant which are the most nutritious, have shattered and fallen
417.In the Soviet Union the attorney’s role is often played by the judge, who not only reserves to hear citizens’ legal complaints and also prepares their cases should the claims be valid.
(A) and also prepares their cases should the claims be
(B) but also does the preparation of their cases if the claims should be
(C) and their cases are prepared if the claims are
(D) but also prepares their cases if the claims are
(E) and prepares their cases if the claims are
418.In the textbook publishing business, the second quarter is historically weak, because revenues are low and marketing expenses are high as companies prepare for the coming school year.
(A) low and marketing expenses are high as companies prepare
(B) low and their marketing expenses are high as they prepare
(C) low with higher marketing expenses in preparation
(D) low, while marketing expenses are higher to prepare
(E) low, while their marketing expenses are higher in preparation
419.In the traditional Japanese household, most clothing could be packed flatly, and so it was not necessary to have elaborated closet facilities.
(A) flatly, and so it was not necessary to have elaborate closet facilities
(B) flat, and so elaborate closet facilities were unnecessary
(C) flatly, and so there was no necessity for elaborate closet facilities
(D) flat, there being no necessity for elaborate closet facilities
(E) flatly, as no elaborate closet facilities were necessary
420.In the United States, trade unions encountered far more intense opposition against their struggle for social legitimacy than the organized labor movements of most other democratic nations.
(A) against their struggle for social legitimacy than
(B) in their struggle for social legitimacy than did
(C) against their struggle for social legitimacy as
(D) in their struggle for social legitimacy as did
(E) when they struggled for social legitimacy than has
421.In theory, international civil servants at the United Nations are prohibited from continuing to draw salaries from their own governments; in practice, however, some governments merely substitute living allowances for their employees’ paychecks, assigned by them to the United Nations.
(A) for their employees’ paychecks, assigned by them
(B) for the paychecks of their employees who have been assigned
(C) for the paychecks of their employees, having been assigned
(D) in place of their employees’ paychecks, for those of them assigned
(E) in place of the paychecks of their employees to have been assigned by them
422.In three centuries—from 1050 to 1350—several million tons of stone were quarried in France for the building of eighty cathedrals, five hundred large churches, and some tens of thousands of parish churches.
(A) for the building of eighty cathedrals, five hundred large churches, and some
(B) in order that they might build eighty cathedrals, five hundred large churches, and some
(C) so as they might build eighty cathedrals, five hundred large churches, and some
(D) so that there could be built eighty cathedrals, five hundred large churches, and
(E) such that they could build eighty cathedrals, five hundred large churches, and
423.In virtually all types of tissue in every animal species, dioxin induces the production of enzymes that are the organism’s trying to metabolize, or render harmless, the chemical that is irritating it.
(A) trying to metabolize, or render harmless, the chemical that is irritating it
(B) trying that it metabolize, or render harmless, the chemical irritant
(C) attempt to try to metabolize, or render harmless, such a chemical irritant
(D) attempt to try and metabolize, or render harmless, the chemical irritating it
(E) attempt to metabolize, or render harmless, the chemical irritant
424.Income in a single year is a very poor guide to income and wealth over even a few years, much less a life; in the longer run, a tax on what people spend is therefore not much different than a tax on their income.
(A) than a tax on their income
(B) from a tax on what they earn
(C) than taxing income
(D) from the income tax
(E) than a tax on what people earn
425.Increases in the cost of energy, turmoil in the international money markets, and the steady erosion of the dollar have altered the investment strategies of United States corporations more radically than those of foreign corporations.
(A) altered the investment strategies of United States corporations more radically than those of
(B) altered the investment strategies of United States corporations more radically than
(C) altered the investment strategies of United States corporations more radically than they have
(D) radically altered the investment strategies of United States corporations more than
(E) radically altered the investment strategies of United States and
426.India is country with at least fifty major regional languages, of whom fourteen have official recognition.
(A) of whom fourteen have official recognition
(B) fourteen that have official recognition
(C) fourteen of which are officially recognized
(D) fourteen that are officially recognized
(E) among whom fourteen have official recognition
427.Industrial pollution has done seriously and possibly irreversible damage to the bronze horses on the facade of the cathedral of St. Mark La Venica.
(A) has done seriously and possibly irreversible damage to
(B) did damage that is seriously and possibly irreversible to
(C) damaged, serious and possibly irreversibly
(D) has done serious and possibly irreversible damage to
(E) did damage, serious and possibly irreversible
428.Inflation has made many Americans reevaluate their assumptions about the future; they still expect to live better than their parents have, but not so well as they once thought they could.
(A) they still expect to live better than their parents have
(B) they still expect to live better than their parents did
(C) they still expect to live better than their parents had
(D) still expecting to live better than their parents had
(E) still expecting to live better than did their parents
429.Inflation in medical costs slowed in 1986 for the fifth consecutive year but were still about 50 percent greater than the rate of price increases for other items included in the consumer price index.
(A) Inflation in medical costs slowed in 1986 for the fifth consecutive year but were
(B) Inflation in medical costs slowed for the fifth consecutive year in 1986 but was
(C) In 1986 inflation in medical costs were slowed for the fifth consecutive year but were
(D) 1986 was the fifth consecutive year in which inflation in medical costs slowed but was
(E) 1986 was the fifth consecutive year that inflation in medical costs were slowed, but they were
430.Initiated five centuries after Europeans arrived in the New World on Columbus Day 1992, Project SETI pledged a $100 million investment in the search for extraterrestrial intelligence.
(A) Initiated five centuries after Europeans arrived in the New World on Columbus Day 1992, Project SETI pledged a $100 million investment in the search for extraterrestrial intelligence.
(B) Initiated on Columbus Day 1992, five centuries after Europeans arrived in the New World, a $100 million investment in the search for extraterrestrial intelligence was pledged by Project SETI.
(C) Initiated on Columbus Day 1992, five centuries after Europeans arrived in the New World, Project SETI pledged a $100 million investment in the search for extraterrestrial intelligence.
(D) Pledging a $100 million investment in the search for extraterrestrial intelligence, the initiation of Project SETI five centuries after Europeans arrived in the New World on Columbus Day 1992.
(E) Pledging a $100 million investment in the search for extraterrestrial intelligence five centuries after Europeans arrived in the New World, on Columbus Day 1992, the initiation of Project SETI took place.
431.Innovative companies are not only skillful in developing new products but also in finding new ways to promote old ones.
(A) are not only skillful in developing new products but also in finding
(B) are skillful not only in developing new products but also in finding
(C) not only are skillful in developing new products, they also find
(D) not only develop new products, but they are also skillful in finding
(E) are both skillful in the development of new products and in finding
432.Intar, the oldest Hispanic theater company in New York, has moved away from the Spanish classics and now it draws on the works both of contemporary Hispanic authors who live abroad and of those in the United States.
(A) now it draws on the works both of contemporary Hispanic authors who live abroad and of those
(B) now draws on the works of contemporary Hispanic authors, both those who live abroad and those who live
(C) it draws on the works of contemporary Hispanic authors now, both those living abroad and who live
(D) draws now on the works both of contemporary Hispanic authors living abroad and who are
(E) draws on the works now of both contemporary Hispanic authors living abroad and those
433.Intended primarily to stimulate family summer travel, the new airfare, which allows both an adult and a child to fly for the price of one ticket, and also shortens the advance-purchase requirement for family travel to a minimum of seven days rather than fourteen.
(A) and also shortens the advance-purchase requirement for family travel to a minimum of seven days rather than
(B) and also lessens the advance-purchase requirement for family travel to a seven-day minimum from
(C) also shortens the advance-purchase requirement for family travel to a minimum of seven days rather than that of
(D) also lessens the advance-purchase requirement for family travel to a seven-day minimum from
(E) also shortens the advance-purchase requirement for family travel to a minimum of seven days rather than
434.International sporting events need not be fiscal disasters, the financial success of the 1992 Olympic Games demonstrates that fact.
(A) the financial success of the 1992 Olympic Games demonstrates that fact
(B) for example, the 1992 Olympic Games were financially successful
(C) like the financial success of the 1992 Olympic Games demonstrates
(D) a fact demonstrated by the 1992 Olympic Games, which were financially successful
(E) as the financial success of the 1992 Olympic Games demonstrates
435.Inuits of the Bering Sea were in isolation from contact with Europeans longer than Aleuts or Inuits of the North Pacific and northern Alaska.
(A) in isolation from contact with Europeans longer than
(B) isolated from contact with Europeans longer than
(C) in isolation from contact with Europeans longer than were
(D) isolated from contact with Europeans longer than were
(E) in isolation and without contacts with Europeans longer than
436.IRS provision 254-B requires that an S corporation with assets of greater than $200,000 send W-2 forms to their full- and part- employees on or before Jan. 31.
(A) that an S corporation with assets of greater than $200,000 send W-2 forms to their full- and part- employees on or before Jan. 31
(B) an S corporation with assets of greater than $200,000 send W-2 forms to their full- and part- employees on or before Jan. 31
(C) that an S corporation with assets of greater than $200,000 send W-2 forms to its full- and part- employees on or before Jan. 31
(D) an S corporation with assets of greater than $200,000 send W-2 forms to their full- and part- employees on Jan. 31 or before
(E) an S corporation with assets of greater than $200,000 send W-2 forms to its full- and part- employees on or before Jan. 31
437.It appears illogical to some people that West Germany, which bans such seeming lesser evils as lawn-mowing on Sundays, still has some 4,000 miles of highway with no speed limit.
(A) which bans such seeming lesser evils as
(B) which bans such seemingly lesser evils as
(C) which is banning such seeming lesser evils like
(D) banning such evils that seem lesser, for example
(E) banning such seeming lesser evils like
438.It has been estimated that the annual cost to the United States of illiteracy in lost industrial output and tax revenues is at least $20 billion a year.
(A) the annual cost to the United States of illiteracy in lost industrial output and tax revenues is at least $20 billion a year
(B) the annual cost of illiteracy to the United States is at least $20 billion a year because of lost industrial output and tax revenues
(C) illiteracy costs the United States at least $20 billion a year in lost industrial output and tax revenues
(D) $20 billion a year in lost industrial output and tax revenues is the annual cost to the United States of illiteracy
(E) lost industrial output and tax revenues cost the United States at least $20 billion a year because of illiteracy
439.It is a special feature of cell aggregation in the developing nervous system that in most regions of the brain the cells not only adhere to one another and also adopt some preferential orientation.
(A) to one another and also adopt
(B) one to the other, and also they adopt
(C) one to the other, but also adopting
(D) to one another but also adopt
(E) to each other, also adopting
440.It is an oversimplified view of cattle raising to say that all one has to do with cattle is leave them alone while they feed themselves, corral them, and to drive them to market when the is ripe.
(A) all one has to do with cattle is leave them alone while they feed themselves, corral them, and to
(B) all one has to do with cattle is to leave them alone to feed themselves, to corral them, and
(C) all one has to do with cattle is leave them alone while they feed themselves and then corral them and
(D) the only thing that has to be done with cattle is leave them alone while they feed themselves, corral them, and
(E) the only thing that has to be done with cattle is to leave them alone while they feed themselves, to corral them, and
441.It is as difficult to prevent crimes against property as those that are against a person.
(A) those that are against a
(B) those against a
(C) it is against a
(D) preventing those against a
(E) it is to prevent those against a
442.It is characteristic of the Metropolitan Museum of Art, as of virtually every great American museum, the taste of local collectors has played at least as large a part in the formation of their collections as has the judgments of the art historian.
(A) of virtually every great American museum, the taste of local collectors has played at least as large a part in the formation of their collections as has
(B) of virtually every great American museum, that the taste of local collectors has played at least as large a part in the formation of their collections as has
(C) it is of virtually every great American museum, that the taste of local collectors has played at least as large a part in the formation of its collections as have
(D) it is of virtually every great American museum, that the taste of local collectors have played at least as large a part in the formation of its collections as have
(E) it is of virtually every great American museum, the taste of local collectors has played at least as large a part in the formation of its collections as has
443.It is not likely that the competitive imbalance that now exists between Japan with all major industrial nations will be redressed during the foreseeable future.
(A) with all major industrial nations will be redressed during
(B) with all other major industrial nations will be redressed within
(C) with all other major industrial nations will be redressed during
(D) and all major industrial nations will be redressed during
(E) and all other major industrial nations will be redressed within
444.It is possible that Native Americans originally have migrated to the Western Hemisphere over a bridge of land that once existed between Siberia and Alaska.
(A) have migrated to the Western Hemisphere over a bridge of land that once existed
(B) were migrating to the Western Hemisphere over a bridge of land that existed once
(C) migrated over a bridge of land to the Western Hemisphere that once existed
(D) migrated to the Western Hemisphere over a bridge of land that once existed
(E) were migrating to the Western Hemisphere over a bridge of land existing once
445.It is well known in the supermarket industry that how items are placed on shelves and the frequency of inventory turnovers can be crucial to profits.
(A) the frequency of inventory turnovers can be
(B) the frequency of inventory turnovers is often
(C) the frequency with which the inventory turns over is often
(D) how frequently is the inventory turned over are often
(E) how frequently the inventory turns over can be
446.It may be another fifteen years before spacecraft from Earth again venture to Mars, a planet now known to be cold, dry, and probably lifeless.
(A) again venture to Mars, a planet now known to be
(B) venture to Mars again, a planet now known for being
(C) will venture to Mars again, a planet now known as being
(D) venture again to Mars, a planet that is known now to be
(E) will again venture to Mars, a planet known now as being
447.It may be that by focusing primarily on a patient’s mental condition and on vague and often very speculative psychodynamic factors, predictions about the patient’s future behavior decrease in accuracy.
(A) predictions about the patient’s future behavior decrease in accuracy
(B) mental health professionals lessen the accuracy of their predictions about the patient’s future behavior
(C) the accuracy of predictions about the patient’s future behavior lessens
(D) a decrease in accuracy is seen in predictions about the patient’s future behavior can be predicted less accurately
(E) the patient’s future behavior can be predicted less accurately
448.It may someday be worthwhile to try to recover uranium from seawater, but at present this process is prohibitively expensive.
(A) It may someday be worthwhile to try to recover uranium from seawater
(B) Someday, it may be worthwhile to try and recover uranium from seawater
(C) Trying to recover uranium out of seawater may someday be worthwhile
(D) To try for the recovery of uranium out of seawater may someday be worthwhile
(E) Recovering uranium from seawater may be worthwhile to try to do someday
449.It seems likely that a number of astronomical phenomena, such as the formation of planetary nebulas, may be caused by the interaction where two stars orbit each other at close range.
(A) may be caused by the interaction where two stars orbit each other
(B) may be caused by the interaction between two stars that each orbit the other
(C) are because of the interaction between two stars that orbit each other
(D) are caused by the interaction of two stars where each is orbiting the other
(E) are caused by the interaction of two stars orbiting each other
450.It takes a deft balance between savings discipline, investment knowledge, risk taking, luck, and to raise a million dollars through investments.
(A) It takes a deft balance between
(B) Deft balancing is needed between
(C) Deftly balanced, it takes
(D) It takes a deft balance of
(E) A deft balance is what one needs among
451.It was an increase in reported cases of malaria along the Gulf Coast that in 1921 led the health authorities’ granting a permit for experimentation with human subjects to the group that later would be called by the name of Unimedco.
(A) authorities’ granting a permit for experimentation with human subjects to the group that later would be called by the name of
(B) authorities’ granting a permit for experimentation with human subjects to the group that later to be called by the name of
(C) authorities’ granting a permit for experimentation using human subjects to the group that later would be called by the name of
(D) authorities to grant a permit for human experimentation to the group later called
(E) authorities to grant a permit for human experimentation to the group that later would be called by the name of
452.It was because she was plagued by injuries that tennis celebrity Tracy Austin, who won the Italian and U.S. Opens at age 16 in 1979, sat out the years between 1984 to 1988, occasionally working as a television commentator.
(A) It was because she was plagued by injuries that tennis celebrity Tracy Austin, who won the Italian and U.S. Opens at age 16 in 1979, sat out the years between
(B) After winning the Italian and U.S. Opens in 1979 at the age of 16, injuries plagued tennis celebrity Tracy Austin and forced her to sit out from
(C) Tracy Austin, the tennis celebrity who, plagued by injuries, won the Italian and U.S. Opens at age 16 in 1979, sat out from
(D) Plagued by injuries, tennis celebrity Tracy Austin, who won the Italian and U.S. Opens at age 16 in 1979, sat out the years from
(E) Tennis celebrity Tracy Austin, who won the Italian and U.S. Opens at age 16 in 1979, was plagued by injuries when she sat out between
453.It was the loss of revenue from declines in tourism that in 1935 led the Saudi
authorities’ granting a concession for oil exploration to the company that would later be known by the name of Aramco.
(A) authorities’ granting a concession for oil exploration to the company that would later be known by the name of
(B) authorities’ granting a concession for oil exploration to the company later to be known as named
(C) authorities granting a concession for oil exploration to the company that would later be known by the name of
(D) authorities to grant a concession for oil exploration to the company that later will be known as being
(E) authorities to grant a concession for oil exploration to the company later to be known as
454.Japan received huge sums of capital from the United States after the Second World War, using it to help build a modern industrial system.
(A) Japan received huge sums of capital from the United States after the Second World War, using it to help build
(B) Japan received huge sums of capital from the United States after the Second World War and used it to help in building
(C) Japan used the huge sums of capital it received from the United States after the Second World War to help build
(D) Japan’s huge sums of capital received from the United States after the Second World War were used to help it in building
(E) Receiving huge sums of capital from the United States after the Second World War, Japan used it to help build
455.Joachim Raff and Giacomo Meyerbeer are examples of the kind of composer who receives popular acclaim while living, often goes into decline after death, and never regains popularity again.
(A) often goes into decline after death, and never regains popularity again
(B) whose reputation declines after death and never regains its status again
(C) but whose reputation declines after death and never regains its former status
(D) who declines in reputation after death and who never regained popularity again
(E) then has declined in reputation after death and never regained popularity
456.Joan of Arc, a young Frenchwoman who claimed to be divinely inspired, turned the tide of English victories in her country by liberating the city of Orleans and she persuaded Charles VII of France to claim his throne.
(A) she persuaded Charles VII of France to claim his throne
(B) persuaded Charles VII of France in claiming his throne
(C) persuading that the throne be claimed by Charles VII of France
(D) persuaded Charles VII of France to claim his throne
(E) persuading that Charles VII of France should claim the throne
457.John Smith provides information on the conditions that lead women to a gynecologist, and he notes that these conditions can, and sometimes are, used in the promotion of surgery which is not needed.
(A) and he notes that these conditions can, and sometimes are, used in the promotion of surgery which is not needed
(B) noting that these conditions can be, and sometimes are, used to promote needless surgery
(C) and notes that they could and sometimes are used unnecessarily in promoting surgery
(D) sometimes promoting needless surgery
(E) which they use sometimes to promote unnecessary surgery
458.Joplin’s faith in his opera “Tremonisha” was unshakable; in 1911 he published the score at his own expense and decided on staging it himself.
(A) on staging it himself
(B) that he himself would do the staging
(C) to do the staging of the work by himself
(D) that he himself would stage it
(E) to stage the work himself
459.Judge Bonham denied a motion to allow members of the jury to go home at the end of each day instead of to confine them to a hotel.
(A) to allow members of the jury to go home at the end of each day instead of to confine them to
(B) that would have allowed members of the jury to go home at the end of each day instead of confined to
(C) under which members of the jury are allowed to go home at the end of each day instead of confining them in
(D) that would allow members of the jury to go home at the end of each day rather than confinement in
(E) to allow members of the jury to go home at the end of each day rather than be confined to
460.It could be argued that the most significant virtue of a popular democracy is not the right to 10 participate in the selection of leaders, but rather that it affirms our importance in the scheme of things.
(A) but rather that it affirms
(B) but rather its affirmation of
(C) but rather it’s affirmation in terms of
(D) but instead of that, its affirming that
(E) affirming rather
461.Judge Forer recognizes that the American judicial system provides more safeguards for accused persons than does the legal system of any other country, but she believes there is a great disparity between the systems of justice accorded rich and poor.
(A) for accused persons than does the legal system of any other country
(B) to accused persons as the legal system of any country
(C) for accused persons as the legal system of any country
(D) to accused persons as the legal system of any other country
(E) for accused persons than the legal system of any country
462.Judge Lois Forer’s study asks why do some litigants have a preferred status over others in the use of a public resource, the courts, which in theory are available to all but in fact are unequally distributed among rich and poor.
(A) do some litigants have a preferred status over others in the use of a public resource, the courts, which in theory are available to all but in fact are unequally distributed among
(B) some litigants have a preferred status over others in the use of a public resource, the courts, which in theory are available to all but in fact are unequally distributed between
(C) do some litigants have a preferred status over another in the use of a public resource, the courts, in theory available to all but in fact are unequally distributed among
(D) some litigants have a preferred status to another in the use of a public resource, the courts, in theory available to all but in fact not equally distributed between
(E) does one litigant have a preferred status over the other in the use of a public resource, the courts, in theory available to all but in fact they are not equally
distributed among
463.Judicial rules in many states require that the identities of all prosecution witnesses are made known to defendants so they can attempt to rebut the testimony, but the Constitution explicitly requires only that the defendant have the opportunity to confront an accuser in court.
(A) that the identities of all prosecution witnesses are made known to defendants so they can attempt to rebut
(B) that the identities of all prosecution witnesses be made known to defendants so that they can attempt to rebut
(C) that the defendants should know the identities of all prosecution witnesses so they can attempt a rebuttal of
(D) the identities of all prosecution witnesses should be made known to defendants so they can attempt rebutting
(E) making known to defendants the identities of all prosecution witnesses so that they can attempt to rebut
464.Just as a writer trying to understand shell life might read Shalom Aleichem or Isaac Bashevis Singer, in the same way writers trying to understand Black life in the American South might well listen to records by the Mississippi Delta bluesman Charlie Patton.
(A) in the same way writers trying to understand Black life in the American South might well listen to records
(B) in the same way writers who try and understand Black life in the American South might well listen to record
(C) so a writer trying to understand black life in the American South might well listen to records
(D) so do writers try and understand Black life in the American South and might well listen to a record
(E) then writers trying to understand Black life in the American South could well listen to records
465.Just as plant species native to regions with browsing mammals evolved many natural anti-browser defenses (such as sharp spines and toxic chemicals), so humans in malarial regions have evolved dozens of chemical defenses against malaria.
(A) so humans in malarial regions have evolved dozens of chemical defenses against malaria
(B) humans in malarial regions have been evolving dozens of chemical defenses against malaria
(C) there has been, in malarial regions, an evolution of dozens of human chemical defenses against malaria
(D) dozens of chemical defenses against malaria have been evolved by humans in malarial regions
(E) similarly, in malarial regions, humans have evolved dozens of chemical defenses against malaria
466.Just as reading Samuel Pepys’s diary gives a student a sense of the seventeenth
century—of its texture and psyche—so Jane Freed’s guileless child narrator takes the
operagoer inside turn-of-the-century Vienna.
(A) so Jane Freed’s guileless child narrator takes the operagoer
(B) so listening to Jane Freed’s guileless child narrator takes the operagoer
(C) so the guileless child narrator of Jane Freed takes the operagoer
(D) listening to Jane Freed’s guileless child narrator takes the operagoer
(E) Jane Freed’s guileless child narrator takes the operagoer to her opera
467.Just as the European countries of the early eighteenth century sought to exploit the resources of our continent, so too are we now attempting to extract energy and minerals from the ocean bed.
(A) Just as the European countries of the early eighteenth century sought to exploit the resources of our continent, so too
(B) The European countries of the early eighteenth century sought to exploit the resources of our continent, and in a similar way
(C) Like the case of the European countries of the early eighteenth century who sought to exploit the resources of our continent, so too
(D) As in the exploitation of the resources of our continent by European countries of the early eighteenth century
(E) Similar to the European countries which sought in the early eighteenth century to exploit the resources of our continent
468.Just because King Alfred occupied and fortified London in 886 did not mean that he also won the loyalty of its citizens: the invading Danes were well aware of this weakness and used it to their advantage in 893.
(A) Just because King Alfred occupied and fortified London in 886 did not mean that he
(B) The fact that King Alfred had occupied and fortified London in 886 did not mean that he had
(C) Just because King Alfred occupied and fortified London in 886, it did not mean he
(D) The fact that King Alfred occupied and fortified London in 886, it did not mean that he
(E) Just because King Alfred had occupied and fortified London in 886, it did not mean he
469.Kansas Republican Nancy Kassebaum, one of only two women in the U.S. Senate in 1992, said she did not so much wish for more women senators but more moderate Republican ones.
(A) did not so much wish for more women senators but more moderate Republican ones
(B) wished not so much for more senators who were women than moderate Republicans
(C) did not wish so much for more women senators as for more moderate Republicans
(D) did not wish for more women senators so much as moderate Republicans
(E) wished for more senators who are moderate Republicans than women
470.Lacking information about energy use, people tend to overestimate the amount of energy used by equipment, such as lights, that are visible and must be turned on and off and underestimate that used by unobtrusive equipment, such as water heaters.
(A) equipment, such as lights, that are visible and must be turned on and off and
underestimate that
(B) equipment, such as lights, that are visible and must be turned on and off and
underestimate it when
(C) equipment, such as lights, that is visible and must be turned on and off and
underestimate it when
(D) visible equipment, such as lights, that must be turned on and off and underestimate that
(E) visible equipment, such as lights, that must be turned on and off and underestimate it when
471.Last spring a Colorado health department survey of 72 playgrounds in private child-care centers found unsafe conditions in 95 percent of them and they ranged from splinters to equipment near collapse.
(A) unsafe conditions in 95 percent of them and they ranged
(B) conditions in 95 percent were unsafe and ranging
(C) the ranging of unsafe conditions in 95 percent of them to be
(D) that 95 percent had unsafe conditions ranging
(E) that 95 percent of them had conditions that were unsafe; the range was
472.Last year, land values in most parts of the pinelands rose almost so fast, and in some parts even faster than what they did outside the pinelands.
(A) so fast, and in some parts even faster than what they did
(B) so fast, and in some parts even faster than, those
(C) as fast, and in some parts even faster than, those
(D) as fast as, and in some parts even faster than, those
(E) as fast as, and in some parts even faster than what they did
473.Lawmakers are examining measures that would require banks to disclose all fees and account requirements in writing, provide free cashing of government checks, and to create basic savings accounts to carry minimal fees and require minimal initial deposits.
(A) provide free cashing of government checks, and to create basic savings accounts to carry
(B) provide free cashing of government checks, and creating basic savings accounts carrying
(C) to provide free cashing of government checks, and creating basic savings accounts that carry
(D) to provide free cashing of government checks, creating basic savings accounts to carry
(E) to provide free cashing of government checks, and to create basic savings accounts that carry
474.Legend has it that when the Mohawk Joseph Brant was presented to George III in 1776, he proudly refused to kiss the King’s hand, inasmuch as he regarded himself an ally, not a subject.
(A) as he regarded himself an ally, not
(B) that he regarded himself to be an ally rather than
(C) as he considered himself an ally, not
(D) that he considered himself to be ally instead of
(E) as he considered himself as an ally rather than
475.Legislation in the Canadian province of Ontario requires of both public and private employers that pay be the same for jobs historically held by women as for jobs requiring comparable skill that are usually held by men.
(A) that pay be the same for jobs historically held by women as for jobs requiring
comparable skill that are
(B) that pay for jobs historically held by women should be the same as for a job requiring comparable skills
(C) to pay the same in jobs historically held by women as in jobs of comparable skill that are
(D) to pay the same regardless of whether a job was historically held by women or is one demanding comparable skills
(E) to pay as much for jobs historically held by women as for a job demanding comparable skills
476.Less detrimental than the effects of bacterial transformation is the effects of
bacterial deterioration: spoilage of food, metals corroding, decay of wood, and other undesirable alterations of substances.
(A) is the effects of bacterial deterioration: spoilage of food, metals corroding
(B) are the effects of bacterial deterioration: spoilage of food, metals corroding
(C) is the effects of bacterial deterioration, which include spoilage of food, metals corroding
(D) are the effects of bacterial deterioration, which includes spoilage of food, corrosion of metals
(E) are the effects of bacterial deterioration: spoilage of food, corrosion of metals
477.Like Auden, the language of James Merrill is chatty, arch, and conversational—given to complex syntactic flights as well as to prosaic free-verse strolls.
(A) Like Auden, the language of James Merrill
(B) Like Auden, James Merrill’s language
(C) Like Auden’s, James Merrill’s language
(D) As with Auden, James Merrill’s language
(E) As is Auden’s the language of James Merrill
478.Like Byron at Missolonghi, Jack London was slowly killed by the mistakes of the medical men who treated him.
(A) Like Byron
(B) Like Byron’s death
(C) Just as Byron died
(D) Similar to Byron
(E) As did Byron
479.Like Edvard Grieg, whom the Scandinavians long refused to recognize, the Italians’ disregard for Verdi persisted for a decade after his critical acclaim in France and Austria.
(A) Like Edvard Grieg, whom the Scandinavians long refused to recognize,
(B) Like Edvard Grieg, who the Scandinavians long refused to recognize,
(C) Just as Edvard Grieg was long refused recognition by the Scandinavians,
(D) Just as the Scandinavians long refused to recognize Edvard Grieg, so
(E) Like the Scandinavians’ long refusal to recognize Edvard Grieg,
480.Like Haydn, Schubert wrote a great deal for the stage, but he is remembered principally for his chamber and concert-hall music.
(A) Like Haydn, Schubert
(B) Like Haydn, Schubert also
(C) As has Haydn, Schubert
(D) As did Haydn, Schubert also
(E) As Haydn did, Schubert also
481.Like John McPhee’s works, Ann Beattie painstakingly assembles in her works an
interesting and complete world out of hundreds of tiny details about a seemingly
uninteresting subject.
(A) Like John McPhee’s works, Ann Beattie painstakingly assembles in her works
(B) Like John McPhee, Ann Beattie’s works painstakingly assemble
(C) Like John McPhee, Ann Beattie painstakingly assembles in her works
(D) Just as John McPhee’s, so Ann Beattie’s works painstakingly assemble
(E) Just as John McPhee, Ann Beattie painstakingly assembles in her works
482.Like many others of his generation of Native American leaders, Joseph Brant lived in two worlds; born into an Iroquois community and instructed in traditional Iroquois ways, he also received an education from English-speaking teachers.
(A) Like many others of his generation of Native American leaders, Joseph Brant lived in two worlds;
(B) Like many others of his generation of Native American leaders, living in two worlds, Joseph Brant was
(C) Like many another of his generation of Native American leaders, Joseph Brant, living in two worlds, was
(D) As with many others of his generation of Native American leaders, living in two worlds, Joseph Brant was
(E) As with many another of his generation of Native American leaders, Joseph Brant lived in two worlds;
483.Like many self-taught artists, Perle Hessing did not begin to paint until she was well into middle age.
(A) Like
(B) As have
(C) Just as with
(D) Just like
(E) As did
484.Like other educators who prefer to substitute anthologies of short stories or
collections of popular essays to dull “basal readers,” Ms. Burton emphasizes how important it is to enjoy good literature.
(A) to dull “basal readers,” Ms. Burton emphasizes how important it is to enjoy
(B) for dull “basal readers,” Ms. Burton emphasizes the importance of enjoying
(C) to dull “basal readers,” Ms. Burton emphasizes that it is important to enjoy
(D) for dull “basal readers,” Ms. Burton’s emphasis is that it is important to enjoy
(E) to dull “basal readers,” Ms. Burton’s emphasis is on the importance of enjoying
485.Like Rousseau, Tolstoi rebelled against the unnatural complexity of human relations in modern society.
(A) Like Rousseau, Tolstoi rebelled
(B) Like Rousseau, Tolstoi’s rebellion was
(C) As Rousseau, Tolstoi rebelled
(D) As did Rousseau, Tolstoi’s rebellion was
(E) Tolstoi’s rebellion, as Rousseau’s, was
486.Like the 1890s Populists who exalted the rural myth, so urban leaders of the 1990s are trying to glorify the urban myth.
(A) Like the 1890s Populists who exalted the rural myth,
(B) Just as the Populists of the 1890s exalted the rural myth,
(C) The Populists of the 1890s having exalted the rural myth,
(D) Just like the rural myth was exalted by the Populists of the 1890s
(E) Populists of the 1890s were exalting the rural myth, and
487.Like the color-discriminating apparatus of the human eye, insects’ eyes depend on recording and comparing light intensities in three regions of the electromagnetic spectrum.
(A) insects’ eyes depend on
(B) an insect eye depends on
(C) that of insects depend on the
(D) that of an insect’s eye depends on
(E) that of an insect’s is dependent on the
488.Like the government that came before it, which set new records for growth, laissez-faire capitalism is the cornerstone of the new government.
(A) laissez-faire capitalism is the cornerstone of the new government
(B) the cornerstone of the new government is laissez-faire capitalism
(C) laissez-faire capitalism is the new government’s cornerstone
(D) the new government has made laissez-faire capitalism its cornerstone
(E) the new government has a laissez-faire cornerstone of capitalism
489.Like the one reputed to live in Loch Ness, also an inland lake connected to the ocean by a river, inhabitants of the area around Lake Champlain claim sightings of a long and narrow “sea monster.”
(A) Like the one reputed to live in Loch Ness, also an inland lake connected to the ocean by a river, inhabitants of the area around Lake Champlain claim sightings of a long and narrow “sea monster.”
(B) Inhabitants of the area around Lake Champlain claim sightings of a long and narrow “sea monster” similar to the one reputed to live in Loch Ness, which, like Lake Champlain, is an inland lake connected to the ocean by a river.
(C) Inhabitants of the area around Lake Champlain claim sightings of a long and narrow “sea monster” similar to Loch Ness’s, which, like Lake Champlain, is an inland lake connected to the ocean by a river.
(D) Like Loch Ness’ reputed monster, inhabitants of the area around Lake Champlain, also an inland lake connected to the ocean by a river, claim sightings of a long and narrow “sea monster.”
(E) Similar to that reputed to live in Loch Ness, inhabitants of the area around Lake Champlain, also an inland lake connected to the ocean by a river, claim sightings of a long and narrow “sea monster.”
490.Like their male counterparts, women scientists are above average in terms of
intelligence and creativity, but unlike men of science, their female counterparts have had to work against the grain of occupational stereotyping to enter a “man’s world.”
(A) their female counterparts have had to work
(B) their problem is working
(C) one thing they have had to do is work
(D) the handicap women of science have had is to work
(E) women of science have had to work
491.Local residents claim that San Antonio, Texas, has more good Mexican American
restaurants than any city does in the United States.
(A) any city does
(B) does any other city
(C) other cities do
(D) any city
(E) other cities
492.Los Angeles has a higher number of family dwellings per capita than any large city.
(A) a higher number of family dwellings per capita than any large city
(B) higher numbers of family dwellings per capita than any other large city
(C) a higher number of family dwellings per capita than does any other large city
(D) higher numbers of family dwellings per capita than do other large cities
(E) a high per capita number of family dwellings, more than does any other large city
493.Lower-level management function most effectively when they have thorough training and unimpeded communication with workers, supervisors, and one another.
(A) management function most effectively when they have
(B) managers function most effectively when they have
(C) management functions most effectively when they have
(D) managers function most effectively when they have received
(E) management functions most effectively when it has
494.Machines replacing human labor, there was wide anticipation that the workweek would continue to become shorter.
(A) Machines replacing human labor, there was wide anticipation that
(B) When machines replaced human labor, there was wide anticipation
(C) As machines replaced human labor, it was widely anticipated that
(D) Insofar as machines replaced human labor, it was widely anticipated
(E) Human labor being replaced by machines, there was wide anticipation that
495.Madagascar was one of the last habitable areas of the earth to undergo human settlement, which has made it an ideal site for researching rare flora and fauna.
(A) Madagascar was one of the last habitable areas of the earth to undergo human settlement, which has made it
(B) Madagascar was one of the last habitable areas of the earth to have undergone human settlement, and that has made it
(C) Madagascar underwent human settlement as one of the last habitable areas of the earth, which makes it
(D) Madagascar, one of the last habitable areas of the earth, underwent human settlement, making it
(E) Because Madagascar was one of the last habitable areas of the earth to undergo human settlement, it is
496.Manifestations of Islamic political militancy in the first period of religious reformism were the rise of the Wahhabis in Arabia, the Sanusi in Cyrenaica, the Fulani in Nigeria, the Mahdi in the Sudan, and the victory of the Usuli “mujtahids” in Shiite Iran and Iraq.
(A) Manifestations of Islamic political militancy in the first period of religious reformism were the rise of the Wahhabis in Arabia, the Sanusi in Cyrenaica, the Fulani in Nigeria, the Mahdi in the Sudan, and
(B) Manifestations of Islamic political militancy in the first period of religious reformism were shown in the rise of the Wahhabis in Arabia, the Sanusi in Cyrenaica, the Fulani in Nigeria, the Mahdi in the Sudan, and also
(C) In the first period of religious reformism, manifestations of Islamic political
militancy were the rise of the Wahhabis in Arabia, of the Sanusi in Cyrenaica, the Fulani in Nigeria, the Mahdi in the Sudan, and
(D) In the first period of religious reformism, manifestations of Islamic political
militancy were shown in the rise of the Wahhabis in Arabia, the Sanusi in Cyrenaica, the Fulani in Nigeria, the Mahdi in the Sudan, and
(E) In the first period of religious reformism, Islamic political militancy was manifested in the rise of the Wahhabis in Arabia, the Sanusi in Cyrenaica, the Fulani in Nigeria, and the Mahdi in the Sudan, and in
497.Many investors base their choice between bonds and stocks on comparing bond yields to the dividends available on common stocks.
(A) between bonds and stocks on comparing bond yields to
(B) among bonds and stocks on comparisons of bond yields to
(C) between bonds and stocks on comparisons of bond yields with
(D) among bonds and stocks on comparing bond yields and
(E) between bonds and stocks on comparing bond yields with
498.Many of them chiseled from solid rock centuries ago, the mountainous regions of northern Ethiopia are dotted with hundreds of monasteries.
(A) Many of them chiseled from solid rock centuries ago, the mountainous regions of northern Ethiopia are dotted with hundreds of monasteries.
(B) Chiseled from solid rock centuries ago, the mountainous regions of northern Ethiopia are dotted with many hundreds of monasteries.
(C) Hundreds of monasteries, many of them chiseled from solid rock centuries ago, are dotting the mountainous regions of northern Ethiopia.
(D) The mountainous regions of northern Ethiopia are dotted with hundreds of monasteries, many of which are chiseled from solid rock centuries ago.
(E) The mountainous regions of northern Ethiopia are dotted with hundreds of monasteries, many of them chiseled from solid rock centuries ago.
499.Many policy experts say that shifting a portion of health-benefit costs back to the workers helps to control the employer’s costs, but also helps to limit medical spending by making patients more careful consumers.
(A) helps to control the employer’s costs, but also helps
(B) helps the control of the employer’s costs, and also
(C) not only helps to control the employer’s costs, but also helps
(D) helps to control not only the employer’s costs, but
(E) not only helps to control the employer’s costs, and also helps
500.Many psychologists and sociologists now contend that the deliberate and even brutal aggression integral to some forms of competitive athletics increase the likelihood of imitative violence that erupts among crowds of spectators dominated by young adult males.
(A) increase the likelihood of imitative violence that erupts
(B) increase the likelihood that there will be an eruption of imitative violence
(C) increase the likelihood of imitative violence erupting
(D) increases the likelihood for imitative violence to erupt
(E) increases the likelihood that imitative violence will erupt
501.Many scholars regard the pre-Hispanic civilizations of Peru as the most impressive in South America.
(A) regard the pre-Hispanic civilizations of Peru as
(B) regard the pre-Hispanic civilizations of Peru to be
(C) regard the pre-Hispanic civilizations of Peru to have been
(D) consider that the pre-Hispanic civilizations of Peru are
(E) consider the pre-Hispanic civilizations of Peru as
502.Many writers of modern English have acquired careless habits that damage the clarity of their prose, but these habits can be broken if they are willing to take the necessary trouble.
(A) but these habits can be broken
(B) but these habits are breakable
(C) but they can break these habits
(D) which can be broken
(E) except that can be broken
503.Margaret Courtney-Clarke has traveled to remote dwellings in the Transvaal to photograph the art of Ndebele women, whose murals are brilliantly colored, their geometrical symmetries embellished with old and new iconography and in a style that varies from woman to woman and house to house.
(A) whose murals are brilliantly colored, their geometrical symmetries embellished with old and new iconography and in a style that varies from woman to woman and house to house
(B) whose murals are brilliantly colored, their geometrical symmetries are embellished with old and new iconography, and their style is varying among women and houses
(C) whose murals are brilliantly colored, their geometrical symmetries are embellished with old and new iconography, and they are in styles that vary from woman to woman and house to house
(D) with murals brilliantly colored, their geometrical symmetries embellished with old and new iconography, and their style varies among women and houses
(E) with murals that are brilliantly colored, their geometrical symmetries embellished with old and new iconography, and their styles vary among women and houses
504.Marketing researchers have found that, because many residents of the Southeast do not share the same ethnic heritage as Northeasterners, the two varieties of commercially prepared coleslaw most popular in New York City and Boston are virtually ignored by consumers in Richmond and Raleigh.
(A) because many residents of the Southeast do not share the same ethnic heritage as
(B) because many residents of the Southeast do not share the same ethnic heritage with
(C) because many residents of the Southeast do not have the same ethnic heritage as
(D) due to many residents of the Southeast not sharing the same ethnic heritage with
(E) whereas many residents of the Southeast do not share the same ethnic heritage as
505.Migraine, the most debilitating common form of headache, afflicts perhaps 18 million Americans, who collectively lose 64 million workdays a year, and they cost the nation $50 billion in medical expenses and lost work .
(A) year, and they cost the nation $50 billion in medical expenses and lost
(B) year and thus cost the nation $50 billion in medical expenses and lost
(C) year, so as to cost the nation $50 billion in medical expenses and lost
(D) year that costs the nation $50 billion in lost medical expenses and
(E) year, which thus cost the nation $50 billion in lost medical expenses and
506.Minnesota is the only one of the contiguous forty-eight states that still has a sizable wolf population, and where this predator remains the archenemy of cattle and sheep.
(A) that still has a sizable wolf population, and where
(B) that still has a sizable wolf population, where
(C) that still has a sizable population of wolves, and where
(D) where the population of wolves is still sizable;
(E) where there is still a sizable population of wolves and where
507.Modern critics are amused by early scholars’ categorizing Tacitus’s Germania as an ethnographic treatise.
(A) scholars’ categorizing Tacitus’s Germania as
(B) scholars’ categorizing Tacitus’s Germania as if
(C) scholars, categorizing of Tacitus’s Germania as
(D) scholars who categorize Tacitus’s Germania as
(E) scholars who categorize Tacitus’s Germania if
508.Monitoring heart patients’ exercise, as well as athletes exercising, is now done by small transmitters broadcasting physiological measurements to nearby recording machines.
(A) Monitoring heart patients’ exercise, as well as athletes exercising, is now done by small transmitters broadcasting physiological measurements to nearby recording machines.
(B) Monitoring the exercise of heart patients, as well as athletes exercising, is now done by small transmitters broadcasting physiological measurements to nearby recording machines.
(C) Small transmitters broadcasting physiological measurements to nearby recording machines are now used to monitor the exercise of both heart patients and athletes.
(D) Broadcasting physiological measurements to nearby recording machines, small transmitters are now used to monitor heart patients’ exercise, as well as athletes exercising.
(E) Both athletes exercising and heart patients’ exercise are now monitored by small
transmitters broadcasting physiological measurements to nearby recording machines.
509.More ancient Egyptian temples were constructed in the reign of Ramses II as in any other.
(A) as in any other
(B) as any other
(C) as in others
(D) than others
(E) than in any other
510.More and more in recent years, cities are stressing the arts as a means to greater economic development and investing millions of dollars in cultural activities, despite strained municipal budgets and fading federal support.
(A) to greater economic development and investing
(B) to greater development economically and investing
(C) of greater economic development and invest
(D) of greater development economically and invest
(E) for greater economic development and the investment of
511.More than ever, paper is expected to be recycled this year, due to new mandatory
recycling laws in municipalities across the nation.
(A) More than ever, paper is expected to be recycled this year
(B) It is expected that more paper than ever will be recycled this year than previously and that is
(C) The paper expected to be recycled this year is more than ever
(D) The amount of paper that will be recycled this year is expected to be greater than ever
(E) A great increase in the amount of paper that will be recycled this year is
512.More than five thousand years ago, Chinese scholars accurately described the flow of blood as a continuous circle controlled by the heart, but it went unnoticed in the West.
(A) but it went
(B) but it was
(C) although it was
(D) but the discovery went
(E) although the discovery was
513.More than thirty years ago Dr. Barbara Mc-Clintock, the Nobel Prize winner, reported that genes can “jump,” as pearls moving mysteriously from one necklace to another.
(A) as pearls moving mysteriously from one necklace to another
(B) like pearls moving mysteriously from one necklace to another
(C) as pearls do that move mysteriously from one necklace to others
(D) like pearls do that move mysteriously from one necklace to others
(E) as do pearls that move mysteriously from one necklace to some other one
514.Most Corporations pay at least twice as much to full- employees, if the value of
benefits, sick days, and paid vacation days are included in earnings, than to part-
employees, whose hourly wages are often higher than those of their full- colleagues.
(A) are included in earnings, than
(B) are included in earnings, as
(C) is included in earnings, than they pay
(D) is included in earnings, as is paid
(E) is included in earnings, as they pay
515.Most energy analysts now agree that the costs of building and maintaining nuclear reactors are too high for nuclear power to likely prove cheaper than coal or oil in the long run.
(A) too high for nuclear power to likely
(B) high enough for nuclear power to be unlikely to
(C) high enough that it is unlikely nuclear power will
(D) so high that nuclear power is unlikely to
(E) so high as to be unlikely that nuclear power will
516.Most large companies prefer customized computer software because it can be molded to fit the way a company does business, when off-the-shelf software often requires the company to alter its procedures to fit the software.
(A) when
(B) since
(C) whereas
(D) because
(E) insofar as
517.Most nations regard their airspace as extending upward as high as an aircraft can fly; no specific altitude, however, has been officially recognized as a boundary.
(A) as extending
(B) as the extent
(C) to be an extent
(D) to be an extension
(E) to extend
518.Most North Carolina ski resorts broadcast music onto the slopes; skiers can choose among hard rock, soft pop, and “beautiful music” slopes, there are no slopes without music.
(A) skiers can choose among hard rock, soft pop, and “beautiful music” slopes, there are
(B) because skiers can choose hard rock, soft pop, or “beautiful music,” there are
(C) however, skiers can choose among hard rock, soft pop, “beautiful music,” and
(D) although skiers can choose among hard rock, soft pop, and “beautiful music” slopes, there are
(E) skiers can choose among hard rock, soft pop, “beautiful music” slopes, but
519.Most primates are immune to feline rhinovirus, but a specific group, this being the golden macaques, are highly susceptible to the organism.
(A) a specific group, this being the golden macaques, are highly susceptible to
(B) a specific group, the golden macaques, is highly susceptible to
(C) one group, specifically golden macaques, is highly susceptible of
(D) a specific group, the golden macaques, which are highly susceptible to
(E) a specific group, the golden macaques, which are highly susceptible of
520.Most state constitutions now mandate that the state budget be balanced each year.
(A) mandate that the state budget be balanced
(B) mandate the state budget to be balanced
(C) mandate that the state budget will be balanced
(D) have a mandate for a balanced state budget
(E) have a mandate to balance the state budget
521.Most teen-agers who work for pay hold jobs that require few skills, little
responsibility, and also no hope for career advancement.
(A) little responsibility, and also
(B) little responsibility, and with
(C) little responsibility, and offer
(D) carry little responsibility, and
(E) carry little responsibility, and offer
522.Most victims of infectious mononucleosis recover after a few weeks of listlessness, but an unlucky few may suffer for years.
(A) but an unlucky few may suffer
(B) and an unlucky few have suffered
(C) that an unlucky few might suffer
(D) that a few being unlucky may suffer
(E) but a few who, being unlucky, suffered
523.Ms. Chambers is among the forecasters who predict that the rate of addition to arable lands will drop while those of loss rise.
(A) those of loss rise
(B) it rises for loss
(C) those of losses rise
(D) the rate of loss rises
(E) there are rises for the rate of loss
524.Ms. Wright tries to get inside Iran to understand how it works, the role it has played in the Middle East and its intricate relationship with the United States, which is complex and depends heavily on understanding of the Farsi language.
(A) which is complex and depends
(B) because it is complex and depends
(C) but they are complex and depend
(D) which are complex and depend
(E) a task that is complex and depends
525.Native American burial sites dating back 5,000 years indicate that the residents of Maine at that were part of a widespread culture of Algonquian-speaking people.
(A) were part of a widespread culture of Algonquian-speaking people
(B) had been part of a widespread culture of people who were Algonquian-speaking
(C) were people who were part of a widespread culture that was Algonquian-speaking
(D) had been people who were part of a widespread culture that was Algonquian-speaking
(E) were a people which had been part of a widespread, Algonquian-speaking culture
526.Neanderthals had a vocal tract that resembled those of the apes and so were probably without language, a shortcoming that may explain why they were supplanted by our own species.
(A) Neanderthals had a vocal tract that resembled those of the apes
(B) Neanderthals had a vocal tract resembling an ape’s
(C) The vocal tracts of Neanderthals resembled an ape’s
(D) The Neanderthal’s vocal tracts resembled the apes’
(E) The vocal tracts of the Neanderthals resembled those of the apes
527.Never before had taxpayers confronted so many changes at once as they had in the Tax Reform Act of 1986.
(A) so many changes at once as they had in
(B) at once as many changes as
(C) at once as many changes that there were with
(D) as many changes at once as they confronted in
(E) so many changes at once that confronted them in
528.Never before in the history of music have musical superstars been able to command so extraordinary fees of the kind they do today.
(A) so extraordinary fees of the kind they do today
(B) so extraordinary fees as they are today
(C) such extraordinary fees as they do today
(D) such extraordinary fees of the kind today’s have
(E) so extraordinary a fee of the kind they can today
529.New hardy varieties of rice show promise of producing high yields without the costly requirements of irrigation and application of commercial fertilizer by earlier high-yielding varieties.
(A) requirements of irrigation and application of commercial fertilizer by earlier
high-yielding varieties
(B) requirements by earlier high-yielding varieties of application of commercial fertilizer and irrigation
(C) requirements for application of commercial fertilizer and irrigation of earlier
high-yielding varieties
(D) application of commercial fertilizer and irrigation that was required by earlier
high-yielding varieties
(E) irrigation and application of commercial fertilizer that were required by earlier high-yielding varieties
530.New Jersey’s is one of the five highest number of reported cases of Lyme disease in the United States.
(A) New Jersey’s is one of the five highest number of reported
(B) New Jersey’s is one of the five highest numbers in reporting
(C) New Jersey has a report of one of the five highest numbers of
(D) New Jersey has one of the five highest numbers of reported
(E) New Jersey reports one of the five highest number of
531.New techniques in thermal-scanning photography, a process that records radiation from surface areas, makes it possible to study the effects of calefaction, or warming, of a river in greater detail than ever before.
(A) makes it possible to study the effects of calefaction, or warming, of a river in greater detail than ever before
(B) make it possible to study, in greater detail, the effects of calefaction, or warming, of a river than ever before
(C) have made it possible to study in greater detail than ever before the effects of
calefaction, or warming, of a river
(D) make possible the study of the effects of calefaction, or warming, of a river in greater detail than it ever was before
(E) has made it more possible than ever before to study in greater detail the effects of calefaction, or warming, of a river
532.New theories propose that catastrophic impacts of asteroids and comets may have caused reversals in the Earth’s magnetic field, the onset of ice ages, splitting apart continents 80 million years ago, and great volcanic eruptions.
(A) splitting apart continents
(B) the splitting apart of continents
(C) split apart continents
(D) continents split apart
(E) continents that were split apart
533.Newcomers to southern California are surprised to discover that, in spite of the
temperate climate, heating bills are as high, or higher than, those in most Midwestern or eastern cities.
(A) as high, or higher than, those in
(B) as high, or higher than,
(C) as high as, or higher than,
(D) at least as high as those in
(E) no lower than
534.No less an authority than Walter Cronkite has reported that half of all Americans never read a book.
(A) No less an authority than
(B) Nonetheless an authority
(C) Nevertheless authoritarian
(D) Not less an authority than
(E) An authority not less than
535.No state law forbids an employer from rejecting a job applicant or to dismiss an
employee based on the results of a lie detector test.
(A) an employer from rejecting a job applicant or to dismiss
(B) an employer to reject a job applicant or dismiss
(C) that employers reject a job applicant or dismiss
(D) the rejection by an employer of a job applicant or dismissal of
(E) rejection by employers of a job applicant or dismissal of
536.None of the attempts to specify the causes of crime explains why most of the people exposed to the alleged causes do not commit crimes and, conversely, why so many of those not so exposed have.
(A) have
(B) has
(C) shall
(D) do
(E) could
537.Not all employment selection mechanisms that have a “disparate effect,” that is, that screens out a percentage of minorities or women disproportionate to their presence in the relevant labor market, are unlawful.
(A) that is, that screens out a percentage of minorities or women disproportionate to
(B) which means, that screens out a percentage of minorities or women disproportionate with
(C) which means, that screen out a percentage of minorities or women disproportionate with
(D) that is, that screen out a percentage of minorities or women disproportionate to
(E) that is, that screens out a percentage of minorities or women disproportionate with
538.Not since Galileo suffered the “scurvy humor” of the Inquisition has a religious
organization so effectively curbed the ability of a major scientist that he could pursue a theory.
(A) has a religious organization so effectively curbed the ability of a major scientist that he could pursue
(B) did a religious organization so effectively curb the ability of a major scientist that he could pursue
(C) has a religious organization so effectively curbed the ability of a major scientist to pursue
(D) did a religious organization so effectively curb the ability of a major scientist to pursue
(E) has a religious organization so effectively curbed whether a major scientist had the ability that he could be pursuing
539.Not until the Enlightenment, some 200 years ago, had society seriously questioned the right of the state that it could execute its citizens.
(A) had society seriously questioned the right of the state that it could execute
(B) did society seriously questioned the right of the state that it could execute
(C) had society seriously questioned the right of the state for the executing of
(D) did society seriously question the right of the state to execute
(E) had society seriously questioned whether the state had a right that it could execute
540.Nowhere in Prakta is the influence of modern European architecture more apparent than their government buildings.
(A) more apparent than their
(B) so apparent as their
(C) more apparent than in its
(D) so apparent than in their
(E) as apparent as it is in its
541.Nuclear fusion is the force that powers the Sun, the stars, and hydrogen bombs, merging the nuclei of atoms and not splitting them apart, as in nuclear reactors.
(A) merging the nuclei of atoms and not splitting them apart, as in nuclear reactors
(B) merging the nuclei of atoms instead of splitting them apart, like nuclear reactors
(C) merging the nuclei of atoms rather than splitting them apart, as nuclear reactors do
(D) and merges the nuclei of atoms but does not split them apart, as is done in nuclear reactors
(E) and merges the nuclei of atoms, unlike atomic reactors that split them apart
542.Oberlin College in Ohio was a renegade institution at its 1833 founding for deciding to accept both men and women as students.
(A) at its 1833 founding for deciding to accept
(B) for the decision at its 1833 founding to accept
(C) when it was founded in 1833 for its decision to accept
(D) in deciding at its founding in 1833 to accept
(E) by deciding at its founding in 1833 on the acceptance of
543.Of all the possible disasters that threaten American agriculture, the possibility of an adverse change in climate is maybe the more difficult for analysis.
(A) is maybe the more difficult for analysis
(B) is probably the most difficult to analyze
(C) is maybe the most difficult for analysis
(D) is probably the more difficult to analyze
(E) is, it may be, the analysis that is most difficult
544.Of all the wild animals in their area, none was more useful to the Delaware tribes than the Virginia white tailed deer: it was a source of meat, and its hide was used for clothing, its antlers and bones for tools, and its sinews and gut for bindings and glue.
(A) deer: it was a source of meat, and its hide was used for clothing, its antlers and bones for tools, and its sinews and gut
(B) deer: it was a source of meat, and its hide used for clothing, with its antlers and bones for tools, and its sinews and gut used
(C) deer: which was a source of meat, with its hide used for clothing, antlers and bones for tools, as well as its sinews and gut used
(D) deer: which, as well as being a source of meat, its hide was used for clothing, its antlers and bones for tools, and its sinews and gut were
(E) deer: with, as well as being a source of meat, its hide used for clothing, its antlers and bones for tools, and its sinews and gut
545.Of the twin supernovas, the most recently discovered is the largest, but it is the small one that has irrevocably altered astronomists’ ideas about the origins of the universe.
(A) the most recently discovered is the largest, but it is the small one
(B) the one most recently discovered is the largest, but it is the small one
(C) the most recently discovered is larger, but it is the smaller
(D) the more recently discovered one is the large one but the small one
(E) the more recently discovered is the larger, but it is the smaller one
546.Often visible as smog, ozone is formed in the atmosphere from hydrocarbons and nitrogen oxides, two major pollutants emitted by automobiles, react with sunlight.
(A) ozone is formed in the atmosphere from
(B) ozone is formed in the atmosphere when
(C) ozone is formed in the atmosphere, and when
(D) ozone, formed in the atmosphere when
(E) ozone, formed in the atmosphere from
547.On stage, the force of Carrick’s personality and the vividness of his acting disguised the fact he was, which his surviving velvet suit shows, a short man.
(A) he was, which his surviving velvet suit shows,
(B) he was, and it is his surviving velvet suit that shows it,
(C) of him being, as his surviving velvet suit shows,
(D) that he was, as his surviving velvet suit shows,
(E) shown in his surviving velvet suit, that he was
548.On the Great Plains, nineteenth-century settlers used mud and grass to build their homes, doing it without timber and nails.
(A) settlers used mud and grass to build their homes, doing it without
(B) settlers used mud and grass to build their homes, did it without
(C) settlers used mud and grass to build their homes, making them while not having
(D) settlers used mud and grass to build their homes, making do without
(E) settlers’ homes were built of mud and grass, making do without
549.Once an apolitical confrontation of the world’s best athletes, the Summer Olympics have been increasingly politicized in recent years as the superpowers have denied many of the best athletes’ access to competition.
(A) been increasingly politicized
(B) been increasing politicization
(C) been of increasing politicization
(D) politicized, increasingly,
(E) increased politicization
550.Once common throughout the Western plains, black-footed ferrets are thought to have declined in number as a result of the poisoning of prairie dogs, their prey.
(A) black-footed ferrets are thought to have declined in number as
(B) it is thought that the decline in number of black-footed ferrets is
(C) the decline in the number of black-footed ferrets is thought of as
(D) that black-footed ferrets have declined in their numbers is thought to be
(E) the numbers of the black-footed ferret are thought to have declined as
551.Once positioned in space, the Hubble Space Telescope will capture light from sources twenty s fainter compared to those that can be detected by ground-based instruments.
(A) compared to those that can be detected
(B) compared to those they can detect
(C) than that can be detected
(D) than those that can be detected
(E) than those detecting
552.Once the economic and social usefulness of the motor car was demonstrated and with its superiority to the horse being proved, much of the early hostility to it in rural regions disappeared.
(A) and with its superiority to the horse being
(B) and its superiority over the horse had been
(C) and its superiority to the horse
(D) its superiority over the horse
(E) with its superiority to the horse having been
553.Once they had seen the report from the medical examiner, the investigators did not doubt whether the body recovered from the river was the man who had attempted to escape from the state prison.
(A) did not doubt whether the body recovered from the river was
(B) have no doubt whether the body recovered from the river was
(C) had not doubted that the body recovered from the river was
(D) have no doubt whether the body recovered from the river was that of
(E) had no doubt that the body recovered from the river was that of
554.One legacy of Madison Avenue’s recent campaign to appeal to people fifty years old and over is the realization that as a person ages, their concerns change as well.
(A) the realization that as a person ages, their
(B) the realization that as people age, their
(C) to realize that when a person ages, his or her
(D) to realize that when people age, their
(E) realizing that as people age, their
555.One noted economist has made a comparison of the Federal Reserve and an automobile as racing through a tunnel, bouncing first off one wall, then the other; the car may get where it is going, but people may be hurt in the process.
(A) made a comparison of the Federal Reserve and an automobile as racing through a tunnel, bouncing
(B) made a comparison between the Federal Reserve and an automobile racing through a tunnel, bouncing
(C) compared the Federal Reserve with an automobile as racing through a tunnel and which bounced
(D) compared the Federal Reserve to an automobile racing through a tunnel, bouncing
(E) compared the Federal Reserve with an automobile that races through a tunnel and it bounces
556.One of Arthur Jessop’s first acts as president of the FHA was to deny a request from the private sector that federal home loan programs should be expanded to cover houses not conforming to civil construction codes.
(A) should be expanded to cover houses not conforming to
(B) be expanded to cover houses not conforming to
(C) should be expand to cover houses not conforming with
(D) would have been expanded to cover houses not conforming to
(E) had to be expanded to cover houses not conforming with
557.One of every two new businesses fail within two years.
(A) fail
(B) fails
(C) should fail
(D) may have failed
(E) has failed
558.One of four babies are now born to mothers aged thirty years or more, compared with just one of six born in 1975.
(A) of four babies are now born to mothers aged thirty years or more, compared with just one of six born
(B) of four babies is now born to a mother whose age is thirty or older, compared to just one of six babies who were born
(C) baby in four are now born to mothers aged thirty or older, compared to just one in six
(D) baby in four is now born to a mother aged thirty or older, compared with just one in six
(E) baby in four is now born to mothers aged thirty years or more, compared to just one in six
559.One of Ronald Reagan’s first acts as President was to rescind President Carter’s
directive that any chemical banned on medical grounds in the United States be prohibited from sale to other countries.
(A) that any chemical banned on medical grounds in the United States be prohibited from sale to other countries
(B) that any chemical be prohibited from sale to other countries that was banned on medical grounds in the United States
(C) prohibiting the sale to other countries of any chemical banned on medical grounds in the United States
(D) prohibiting that any chemical banned on medical grounds in the United States is sold to other countries
(E) that any chemical banned in the United States on medical grounds is prohibited from being sold to other countries
560.One of the most powerful driving forces behind recycling is the threat of legislation that would require companies that they take more responsibility for the disposal of its products.
(A) that they take more responsibility for the disposal of its products
(B) that they should take more responsibility for disposing of products
(C) having to take more responsibility for disposing of their products
(D) to take more responsibility for the disposal of their products
(E) taking more responsibility for their product’s disposal
561.One pervasive theory explains the introduction of breakfast cereals in the early 1900s as a result of the growing number of automobiles, which led to a decline in horse ownership and a subsequent grain glut; by persuading people to eat what had previously been horse feed, market equilibrium was restored.
(A) by persuading people to eat what had previously been horse feed, market equilibrium was restored
(B) persuading people to eat what had previously been horse feed restored market equilibrium
(C) by persuading people to eat what had previously been horse feed, it restored market equilibrium
(D) the persuasion of people to eat what had previously been horse feed restored market equilibrium
(E) market equilibrium was restored when people were persuaded to eat former horse feed
562.One reason why more young people lose their virginity during the summer than at other s of the year undoubtedly is because school vacations give adolescents more free .
(A) One reason why more young people lose their virginity during the summer than at other s of the year undoubtedly is because
(B) If young people lose their virginity more during the summer than other seasons, it is undoubtedly because
(C) One undoubtable reason that young people lose their virginity more during the summer than other s is
(D) One reason more young people lose their virginity during the summer than at other s of the year is undoubtedly that
(E) Young people lose their virginity more often during the summer undoubtedly because, for one reason,
563.One view of the economy contends that a large drop in oil prices should eventually lead to lowering interest rates, as well as lowering fears about inflation, a rally in stocks and bonds, and a weakening of the dollar.
(A) lowering interest rates, as well as lowering fears about inflation,
(B) a lowering of interest rates and of fears about inflation,
(C) a lowering of interest rates, along with fears about inflation,
(D) interest rates being lowered, along with fears about inflation,
(E) interest rates and fears about inflation being lowered, with
564.Opened in 1683, the Ashmolean was the first public museum in Great Britain, while earlier the Bodleian has been the first truly public library.
(A) while earlier the Bodleian has been
(B) where the Bodleian earlier is
(C) just as earlier the Bodleian had been
(D) as the earlier Bodleian has been
(E) the Bodleian earlier being
565.Opening with tributes to jazz-age divas like Bessie Smith and closing with Koko Taylor’s electrified gravel-and-thunder songs, the program will trace the blues’ vigorous matriarchal line over more than 50 years.
(A) the program will trace
(B) the program shall trace
(C) there will be a program tracing
(D) it is a program that traces
(E) it will be a program tracing
566.Organized in 1966 by the Fish and Wildlife Service, the Breeding Bird Survey uses annual roadside counts along established routes for monitoring of population changes of as many as, or of more than 250 bird species, including 180 songbirds.
(A) for monitoring of population changes of as many as, or of
(B) to monitor population changes of as many, or
(C) to monitor changes in the populations of
(D) that monitors population changes of
(E) that monitors changes in populations of as many as, or
567.Originally developed for detecting air pollutants, a technique called proton-induced x-ray emission, which can quickly analyze the chemical elements in almost any substance without destroying it, is finding uses in medicine, archaeology, and criminology.
(A) Originally developed for detecting air pollutants, a technique called proton-induced x-ray emission, which can quickly analyze the chemical elements in almost any substance without destroying it,
(B) Originally developed for detecting air pollutants, having the ability to analyze the chemical elements in almost any substance without destroying it, a technique called proton induced x-ray emission
(C) A technique originally developed for detecting air pollutants, called proton-induced x-ray emission, which can quickly analyze the chemical elements in almost any substance without destroying it,
(D) A technique originally developed for detecting air pollutants, called proton-induced x-ray emission, which has the ability to analyze the chemical elements in almost any substance quickly and without destroying it,
(E) A technique that was originally developed for detecting air pollutants and has the ability to analyze the chemical elements in almost any substance quickly and without destroying the substance, called proton-induced x-ray emission,
568.Originally published in 1950, Some Tame Gazelle was Barbara Pym’s first novel, but it does not read like an apprentice work.
(A) does not read like an apprentice work
(B) seems not to read as an apprentice work
(C) does not seem to read as an apprentice work would
(D) does not read like an apprentice work does
(E) reads unlike an apprentice work
569.Our rate of teenage pregnancies is among the highest in the industrialized world, being exceeded only by Chile, Hungary, Romania, Cuba, and Bulgaria.
(A) is among the highest in the industrialized world, being exceeded only by
(B) is among the highest in the industrialized world, exceeded only by that of
(C) are among the highest in the industrialized world, only exceeded by
(D) is among the highest in the industrialized world, and exceeds only
(E) are among the highest in the industrialized world, and they exceed those of only
570.Out of America’s fascination with all things antique have grown a market for bygone styles of furniture and fixtures that are bringing back the chaise lounge, the overstuffed sofa, and the claw-footed bathtub.
(A) things antique have grown a market for bygone styles of furniture and fixtures that are bringing
(B) things antique has grown a market for bygone styles of furniture and fixtures that is bringing
(C) things that are antiques has grown a market for bygone styles of furniture and fixtures that bring
(D) antique things have grown a market for bygone styles of furniture and fixtures that are bringing
(E) antique things has grown a market for bygone styles of furniture and fixtures that bring
571.Over five thousand years ago, archaeologists have discovered that Mesopotamian cooks were preparing chicken with coriander and tamarind, just as Egyptian cooks are still doing today.
(A) Over five thousand years ago, archaeologists have discovered that
(B) Over five thousand years ago, archaeologists had discovered that
(C) Archaeologists have discovered that, over five thousand years ago,
(D) Over five thousand years ago, as archaeologists have discovered it,
(E) Archaeologists have discovered, over five thousand years ago, that
572.Pablo Picasso, the late Spanish painter, credited African art with having had a strong influence on his work.
(A) with having had
(B) for its having
(C) to have had
(D) for having
(E) in that it had
573.Paleontologists believe that fragments of a primate jawbone unearthed in Burma and estimated at 40 to 44 million years old provide evidence of a crucial step along the evolutionary path that led to human beings.
(A) at 40 to 44 million years old provide evidence of
(B) as being 40 to 44 million years old provides evidence of
(C) that it is 40 to 44 million years old provides evidence of what was
(D) to be 40 to 44 million years old provide evidence of
(E) as 40 to 44 million years old provides evidence of what was
574.Parliament did not accord full refugee benefits to twelve of the recent immigrants because it believed that to do it rewards them for entering the country illegally.
(A) to do it rewards
(B) doing it rewards
(C) to do this would reward
(D) doing so would reward
(E) to do it would reward
575.Pensions are now viewed as a deferred payment of salary, money a worker is compelled to put away to take care of one’s later years.
(A) a worker is compelled to put away to take care of one’s
(B) that a worker is compelled to put away to take care of oneself in
(C) a worker is compelled to put away to take care of oneself in
(D) workers are compelled to put away to take care of them in
(E) workers are compelled to put away to take care of themselves in
576.People have discovered the principles of solar energy whenever fuel becomes scarce and expensive but will forget them every a new source of cheap energy is developed.
(A) have discovered the principles of solar energy whenever fuel becomes scarce and
expensive but will forget
(B) have discovered the principles of solar energy whenever fuel has become scarce and expensive but they forget
(C) discovered the principles of solar energy every fuel becomes scarce and expensive, forgetting
(D) discover the principles of solar energy every fuel became scarce and expensive, but they forget
(E) discover the principles of solar energy whenever fuel becomes scarce and expensive but forget
577.People who inherit the sickle cell anemia gene from only one parent seem to be resistant to malaria, an evolutionary advantage that may explain why a genetic condition so debilitating to many individuals has survived in the human population.
(A) seem to be resistant to malaria,
(B) seemingly are resistant to malaria,
(C) seem to be resistant to malaria and have
(D) seemingly are resistant to malaria and to have
(E) are, it seems, resistant to malaria, and they have
578.Personal trainers and fitness coaches are to the narcissistic 1980’s just like a private fencing master and dancing teacher was to an earlier .
(A) just like a private fencing master and dancing teacher was
(B) as have been a private fencing master and dancing teacher
(C) what private fencing masters and dancing teachers were
(D) what private fencing masters and dancing teachers are
(E) just the same as private fencing masters and dancing teachers had been
579.Persons, suffering from a deficiency of the blood enzyme G6PD, eat fava beans and discover that they trigger hemolytic anemia.
(A) Persons, suffering from a deficiency of the blood enzyme G6PD, eat fava beans and discover that they trigger hemolytic anemia.
(B) When persons suffering a deficiency of enzyme G6PD, a blood enzyme, eat fava beans, they discover that it triggers hemolytic anemia.
(C) If people suffer deficiencies from the blood enzyme G6PD, they discover that fava beans trigger hemolytic anemia when eaten.
(D) Hemolytic anemia is triggered by people with a deficiency of the blood enzyme G6PD, who discover it eating fava beans.
(E) Persons who suffer from a deficiency of the blood enzyme G6PD discover that eating fava beans triggers hemolytic anemia.
580.Physics professors, the Arnmore Laboratories and Arnmore Research Facilities were founded by Leo and Fontove Arnmore in 1989 after ten years of fundraising.
(A) the Arnmore Laboratories and Arnmore Research Facilities were founded by Leo and Fontove Arnmore in 1989 after ten years of fundraising
(B) Leo and Fontove Arnmore founded the Arnmore Laboratories and Arnmore Research Facilities in 1989, after ten years of fundraising
(C) after ten years of fundraising, the Arnmore Laboratories and Arnmore Research Facilities were founded by Leo and Fontove Arnmore in 1989
(D) the Arnmore Laboratories and Arnmore Research Facilities were founded in 1989 by Leo and Fontove Arnmore after ten years of fundraising
(E) Leo and Fontove Arnmore founded after ten years of fundraising the Laboratories and Arnmore Research Facilities in 1989
581.Piaget’s research revealed that children can learn to count long before the recognition that a pint of water poured from a small glass into a large one remains the same amount of water.
(A) the recognition that
(B) they can recognize that
(C) they would recognize
(D) they could have the recognition of
(E) having the recognition of
582.Plausible though it sounds, the weakness of the hypothesis is that it does not
incorporate all relevant evidence.
(A) Plausible though it sounds, the weakness of the hypothesis
(B) Even though it sounds plausible, the weakness of the hypothesis
(C) Though plausible, the hypothesis’ weakness
(D) Though the hypothesis sounds plausible, its weakness
(E) The weakness of the hypothesis which sounds plausible
583.Poor management, outdated technology, competition from overseas, and steel’s replacement to materials like aluminum and fiber-reinforced plastics have all been cited as causes for the decline of the United States steel industry.
(A) steel’s replacement to materials like
(B) the replacement of steel by such materials as
(C) the replacing of steel with materials of
(D) the replacing of steel by means of materials like
(E) to replace steel by materials such as
584.Presenters at the seminar, one who is blind, will demonstrate adaptive equipment that allows visually impaired people to use computers.
(A) one who
(B) one of them who
(C) and one of them who
(D) one of whom
(E) one of which
585.Promotions, retirements, deaths, and other actions approved by the board of directors at its May meeting will be reported in the July 15 issue of the company paper.
(A) Promotions, retirements, deaths, and other actions approved by the board of directors at its May meeting will be reported in the July 15 issue of the company paper.
(B) Promotions, retirements, and other actions which have been approved at the May meeting of the board of directors along with deaths, with be reported in the July 15 issue of the company paper.
(C) To be reported in the July 15 issue of the company paper are the promotions,
retirements, deaths, and other actions which were approved at the board of directors’ May meeting.
(D) Meeting in May, the promotions, retirements, and other actions approved by the board of directors, including obituaries, will be reported in the July 15 issue of the company paper.
(E) The July 15 issue of the company paper will report on promotions, retirements, and other actions approved by the board of directors at its May meeting; the paper will also include obituaries.
586.Prompted by new evidence that the health risk posed by radon gas is far more serious than was previously thought, property owners are being advised by authorities to test all dwellings below the third floor for radon gas and to make repairs as needed.
(A) property owners are being advised by authorities to
(B) property owners are advised by authorities that they should
(C) authorities are advising property owners to
(D) authorities are advising property owners they
(E) authorities’ advice to property owners is they should
587.Proponents of artificial intelligence say they will be able to make computers that can understand English and other human languages, recognize objects, and reason as an expert does—computers that will be used to diagnose equipment breakdowns, deciding whether to authorize a loan, or other purposes such as these.
(A) as an expert does—computers that will be used to diagnose equipment breakdowns, deciding whether to authorize a loan, or other purposes such as these
(B) as an expert does, which may be used for purposes such as diagnosing equipment
breakdowns or deciding whether to authorize a loan
(C) like an expert—computers that will be used for such purposes as diagnosing equipment breakdowns or deciding whether to authorize a loan
(D) like an expert, the use of which would be for purposes like the diagnosis of equipment breakdowns or the decision whether or not a loan should be authorized
(E) like an expert, to be used to diagnose equipment breakdowns, deciding whether to
authorize a loan or not, or the like
588.Psychologists now contend that the way adults think and feel are determined as much by their peers in early childhood than by their parents.
(A) are determined as much by their peers in early childhood than by their
(B) are determined as much by peers in early childhood as do their
(C) is determined as much by their early childhood peers as by their
(D) have been determined by childhood peers as much as their
(E) was determined as much by one’s peers in childhood as by one’s
589.Published during the late eighteenth century, Diderot’s factual Encyclopedia and his friend Voltaire’s fictional Candide were the cause of such a sensational scandal, and both men prudently chose to embark on extended vacations in nearby Austria.
(A) Diderot’s factual Encyclopedia and his friend Voltaire’s fictional Candide were the cause of such a sensational scandal, and
(B) Diderot and his friend Voltaire’s caused such a sensational scandal with their factual Encyclopedia and fictional Candide, respectively, that
(C) Diderot’s factual Encyclopedia and his friend Voltaire’s fictional Candide were the cause of a scandal so sensational that
(D) the scandal caused by Diderot’s factual Encyclopedia and his friend Voltaire’s fictional Candide was so sensational
(E) a factual Encyclopedia by Diderot and the fictional Candide, by his friend Voltaire, caused a sensational scandal, which
590.Published in Harlem, the owner and editor of the Messenger were two young journalists, Chandler Owen and A. Philip Randolph, who would later make his reputation as a labor leader.
(A) Published in Harlem, the owner and editor of the Messenger were two young journalists, Chandler Owen and A. Philip Randolph, who would later make his reputation as a labor leader.
(B) Published in Harlem, two young journalists, Chandler Owen and A. Philip Randolph, who would later make his reputation as a labor leader, were the owner and editor of the Messenger.
(C) Published in Harlem, the Messenger was owned and edited by two young journalists, A. Philip Randolph, who would later make his reputation as a labor leader, and Chandler Owen.
(D) The Messenger was owned and edited by two young journalists, Chandler Owen and A. Philip Randolph, who would later make his reputation as a labor leader, and published in Harlem.
(E) The owner and editor being two young journalists, Chandler Owen and A. Philip Randolph, who would later make his reputation as a labor leader, the Messenger was published in Harlem.
591.Quasars are so distant that their light has taken billions of years to reach the Earth; consequently, we see them as they were during the formation of the universe.
(A) we see them as they were during
(B) we see them as they had been during
(C) we see them as if during
(D) they appear to us as they did in
(E) they appear to us as though in
592.Quasars, at billions of light-years from Earth the most distant observable objects in the universe, believed to be the cores of galaxies in an early stage of development.
(A) believed to be
(B) are believed to be
(C) some believe them to be
(D) some believe they are
(E) it is believed that they are
593.Ralph Eilison and Amiri Baraka both argued that music was perhaps the ultimate
expression of Afro-American culture, that it was the one vector of African culture that there was no possibility to eradicate.
(A) that it was the one vector of African culture that there was no possibility to eradicate
(B) the one vector of African culture that could not be eradicated
(C) for it was the one vector of African culture, and that it was impossible to eradicate
(D) a vector of African culture that there was no possibility to eradicate
(E) as being the one vector that could not be eradicated from African culture
594.Ranked as one of the most important of Europe’s young playwrights, Franz Xaver Kroetz has written forty plays; his works—translated into over thirty languages—are produced more often than any contemporary German dramatist.
(A) than any
(B) than any other
(C) than are any
(D) than those of any other
(E) as are those of any
595.Rather than continue to produce most of the items necessary for subsistence, a growing number of farm families during the first decades of the nineteenth century began to specialize in the production of grain or cotton and to use the cash proceeds from selling their crops for buying necessities.
(A) selling their crops for buying
(B) the sales of their crops for buying
(C) their selling of crops so as to buy
(D) their selling crops for buying of
(E) the sale of their crops to buy
596.Real similar to a Hollywood movie set with nothing behind the building fronts, the country’s apparent new-found affluence masks a very different reality: most citizens are not living at all well.
(A) Real similar to
(B) Much as if it was
(C) As though
(D) Like
(E) Just as
597.Reared apart from each other, a recent United States study showed striking similarities in identical twins, including many idiosyncrasies of behavior.
(A) Reared apart from each other, a recent United States study showed striking similarities in identical twins, including many idiosyncrasies of behavior.
(B) Reared apart from each other, striking similarities between identical twins that include many idiosyncrasies of behavior were shown in a recent United States study.
(C) A recent United States study showed striking similarities in identical twins reared apart from each other that include many idiosyncrasies of behavior.
(D) According to a recent United States study, identical twins reared apart from each other showed striking similarities, including many idiosyncrasies of behavior.
(E) According to a recent United States study, identical twins showed striking similarities reared apart from each other, including many idiosyncrasies of behavior.
598.Recent essays by Garrison Keillor, the humorist and host of the public radio show A Prairie Home Companion, describes what might happen if Huns, Goths, and Visigoths were to invade Chicago.
(A) describes what might happen if Huns, Goths, and Visigoths were to invade
(B) describes what would happen if Huns, Goths, and Visigoths invaded
(C) describes what would happen if Huns, Goths, and Visigoths would have invaded
(D) describe what might happen if Huns, Goths, and Visigoths were to invade
(E) describe what happen if Huns, Goths, and Visigoths invade
599.Recent excavations suggest that the ancient peoples of the Italian peninsula merged the cult of Damia—a goddess of fertility and the harvest—with Venus.
(A) with Venus
(B) and Venus
(C) with that of Venus
(D) and Venus’
(E) and Venus’ cult
600.Recently discovered fossil remains strongly suggest that the Australian egg-laying mammals of today are a branch of the main stem of mammalian evolution rather than developing independently from a common ancestor of mammals more than 220 million years ago.
(A) rather than developing independently from
(B) rather than a type that developed independently from
(C) rather than a type whose development was independent of
(D) instead of developing independently from
(E) instead of a development that was independent of
601.Recently implemented “shift-work equations” based on studies of the human sleep cycle have reduced sickness, sleeping on the job, fatigue among shift workers, and have raised production efficiency in various industries.
(A) fatigue among shift workers, and have raised
(B) fatigue among shift workers, and raised
(C) and fatigue among shift workers while raising
(D) lowered fatigue among shift workers, and raised
(E) and fatigue among shift workers was lowered while raising
602.Recently there has been increased debate over if a budget surplus should go towards lower taxes or increased spending on social programs.
(A) over if a budget surplus should go towards lower taxes or increased spending
(B) over whether a budget surplus should go towards lowering taxes or increasing spending
(C) about a budget surplus going towards lower taxes or increasing spending
(D) about if lower taxes should come from a budget surplus or spending increases
(E) concerning a budget surplus and its going towards lower taxes or increased spending
603.Rembrandt so treasured his collection of Islamic portraits that when forced to sell them in order to raise money, he first made copies of more than twenty.
(A) he first made copies of more than twenty
(B) first he made copies of more than twenty
(C) more than twenty were copied
(D) copies of more than twenty were made
(E) he copies more than twenty of them first
604.Repelled by bodily punishments such as maiming and branding, the idea of penitentiaries were reforms of the penal system by Quakers.
(A) the idea of penitentiaries were reforms of the penal system by Quakers
(B) penitentiaries were ideas for reform of the penal system suggested by Quakers
(C) Quakers suggested the penitentiary as a reform of the penal system
(D) Quakers suggested that the penal system be reformed as penitentiaries
(E) the penitentiary was suggested to be a reform of the penal system by Quakers
605.Reporting that one of its many problems had been the recent extended sales slump in women’s apparel, the seven-store retailer said it would start a three-month liquidation sale in all of its stores.
(A) its many problems had been the recent
(B) its many problems has been the recently
(C) its many problems is the recently
(D) their many problems is the recent
(E) their many problems had been the recent
606.Research during the past several decades on the nature of language and the processes that produce and make it understandable has revealed great complexity instead of underlying simplicity.
(A) that produce and make it understandable has revealed great complexity instead of
underlying simplicity
(B) of producing and understanding it have revealed not underlying simplicity but great complexity
(C) by which it is produced and understood has revealed not underlying simplicity but great complexity
(D) by which it is produced and understood have revealed great complexity rather than underlying simplicity
(E) by which one produces and understands it have revealed great complexity instead of underlying simplicity
607.Researchers at Cornell University have demonstrated that homing pigeons can sense changes in the earth’s magnetic field, see light waves that people cannot see, detect low-frequency sounds from miles away, sense changes in air pressure, and can identify familiar odors.
(A) sense changes in air pressure, and can identify familiar odors
(B) can sense changes in air pressure, and can identify familiar odors
(C) sense changes in air pressure, and identify familiar odors
(D) air pressure changes can be sensed, and familiar odors identified
(E) air pressure changes are sensed, and familiar odors identified
608.Researchers have questioned the use of costly and experimental diagnostic tests to identify food allergies, such as milk, that supposedly disrupt normal behavior.
(A) to identify food allergies, such as
(B) to identify food allergies, like
(C) to identify food allergies, such as to
(D) for identifying food allergies, like that of
(E) for identifying food allergies, such as for
609.Revered by an ill-informed citizenry, the Duke of York was feted opulently for several months before there was denunciation and exile.
(A) there was denunciation and exile
(B) he was to be denounced with exile
(C) being denounced and exiled
(D) denunciation and his exile
(E) being exiled, having been denounced
610.Rising inventories, when unaccompanied correspondingly by increases in sales, can lead to production cutbacks that would hamper economic growth.
(A) when unaccompanied correspondingly by increases in sales, can lead
(B) when not accompanied by corresponding increases in sales, possibly leads
(C) when they were unaccompanied by corresponding sales increases, can lead
(D) if not accompanied by correspondingly increased sales, possibly leads
(E) if not accompanied by corresponding increases in sales, can lead
611.Roy Wilkins was among the last of a generation of civil rights activists who led the nation through decades of change so profound many young Americans are not able to imagine, even less to remember, what segregation was like.
(A) so profound many young Americans are not able to imagine, even less to remember
(B) so profound that many young Americans cannot imagine, much less remember
(C) so profound many young Americans cannot imagine nor even less remember
(D) of such profundity many young Americans cannot imagine, even less can they remember
(E) of such profundity that many young Americans are not able to imagine, much less to remember
612.Rules banning cancer-causing substances from food apply to new food additives and not to natural constituents of food because their use as additives is entirely avoidable.
(A) their use as additives is
(B) as additives, their use is
(C) the use of such additives is
(D) the use of such additives are
(E) the use of them as additives is
613.Sales of United States manufactured goods to nonindustrialized countries rose to $167 billion in 1992, which is 14 percent more than the previous year and largely offsets weak demand from Europe and Japan.
(A) which is 14 percent more than the previous year
(B) which is 14 percent higher than it was the previous year
(C) 14 percent higher than the previous year’s figure
(D) an amount that is 14 percent more than the previous year was
(E) an amount that is 14 percent higher than the previous year’s figure
614.Salt deposits and moisture threaten to destroy the Mohenjo-Daro excavation in Pakistan, the site of an ancient civilization that flourished at the same as the civilizations in the Nile delta and the river valleys of the Tigris and Euphrates.
(A) that flourished at the same as the civilizations
(B) that had flourished at the same as had the civilizations
(C) that flourished at the same those had
(D) flourishing at the same as those did
(E) flourishing at the same as those were
615.Samuel Sewall viewed marriage, as other seventeenth-century colonists, like a property arrangement rather than an emotional bond based on romantic love.
(A) Samuel Sewall viewed marriage, as other seventeenth-century colonists, like a property arrangement rather than
(B) As did other seventeenth-century colonists, Samuel Sewall viewed marriage to be a property arrangement rather than viewing it as
(C) Samuel Sewall viewed marriage to be a property arrangement, like other
seventeenth-century colonists, rather than viewing it as
(D) Marriage to Samuel Sewall, like other seventeenth-century colonists, was viewed as a property arrangement rather than
(E) Samuel Sewall, like other seventeenth-century colonists, viewed marriage as a property arrangement rather than
616.Sartre believed each individual is responsible to choose one course of action over another one, that it is the choice that gives value to the act, and that nothing that is not acted upon has value.
(A) each individual is responsible to choose one course of action over another one
(B) that each individual is responsible for choosing one course of action over another
(C) that each individual is responsible, choosing one course of action over another
(D) that each individual is responsible to choose one course of action over the other
(E) each individual is responsible for choosing one course of action over other ones
617.School desegregation has worked well in Buffalo, New York, in part because parents and teachers were given major roles in designing the city’s magnet schools, because extra federal funds were allocated to make each school unique, and because the federal judge enforced desegregation orders.
(A) because extra federal funds were allocated to make each school unique
(B) because of the allocation of extra federal funds that make each school unique
(C) because each school is made unique by allocating it extra federal funds
(D) extra federal funds were allocated in order to make each school unique
(E) extra federal funds were allocated for making each school unique
618.School integration plans that involve busing between suburban and central-city areas have contributed, according to a recent study, to significant increases in housing integration, which, in turn, reduces any future need for busing.
(A) significant increases in housing integration, which, in turn, reduces
(B) significant integration increases in housing, which, in turn, reduces
(C) increase housing integration significantly, which, in turn, reduces
(D) increase housing integration significantly, in turn reducing
(E) significantly increase housing integration, which, in turn, reduce
619.Scientific interest in providing suitable habitats for bottom-dwelling animals such as river clams arises not because they are important sources of human food but from their role as an integral link in the Aquatic food chain.
(A) not because they are important sources of human food
(B) although they are not an important sources of human food
(C) not in that they are important as human sources of food
(D) not from their importance as a source of human food
(E) not from being important sources of food for human beings
620.Scientists believe that unlike the males of most species of moth, the male whistling moths of Nambung, Australia, call female moths to them by the use of acoustical signals, but not olfactory ones, and they attract their mates during the day, rather than at night.
(A) by the use of acoustical signals, but not olfactory ones, and they attract
(B) by the use of acoustical signals instead of using olfactory ones, and attracting
(C) by using acoustical signals, not using olfactory ones, and by attracting
(D) using acoustical signals, rather than olfactory ones, and attract
(E) using acoustical signals, but not olfactory ones, and attracting
621.Scientists calculated that the asteroid, traveling at 46,000 miles an hour, is on an elliptical path that orbits the Sun once a year and regularly brings it back toward Earth.
(A) hour, is on an elliptical path that orbits the Sun once a year and regularly brings it
(B) hour, is orbiting the Sun once a year on an elliptical path that regularly brings it
(C) hour, once a year orbits the Sun, regularly bringing it on an elliptical path
(D) hour and orbiting the Sun once a year on an elliptical path, regularly bringing it
(E) hour, orbits the Sun on an elliptical path once a year and that regularly brings it
622.Scientists have observed large concentrations of heavy-metal deposits in the upper twenty centers of Baltic Sea sediments, which are consistent with the growth of industrial activity there.
(A) Baltic Sea sediments, which are consistent with the growth of industrial activity there
(B) Baltic Sea sediments, where the growth of industrial activity is consistent with these findings
(C) Baltic Sea sediments, findings consistent with its growth of industrial activity
(D) sediments from the Baltic Sea, findings consistent with the growth of industrial
activity in the area
(E) sediments from the Baltic Sea, consistent with the growth of industrial activity there
623.Scientists have recently discovered what could be the largest and oldest living organism on Earth, a giant fungus that is an interwoven filigree of mushrooms and rootlike tentacles spawned by a single fertilized spore some 10,000 years ago and extending for more than 30 acres in the soil of a Michigan forest.
(A) extending
(B) extends
(C) extended
(D) it extended
(E) is extending
624.Scientists have suggested that once every 10 million years or so a truly colossal object from space cuts through the atmosphere and slams into Earth, sending up a global pall of dust that blots out the Sun, alters the climate, and changes the course of evolution by killing off many plant and animal species.
(A) sending up a global pall of dust that blots
(B) thus sending up a global pall of dust to blot
(C) thereby sending up a global pall of dust to blot
(D) and that sends up a global pall of dust, blotting
(E) which sends up a global pall of dust, blots
625.Scientists who studied the famous gold field known as Serra Pelada concluded that the rich lode was not produced by the accepted methods of ore formation but that swarms of microbes over millions of years concentrated the gold from jungle soils and rivers and rocks.
(A) not produced by the accepted methods of ore formation but that swarms of microbes over millions of years
(B) not produced by the accepted methods of ore formation but instead swarms of microbes over millions of years that
(C) not produced by the accepted methods of ore formation but swarms of microbes over millions of years that
(D) produced not by the accepted methods of ore formation but by swarms of microbes that over millions of years
(E) produced not by the accepted methods of ore formation but that swarms of microbes over millions of years
626.Scorched by fire, stained by water, and inscribed in cramped handwriting, the
seventeenth-century Dutch documents on the beginnings of New York City were long ignored by historians, depending instead on English sources for information.
(A) the seventeenth-century Dutch documents on the beginnings of New York City were long ignored by historians, depending
(B) the seventeenth-century Dutch documents on the beginnings of New York City were long ignored by historians, who depended
(C) historians long ignored the seventeenth-century Dutch documents on the beginnings of New York City, depending
(D) historians long ignored the seventeenth-century Dutch documents on the beginnings of New York City and depended
(E) historians long ignored the seventeenth-century Dutch documents on the beginnings of New York City, they depended
627.Sculptor Alexander Calder, who often made use of old pieces of junk in his art and also believed in recycling at home; he once turned a broken goblet into a dinner bell and a cake mold into a lamp.
(A) Sculptor Alexander Calder, who often made use of old pieces of junk in his art and also believed in recycling at home; he
(B) Alexander Calder, for whom old pieces of junk was often made into sculpture, believed in recycling at home and
(C) A believer in recycling at home, sculptor Alexander Calder often made use for old pieces of junk in his art; he
(D) Alexander Calder, for whom sculpture was often made from old pieces of junk, also believed in recycling at home, for example, he
(E) Sculptor Alexander Calder, who often made use of old pieces of junk in his art, also believed in recycling at home; he
628. 13(d) of the Securities Exchange Act of 1934 requires anyone who buys more than 5 percent of a company’s stock make a public disclosure of the purchase.
(A) make
(B) will also make
(C) to make
(D) must make
(E) must then make
629.Seeming to be one of the few corporations diversified enough to survive the recession, many shareholders ignored the drop in third-quarter profits and invested even more heavily in Emco.
(A) Seeming to be
(B) As if
(C) In that they seemed
(D) Although it seemed
(E) Because it seemed to be
630.Seeming to be the only organization fighting for the rights of poor people in the South, Hosea Hudson, a laborer in Alabama, joined the Communist party in 1931.
(A) Seeming to be
(B) As
(C) In that they seemed
(D) Since it seemed
(E) Because it seemed to be
631.Seismologists studying the earthquake that struck northern California in October 1989 are still investigating some of its mysteries: the unexpected power of the seismic waves, the upward thrust that threw one man straight into the air, and the strange electromagnetic signals detected hours before the temblor.
(A) the upward thrust that threw one man straight into the air, and the strange
electromagnetic signals detected hours before the temblor
(B) the upward thrust that threw one man straight into the air, and strange electromagnetic signals were detected hours before the temblor
(C) the upward thrust threw one man straight into the air, and hours before the temblor strange electromagnetic signals were detected
(D) one man was thrown straight into the air by the upward thrust, and hours before the temblor strange electromagnetic signals were detected
(E) one man who was thrown straight into the air by the upward thrust, and strange
electromagnetic signals that were detected hours before the temblor
632.Selling several hundred thousand copies in six months, the publication of “Maple Leaf Rag” in 1899 was an instant hit, helping to establish Scott Joplin as the preeminent rag composer.
(A) Selling several hundred thousand copies in six months, the publication of “Maple Leaf Rag” in 1899 was an instant hit, helping to establish Scott Joplin as the preeminent rag composer.
(B) The publication in 1899 of “Maple Leaf Rag” was an instant hit: in six months they sold several hundred thousand copies and it helped establish Scott Joplin as the preeminent rag composer.
(C) Helping to establish Scott Joplin as the preeminent rag composer was the publication of “Maple Leaf Rag” in 1899, which was an instant hit: it sold several hundred thousand copies in six months.
(D) “Maple Leaf Rag” was an instant hit: it helped establish Scott Joplin as the preeminent rag composer, published in 1899 and selling several hundred thousand copies in six months.
(E) Published in 1899, “Maple Leaf Rag” was an instant hit, selling several hundred thousand copies in six months: it helped establish Scott Joplin as the preeminent rag composer.
633.Senator Lasker has proposed legislation requiring that employers should retain all older workers indefinitely or show just cause for dismissal.
(A) that employers should retain all older workers
(B) that all older workers be retained by employers
(C) the retaining by employers of all older workers
(D) employers’ retention of all older workers
(E) employers to retain all older workers
634.Senior executives had a larger percentage increase in pay in 1990 than the wages of other salaried workers.
(A) Senior executives had a larger percentage increase in pay in 1990 than
(B) The percentage of senior executives’ pay increase in 1990 was larger than for
(C) The 1990 increase in pay for senior executives was larger in terms of percentage than
(D) In 1990 senior executives had a larger pay increase in terms of percentage than did
(E) The pay of senior executives increased in 1990 by a larger percentage than did
635.Several recent studies suggest that a child born into a family whose members have allergies will probably themselves develop allergies following the onset of a minor viral infection.
(A) a child born into a family whose members have allergies will probably themselves develop allergies
(B) children born into families whose members have allergies will probably themselves develop allergies
(C) a child born into a family the members of which have allergies will probably develop an allergy
(D) in those families where members have allergies, children will probably develop allergies themselves
(E) children born into a family in which there are allergies will themselves probably develop an allergy
636.Several senior officials spoke to the press on condition that they not be named in the story.
(A) that they not be named
(B) that their names will not be used
(C) that their names are not used
(D) of not being named
(E) they will not be named
637.Several studies have found that the coronary patients who exercise most actively have half or less than half the chance of dying of a heart attack as those who are sedentary.
(A) have half or less than half the chance of dying of a heart attack as those who are sedentary
(B) have half the chance, or less, of dying of a heart attack than those who are sedentary do
(C) have half the chance that they will die of a heart attack, or less, than those who are sedentary do
(D) are at least fifty percent less likely to die of a heart attack as those who are
sedentary
(E) are at least fifty percent less likely than those who are sedentary to die of a heart attack
638.Several years ago the diet industry introduced a variety of appetite suppressants, but some of these drugs caused stomach disorders severe enough to have them banned by the Food and Drug Administration.
(A) stomach disorders severe enough to have them
(B) stomach disorders that were severe enough so they were
(C) stomach disorders of such severity so as to be
(D) such severe stomach disorders that they were
(E) such severe stomach disorders as to be
639.Severe and increasing numerous critics are pointing to deficiencies in the British legal system, deficiencies that seem to deny a proper defense to many clients who are charged with crimes.
(A) Severe and increasing numerous critics are pointing to deficiencies in the British legal system, deficiencies that seem
(B) Severe and increasing numerous critics point to deficiencies in the British legal system, deficiencies seeming
(C) Severe and increasingly numerous critics are pointing to deficiencies in the British legal system that seem
(D) Severely and increasingly numerous critics point to deficiencies in the British legal system seeming
(E) Severely and increasingly numerous critics are pointing to deficiencies in the British legal system that seem
640.She was a child prodigy, and Clara Schumann developed into one of the greatest pianists of her .
(A) She was a child prodigy, and Clara Schumann
(B) A child prodigy, Clara Schumann
(C) Child prodigy that she was, Clara Schumann
(D) Clara Schumann has been a child prodigy, and she
(E) Being a child prodigy, Clara Schumann
641.Similar to rising interest rates, consumer and producer prices have been rising.
(A) Similar to rising interest rates, consumer and producer prices have been rising.
(B) Consumer and producer prices have been rising, as have interest rates.
(C) As interest rates are rising, so have consumer and producer prices.
(D) Consumer and producer prices have been rising, like interest rates do.
(E) Consumer and producer prices, as interest rates, have been rising.
642.Since 1965 there are four s as many Black college students enrolled, and the one million Black people in college today represent 11 percent of all college students.
(A) Since 1965 there are four s as many Black college students enrolled
(B) The enrollment of Black college students was only one-fourth in 1965
(C) The enrollment of Black college students has increased four s from 1965 on
(D) Quadrupling since 1965, there are now four s as many Black college students enrolled
(E) The enrollment of Black college students has quadrupled since 1965
643.Since 1970 the number of Blacks elected to state and federal offices in the United States has multiplied nearly four s.
(A) has multiplied nearly four s
(B) has almost quadrupled
(C) has almost multiplied by four
(D) is almost four s as great
(E) is nearly fourfold what it was
644.Since 1975 the number of women in upper-level management in American corporations have increased by 25 percent; female executives’ salaries, however, still lag behind those of their male counterparts.
(A) have increased by 25 percent; female executives’ salaries, however, still lag
(B) has increased by 25 percent, however much their salaries lag
(C) have increased 25 percent; female executives’ salaries, however, still have lagged
(D) has increased by 25 percent; female executives’ salaries, however, still lag
(E) have increased 25 percent; their salaries, however, still lag
645.Since 1981, when the farm depression began, the number of acres overseen by professional farm-management companies have grown from 48 million to nearly 59 million, an area that is about Colorado’s size.
(A) have grown from 48 million to nearly 59 million, an area that is about Colorado’s size
(B) have grown from 48 million to nearly 59 million, about the size of Colorado
(C) has grown from 48 million to nearly 59 million, an area about the size of Colorado
(D) has grown from 48 million up to nearly 59 million, an area about the size of Colorado’s
(E) has grown from 48 million up to nearly 59 million, about Colorado’s size
646.Since 1986 enrollments of African Americans, American Indians, and Hispanic Americans in full engineering programs in the United States has steadily increased, while the number of other students who enter the field has fallen.
(A) has steadily increased, while the number of other students who enter the field has fallen
(B) has steadily increased, while other students entering the field have declined in number
(C) increased steadily, while there was a decline in the number of other students entering the field
(D) have steadily increased, while the number of other students entering the field has fallen
(E) have steadily increased, while that of other students who enter the field fell
647.Since 1986 when the Department of Labor began to allow investment officers’ fees to be based on how the funds they manage perform, several corporations began paying their investment advisers a small basic fee, with a contract promising higher fees if the managers perform well.
(A) investment officers’ fees to be based on how the funds they manage perform, several corporations began
(B) investment officers’ fees to be based on the performance of the funds they manage, several corporations began
(C) that fees of investment officers be based on how the funds they manage perform, several corporations have begun
(D) fees of investment officers to be based on the performance of the funds they manage, several corporations have begun
(E) that investment officers’ fees be based on the performance of the funds they manage, several corporations began
648.Since chromosome damage may be caused by viral infections, medical x-rays, and exposure to sunlight, it is important that the chromosomes of a population to be tested for chemically induced damage be compared with those of a control population.
(A) to be tested for chemically induced damage be compared with
(B) being tested for damage induced chemically are compared with
(C) being tested for chemically induced damage should be compared to
(D) being tested for chemically induced damage are to be compared to
(E) that is to be tested for chemically induced damage are to be comparable with
649.Since conscious patients often died of shock on the operating table, the invention of anesthesia was essential to the development of surgery as the invention of the propeller was to powered flight.
(A) the invention of anesthesia was essential to the development of surgery as the invention of the propeller was to powered flight
(B) inventing anesthesia was as essential for the development of surgery as the invention of the propeller was for powered flight
(C) the invention of anesthesia was as essential in the development of surgery much as the invention of the propeller had been for powered flight
(D) the invention of anesthesia was as essential to the development of surgery as the invention of the propeller was to powered flight
(E) the invention of anesthesia was essential to the development of surgery, so was the invention of the propeller essential to powered flight
650.Since savings banks have to use short-term deposits to finance long-term fixed-rate mortgage loans, they sometimes lose money when there is a rise in short-term rates and, on the other hand, they are unable to raise the rates on their mortgages.
(A) when there is a rise in short-term rates and, on the other hand, they are unable to raise
(B) when short-term rates rise and they are unable to raise
(C) when a rise in short-term rates occurs and, correspondingly, there is no rise possible in
(D) with a rise in short-term rates, and they are unable to raise
(E) with short-term rates on the rise and no rise possible in
651.Since the 1930’s aircraft manufacturers have tried to build airplanes with frictionless wings, shaped so smoothly and perfectly that the air passing over them would not become turbulent.
(A) wings, shaped so smoothly and perfectly
(B) wings, wings so smooth and so perfectly shaped
(C) wings that are shaped so smooth and perfect
(D) wings, shaped in such a smooth and perfect manner
(E) wings, wings having been shaped smoothly and perfectly so
652.Since the movie was released seventeen UFOs have been sighted in the state, which is more than had been sighted in the past ten years together.
(A) which is more than had been sighted
(B) more than had been sighted
(C) more than they had sighted
(D) more than had reported sightings
(E) which is more than had reported sightings
653.Slips of the tongue do not necessarily reveal concealed beliefs or intentions but rather are the result from the competition between various processing mechanisms in the brain.
(A) but rather are the result from
(B) and instead are the result from
(C) being rather the result of
(D) and rather result from
(E) but rather result from
654.So poorly educated and trained are many young recruits to the United States work force that many business executives fear this country will lose its economic preeminence.
(A) So poorly educated and trained are many young recruits to the United States work force that
(B) As poorly educated and trained as many young recruits to the United States work force are
(C) Because of many young recruits to the United States work force who are so poorly
educated and trained.
(D) That many young recruits to the United States work force are so poorly educated and trained is why
(E) Many young recruits to the United States work force who are so poorly educated and trained explains why
655.Socrates could have fled from Athens after he was sentenced to death, but he refused to do it.
(A) Socrates could have fled from Athens after he was sentenced to death, but he refused to do it.
(B) After he was sentenced to death, Socrates might have fled from Athens, but he refused to do it.
(C) After he was sentenced to death, Socrates could have fled from Athens, but he refused to do so.
(D) After he was sentenced to death, Socrates could have done so after he was sentenced to death.
(E) Socrates could have fled from Athens but refused to after he was sentenced to death.
656.Some analysts contend that true capitalism exists only when the ownership of both property and the means of production is regarded as an inalienable right of an individual’s, and it is not a license granted by government and revokable at whim.
(A) is regarded as an inalienable right of an individual’s, and it is not
(B) are regarded as individuals’ inalienable rights, and that it not be
(C) is regarded as an individual’s inalienable right, not as
(D) are regarded as an individual’s inalienable rights, not when they are
(E) is regarded as the inalienable rights of an individual, not when it is
657.Some analysts of retirement problems insist that the only way to provide equity and protection for all Americans is to amalgamate all retirement-income systems, including Social Security, into one central system.
(A) insist that the only way to provide equity and protection for all Americans is to amalgamate
(B) have insisted the only way equity and protection can be provided for all Americans is amalgamating
(C) insist the only way to provide equity and protection for all Americans is the
amalgamation of
(D) are insistent that the only way equity and protection can be provided for all Americans is the amalgamation of
(E) insist that the only way for the provision of equity and protection for all Americans is to amalgamate
658.Some analysts point out that because people are becoming accustomed to a steady
inflation rate of four to five percent, businesses found that they could raise prices according to this amount without thereby provoking strong public reaction.
(A) found that they could raise prices according to this amount without thereby provoking
(B) found that they were capable of raising prices by this amount and not provoke
(C) find that they are capable of raising prices by this amount and not provoke
(D) are finding that they can raise prices by this amount without provoking
(E) are finding that they can raise prices according to this amount and will not thereby provoke
659.Some bat caves, like honeybee hives, have residents that take on different duties such as defending the entrance, acting as sentinels and to sound a warning at the approach of danger, and scouting outside the cave for new food and roosting sites.
(A) acting as sentinels and to sound
(B) acting as sentinels and sounding
(C) to act as sentinels and sound
(D) to act as sentinels and to sound
(E) to act as a sentinel sounding
660.Some biographers have not only disputed the common notion that Edgar Allan Poe drank to excess but also questioned whether he drank at all.
(A) have not only disputed the common notion that Edgar Allan Poe drank to excess but also questioned whether he drank
(B) not only have disputed the common notion that Edgar Allan Poe drank to excess but also over whether he drank
(C) have disputed not only the common notion that Edgar Allan Poe drank to excess but also whether he may not have drunk
(D) not only have disputed the common notion that Edgar Allan Poe drank to excess but also questioned whether or not he had drunk
(E) have disputed the common notion not only that Edgar Allan Poe drank to excess but also questioned whether he may not have drunk
661.Some buildings that were destroyed and heavily damaged in the earthquake last year were constructed in violation of the city’s building code.
(A) Some buildings that were destroyed and heavily damaged in the earthquake last year were
(B) Some buildings that were destroyed or heavily damaged in the earthquake last year had been
(C) Some buildings that the earthquake destroyed and heavily damaged last year have been
(D) Last year the earthquake destroyed or heavily damaged some buildings that have been
(E) Last year some of the buildings that were destroyed or heavily damaged in the earth quake had been
662.Some historians of science have argued that science moves forward not so much because of the insights of great thinkers but because of more mundane developments, such as improved tools and technologies.
(A) because of the insights of great thinkers but because of
(B) because of the insights of great thinkers as the result of
(C) because of the insights of great thinkers as because of
(D) through the insights of great thinkers but through
(E) through the insights of great thinkers but result from
663.Some of the tenth-century stave churches of Norway are still standing, demonstrating that with sound design and maintenance, wooden buildings can last indefinitely.
(A) standing, demonstrating that with sound design and maintenance, wooden buildings can last indefinitely
(B) standing, demonstrating how wooden buildings, when they have sound design and
maintenance, can last indefinitely
(C) standing; they demonstrate if a wooden building has sound design and maintenance it can last indefinitely
(D) standing, and they demonstrate wooden buildings can last indefinitely when there is sound design and maintenance
(E) standing, and they demonstrate how a wooden building can last indefinitely when it has sound design and maintenance
664.Some psychiatric studies indicate that among distinguished artists the rates of manic depression and major depression are ten to thirteen s as prevalent as in the population at large.
(A) the rates of manic depression and major depression are ten to thirteen s as prevalent as in
(B) the rates of manic depression and major depression are ten to thirteen s more prevalent than in
(C) the rates of manic depression and major depression are ten to thirteen s more prevalent when compared to
(D) manic depression and major depression are ten to thirteen s as prevalent when compared to
(E) manic depression and major depression are ten to thirteen s more prevalent than in
665.Some scientists have been critical of the laboratory tests conducted by the Federal Drug Administration on the grounds that the amounts of suspected carcinogens fed to animals far exceeds those that humans could consume.
(A) far exceeds those that humans could consume
(B) exceeds by far those humans can consume
(C) far exceeds those humans are able to consume
(D) exceed by far those able to be consumed by humans
(E) far exceed those that humans could consume
666.South Korea has witnessed the world’s most dramatic growth of Christian congregations; church membership is expanding by 6.6 percent a year, fully two-thirds of the growth coming from conversions rather than the population increasing.
(A) coming from conversions rather than the population increasing
(B) coming from conversions rather than increases in the population
(C) coming from conversions instead of the population’s increasing
(D) is from conversions instead of population increases
(E) is from conversions rather than increasing the population
667.Spanning more than fifty years, Friedrich Muller began his career in an unpromising apprenticeship as a Sanskrit scholar and culminated in virtually every honor that European governments and learned societies could bestow.
(A) Muller began his career in an unpromising apprenticeship as
(B) Muller’s career began in an unpromising apprenticeship as
(C) Muller’s career began with the unpromising apprenticeship of being
(D) Muller had begun his career with the unpromising apprenticeship of being
(E) the career of Muller has begun with an unpromising apprenticeship of
668.St. John’s, Newfoundland, lies on the same latitude as Paris, France, but in spring St. John’s residents are less likely to be sitting at outdoor cafes than to be bracing themselves against arctic chills, shoveling snow, or seeking shelter from a raging northeast storm.
(A) residents are less likely to be sitting at outdoor cafes than to be bracing themselves against arctic chills, shoveling snow, or seeking
(B) residents are less likely to sit at outdoor cafes, and more to brace themselves against arctic chills, shovel snow, or be seeking
(C) residents are less likely to be sitting at outdoor cafes, and more likely to be bracing themselves against arctic chills, shoveling snow, or to be seeking
(D) residents, instead of their sitting at outdoor cafes, they are more likely to brace themselves against arctic chills, shovel snow, or seek
(E) residents, instead of sitting at outdoor cafes, are more likely to brace themselves against arctic chills, shovel snow, or to be seeking
669.Stable interest rates on long-term bonds are the financial market’s vote of confidence in the Federal Reserve keeping in control of inflation.
(A) in the Federal Reserve keeping in control of inflation
(B) that the Federal Reserve will keep inflation under control
(C) for the Federal Reserve, that it would keep control of inflation
(D) that inflation will be kept control of by the Federal Reserve
(E) that inflation would be kept control of by the Federal Reserve
670.Stars like the sun can continue to shine steadily for billions of years because its light and heat are produced by nuclear fusion, in which titanic releases of energy result in the loss of only tiny amounts of mass.
(A) its light and heat are produced by nuclear fusion, in which
(B) nuclear fusion produces its light and heat, in which
(C) nuclear fusion produces their light and heat, where
(D) its light and heat are produced by nuclear fusion, where
(E) their light and heat are produced by nuclear fusion, in which
671.State officials report that soaring rates of liability insurance have risen to force cutbacks in the operations of everything from local governments and school districts to day-care centers and recreational facilities.
(A) rates of liability insurance have risen to force
(B) rates of liability insurance are a force for
(C) rates for liability insurance are forcing
(D) rises in liability insurance rates are forcing
(E) liability insurance rates have risen to force
672.Students in the metropolitan school district lack math skills to such a large degree as to make it difficult to absorb them into a city economy becoming ever more dependent on information-based industries.
(A) lack math skills to such a large degree as to make it difficult to absorb them into a city economy becoming
(B) lack math skills to a large enough degree that they will be difficult to absorb into a city’s economy that becomes
(C) lack of math skills is so large as to be difficult to absorb them into a city’s economy that becomes
(D) are lacking so much in math skills as to be difficult to absorb into a city’s economy becoming
(E) are so lacking in math skills that it will be difficult to absorb them into a city economy becoming
673.Studies of circadian rhythms can be applied to problems ranging from chronic insomnia in frequent transcontinental air travelers to workers in all-night plants being inefficient.
(A) to workers in all-night plants being inefficient
(B) toward workers in all-night plants being inefficient
(C) to workers in all-night plants who are inefficient
(D) to combat worker inefficiency in all-night plants
(E) to decreased efficiency among workers in all-night plants
674.Studies of the human “sleep-wake cycle” have practical relevance for matters ranging from duty assignments in nuclear submarines and air-traffic control towers to the staff of shifts in 24-hour factories.
(A) to the staff of
(B) to those who staff
(C) to the staffing of
(D) and staffing
(E) and the staff of
675.Studies show that young people with higher-than-average blood pressure and their
families have a history of high blood pressure are more likely than others to develop a severe form of the condition.
(A) and their families have a history of high blood pressure
(B) whose families have a history of high blood pressure
(C) and a history of high blood pressure runs in the family
(D) whose families have a history of high blood pressure running in them
(E) with a history of high blood pressure running in their family
676.Sulfur dioxide, a major contributor to acid rain, is an especially serious pollutant because it diminishes the respiratory system’s ability to deal with all other pollutants.
(A) an especially serious pollutant because it diminishes the respiratory system’s ability to deal
(B) an especially serious pollutant because of diminishing the respiratory system’s
capability of dealing
(C) an especially serious pollutant because it diminishes the capability of the respiratory system in dealing
(D) a specially serious pollutant because it diminishes the capability of the respiratory system to deal
(E) a specially serious pollutant because of diminishing the respiratory system’s ability to deal
677.Sunspots, vortices of gas associated with strong electromagnetic activity, are visible as dark spots on the surface of the Sun but have never been sighted on the Sun’s poles or equator.
(A) are visible as dark spots on the surface of the Sun but have never been sighted on
(B) are visible as dark spots that never have been sighted on the surface of the Sun
(C) appear on the surface of the Sun as dark spots although never sighted at
(D) appear as dark spots on the surface of the Sun, although never having been sighted at
(E) appear as dark spots on the Sun’s surface, which have never been sighted on
678.Television programs developed in conjunction with the marketing of toys, which was once prohibited by federal regulations, are thriving in the free market conditions permitted by the current Federal Communications Commission.
(A) Television programs developed in conjunction with the marketing of toys, which was once prohibited by federal regulations, are
(B) Television programs developed in conjunction with the marketing of toys, a practice that federal regulations once prohibited, is
(C) Developing television programs in conjunction with the marketing of toys, as once prohibited by federal regulations, is
(D) Federal regulations once prohibited developing television programs in conjunction with the marketing of toys, but they are
(E) Federal regulations once prohibited developing television programs in conjunction with the marketing of toys, but such programs are
679.Temporary-employment agencies benefit not only from the increasing demand for clerical workers but also the higher profits made when highly paid professionals are placed, requests for whom have increased in the recent wave of corporate takeovers.
(A) the higher profits made when highly paid professionals are placed, requests for whom
(B) the higher profits that are made in the placement of highly paid professionals, requests for whom
(C) from the requests for highly paid professionals, who make higher profits for the
agencies when placed and whose requests
(D) from highly paid professionals, whose placement makes higher profits for the agencies and whose requests
(E) from the higher profits made in placing highly paid professionals, requests for whom
680.Tenor George Shirley sang more than 20 leading roles at the Metropolitan Opera since having his debut there as Fernando in Cosifan tulle on October 24, 1961.
(A) sang more than 20 leading roles at the Metropolitan Opera since having
(B) sang more than 20 leading roles at the Metropolitan Opera since having had
(C) has sung more than 20 leading roles at the Metropolitan Opera since
(D) has sung more than 20 leading roles at the Metropolitan Opera after
(E) has sung more than 20 leading roles at the Metropolitan Opera subsequently to
681.Teratomas are unusual forms of cancer because they are composed of tissues such as tooth and bone not normally found in the organ in which the tumor appears.
(A) because they are composed of tissues such as tooth and bone
(B) because they are composed of tissues like tooth and bone that are
(C) because they are composed of tissues, like tooth and bone, tissues
(D) in that their composition, tissues such as tooth and bone, is
(E) in that they are composed of tissues such as tooth and bone, tissues
682.Textbooks for the used book sale should be in good condition and should have no writing in them or be underlined.
(A) and should have no writing in them or be underlined
(B) and should not have writing in them or not be underlined
(C) and contain no writing or underlining
(D) without containing writing nor be underlined
(E) without having any writing or no underlining in them
683.That educators have not anticipated the impact of microcomputer technology can hardly be said that it is their fault: Alvin Toffler, one of the most prominent students of the future, did not even mention microcomputers in Future Shock, published in 1970.
(A) That educators have not anticipated the impact of microcomputer technology can hardly be said that it is their fault
(B) That educators have not anticipated the impact of microcomputer technology can hardly be said to be at fault
(C) It can hardly be said that it is the fault of educators who have not anticipated the impact of microcomputer technology
(D) It can hardly be said that educators are at fault for not anticipating the impact of microcomputer technology
(E) The fact that educators are at fault for not anticipating the impact of microcomputer technology can hardly be said
684.That the new managing editor rose from the publication’s “soft” news s to a leadership position is more of a landmark in the industry than her being a woman.
(A) her being a woman
(B) being a woman is
(C) her womanhood
(D) that she was a woman
(E) that she is a woman
685.The 19th-century proponents of the school of thought known as mechanism held that life process are not the products of some mysterious life force, but are the same chemical and physical processes that operate in inorganic systems, which is still a subject of debate between biologists today.
(A) systems, which is still a subject of debate between
(B) systems, which are still debated between
(C) systems still debated among
(D) systems, a theory still debated by
(E) systems, a theory still debated between
686.The aim of the new regulations is to make it easier for prospective homeowners to have funds available for energy improvement in their new houses.
(A) to make it easier for prospective homeowners to have
(B) to make easier for prospective homeowners the having of
(C) making it easier for prospective homeowners so they can have
(D) that prospective homeowners more easily can have
(E) for prospective homeowners to more easily have
687.The airline industry is cutting its lowest discount fares more widely, more
substantially, and earlier this year than it normally does at the end of the summer, a during which travel usually decreases and the industry uses some lower fares for the attraction of passengers.
(A) during which travel usually decreases and the industry uses some lower fares for the attraction of
(B) during which travel usually decreases and therefore the industry will use some lower fares for the attraction of
(C) in which travel usually decreases and in which the industry therefore uses some lower fares attracting
(D) when travel usually decreases and the industry uses some lower fares to attract
(E) when travel usually decreases and therefore the industry will use lower fares for the attraction of
688.The American Medical Association has argued that the rapidly rising costs associated with malpractice litigation are driving doctors from the profession and that reform of the tort system is imperative for bringing malpractice insurance premiums under control.
(A) that reform of the tort system is imperative for bringing malpractice insurance premiums
(B) that reform of the tort system is imperative if malpractice insurance premiums are to be brought
(C) that reform of the tort system is imperative to bring malpractice insurance premiums
(D) reform of the tort system is necessary in bringing malpractice insurance premiums
(E) the tort system needs to be reformed so that malpractice insurance premiums are brought
689.The animosity between those who regulate and those who are regulated, never more
pronounced than in recent debates over environmentalism and pollution control.
(A) The animosity between those who regulate and those who are regulated, never
(B) The animosity between those who regulate and those who are regulated, never being
(C) The animosity between those who regulate and those who are regulated has never been
(D) Between those who regulate and those who are regulated, such animosity was never
(E) Between those who regulate and between those regulated, such animosity has never been
690.The argument that the dominant form of family structure today is not the nuclear family, but rather it is a family which is modified and extended, is based on a number of facts: the existence of three-generational families, the amount of vertical and horizontal communication between family subunits, and the extent to which family members offer assistance to one another.
(A) family, but rather it is a family which is modified and extended, is based on a number of facts: the existence of three-generational families
(B) family, but is instead a family that is modified and extended, is based on a number of facts: three-generational families exist
(C) family, which has been modified and extended, is based on a number of facts: the
existence of three-generational families
(D) family but a modified extended family is based on a number of facts: the existence of three-generational families
(E) family, but also a modified and extended family, is based on a number of facts:
three-generational families exist
691.The aristocratic values expressed in the writings of Marguerite Yourcenar place her within the French classical tradition, as does her passionate interest in history, particularly Roman history.
(A) as does
(B) so do
(C) as do
(D) so is the case with
(E) similarly, does
692.The attorney turned down the law firm’s offer of a position because she suspected that it was meant merely to fill an affirmative action quota with no commitment to minority hiring and eventually promoting.
(A) quota with no commitment to minority hiring and eventually promoting
(B) quota, having no commitment to minority hiring and eventually promoting
(C) quota and did not reflect a commitment to minority hiring and eventual promotion
(D) quota, not reflecting a commitment to minority hiring and eventual promotion
(E) quota, not one that reflected that minority hiring and eventual promotion was a
commitment
693.The Audubon Society and other conservation groups, concerned over what they have
perceived to be the serious threatening of the environment as posed by the policies of the government, are preparing for a major political effort.
(A) have perceived to be the serious threatening of the environment as
(B) perceived as the serious threat to the environment as
(C) perceive being the serious threat to the environment
(D) are perceiving as the serious threatening of the environment
(E) perceive as the serious threat to the environment
694.The auto industry has experienced one of its most significant trends in the last 50 years, which is the migration of motorists from passenger cars to minivans, sport utility vehicles, and pickups.
(A) The auto industry has experienced one of its most significant trends in the last 50 years, which is
(B) Of the trends the auto industry experienced in the last 50 years has been one of the most significant
(C) In the last 50 years, one of the most significant trends that the auto industry has been experiencing has been
(D) One of the most significant trends that the auto industry has experienced in the last 50 years is
(E) In the last 50 years, the auto industry experienced one of the most significant trends that it has had, that of
695.The average weekly wage nearly doubled in the 1970’s, rising from $114 to $220, yet the average worker ended the decade with a decrease in what their pay may buy.
(A) with a decrease in what their pay may buy
(B) with what was a decrease in what they were able to buy
(C) having decreased that which they could buy
(D) decreasing in purchasing power
(E) with a decrease in purchasing power
696.The Baldrick Manufacturing Company has for several years followed a policy aimed at decreasing operating costs and improving the efficiency of its distribution system.
(A) aimed at decreasing operating costs and improving
(B) aimed at the decreasing of operating costs and to improve
(C) aiming at the decreasing of operating costs and improving
(D) the aim of which is the decreasing of operating costs and improving
(E) with the aim to decrease operating costs and to improve
697.The bank acknowledged that they are and will continue to experience difficulties as it attempts to deal with the precipitous fall of the dollar against the yen and the dislocations reflected in the stock market decline.
(A) they are and will continue to experience difficulties as it attempts
(B) they are and will continue to experience difficulties as they attempt
(C) it is and will continue to experience difficulties as it attempts
(D) it is experiencing and will continue to experience difficulties as they make an attempt
(E) its difficulties are likely to continue as it attempts
698.The bank holds $3 billion in loans that are seriously delinquent or in such trouble that they do not expect payments when due.
(A) they do not expect payments when
(B) it does not expect payments when it is
(C) it does not expect payments to be made when they are
(D) payments are not to be expected to be paid when
(E) payments are not expected to be paid when they will be
699.The black hole has entered the popular imagination as an object too massive that neither light nor matter can escape its gravitational pull.
(A) too massive that neither light nor matter can escape its
(B) too massive for either allowing light or matter to escape its
(C) massive enough that either light or matter cannot escape their
(D) so massive that neither light nor matter could escape their
(E) so massive that neither light nor matter can escape its
700.The boom in agricultural exports in the early 1970’s emptied United States grain bins and many were led to thinking that overproduction was now a problem of the past.
(A) many were led to thinking that overproduction was now
(B) many had been led to thinking of overproduction as if it were
(C) the thought this led to was that overproduction had become
(D) led many to the thought of overproduction as if it were
(E) led many to think that overproduction had become
701.The brain is something of a stimulus reduction system, a means to reduce, in order to comprehend, the nearly infinite amount of stimuli that reach the senses at any given moment.
(A) a means to reduce, in order to comprehend, the nearly infinite amount
(B) a means to reduce, in order to comprehend, the nearly infinite number
(C) the means of reducing for comprehending the nearly infinite number
(D) the means that reduces, in order to comprehend, the nearly infinite amount
(E) the means for reducing in order to comprehend the nearly infinite amount
702.The British Admiralty and the War Office met in March 1892 to consider a possible Russian attempt to seize Constantinople and how they would have to act militarily to deal with them.
(A) how they would have to act militarily to deal with them
(B) how to deal with them if military action would be necessary
(C) what would be necessary militarily for dealing with such an event
(D) what military action would be necessary in order to deal with such an event
(E) the necessity of what kind of military action in order to take for dealing with it
703.The British sociologist and activist Barbara Wootton once noted as a humorous example of income maldistribution that the elephant that gave rides to children at the Whipsnade Zoo was earning annually exactly what she then earned as director of adult education for London.
(A) that the elephant that gave rides to children at the Whipsnade Zoo was earning
(B) that the elephant, giving rides to children at the Whipsnade Zoo, had been earning
(C) that there was an elephant giving rides to children at the Whipsnade Zoo, and it earned
(D) the elephant that gave rides to children at the Whipsnade Zoo and was earning
(E) the elephant giving rides to children at the Whipsnade Zoo and that it earned
704.The brochure notes that in the seminar the importance that communication is a two-way process will be emphasized.
(A) importance that communication is a two-way process will be emphasized
(B) importance of communication as a two-way process will be emphasized
(C) importance of communication being a two-way process will be the emphasis
(D) fact will be emphasized that communication is a two-way process and of importance
(E) emphasis will be that communication being a two-way process is important
705.The cameras of the Voyager II spacecraft detected six small, previously unseen moons circling Uranus, which doubles to twelve the number of satellites now known as orbiting the distant planet.
(A) which doubles to twelve the number of satellites now known as orbiting
(B) doubling to twelve the number of satellites now known to orbit
(C) which doubles to twelve the number of satellites now known in orbit around
(D) doubling to twelve the number of satellites now known as orbiting
(E) which doubles to twelve the number of satellites now known that orbit
706.The capital of Bosnia-Herzegovina, Sarajevo’s population on the eve of the First World War was 51,919.
(A) Bosnia-Herzegovina, Sarajevo’s population
(B) Bosnia-Herzegovina is Sarajevo, whose population
(C) Bosnia-Herzegovina is Sarajevo, with a population
(D) Bosnia-Herzegovina, Sarajevo having a population that
(E) Bosnia-Herzegovina, the population of Sarajevo
707.The caterpillar of the geometrid moth strikes when special tactile hairs on its body are disturbed, after capturing its prey, holds the victim so that it cannot escape.
(A) strikes when special tactile hairs on its body are disturbed,
(B) striking when special tactile hairs on its body are disturbed, but
(C) which strikes when special tactile hairs on its body are disturbed,
(D) which, striking when special tactile hairs on its body are disturbed,
(E) strikes when special tactile hairs on its body are disturbed and,
708.The Coast guard is conducting tests to see whether pigeons can be trained to help find survivors of wrecks at sea.
(A) to see whether pigeons can be trained to help find
(B) to see whether pigeons can be trained as help to find
(C) to see if pigeons can be trained for helping to find
(D) that see if pigeons are able to be trained in helping to find
(E) that see whether pigeons are able to be trained for help in finding
709.The Coast Guard recently redesigned channel markers in the Chesapeake Bay to accommodate large numbers of ospreys, whose huge nests formerly obstructed the lights.
(A) numbers of ospreys, whose huge nests
(B) numbers of ospreys, their huge nests
(C) amounts of ospreys, the huge nests of which
(D) nests of ospreys they
(E) numbers of ospreys, and their huge nests
710.The colorization of black-and-white films by computers is defended by those who own the film rights, for the process can mean increased revenues for them; many others in the film industry, however, contend that the technique degrades major works of art, which they liken to putting lipstick on a Greek statue.
(A) which they liken to putting lipstick on a Greek statue
(B) which they liken to a Greek statue with lipstick put on it
(C) which they liken to lipstick put on a Greek statue
(D) likening it to a Greek statue with lipstick put on it
(E) likening it to putting lipstick on a Greek statue
711.The Commerce Department announced that the economy grew during the second quarter at a 7.5 percent annual rate, while inflation eased when it might have been expected for it to rise.
(A) it might have been expected for it to rise
(B) it might have been expected to rise
(C) it might have been expected that it should rise
(D) its rise might have been expected
(E) there might have been an expectation it would rise
712.The commission acknowledged that no amount of money or staff members can ensure the safety of people who live in the vicinity of a nuclear plant, but it approved the installation because it believed that all reasonable precautions had been taken.
(A) no amount of money or staff members
(B) neither vast amounts of money nor staff members
(C) neither vast amounts of money nor numbers of staff members
(D) neither vast amounts of money nor a large staff
(E) no matter how large the staff or how vast the amount of money
713.The commission has directed advertisers to restrict the use of the word “natural” to foods that do not contain color or flavor additives, chemical preservatives, or nothing that has been synthesized.
(A) or nothing that has been
(B) or that has been
(C) and nothing that is
(D) or anything that has been
(E) and anything
714.The commission proposed that funding for the park’s development, which could be open to the public early next year, is obtained through a local bond issue.
(A) that funding for the park’s development, which could be open to the public early next year, is
(B) that funding for development of the park, which could be open to the public early next year, be
(C) funding for the development of the park, perhaps open to the public early next year, to be
(D) funds for the park’s development, perhaps open to the public early next year, be
(E) development funding for the park, which could be open to the public early next year, is to be
715.The commissior’s office of compliance, inspections, and investigations plans to
intensify its scrutiny of stock analysts to investigate not only whether research is an independent function at brokerage firms, but also whether conflicts result when analysts own the stocks they write about or when they are paid for their work by a firm’s investment.
(A) to investigate not only whether research is an independent function at brokerage firms, but also whether conflicts result when analysts own the stocks they write about or when they are
(B) to investigate not only whether research is an independent function at brokerage firms, but also if conflicts result when analysts own the stocks they write about or they are
(C) to not only investigate whether or not research is an independent function at brokerage firms, but also if conflicts result when analysts own the stocks they write about or are
(D) not only to investigate whether or not research is an independent function at brokerage firms, but also whether conflicts result when analysts own the stocks they write about or are
(E) not only to investigate whether research is an independent function at brokerage firms, but also whether conflicts result when analysts own the stocks they write about or when
716.The company announced that its profits declined much less in the second quarter than analysts had expected it to and its business will improve in the second half of the year.
(A) had expected it to and its business will improve
(B) had expected and that its business would improve
(C) expected it would and that it will improve its business
(D) expected them to and its business would improve
(E) expected and that it will have improved its business
717.The company is negotiating to sell its profitable credit card subsidiary, which it plans to use money from to acquire some of the mortgage-servicing operations that are being sold by troubled savings institutions.
(A) subsidiary, which it plans to use money from
(B) subsidiary, from which it plans to use money
(C) subsidiary, and it plans the use of money from that
(D) subsidiary and plans to use money from that sale
(E) subsidiary and plans the use of money from that sale
718.The complex tax dispute between the Covered Bridge Mall and Harris Township is not likely to be adjudicated for several years, and, in the mean, both sides are intent on creating difficulties for the other.
(A) both sides are intent on creating difficulties for the other
(B) both sides are intent on creating difficulties for each other
(C) each side is intent on creating difficulties for the other
(D) each side is intent on creating difficulties for one another
(E) the sides are both intent on creating difficulties for each other
719.The computer software being designed for a project studying Native American access to higher education will not only meet the needs of that study, but also has the versatility and power of facilitating similar research endeavors.
(A) but also has the versatility and power of facilitating
(B) but also have the versatility and power to facilitate
(C) but it also has the versatility and power to facilitate
(D) and also have the versatility and power of facilitating
(E) and it also has such versatility and power that it can facilitate
720.The concept of the grand jury dates from the twelfth century, when Henry II of England ordered panels of common citizens should prepare lists of who were their communities’ suspected criminals.
(A) should prepare lists of who were their communities’ suspected criminals
(B) would do the preparation of lists of their communities’ suspected criminals
(C) preparing lists of suspected criminals in their communities
(D) the preparing of a list of suspected criminals in their communities
(E) to prepare lists of suspected criminals in their communities
721.The current administration, being worried over some foreign trade barriers being removed and our exports failing to increase as a result of deep cuts in the value of the dollar, has formed a group to study ways to sharpen our competitiveness.
(A) being worried over some foreign trade barriers being removed and our exports failing
(B) worrying over some foreign trade barriers being removed, also over the failure of our exports
(C) worried about the removal of some foreign trade barriers and the failure of our exports
(D) in that they were worried about the removal of some foreign trade barriers and also about the failure of our exports
(E) because of its worry concerning the removal of some foreign trade barriers, also
concerning the failure of our exports
722.The data collected by weather airplanes that fly into the heart of a hurricane are useful mainly for gauging the storm’s structure and strength, not for the speed and the path of their movement.
(A) not for the speed and the path of their movement
(B) not for the speed and path of its movement
(C) not the speed and path of its movement
(D) and not the speed and path of their movements
(E) and not for the speed and the path of its movements
723.The debate over bilingual education centers on the issue of whether the United States should foster the idea of single common language, an idea, some believe, that has in the past been crucial in binding diverse constituencies together.
(A) been crucial in binding diverse constituencies together
(B) been crucial as a binding together of diverse constituencies
(C) been crucial to bind together constituencies that are diverse
(D) become crucial in binding together diverse constituencies
(E) become crucial to bind together constituencies that are diverse
724.The decision by one of the nation’s largest banks to admit to $3 billion in potential losses on foreign loans could mean less lending by commercial banks to developing countries and increasing the pressure on multigovernment lenders to supply the funds.
(A) increasing the pressure
(B) the increasing pressure
(C) increased pressure
(D) the pressure increased
(E) the pressure increasing
725.The decisions of John Marshall, the fourth chief justice, have had a greater influence than any chief justice in history.
(A) than any chief justice in history
(B) historically than any other chief justice
(C) than have those of any chief justice in history
(D) in history as any other chief justice has had
(E) than those of any other chief justice in history
726.The delinquency rates on mortgages for office buildings, hotels, shopping malls, and other commercial properties held by the nation’s insurance companies in recent months have increased sharply, leading to predictions that foreclosures on these types of loan could double over the next three years.
(A) in recent months have increased sharply, leading to predictions that foreclosures on these types of loan could double over the next three years
(B) have increased sharply in recent months, leading to predictions that foreclosures on these types of loans could double over the next three years
(C) have increased sharply in recent months, leading to predictions that doubling of
foreclosures on these types of loans could occur over the next three years
(D) has increased sharply in recent months, and lead to predictions that over the next three years doubling of foreclosures on this type of loan could occur
(E) in recent months has increased sharply, and this leads to predictions that foreclosures on that type of loan in the next three years could double
727.The demand for airplane mechanics is expected to grow about ten percent a year in the next decade, largely because new federal rules calling for major modifications to older planes and because the airlines are adding hundreds of new jets.
(A) new federal rules calling for major modifications to older planes
(B) new federal rules, which call for major modifications to older planes
(C) new federal rules call for major modifications to older planes
(D) major modifications to older planes called for by new federal rules
(E) major modifications to older planes are called for according to new federal rules
728.The department defines a private passenger vehicle as one registered to an individual with a gross weight of less than 8,000 pounds.
(A) as one registered to an individual with a gross weight of less than 8,000 pounds
(B) to be one that is registered to an individual with a gross weight of less than 8,000 pounds
(C) as one that is registered to an individual and that has a gross weight of less than 8,000 pounds
(D) to have a gross weight less than 8,000 pounds and being registered to an individual
(E) as having a gross weight of less than 8,000 pounds and registered to an individual
729.The development of a new jumbo rocket that is expected to carry the United States into its next phase of space exploration will be able to deliver a heavier load of instruments into orbit than the space shuttle and at a lower cost.
(A) The development of a new jumbo rocket that is expected to carry the United States into its next phase of space exploration will be able to deliver a heavier load of instruments into orbit than the space shuttle and at a lower cost.
(B) The development of a new jumbo rocket is expected to carry the United States into its next phase of space exploration and be able to deliver a heavier load of instruments into orbit at a lower cost than the space shuttle.
(C) The new development of a jumbo rocket, which is expected to carry the United States into its next phase of space exploration, will be able to deliver a heavier load of instruments into orbit at a lower cost than the space shuttle.
(D) A newly developed jumbo rocket, which is expected to carry the United States into its next phase of space exploration, will be able to deliver a heavier load of instruments into orbit than the space shuttle can, and at a lower cost.
(E) A newly developed jumbo rocket, which is expected to carry the United States into its next phase of space exploration, will be able to deliver a heavier load of instruments into orbit than the space shuttle and to cost less.
730.The diet of the ordinary Greek in classical s was largely vegetarian—vegetables, fresh cheese, oatmeal, and meal cakes, and meat rarely.
(A) and meat rarely
(B) and meat was rare
(C) with meat as rare
(D) meat a rarity
(E) with meat as a rarity
731.The direction in which the Earth and the other solid planets—Mercury, Venus, and
Mars—spins were determined from collisions with giant celestial bodies in the early history of the Solar System.
(A) spins were determined from
(B) spins were determined because of
(C) spins was determined through
(D) spin was determined by
(E) spin was determined as a result of
732.The distinctive hereditary tartans that are alleged to be worn since antiquity by members of the Highland clans were in fact designed by Scottish woolen manufacturers in the eighteenth and nineteenth centuries.
(A) that are alleged to be worn
(B) alleged to have been worn
(C) that are worn, it was alleged
(D) alleged as having been worn
(E) that are worn, allegedly
733.The distribution of mass within the core of the Earth, like the mantle that surrounds the core, has been deduced from the orbital behavior of the Earth and the motions of satellites controlled by the gravity of the Earth.
(A) the mantle that surrounds the core
(B) that within the mantle surrounding the core
(C) that of the mantle surrounding the core
(D) the mantle the core surrounds
(E) the distribution of mass within the mantle that surrounds the core
734.The domesticated camel, which some scholars date around the twelfth century B.C., was the key to the development of the spice trade in the ancient world.
(A) The domesticated camel, which some scholars date
(B) The domesticated camel, which some scholars having thought to occur
(C) Domesticating the camel, dated by some scholars at
(D) The domestication of the camel, thought by some scholars to have occurred
(E) The camel’s domestication, dated by some scholars to have been
735.The earnings of women are well below that of men in spite of educational differences that are diminishing between the sexes.
(A) well below that of men in spite of educational differences that are diminishing
(B) much below that of men’s despite educational differences diminishing
(C) much below men in spite of diminishing educational differences
(D) well below those of men in spite of diminishing educational differences
(E) below men’s despite their educational differences that are diminishing
736.The economic forces which may affect the new public offering of stock include sudden downturns in the market, hedging and other investor strategies for preventing losses, loosening the interest rates in Washington, and fearing that the company may still be undercapitalized.
(A) loosening the interest rates in Washington, and fearing that the company may still be undercapitalized
(B) loosening the interest rates in Washington, and a fear of the company still being undercapitalized
(C) a loosening of the interest rates in Washington, and fearing that the company may still be undercapitalized
(D) a loosening of the interest rates in Washington, and a fear of the still
undercapitalized company
(E) a loosening of the interest rates in Washington, and a fear that the company may still be undercapitalized
737.The efforts of the bipartisan committee were marked as much by frustration as it was by success.
(A) as it was by
(B) and also by
(C) as by
(D) and equally by
(E) as there was
738.The Emperor Augustus, it appears, commissioned an idealized sculptured portrait, the features of which are so unrealistic as to constitute what one scholar calls an “artificial face.”
(A) so unrealistic as to constitute
(B) so unrealistic they constituted
(C) so unrealistic that they have constituted
(D) unrealistic enough so that they constitute
(E) unrealistic enough so as to constitute
739.The end of the eighteenth century saw the emergence of prize-stock breeding, with individual bulls and cows receiving awards, fetching unprecedented prices, and excited enormous interest whenever they were put on show.
(A) excited
(B) it excited
(C) exciting
(D) would excite
(E) it had excited
740.The endurance and consistency of baseball star Lou Gehrig, known as “The Iron Horse,” are legendary.
(A) The endurance and consistency of baseball star Lou Gehrig, known as “The Iron Horse,” are legendary.
(B) The endurance and consistency of Lou Gehrig, a baseball star known as “The Iron Horse,” is legendary.
(C) Known as “The Iron Horse,” the endurance and consistency of Lou Gehrig, the baseball star, is legendary.
(D) The reason baseball star Lou Gehrig is known as “The Iron Horse” is because of his legendary endurance and consistency.
(E) Known as “The Iron Horse,” baseball star Lou Gehrig’s endurance and consistency are legendary.
741.The energy source on Voyager 2 is not a nuclear reactor, in which atoms are actively broken apart; rather a kind of nuclear battery that uses natural radioactive decay to produce power.
(A) apart; rather
(B) apart, but rather
(C) apart, but rather that of
(D) apart, but that of
(E) apart; it is that of
742.The evolution of the technology of microelectronics over the past decade has been so rapid that it is sometimes called a revolution.
(A) has been so rapid that it is sometimes
(B) has been rapid enough sometimes to be
(C) has been rapid enough it is sometimes
(D) is so rapid it has sometimes been
(E) is so rapid that it is sometimes
743.The exhibition of art from Nubians, the site of a Black civilization that goes back to the fourth millennium B.C., makes clear the Nubians combined artistic elements from Egypt to that of sub-Saharan Africa.
(A) the Nubians combined artistic elements from Egypt to that
(B) that the Nubians combined artistic elements from Egypt to that
(C) the Nubians combined artistic elements from Egypt with that
(D) that the Nubians combined artistic elements from Egypt with those
(E) that Nubians combined artistic elements from Egypt and those
744.The exigencies of dramatic art, as shown even by the history plays of Shakespeare, makes the foreshortening of dramatized historical events inevitable.
(A) makes the foreshortening of dramatized historical events inevitable
(B) made dramatized historical events inevitably foreshortened
(C) make the foreshortening of dramatized historical events inevitable
(D) has inevitably foreshortened dramatized historical events
(E) inevitably foreshortens dramatized historical events
745.The expected rise in the price of oil could be a serious impact to industrialized nations and severely diminish the possibility to have an economy free of inflation.
(A) be a serious impact to industrialized nations and severely diminish the possibility to have
(B) seriously impact on industrialized nations and severely impede the possibility to have
(C) seriously impact on industrialized nations and severely impede the possibility of having
(D) have a serious impact on industrialized nations and severely impede the possibility to have
(E) have a serious impact on industrialized nations and severely diminish the possibility of having
746.The extraordinary diary of William Lyon Mackenzie King, prime minister of Canada for over twenty years, revealed that this most bland and circumspect of men was a mystic guided in both public and private life by omens, messages received at séances, and signs from heaven.
(A) that this most bland and circumspect of men was a mystic guided in both public and
(B) that this most bland and circumspect of men was a mystic and also guided both in public as well as
(C) this most bland and circumspect of men was a mystic and that he was guided in both public and
(D) this most bland and circumspect of men was a mystic and that he was guided in both public as well as
(E) this most bland and circumspect of men to have been a mystic and that he guided himself both in public as well as
747.The fastest of mammals, cheetah’s bodies are geared to accelerate from one to forty miles per hour in less than two seconds and reach speeds of seventy miles per hour.
(A) The fastest of mammals, cheetah’s bodies are geared to
(B) The fastest of mammals, the body of cheetah’s bodies is able to
(C) Faster than other mammals, the body of the cheetah is geared to
(D) The fastest of mammals, the cheetah can
(E) The cheetah, the fastest of mammals, have bodies that can
748.The fear of rabies is well founded; few people are known to recover from the disease after the appearance of the clinical symptoms.
(A) few people are known to recover from the disease after the appearance of the clinical symptoms
(B) few people are known to have recovered from the disease once the clinical symptoms have appeared
(C) there are few known people who have recovered from the disease once the clinical
symptoms have appeared
(D) after the clinical symptoms appear, there are few known people who have recovered from the disease
(E) recovery from the disease is known for only a few people after the clinical symptoms appear
749.The federal government requires hospitals to tell a Medicare patient of their legal right of challenge their discharge if they feel they are being sent home prematurely.
(A) hospitals to tell a Medicare patient of their
(B) hospital to tell Medicare patients that they have a
(C) hospitals to tell Medicare patients that there is a
(D) that hospitals tell a Medicare patient of their
(E) that hospitals tell a Medicare patient that they have a
750.The Federal Reserve Board’s reduction of interest rates on loans to financial
institutions is both an acknowledgement of past economic trends and an effort to influence their future direction.
(A) reduction of interest rates on loans to financial institutions is both an
acknowledgement of past economic trends and an effort
(B) reduction of interest rates on loans to financial institutions is an acknowledgement both of past economic trends as well as an effort
(C) reduction of interest rates on loans to financial institutions both acknowledge past economic trends and attempt
(D) reducing interest rates on loans to financial institutions is an acknowledgement both of past economic trends and an effort
(E) reducing interest rates on loans to financial institutions both acknowledge past
economic trends as well as attempt
751.The Federalist papers, a strong defense of the United States Constitution and important as a body of work in political science as well, represents the handiwork of three different authors.
(A) and important as a body of work in political science as well, represents
(B) as well as an important body of work in political science, represent
(C) and also a body of work of importance in political science is representing
(D) an important body of work in political science and has been representative of
(E) and as political science an important body of work too, represent
752.The figure of the jaguar, being a recurring symbol within Olmec art, is prominent among the hieroglyphics inscribed on a monument that was discovered in the Mexican state of Veracruz.
(A) being a recurring symbol within
(B) a symbol having recurred within
(C) a recurring symbol in
(D) having been a symbol that recurred in
(E) recurring as it is, a symbol in
753.The filibuster, a parliamentary device that slows the snail’s pace that prevails even in the best of s in congressional sessions and tests the endurance of everyone associated with it, seems more and more an anachronism in the age of telecommunications.
(A) sessions and tests the endurance of everyone associated with it, seems
(B) sessions and tests the endurance of everyone who is associated with it, seeming to be
(C) sessions, tests the endurance of everyone associated with it, seems
(D) sessions, that tests the endurance of everyone associated with it and seems
(E) sessions, testing the endurance of everyone associated with it and seeming
754.The financial crash of October 1987 demonstrated that the world’s capital markets are integrated more closely than never before and events in one part of the global village may be transmitted to the rest of the village—almost instantaneously.
(A) integrated more closely than never before and
(B) closely integrated more than ever before so
(C) more closely integrated as never before while
(D) more closely integrated than ever before and that
(E) more than ever before closely integrated as
755.The first decision for most tenants living in a building undergoing being converted to cooperative ownership is if to sign a no-buy pledge with the other tenants.
(A) being converted to cooperative ownership is if to sign
(B) being converted to cooperative ownership is whether they should be signing
(C) being converted to cooperative ownership is whether or not they sign
(D) conversion to cooperative ownership is if to sign
(E) conversion to cooperative ownership is whether to sign
756.The five hundred largest manufacturing firms in the United States produce goods that are worth almost as much as that of the four hundred thousand firms that remain.
(A) that are worth almost as much as that of the four hundred thousand firms that remain
(B) of a worth that is almost as much as that of the four hundred thousand firms that remain
(C) almost as much in worth as those of the remaining four hundred thousand firms
(D) almost as much in worth as that of the remaining four hundred thousand firms
(E) worth almost as much as those of the remaining four hundred thousand firms
757.The Forbidden City in Beijing, from which the emperors ruled by heavenly mandate, was a site which a commoner or foreigner could not enter without any permission, on pain of death.
(A) which a commoner or foreigner could not enter without any permission,
(B) which a commoner or foreigner could enter without any permission only
(C) which no commoner or foreigner could enter without permission,
(D) which, without permission, neither commoner or foreigner could only enter,
(E) which, to enter without permission, neither commoner or foreigner could do,
758.The golden crab of the Gulf of Mexico has not been fished commercially in great numbers, primarily on account of living at great depths—2,500 to 3,000 feet down.
(A) on account of living
(B) on account of their living
(C) because it lives
(D) because of living
(E) because they live
759.The Gorton-Dodd bill requires that a bank disclose to their customers how long they will delay access to funds from deposited checks.
(A) that a bank disclose to their customers how long they will delay access to funds from deposited checks
(B) a bank to disclose to their customers how long they will delay access to funds from a deposited check
(C) that a bank disclose to its customers how long it will delay access to funds from deposited checks
(D) a bank that it should disclose to its customers how long it will delay access to funds from a deposited check
(E) that banks disclose to customers how long access to funds from their deposited check is to be delayed
760.The group called the Teton Sioux inhabits parts of North and South Dakota; their
language and customs differ, however, from the Yankton, Sisseton, and Dakota Sioux.
(A) their language and customs differ, however, from
(B) its language and customs differ, however, from
(C) however, they have different language and customs than
(D) however, their language and customs differ from those of
(E) its language and customs differ, however, from those of
761.The growing demand for housing, traffic congestion, and longer commuting trips has all but eliminated the cost advantage of owning a house in many rural communities.
(A) The growing demand for housing, traffic congestion, and longer commuting trips has
(B) Traffic congestion, the growing demand for housing, and longer commuting trips has
(C) Longer commuting trips, traffic congestion, and the growing demand for housing has
(D) Traffic congestion, longer commuting trips, and the growing demand for housing have
(E) The growing demand for housing, as well as traffic congestion and longer commuting trips, have
762.The growth of the railroads led to the abolition of local s, which was determined by when the sun reached the observer’s meridian and differing from city to city, and to the establishment of regional s.
(A) which was determined by when the sun reached the observer’s meridian and differing
(B) which was determined by when the sun reached the observer’s meridian and which differed
(C) which were determined by when the sun reached the observer’s meridian and differing
(D) determined by when the sun reached the observer’s meridian and differed
(E) determined by when the sun reached the observer’s meridian and differing
763.The guiding principles of the tax plan released by the Treasury Department could have even a greater significance for the economy than the particulars of the plan.
(A) even a greater significance for the economy than
(B) a significance that is even greater for the economy than
(C) even greater significance for the economy than have
(D) even greater significance for the economy than do
(E) a significance even greater for the economy than have
764.The gyrfalcon, an Arctic bird of prey, has survived a close brush with extinction; its numbers are now five s greater than when the use of DDT was sharply restricted in the early 1970’s.
(A) extinction; its numbers are now five s greater than
(B) extinction; its numbers are now five s more than
(C) extinction, their numbers now fivefold what they were
(D) extinction, now with fivefold the numbers they had
(E) extinction, now with numbers five s greater than
765.The herbicide Oryzalin was still being produced in 1979, three years after the wives of workers producing the chemical in Rensselaer, New York, were found to have borne children with heart defects or miscarriages, and none of their pregnancies was normal.
(A) to have borne children with heart defects or miscarriages, and none of their pregnancies was
(B) to have had children born with heart defects or miscarriages, and none of the
pregnancies was
(C) either to have had children with heart defects or miscarriages, without any of their pregnancies being
(D) either to have had miscarriages or to have borne children with heart defects; none of the pregnancies was
(E) either to have had miscarriages or children born with heart defects, without any of their pregnancies being
766.The human growth hormone, made by the pituitary gland, is secreted during sleep in higher concentrations than when awake.
(A) during sleep in higher concentrations than when awake
(B) when sleeping in higher concentrations than waking hours
(C) in higher concentrations during sleeping than waking
(D) in higher concentrations during sleep than during waking hours
(E) in higher concentrations when asleep than when awake
767.The increased popularity and availability of televisions has led to the decline of regional dialects, language variations which originate from diverse ethnic and cultural heritages and perpetuated by geographic isolation.
(A) which originate from diverse ethnic and cultural heritages and perpetuated
(B) that originated from diverse ethnic and cultural heritages and perpetuated
(C) originated from diverse ethnic and cultural heritages and perpetuated
(D) originating from diverse ethnic and cultural heritages and perpetuated
(E) originating from diverse ethnic and cultural heritages and perpetuating
768.The investor who is uncertain about the future is more likely to put money into
blue-chip stocks or treasury bills than into gold.
(A) than into
(B) than they do
(C) than they are
(D) as into
(E) as
769.The Iroquois were primarily planters, but supplementing their cultivation of maize, squash, and beans with fishing and hunting.
(A) but supplementing
(B) and had supplemented
(C) and even though they supplemented
(D) although they supplemented
(E) but with supplementing
770.The key to control over the Eurasian steppes lay in the nomad’s ability to use the horse both as a means of transport but also as an effective military tool.
(A) but also as
(B) or as
(C) and as
(D) or
(E) and also
771.The labor agreement permits staff reductions through attrition with increased pension benefits and a special early-retirement program for speeding it up.
(A) attrition with increased pension benefits and a special early-retirement program for speeding it up
(B) attrition and provides increased pension benefits and a special early-retirement program to speed the attrition process
(C) attrition, which will be speeded up by providing increased pension benefits and a special early-retirement program
(D) attrition, which, by their providing increased pension benefits and a special
early-retirement program, will speed the process
(E) attrition, which provides increased pension benefits and a special early-retirement program for speeding the attrition process
772.The lack of complete historical records from the mid-to-late 1800’s have made some Black inventions difficult to trace to their originators.
(A) have made some Black inventions difficult to trace to their originators
(B) have made for difficulties in tracing some inventions by Blacks to their originators
(C) have made it difficult to trace some inventions by Blacks to their originators
(D) has made it difficult to trace some inventions to their Black originators
(E) has made it difficult in tracing some Black inventions to their originators
773.The last “wild” Indian in North America, according to anthropologist Alfred Kroeber, was the lone survivor of California’s lost Yahi tribe, which staggered out of the mountains near Lassen Peak in 1912, deep in mourning for the last of his companions, expecting to be butchered and eaten by white ranchers.
(A) which
(B) who
(C) that
(D) the survivor having
(E) having
774.The lawyer for the defense charged that she suspected the police of having illegally taped her confidential conversations with her client and then used the information obtained to find evidence supporting their murder charges.
(A) used the information obtained to find evidence supporting
(B) used such information as they obtained to find evidence supporting
(C) used the information they had obtained to find evidence that would support
(D) of using the information they had obtained to find evidence that would support
(E) of using such information as they obtained to find evidence that would be supportive of
775.The main reasons for the phenomenal growth of agribusiness over the last fifteen years are because of elevated land prices, the basing of retail marketing networks on produce imported from other states, and decreased competition from developers.
(A) because of elevated land prices, the basing of retail marketing networks
(B) a result of elevated land prices, retail marketing networks based
(C) that land prices are elevated, retail marketing networks are based
(D) elevated land prices, retail marketing networks based
(E) because land prices are elevated, the basing of retail marketing networks
776.The major areas of medicine in which lasers are effective is in the cutting and closing of blood vessels, and in the destruction of tumors.
(A) is in the cutting and closing of blood vessels, and in the destruction
(B) are the cutting and closing of blood vessels, and also the case of destroying
(C) are the cutting, closing of blood vessels, and in the destroying
(D) are the cutting and closing of blood vessels, and the destruction
(E) is in the cutting and closing of blood vessels, and the destroying
777.The man was always aware, sometimes proudly and sometimes resentfully, that he was a small-town Midwesterner who was thrust into a world that was dominated by wealthier, better-educated, and more polished people than him.
(A) who was thrust into a world that was dominated by wealthier, better-educated, and more polished people than him
(B) who had been thrust into a world that was dominated by more wealthy, educated, and polished people than him
(C) who had been thrust into a world dominated by wealthier, better-educated, and people more polished than he was
(D) thrust into a world dominated by more wealthy, educated, and polished people than him
(E) thrust into a world dominated by wealthier, better-educated, and more polished people than he
778.The medieval scholar made almost no attempt to investigate the anatomy of plants, their mechanisms of growth, nor the ways where each was related to the other.
(A) nor the ways where each was related to the other
(B) nor how each was related to some other
(C) or the way where one is related to the next
(D) or the ways in which they are related to one another
(E) or the ways that each related to some other
779.The metabolic rate of sharks is low compared with the rates of most other fishes.
(A) with the rates of most other fishes
(B) to most other fishes’ rate
(C) to that of rates for most other fishes
(D) to most other fishes
(E) with most other fishes
780.The mistakes children make in learning to speak tell linguists more about how they learn language than the correct forms they use.
(A) how they learn language than
(B) how one learns language than
(C) how children learn language than do
(D) learning language than
(E) their language learning than do
781.The modernization program for the steel mill will cost approximately 51 million dollars, which it is hoped can be completed in the late 1980’s.
(A) The modernization program for the steel mill will cost approximately 51 million dollars, which it is hoped can be completed in the late 1980’s.
(B) The modernization program for the steel mill, hopefully completed in the late 1980’s, will cost approximately 51 million dollars.
(C) Modernizing the steel mill, hopefully to be completed in the late 1980’s, will cost approximately 51 million dollars.
(D) The program for modernizing the steel mill, which can, it is hoped, be completed in the late 1980’s and cost approximately 51 million dollars.
(E) Modernizing the steel mill, a program that can, it is hoped, be completed in the late 1980’s, will cost approximately 51 million dollars.
782.The most common reasons for an employee’s unwillingness to accept a transfer are that mortgage rates are high, housing in the new location costs more, and the difficulty of selling the old home.
(A) that mortgage rates are high, housing in the new location costs more, and the difficulty of selling the old home
(B) that mortgage rates are high, housing in the new location costs more, and that it is difficult to sell the old home
(C) high mortgage rates, the greater cost of housing in the new location, and that the old home is difficult to sell
(D) high mortgage rates, the greater cost of housing in the new location, and it is
difficult to sell the old home
(E) high mortgage rates, the greater cost of housing in the new location, and the difficulty of selling the old home
783.The most favorable locations for the growth of glaciers, rather than being the cold, dry polar regions, would be instead the cool, moist middle latitudes, where there is abundant precipitation and where it is cold enough to allow some snow to accumulate year by year.
(A) glaciers, rather than being the cold, dry polar regions, would be instead the cool, moist middle latitudes
(B) glaciers are not the cold, dry polar regions but the cool, moist middle latitudes
(C) glaciers are the cool, moist middle latitudes rather than the cold, dry polar regions
(D) glaciers, instead of being the cold, dry polar regions, would be the cool, moist middle latitudes
(E) glaciers are, instead of the cold, dry polar regions, rather the cool, moist middle latitudes
784.The nation’s three military academies have seen a dramatic rise in applications, one fueled by a resurgence of patriotism, increasing tuition costs at private colleges, and improved recruiting by the academies.
(A) one fueled by a resurgence of patriotism, increasing tuition costs at private colleges, and improved recruiting by the academies
(B) one fueled by a resurgence of patriotism, tuition costs that have increased at private colleges, and academies improving their recruiting
(C) one fueled by a resurgence of patriotism, private colleges that increased their tuition costs, and recruiting improvements by the academies
(D) fueled by a resurgence of patriotism, tuition costs increasing at private colleges, and academies improving their recruiting
(E) fueled by a resurgence of patriotism, increasing tuition costs at private colleges, and academies improving their recruiting
785.The National Labor Relations Act expressly forbids unions from engaging in secondary boycotts against companies not directly involved in a labor dispute.
(A) unions from engaging in
(B) the engagement by unions of
(C) unions to engage in
(D) unions from becoming engaged with
(E) that unions engage upon
786.The National Transportation Safety Board has recommended the use of fail-safe mechanisms on airliner cargo door latches assuring the doors are properly closed before takeoff and to prevent them from popping open in flight.
(A) assuring the doors are properly closed
(B) for the assurance of proper closing
(C) assuring proper closure
(D) to assure closing the doors properly
(E) to assure that the doors are properly closed
787.The nephew of Pliny the Elder wrote the only eyewitness account of the great eruption of Vesuvius in two letters to the historian Tacitus.
(A) The nephew of Pliny the Elder wrote the only eyewitness account of the great eruption of Vesuvius in two letters to the historian Tacitus.
(B) To the historian Tacitus, the nephew of Pliny the Elder wrote two letters, being the only eyewitness accounts of the great eruption of Vesuvius.
(C) The only eyewitness account is in two letters by the nephew of Pliny the Elder writing to the historian Tacitus an account of the great eruption of Vesuvius.
(D) Writing the only eyewitness account, Pliny the Elder’s nephew accounted for the great eruption of Vesuvius in two letters to the historian Tacitus.
(E) In two letters to the historian Tacitus, the nephew of Pliny the Elder wrote the only eyewitness account of the great eruption of Vesuvius.
788.The new contract forbids a strike by the transportation union.
(A) forbids a strike by the transportation union
(B) forbids the transportation union from striking
(C) forbids that there be a strike by the transportation union
(D) will forbid the transportation union from striking
(E) will forbid that the transportation union strikes
789.The new regulations mandate that a company allows their retiring employees who would otherwise lose group health care coverage to continue the same insurance at their own expense for a specified period.
(A) that a company allows their retiring employees who would otherwise lose group health care coverage to continue
(B) companies to allow their retiring employees who would otherwise lose group health care coverage that they can continue
(C) that a company allow its retiring employees who would otherwise lose group health care coverage to continue
(D) companies allowing a retiring employee whose group health care coverage would otherwise be lost the continuation of
(E) companies to allow a retiring employee whose group health care coverage would otherwise be lost he continuation of
790.The number of mountain gorillas is declining with such rapidity that the population is one-half in the twenty years between a count made by George Schaller in 1960 and the one made by Dian Fossey in 1980.
(A) with such rapidity that the population is one-half
(B) with such rapidity that the population was one-half
(C) so rapidly the population divided in half
(D) so rapidly that the population was halved
(E) in such rapidity that the population is halved
791.The number of undergraduate degrees in engineering awarded by colleges and universities in the United States increased by more than twice from 1978 to 1985.
(A) increased by more than twice
(B) increased more than two s
(C) more than doubled
(D) was more than doubled
(E) had more than doubled
792.The odds are about 4 to 1 against surviving a takeover offer, and many business
consultants therefore advise that a company’s first line of defense in eluding offers like these be to even refuse to take calls from likely corporate raiders.
(A) that a company’s first line of defense in eluding offers like these be to even refuse
(B) that a company’s first line of defense in eluding such offers be to refuse even
(C) a company defending itself against offers of this kind that, as a first line of defense, they should even refuse
(D) companies which are defending themselves against such an offer that, as a first line of defense, they should even refuse
(E) that the first line of defense for a company who is eluding offers like these is the refusal even
793.The official languages are of India, Hindi, and of Pakistan, Urdu, but neither are spoken by a majority of the population.
(A) The official languages are of India, Hindi, and of Pakistan, Urdu, but neither are
(B) The official languages are of India, Hindi, and of Pakistan, Urdu, but neither is
(C) Officially, the languages are Hindi for India and for Pakistan, Urdu, but neither are
(D) The official language of India is Hindi, and that of Pakistan is Urdu, but neither is
(E) The official language of India is Hindi, and Urdu in Pakistan, but none is
794.The Olympic Games helped to keep peace among the pugnacious states of the Greek world in that a sacred truce was proclaimed during the festival’s month.
(A) world in that a sacred truce was proclaimed during the festival’s month
(B) world, proclaiming a sacred truce during the festival’s month
(C) world when they proclaimed a sacred truce for the festival month
(D) world, for a sacred truce was proclaimed during the month of the festival
(E) world by proclamation of a sacred truce that was for the month of the festival
795.The only way for growers to salvage frozen citrus is to process them quickly into juice concentrate before they rot when warmer weather returns.
(A) to process them quickly into juice concentrate before they rot when warmer weather returns
(B) if they are quickly processed into juice concentrate before warmer weather returns to rot them
(C) for them to be processed quickly into juice concentrate before the fruit rots when warmer weather returns
(D) if the fruit is quickly processed into juice concentrate before they rot when warmer weather returns
(E) to have it quickly processed into juice concentrate before warmer weather returns and rots the fruit
796.The ordinance is intended to force householders to separate such hazardous waste like pesticides, batteries, fertilizers, and oil-based paints from the general stream of household trash.
(A) to separate such hazardous waste like
(B) that they should separate such hazardous waste like
(C) separating such hazardous wastes as
(D) that they should separate such hazardous wastes as
(E) to separate such hazardous wastes as
797.The original building and loan associations were organized as limited life funds, whose members made monthly payments on their share subscriptions, then taking turns drawing on the funds for home mortgages.
(A) subscriptions, then taking turns drawing
(B) subscriptions, and then taking turns drawing
(C) subscriptions and then took turns drawing
(D) subscriptions and then took turns, they drew
(E) subscriptions and then drew, taking turns
798.The original employees hired, who had been there over twenty years, were fiercely loyal to the firm, and it offered no retirement benefits or profit sharing to any employees.
(A) and it offered no
(B) and it offered neither
(C) still it offered no
(D) though it offered no
(E) though it offered neither
799.The owner of Steele’s Grocery in Osage, Ohio, saved $600 monthly on heat during the winter by putting all his refrigerator air compressors together in an insulated compartment, then installing two fans and a duct that carried waste heat from the compressors into the main part of the store.
(A) then installing two fans and a duct that carried
(B) then he installing two fans and a duct that carried
(C) then two fans were installed with a duct that carried
(D) installing two fans, and then carrying through a duct
(E) installing two fans, and a duct carrying
800.The Parthenon was a church from 1204 until 1456, when Athens was taken by General Mohammed the Conqueror, the Turkish sultan, who established a mosque in the building and used the Acropolis as a fortress.
(A) who established a mosque in the building and used the Acropolis as
(B) who, establishing a mosque in the building, used the Acropolis like
(C) who, when he had established a mosque in the building, used the Acropolis like
(D) who had established a mosque in the building, using the Acropolis to be
(E) establishing a mosque in the building and using the Acropolis as
801.The pattern of whisker spot on the face of a male lion, like human fingerprints, are a lifelong means of identification, since they are both unique and unchanging.
(A) like human fingerprints, are a lifelong means of identification, since they are both unique and unchanging
(B) like human fingerprints, is a lifelong means of identification, since it is both unique and unchanging
(C) like human fingerprints, is a means of identification for life, being both unique and unchanging
(D) since they are both unique and unchanging, are, like human fingerprints, are a means of identification for life
(E) both unique and unchanging, are, like human fingerprints, a lifelong means of
identification
802.The peaks of a mountain range, acting like rocks in a streambed, produce ripples in the air flowing over them: the resulting flow pattern, with crests and troughs that remain stationary although the air that forms them is moving rapidly, are known as “standing waves.”
(A) crests and troughs that remain stationary although the air that forms them is moving rapidly, are
(B) crests and troughs that remain stationary although they are formed by rapidly moving air, are
(C) crests and troughs that remain stationary although the air that forms them is moving rapidly, is
(D) stationary crests and troughs although he air that forms them is moving rapidly, are
(E) stationary crests and troughs although they are formed by rapidly moving air, is
803.The period when the great painted caves at Lascaux and Altamira were occupied by Upper Paleolithic people has been established by carbon-14 dating, but what is much more difficult to determine are the reason for their decoration, the use to which primitive people put the caves, and the meaning of the magnificently depicted animals.
(A) has been established by carbon-14 dating, but what is much more difficult to determine are
(B) has been established by carbon-14 dating, but what is much more difficult to determine is
(C) have been established by carbon-14 dating, but what is much more difficult to determine is
(D) have been established by carbon-14 dating, but what is much more difficult to determine are
(E) are established by carbon-14 dating, but that which is much more difficult to determine is
804.The physical structure of the human eye enables it to sense light of wavelengths up to 0.0005 millimeters; infrared radiation, however, is invisible because its wavelength—0.1 millimeters—is too long to be registered by the eye.
(A) infrared radiation, however, is invisible because its wavelength—0.1 millimeters—is too long to be registered by the eye
(B) however, the wavelength of infrared radiation—0.1 millimeters—is too long to be
registered by the eye making it invisible
(C) infrared radiation, however, is invisible because its wavelength—0.1 millimeters—is too long for the eye to register it
(D) however, because the wavelength of infrared radiation is 0.1 millimeters, it is too long for the eye to register and thus invisible
(E) however, infrared radiation has a wavelength of 0.1 millimeters that is too long for the eye to register, thus making it invisible
805.The plot of The Bostonians centers on the rivalry between Olive Chancellor, an active feminist, with her charming and cynical cousin, Basil Ransom, when they find themselves drawn to the same radiant young woman whose talent for public speaking has won her an ardent following.
(A) rivalry between Olive Chancellor, an active feminist, with her charming and cynical cousin, Basil Ransom
(B) rivals Olive Chancellor, an active feminist, against her charming and cynical cousin, Basil Ransom
(C) rivalry that develops between Olive Chancellor, an active feminist, and Basil Ransom, her charming and cynical cousin
(D) developing rivalry between Olive Chancellor, an active feminist, with Basil Ransom, her charming and cynical cousin
(E) active feminist, Olive Chancellor, and the rivalry with her charming and cynical cousin Basil Ransom
806.The president of the block association tried to convince her neighbors they should join forces to prevent crime in the neighborhood rather than continuing to be victimized.
(A) they should join forces to prevent crime in the neighborhood rather than continuing to be victimized
(B) that they should join forces to prevent crime in the neighborhood rather than continue to be victimized
(C) about joining forces to prevent crime in the neighborhood instead of continuing to be victimized
(D) for the joining of forces to prevent crime in the neighborhood rather than continue to be victimized
(E) to join forces to prevent crime in the neighborhood rather than continuing to be
victimized
807.The prime lending rate is a key rate in the economy: not only are the interest rates on most loans to small and medium-sized businesses tied to the prime, but also on a growing number of consumer loans, including home equity loans.
(A) not only are the interest rates on most loans to small and medium-sized businesses tied to the prime, but also on
(B) tied to the prime are the interest rates not only on most loans to small and
medium-sized businesses, but also on
(C) the interest rates not only on most loans to small and medium-sized businesses are tied to the prime, but also
(D) not only the interest rates on most loans to small and medium-sized businesses are tied to the prime, but also on
(E) the interest rates are tied to the prime, not only on most loans to small and
medium-sized businesses, but also
808.The proposed health care bill would increase government regulation of health insurance, establish standards that would guarantee wider access to people with past health problems and to workers changing jobs who otherwise could be uncovered for months.
(A) establish standards that would guarantee wider access to people with past health
problems and to workers changing jobs who
(B) establishing standards that would guarantee wider access to people with past health problems and to workers who are changing jobs and
(C) to establish standards that would guarantee wider access to people with past health problems and to workers who change jobs that
(D) for establishing standards that would guarantee wider access for people with past health problems and workers changing jobs who
(E) for the establishment of standards that would guarantee wider access for people with past health problems and workers who are changing jobs that
809.The proposed urban development zones do not represent a new principle; it was employed in “Operation Bootstrap” in Puerto Rico.
(A) do not represent a new principle; it
(B) represent not a new principle, but one that
(C) are not a new principle; the same one
(D) are not a new principle, but one that
(E) are not new in principle; it
810.The prospect of a new wave of automobile imports has prompted domestic manufacturers to reduce staff, close plants, and offering buyers financial incentives so they stay competitive.
(A) reduce staff, close plants, and offering buyers financial incentives so they
(B) reduce staff, close plants, and offer financial incentives to buyers in order to
(C) reducing staff, closing plants, and the offer of financial incentives to buyers so they can
(D) staff reductions, closing plants, and offering buyers financial incentives in order to
(E) a reduction of staff, plant closings, and offering financial incentives to buyers to
811.The psychologist William James believed that facial expressions not only provide a visible sign of an emotion, actually contributing to the feeling itself.
(A) emotion, actually contributing to the feeling itself
(B) emotion but also actually contributing to the feeling itself
(C) emotion but also actually contribute to the feeling itself
(D) emotion; they also actually contribute to the feeling of it
(E) emotion; the feeling itself is also actually contributed to by them
812.The Quechuans believed that all things participated in both the material level and the mystical level of reality, and many individual Quechuans claimed to have contact with it directly with an ichana (dream) experience.
(A) contact with it directly with
(B) direct contact with it by way of
(C) contact with the latter directly through
(D) direct contact with the latter by means of
(E) contact directly with the mystical level due to
813.The question of whether to divest themselves of stock in companies that do business in South Africa is particularly troublesome for the nation’s 116 private Black colleges because their economic bases are often more fragile than most predominantly White colleges.
(A) than
(B) than those of
(C) than is so of
(D) compared to
(E) compared to those of
814.The recent surge in the number of airplane flights has clogged the nation’s air-traffic control system, to lead to 55 percent more delays at airports, and prompts fears among some officials that safety is being compromised.
(A) to lead to 55 percent more delays at airports, and prompts
(B) leading to 55 percent more delay at airports and prompting
(C) to lead to a 55 percent increase in delay at airports and prompt
(D) to lead to an increase of 55 percent in delays at airports, and prompted
(E) leading to a 55-percent increase in delays at airports and prompting
815.The record of the past is always incomplete, and the historian who writes about it inevitably reflects the preoccupations of their own .
(A) the historian who writes about it inevitably reflects
(B) the historian writing about it will inevitably reflect
(C) a historian writing about it inevitably reflects
(D) writing about it, it is inevitable for historians to reflect
(E) historians in writing about it inevitably reflect
816.The relationship between corpulence and disease remain controversial, although
statistics clearly associate a reduced life expectancy with chronic obesity.
(A) remain controversial, although statistics clearly associate a reduced life expectancy with
(B) remain controversial, although statistics clearly associates a reduced life expectancy with
(C) remain controversial, although statistics clearly associates reduced life expectancy to
(D) remains controversial, although statistics clearly associate a reduced life expectancy with
(E) remains controversial, although statistics clearly associates reduced life expectancy to
817.The report on the gross national product—the nation’s total production of goods and services—showed that second-quarter inflation was somewhat lower than a previous estimation and the savings rate slightly higher.
(A) a previous estimation and the savings rate slightly higher
(B) a previous estimation and with a slightly higher savings rate
(C) a previous estimate and that the savings rate is slightly higher
(D) previously estimated and a slightly higher savings rate
(E) previously estimated and that the savings rate was slightly higher
818.The reports from the Department of Commerce indicated that the economy had grown at an annual rate much higher than most economists had predicted may occur.
(A) had predicted may occur
(B) had predicted
(C) predicted the occurrence of
(D) predicted may occur
(E) predicted
819.The residents’ opposition to the spraying program has rekindled an old debate among those who oppose the use of pesticides and those who feel that the pesticides are necessary to save the trees.
(A) among those who oppose the use of pesticides and
(B) between those who oppose the use of pesticides and
(C) among those opposing the use of pesticides with
(D) between those who oppose the use of pesticides with
(E) among those opposing the use of pesticides and
820.The rise in the Commerce Department’s index of leading economic indicators suggest that the economy should continue its expansion into the coming months, but that the mixed performance of the index’s individual components indicates that economic growth will proceed at a more moderate pace than in the first quarter of this year.
(A) suggest that the economy should continue its expansion into the coming months, but that
(B) suggest that the economy is to continue expansion in the coming months, but
(C) suggests that the economy will continue its expanding in the coming months, but that
(D) suggests that the economy is continuing to expand into the coming months, but that
(E) suggests that the economy will continue to expand in the coming months, but
821.The rising of costs of data-processing operations at many financial institutions has created a growing opportunity for independent companies to provide these services more efficiently and at lower cost.
(A) The rising of costs
(B) Rising costs
(C) The rising cost
(D) Because the rising cost
(E) Because of rising costs
822.The root systems of most flowering perennials either become too crowded, which results in loss in vigor, and spread too far outward, producing a bare center.
(A) which results in loss in vigor, and spread
(B) resulting in loss in vigor, or spreading
(C) with the result of loss of vigor, or spreading
(D) resulting in loss of vigor, or spread
(E) with a resulting loss of vigor, and spread
823.The Rorschzch test is gaining new respect as a diagnostic tool because it takes only one hour to expose behavior and thought processes that may be unlikely to emerge in other procedures or weeks of ordinary interviewing.
(A) that may be unlikely to emerge in other procedures or weeks of ordinary interviewing
(B) whose emergence is unlikely in other procedures or weeks of ordinary interviews
(C) that might not emerge in other procedures or in weeks of ordinary interviews
(D) that may not emerge under other procedures or weeks of ordinary interviews
(E) unlikely not to emerge during weeks of ordinary interviewing or in other procedures
824.The rules that govern political contributions are less stringent in local elections than they are in national elections because they typically involve smaller amounts of money and present less opportunity for abuse.
(A) The rules that govern political contributions are less stringent in local elections than they are in national elections because they typically involve smaller amounts of money and present less opportunity for abuse.
(B) Because they typically involve smaller amounts of money and present less opportunity for abuse, the rules that govern political contributions are less stringent in local elections than the rules are less stringent in local elections than the rules are in national elections.
(C) The rules that govern political contributions are less stringent in local elections than national elections because they typically involve smaller amounts of money and present less opportunity for abuse.
(D) Because local elections typically involve smaller amounts of money and present less opportunity for abuse than national elections, the rules that govern local political contributions are less stringent than national contributions.
(E) The rules that govern political contributions are less stringent in local elections than they are in national elections because local elections typically involve smaller amounts of money and present less opportunity for abuse.
825.The sale of government surplus machinery will begin at 9 a.m. and continue until the supply lasts.
(A) will begin at 9 a.m. and continue until the supply lasts
(B) begins at 9 a.m., continuing until the supply lasts
(C) will begin at 9 a.m. and, until the supply lasts, will continue
(D) begins at 9 a.m. and, as long as the supply may last, it continues
(E) will begin at 9 a.m. and continue as long as the supply lasts
826.The school board ruling mandating that physically handicapped students be placed in regular classroom settings whenever possible also assured all children who have a reading problem of special aid.
(A) be placed in regular classroom settings whenever possible also assured all children who have a reading problem
(B) should be placed in regular classroom settings whenever possible also assures all children that have a reading problem
(C) are placed in regular classroom settings whenever possible also assures those children who are having reading problems
(D) be placed in regular classroom settings whenever possible also assured children with reading problems
(E) should be placed in regular classroom settings whenever possible also has assured all those children with a reading problem
827.The science of economics, which for four decades was dominated by Keynesians, who at first stressed the government’s role in stimulating the economy, but who were ultimately led away from solutions based on government intervention.
(A) economics, which for four decades was
(B) economics that was to be
(C) economics, one which has, for four decades, been
(D) economics is one that for four decades has been
(E) economics, for four decades, is one that was
828.The Senate approved immigration legislation that would grant permanent residency to millions of aliens currently residing here and if employers hired illegal aliens they would be penalized.
(A) if employers hired illegal aliens they would be penalized
(B) hiring illegal aliens would be a penalty for employers
(C) penalize employers who hire illegal aliens
(D) penalizing employers hiring illegal aliens
(E) employers to be penalized for hiring illegal aliens
829.The speculative fever of the Roaring Twenties infected rich and poor alike; vast
quantities of people were dangerously overextended, credit was absurdly easy to obtain, and most brokerage houses required only ten percent cash for stocks bought on “margin.”
(A) rich and poor alike; vast quantities of people were dangerously overextended
(B) both rich and poor alike; large amounts of people dangerously overextended themselves
(C) rich and poor alike; great numbers of people were dangerously overextended
(D) both rich and poor alike; vast amounts of people dangerously overextended themselves
(E) both rich and poor; great quantities of people were dangerously overextended
830.The spraying of pesticides can be carefully planned, but accidents, weather conditions that could not be foreseen, and pilot errors often cause much larger deposits of spray than they had anticipated.
(A) weather conditions that could not be foreseen, and pilot errors often cause much larger deposits of spray than they had
(B) weather conditions that cannot be foreseen, and pilot errors often cause much larger deposits of spray than
(C) unforeseeable weather conditions, and pilot errors are the cause of much larger deposits of spray than they had
(D) weather conditions that are not foreseeable, and pilot errors often cause much larger deposits of spray than
(E) unforeseeable weather conditions, and pilot errors often cause much larger deposits of spray than they had
831.The study undertaken by Department of the Interior will involve examination and tagging of the California condor in order to obtain information about their daily movements, foraging habits, and sites where they nest.
(A) their daily movements, foraging habits, and sites where they nest
(B) their daily movements, foraging habits, and their nesting sites
(C) its daily movements, foraging habits, and nesting sites
(D) its daily movements, foraging habits, and about nesting sites
(E) daily movements, foraging habits, and sites in which there are nests
832.The supply of oil being finite has become an economical and political consideration of the first magnitude for all modern industrial nations.
(A) The supply of oil being finite has become an economical
(B) The finite supply of oil has become an economical
(C) That the supply of oil is finite has become an economical
(D) The supply of oil being finite has become an economic
(E) That the supply of oil is finite has become an economic
833.The Supreme Court’s concern with legitimacy is not for the sake of the court but the nation to which it is responsible.
(A) but the nation to which it is responsible
(B) but for the sake of the nation to which it is responsible
(C) so much as the nation it is responsible to
(D) as the nation it is responsible to
(E) but the nation
834.The suspect in the burglary was advised of his right to remain silent, told he could not leave, and was interrogated in a detention room.
(A) of his right to remain silent, told he could not leave, and was
(B) of his right to remain silent, told he could not leave, and
(C) of his right to remain silent and that he could not leave and
(D) that he had a right to remain silent, could not leave, and was
(E) that he had a right to remain silent, that he could not leave, and was
835.The task force is reviewing the company’s hiring practices for the determination of whether they are meeting the requirements set by the Office of Equal Opportunity.
(A) for the determination of whether they are meeting the requirements set by the Office of Equal Opportunity
(B) for the determining of whether or not it meets the requirement set by the Office of Equal Opportunity
(C) for the determining of whether the requirements set by the Office of Equal Opportunity are being met or not
(D) determining whether the requirements set by the Office of Equal Opportunity are met
(E) to determine whether they meet the requirements set by the Office of Equal Opportunity
836.The technical term “pagination” is a process that leaves editors, instead of printers, assemble the page images that become the metal or plastic plates used in printing.
(A) is a process that leaves editors, instead of printers, assemble
(B) refers to a process that allows editors, rather than printers, to assemble
(C) is a process leaving the editors, rather than printers, to assemble
(D) refers to a process which allows editors, but not to printers, the assembly of
(E) has reference to the process leaving to editors, instead of the printer, assembling
837.The themes that Rita Dove explores in her poetry is universal, encompassing much of the human condition while occasionally deals with racial issues.
(A) is universal, encompassing much of the human condition while occasionally deals
(B) is universal, encompassing much of the human condition, also occasionally it deals
(C) are universal, they encompass much of the human condition and occasionally deals
(D) are universal, encompassing much of the human condition while occasionally dealing
(E) are universal, they encompass much of the human condition, also occasionally are dealing
838.The underlying physical principles that control the midair gyrations of divers and gymnasts are the same as the body orientation controlling astronauts in a weightless environment.
(A) as the body orientation controlling
(B) as the body orientation which controls
(C) as those controlling the body orientation of
(D) ones to control the body orientation of
(E) ones used in controlling the body orientation of
839.The United States government employs a much larger proportion of women in trade
negotiations than any government.
(A) a much larger proportion of women in trade negotiations than any
(B) a much larger proportion of women in trade negotiations than does any other
(C) much larger proportions of women in trade negotiations than has any
(D) proportions of women in trade negotiations that are much larger than any
(E) proportions of women in trade negotiations that are much larger than any other
840.The United States petroleum industry’s cost to meet environmental regulations is
projected at ten percent of the price per barrel of refined petroleum by the end of the decade.
(A) The United States petroleum industry’s cost to meet environmental regulations is
projected at ten percent of the price per barrel of refined petroleum by the end of the decade.
(B) The United States petroleum industry’s cost by the end of the decade to meet
environmental regulations is estimated at ten percent of the price per barrel of refined petroleum.
(C) By the end of the decade, the United States petroleum industry’s cost of meeting
environmental regulations is projected at ten percent of the price per barrel of refined petroleum.
(D) To meet environmental regulations, the cost to the United States petroleum industry is estimated at ten percent of the price per barrel of refined petroleum by the end of the decade.
(E) It is estimated that by the end of the decade the cost to the United States petroleum industry of meeting environmental regulations will be ten percent of the price per barrel of refined petroleum.
841.The unprecedented increases in the prime lending rate this year has probably been brought about by business community’s uncertainty about the President’s position on the budget deficit.
(A) in the prime lending rate this year has
(B) this year in the prime lending rate has
(C) this year in the prime lending rate having
(D) in the prime lending rate this year had
(E) in the prime lending rate this year have
842.The unskilled workers at the Allenby plant realized that their hourly rate of $4.11 to $4.75 was better than many nearby factory wages.
(A) many nearby factory wages
(B) many wages in nearby factories
(C) what are offered by many nearby factories
(D) it is in many nearby factories
(E) that offered by many nearby factories
843.The use of chemical pesticides in this country is equally extensive or more so than ten years ago.
(A) equally extensive or more so than ten years ago
(B) equal to or more extensive than ten years ago
(C) as extensive as ten years ago or more
(D) equal to, if not more, than ten years ago
(E) as extensive as it was ten years ago, if not more so
844.The use of gravity waves, which do not interact with matter in the way electromagnetic waves do, hopefully will enable astronomers to study the actual formation of black holes and neutron stars.
(A) in the way electromagnetic waves do, hopefully will enable
(B) in the way electromagnetic waves do, will, it is hoped, enable
(C) like electromagnetic waves, hopefully will enable
(D) like electromagnetic waves, would enable, hopefully
(E) such as electromagnetic waves do, will, it is hoped, enable
845.The utility company has announced that it will permanently close its Unit I nuclear power plant, the first plant that had been built by private industry and the model for a generation of modern nuclear reactors.
(A) that had been built by private industry and
(B) built by private industry and which was
(C) to be built by private industry and which was
(D) built by private industry and
(E) to have been built by private industry and was
846.The visiting pharmacologists concluded that the present amalgam of Chinese and Western medicine is probably as good, or better than, any system that might be devised for the patients who are treated at the Nan Kai hospital in Tian-jing.
(A) as good, or better than, any system that might be devised for the patients who are
(B) as good, or better, than any system that might be devised for patients being
(C) as good, or better than, any system that might be devised for patients which are being
(D) good as, or even better than, any other system that may be devised for the patients who are
(E) as good as, or better than, any other system that might be devised for the patients
847.The voluminous personal papers of Thomas Alva Edison reveal that his inventions
typically sprang to life not in a flash of inspiration but evolved slowly from previous works.
(A) sprang to life not in a flash of inspiration but evolved slowly
(B) sprang to life not in a flash of inspiration but were slowly evolved
(C) did not spring to life in a flash of inspiration but evolved slowly
(D) did not spring to life in a flash of inspiration but had slowly evolved
(E) did not spring to life in a flash of inspiration but they were slowly evolved
848.The Wallerstein study indicates that even after a decade young men and women still experience some of the effects of a divorce occurring when a child.
(A) occurring when a child
(B) occurring when children
(C) that occurred when a child
(D) that occurred when they were children
(E) that has occurred as each was a child
849.The Western world’s love affair with chocolate is well-documented: few people have been known to have tasted it for the first without requesting more.
(A) few people have been known to have tasted it
(B) few having been known to taste it
(C) it has been tasted by few people
(D) few people have been known to taste it
(E) few people having tasted it
850.The winds that howl across the Great Plains not only blow away valuable topsoil, thereby reducing the potential crop yield of a tract of land, and also damage or destroy young plants.
(A) and also damage or destroy
(B) as well as damaging or destroying
(C) but they also cause damage or destroy
(D) but also damage or destroy
(E) but also causing damage or destroying
851.The work of mathematician Roger Penrose in the early 1970s, on the geometry of what are called aperiodic tiles, turned out to describe the architecture of a previously unknown class of crystals.
(A) what are called aperiodic tiles, turned out to describe
(B) what is called aperiodic tiles, describes
(C) aperiodic tiles, describing
(D) so-called aperiodic tiles, describe
(E) aperiodic tiles, it turned out to describe
852.There are more than forty newspapers published in the cities of Kerala, a state on the Malabar Coast, which reflects the fact that Keralans are by far India’s most literate citizens.
(A) which reflects
(B) and that number reflects
(C) which reflect
(D) that number reflects
(E) that reflects
853.There has been a 30- to 40-fold increase in the incidence of malaria caused by
increasing mosquito resistance against pesticides.
(A) increase in the incidence of malaria caused by increasing mosquito resistance against
(B) increase in the incidence of malaria because of increasing resistance of mosquitoes to
(C) increasing malaria incidence because of increasing resistance of mosquitoes to
(D) incidence of malaria increase caused by increasing mosquito resistance against
(E) incidence of malaria increase because of increased mosquito resistance to
854.There is ample evidence, derived from the lore of traditional folk medicine, that naturally occurring antibiotics are usually able to be modified to make them a more effective drug.
(A) are usually able to be modified to make them a more effective drug.
(B) are usually able to be modified to make them more effective drugs
(C) are usually able to be modified, which makes them more effective drugs
(D) can usually be modified to make them a more effective drug
(E) can usually be modified to make them more effective drugs
855.There is growing demand in the state for “initiative and referendum,” a procedure that allows voters to propose and pass laws, as well as to repeal them.
(A) allows voters to propose and pass laws, as well as to repeal them
(B) allows voters to propose, pass, and to repeal laws
(C) allows voters to propose, to pass, and repeal laws
(D) will allow the voter to propose, pass, as well as to repeal laws
(E) will allow laws to be proposed, passed, as well as repealed by voters
856.There is no consensus on what role, if any, is played by acid rain in slowing the growth or damaging forests in the eastern United States.
(A) slowing the growth or damaging
(B) the damage or the slowing of the growth of
(C) the damage to or the slowness of the growth of
(D) damaged or slowed growth of
(E) damaging or slowing the growth of
857.There is speculation that increasing cold weather was what may have been responsible for the Anasazi move from Mesa Verde to sites in other canyons.
(A) that increasing cold weather was what may have been
(B) whether increasing cold weather was what was
(C) that increasingly cold weather was what had been
(D) whether increasingly cold weather may have been what was
(E) that increasingly cold weather may have been
858.There is substantial evidence that certain forms of solar energy either now or within a few years will be economically competitive with conventional sources of heat and power.
(A) either now or within a few years will be economically competitive with conventional sources of heat and power
(B) will either be economically competitive with conventional sources of heat and power within a few years or are so now
(C) will be economically competitive with conventional sources of heat and power either now or within a few years
(D) either are now economically competitive with conventional sources of heat and power or will be so within a few years
(E) are either now or will be within a few years economically competitive with conventional sources of heat and power
859.Thomas Eakins’ powerful style and his choices of subject—the advances in modern surgery, the discipline of sport, the strains of individuals in tension with society or even with themselves—was as disturbing to his own as it is compelling for ours.
(A) was as disturbing to his own as it is
(B) were as disturbing to his own as they are
(C) has been as disturbing in his own as they are
(D) had been as disturbing in his own as it was
(E) have been as disturbing in his own as
860.Those who come to church with a predisposition to religious belief will be happy in an auditorium or even a storefront, and there is no doubt that religion is sometimes better served by adapted spaces of this kind instead of by some of the buildings actually designed for it.
(A) adapted spaces of this kind instead of by some of the buildings actually designed for it
(B) adapted spaces like these rather than some of the buildings actually designed for them
(C) these adapted spaces instead of by some of the buildings actually designed for it
(D) such adapted spaces rather than by some of the buildings actually designed for them
(E) such adapted spaces than by some of the buildings actually designed for it
861.Those who enter marathons soon learn that, to succeed in these grueling competitive events, runners must be in excellent condition, have unshakable self-confidence, and, most important of all, know how to pace yourself.
(A) know how to pace yourself
(B) is knowing how to pace yourself
(C) know how to pace themselves
(D) you must pace yourself
(E) they must know how to pace themselves
862.Those with a cynical turn of mind might speculate if the new corporation, eager for profit, might not have started the rumor that caused its competitor to declare bankruptcy.
(A) speculate if the new corporation, eager for profit, might not have started
(B) speculate if the new corporation, eager for profit, had not started
(C) speculate if, in its eagerness for profit, the new corporation started
(D) wonder as to whether, in its eagerness for profit, the new corporation did not start
(E) wonder whether the new corporation, eager for profit, had started
863.Though the term “graphic design” may suggest laying out corporate brochures and annual reports, they have come to signify widely ranging work, from package designs and company logotypes to signs, book jackets, computer graphics, and film titles.
(A) suggest laying out corporate brochures and annual reports, they have come to signify widely ranging
(B) suggest laying out corporate brochures and annual reports, it has come to signify a wide range of
(C) suggest corporate brochure and annual report layout, it has signified widely ranging
(D) have suggested corporate brochure and annual report layout, it has signified a wide range of
(E) have suggested laying out corporate brochures and annual reports, they have come to signify widely ranging
864.Three out of every four automobile owners in the United States also own a bicycle.
(A) Three out of every four automobile owners in the United States also own a bicycle.
(B) Out of every four, three automobile owners in the United States also owns a bicycle.
(C) Bicycles are owned by three out of every four owners of automobiles in the United States.
(D) In the United States, three out of every four automobile owners owns bicycles.
(E) Out of every four owners of automobiles in the United States, bicycles are also owned by three.
865.Through the years, the exquisitely subtle flavors and superb richness of Kenyan coffee has attracted an international following of discerning consumers.
(A) the exquisitely subtle flavors and superb richness of Kenyan coffee has
(B) the coffee of Kenya, with its exquisitely subtle and superbly rich flavors, have
(C) the exquisitely subtle, superbly rich flavors of Kenyan coffee are what has
(D) Kenyan coffee’s superb richness and exquisite subtlety of flavor has
(E) the exquisitely subtle flavors and superb richness of Kenyan coffee have
866.Tiny quantities of more than thirty rare gases, most of them industrial by-products, threaten to warm the Earth’s atmosphere even more rapidly than carbon dioxide during the next fifty years.
(A) to warm the Earth’s atmosphere even more rapidly than carbon dioxide during the next fifty years
(B) to warm the Earth’s atmosphere even more rapidly over the next fifty years than carbon dioxide will
(C) during the next fifty years to warm the Earth’s atmosphere even more rapidly than carbon dioxide
(D) a warming of the Earth’s atmosphere during the next fifty years even more rapid than carbon dioxide’s
(E) a warming of the Earth’s atmosphere even more rapid than carbon dioxide’s will be over the next fifty years
867.To compare the lightning-fast genius of playwright Tom Stoppard with the pedestrian efforts of some of his contemporaries is to compare the exquisite bouquet of a fine wine with that of ordinary grape juice.
(A) To compare the lightning-fast genius of playwright Tom Stoppard with the pedestrian efforts of some of his contemporaries is to compare the exquisite bouquet of a fine wine with that of ordinary grape juice.
(B) To compare the lightning-fast genius of playwright Tom Stoppard with the pedestrian efforts of some of his contemporaries is comparing the exquisite bouquet of a fine wine with that of ordinary grape juice.
(C) Comparing the lightning-fast genius of playwright Tom Stoppard with the pedestrian efforts of some of his contemporaries is to compare the exquisite bouquet of a fine wine with ordinary grape juice.
(D) Comparing the lightning-fast genius of playwright Tom Stoppard with the pedestrian efforts of some of his contemporaries is like comparing the exquisite bouquet of a fine wine with ordinary grape juice.
(E) To compare the lightning-fast genius of playwright Tom Stoppard with the pedestrian efforts of some of his contemporaries is to compare a fine wine’s bouquet with ordinary grape juice’s bouquet.
868.To ensure consistently high quality in its merchandise, the chain of retail stores became involved in every aspect of their suppliers’ operations, dictating not only the number of stitches and the width of the hem in every garment as well as the profit margins of those suppliers.
(A) their suppliers’ operations, dictating not only the number of stitches and the width of the hem in every garment as well as
(B) its suppliers’ operations, dictating not only the number of stitches and the width of the hem in every garment as well as
(C) their suppliers’ operations, dictating not only the number of stitches and the width of the hem in every garment but also
(D) its suppliers’ operations, dictating not only the number of stitches and the width of the hem in every garment but also
(E) their suppliers’ operations, dictating the number of stitches, the width of the hem in every garment, and
869.To help preserve ancient Egyptian monuments threatened by high water tables, a Swedish engineering firm has proposed installing pumps, perhaps solar powered, to lower the underground water level and dig trenches around the bases of the stone walls.
(A) to lower the underground water level and dig trenches
(B) to lower the underground water level and to dig trenches
(C) to lower the underground water level and digging trenches
(D) that lower the underground water level and that trenches be dug
(E) that lower the underground water level and trench digging
870.To maintain a high demand for their product, the manufacturers first took over the marketing and sales functions previously performed by outside agents; next, they began changing their advertising campaigns monthly to keep pace with the rapid changes in consumers’ lives.
(A) they began changing
(B) this began changing
(C) the former began changing
(D) to begin changing
(E) to change
871.To read of Abigail Adams’ lengthy separation from her family, her difficult travels, and her constant battles with illness is to feel intensely how harsh life was even for the so-called aristocracy of Revolutionary s.
(A) To read of
(B) Reading about
(C) Having read about
(D) Once one reads of
(E) To have read of
872.To speak habitually of the “truly needy” is gradually instilling the notion that many of those who are just called “needy” actually have adequate resources; such a conclusion is unwarranted.
(A) To speak habitually of the “truly needy” is gradually instilling the notion
(B) To speak habitually of the “truly needy” is instilling the notion gradually
(C) To speak habitually of the “truly needy” is gradually to instill the notion
(D) Speaking habitually of the “truly needy” is to instill the gradual notion
(E) Speaking habitually of the “truly needy” is instilling the gradual notion
873.To spread the word about mortgage servicers, a fact sheet that outlines one’s legal rights if you get caught in a mortgage serving mess has been put together by the Federal Trade Commission.
(A) a fact sheet that outlines one’s legal rights if you get caught in a mortgage serving mess has been put together by the Federal Trade Commission
(B) an outline of one’s legal rights has been put together by the Federal Trade Commission if one gets caught up in a mortgaging mess on a fact sheet
(C) should you get caught in a mortgaging mess, a fact sheet outlining your legal rights has been put together by the Federal Trade Commission
(D) there is a fact sheet put together by the Federal Trade Commission, which outlines one’s legal rights in a mortgage servicing mess
(E) the Federal Trade Commission has put together a fact sheet that outlines your legal rights if you get caught in a mortgage servicing mess
874.Today, because of improvements in agricultural technology, the same amount of acreage produces double the apples that it has in 1910.
(A) double the apples that it has
(B) twice as many apples as it did
(C) as much as twice the apples it has
(D) two s as many apples as there were
(E) a doubling of the apples that it did
875.Today’s technology allows manufacturers to make small cars more fuel-efficient now than at any in their production history.
(A) small cars more fuel-efficient now than at any in their
(B) small cars that are more fuel-efficient than they were at any in their
(C) small cars that are more fuel-efficient than those at any other in
(D) more fuel-efficient small cars than those at any other in their
(E) more fuel-efficient small cars now than at any in
876.Too old to bear arms himself, Frederick Douglass served as a recruiting agent, traveled through the North to exhort Black men to join the Union army.
(A) traveled through the North to exhort
(B) and he traveled through the North and exhorted
(C) and traveling through the North exhorted
(D) traveling through the North and exhorted
(E) traveling through the North and exhorting
877.Traveling the back roads of Hungary, in 1905 Béla Bartók and Zoltán Kodály began their pioneering work in ethnomusicology, and they were armed only with an Edison phonograph and insatiable curiosity.
(A) Traveling the back roads of Hungary, in 1905 Béla Bartók and Zoltán Kodály began their pioneering work in ethnomusicology, and they were armed only
(B) In 1905, Béla Bartók and Zoltán Kodály, traveling the back roads of Hungary, began their pioneering work in ethnomusicology, and they were only armed
(C) In 1905 Béla Bartók and Zoltán Kodály began their pioneering work in ethnomusicology, traveling the back roads of Hungary armed only
(D) Having traveled the back roads of Hungary, in 1905 Béla Bartók and Zoltán Kodály began their pioneering work in ethnomusicology; they were only armed
(E) Béla Bartók and Zoltán Kodály, in 1905 began their pioneering work in ethnomusicology, traveling the back roads of Hungary, arming themselves only
878.Trying to learn some of the basics of programming is the same as to tinker with a car when one is a teenager: some people end up going to engineering school, and others, twenty years later, remember nothing of the experience.
(A) the same as to tinker with a car when one is a teenager
(B) similar to a teenager tinkering with a car
(C) like tinkering with a car as a teenager
(D) the same as a teenager tinkering with a car
(E) like the teenager’s tinkering with a car
879.Two new studies indicate that many people become obese more due to the fact that their bodies burn calories too slowly than overeating.
(A) due to the fact that their bodies burn calories too slowly than overeating
(B) due to their bodies burning calories too slowly than to eating too much
(C) because their bodies burn calories too slowly than that they are overeaters
(D) because their bodies burn calories too slowly than because they eat too much
(E) because of their bodies burning calories too slowly than because of their eating too much
880.Two valence states of uranium, one with a deficit of four electrons and the other one with a deficit of six, occurs in nature and contributes to the diversity of uranium’s behavior.
(A) the other one with a deficit of six, occurs in nature and contributes
(B) the other one a deficit of six, occur in nature and contribute
(C) the other with a deficit of six, occurs in nature and contributes
(D) the other with a deficit of six, occur in nature and contribute
(E) one with six, occurs in nature and contributes
881.Two week notice being given to employers before leaving a job is the generally accepted protocol.
(A) Two week notice being given to employers before leaving
(B) Giving notice to employers of two weeks before having to leave
(C) Two week’s notice to give to employers before leaving
(D) Giving notice to employers two weeks before leaving
(E) To give two weeks’ worth of notice before having to leave
882.Under a provision of the Constitution that was never applied, Congress has been required to call a convention for considering possible amendments to the document when formally asked to do it by the legislatures of two-thirds of the states.
(A) was never applied, Congress has been required to call a convention for considering possible amendments to the document when formally asked to do it
(B) was never applied, there has been a requirement that Congress call a convention for consideration of possible amendments to the document when asked to do it formally
(C) was never applied, whereby Congress is required to call a convention for considering possible amendments to the document when asked to do it formally
(D) has never been applied, whereby Congress is required to call a convention to consider possible amendments to the document when formally asked to do so
(E) has never been applied, Congress is required to call a convention to consider possible amendments to the document when formally asked to do so
883.Under Napoleon the French were not able to organize an adequate supply system, and it was a major cause of the failure of their invasion of Russia.
(A) Under Napoleon the French were not able to organize an adequate supply system, and it
(B) The French being unable to organize an adequate supply system under Napoleon
(C) For the French under Napoleon, to be unable to organize an adequate supply system
(D) The inability of the French under Napoleon to organize an adequate supply system
(E) The French inability under Napoleon of organizing an adequate supply system.
884.Under the new corporate insurance policy, when an employer is charged for damages to a third party wholly or largely as a result of actions by an employee, he is entitled to recoup the amount of the damages.
(A) he is entitled to recoup
(B) the employer is entitled to recoup
(C) he or she is entitled to recoup
(D) he is entitled to recoup for
(E) the employer is entitled to recoup for
885.Under the restructuring, the huge organization that operates the company’s basic
businesses will be divided into five groups, each with its own executive.
(A) each with its own executive
(B) all having their own executive
(C) each having their own executive
(D) with its own executive for each
(E) every one with an executive of their own
886.Under the Safe Drinking Water Act, the Environmental Protection Agency is required either to approve individual state plans for controlling the discharge of wastes into underground water or that they enforce their own plan for states without adequate regulations.
(A) that they enforce their
(B) for enforcing their
(C) they should enforce their
(D) it should enforce its
(E) to enforce its
887.Unlike a funded pension system, in which contributions are invested to pay future beneficiaries, a pay-as-you-go approach is the foundation of Social Security.
(A) a pay-as-you-go approach is the foundation of Social Security
(B) the foundation of Social Security is a pay-as-you-go approach
(C) the approach of Social Security is pay-as-you-go
(D) Social Security’s approach is pay-as-you-go
(E) Social Security is founded on a pay-as-you-go approach
888.Under the 1986 tax law, interest payments on a refinanced home loan are deductible only if the amount of the loan does not exceed the purchase price of the home, the cost of improvements, and any additional amount borrowed against the home to pay for medical or educational expenses.
(A) any additional amount borrowed against the home to pay for medical or educational expenses
(B) borrowing any additional amount against the home for payment of medical or educational expenses
(C) also borrowing any additional amount against the home to pay for medical or educational expenses
(D) any additional payment of medical or educational expenses that were borrowed against the home
(E) any additional payment borrowed against the home for medical or educational expenses
889.Unlike a hurricane, which can be observed from within, a tornado is so small that such a study has not been practical.
(A) that such a study has not been practical
(B) that studying it that way has not been impractical
(C) for such studies as this to have been impractical
(D) as to not make such a study practical
(E) as to be impractical of study
890.Unlike a typical automobile loan, which requires a fifteen- to twenty-percent down payment, the lease-loan buyer is not required to make an initial deposit on the new vehicle.
(A) the lease-loan buyer is not required to make
(B) with lease-loan buying there is no requirement of
(C) lease-loan buyers are not required to make
(D) for the lease-loan buyer there is no requirement of
(E) a lease-loan does not require the buyer to make
891.Unlike auto insurance, the frequency of claims does not affect the premiums for personal property coverage, but if the insurance company is able to prove excessive loss due to owner negligence, it may decline to renew the policy.
(A) Unlike auto insurance, the frequency of claims does not affect the premiums for personal property coverage
(B) Unlike with auto insurance, the frequency of claims do not affect the premiums for personal property coverage
(C) Unlike the frequency of claims for auto insurance, the premiums for personal property coverage are not affected by the frequency of claims
(D) Unlike the premiums for auto insurance, the premiums for personal property coverage are not affected by the frequency of claims
(E) Unlike with the premiums for auto insurance, the premiums for personal property coverage is not affected by the frequency of claims
892.Unlike computer skills or other technical skills, there is a disinclination on the part of many people to recognize the degree to which their analytical skills are weak.
(A) Unlike computer skills or other technical skills, there is a disinclination on the part of many people to recognize the degree to which their analytical skills are weak.
(B) Unlike computer skills or other technical skills, which they admit they lack, many people are disinclined to recognize that their analytical skills are weak.
(C) Unlike computer skills or other technical skills, analytical skills bring out a
disinclination in many people to recognize that they are weak to a degree.
(D) Many people, willing to admit that they lack computer skills or other technical skills, are disinclined to recognize that their analytical skills are weak.
(E) Many people have a disinclination to recognize the weakness of their analytical skills while willing to admit their lack of computer skills or other technical skills.
893.Unlike Gertrude Stein, Ezra Pound, and other expatriates, William Carlos Williams insisted that poets honor their own regions and employ specifically American rhythms.
(A) Unlike Gertrude Stein, Ezra Pound, and other expatriates, William Carlos Williams insisted
(B) Although Gertrude Stein, Ezra Pound, and other expatriates did not, William Carlos Williams’ insistence was
(C) Contrary to Gertrude Stein, Ezra Pound, and other expatriates, it was William Carlos Williams who insisted
(D) As opposed to what Gertrude Stein, Ezra Pound, and other expatriates did, William Carlos Williams was to insist
(E) While Gertrude Stein, Ezra Pound, and other expatriates did not, William Carlos Williams was insistent
894.Unlike in other transportation industries, there are no minimum standards set by
government or industry for qualifying for being an engineer on a train.
(A) Unlike in other transportation industries, there are no minimum standards set by
government or industry for qualifying for being
(B) Unlike other transportation industries that have minimum standards set by government or industry, there are none for qualifying to be
(C) Although the government or industry usually sets minimum standards for transportation industries, no such one has been set for qualifying as to being
(D) Although the government or industry usually sets minimum standards for transportation industries, no such standard has been set for qualifying to be
(E) Although there are usually minimum standards set by government or industry for
transportation industries, there is none for qualifying and being
895.Unlike most warbler species, the male and female blue-winged warbler are very difficult to tell apart.
(A) Unlike most warbler species, the male and female blue-winged warbler are very difficult to tell apart.
(B) Unlike most warbler species, the gender of the blue-winged warbler is very difficult to distinguish.
(C) Unlike those in most warbler species, the male and female blue-winged warblers are very difficult to distinguish.
(D) It is very difficult, unlike in most warbler species, to tell the male and female blue-winged warbler apart.
(E) Blue-winged warblers are unlike most species of warbler in that it is very difficult to tell the male and female apart.
896.Unlike other arachnids, which have their nerve cells evenly distributed along their bodies, the scorpion’s nerve cells are clustered in its head, like a mammal’s.
(A) bodies, the scorpion’s nerve cells are clustered in its head, like a mammal’s
(B) bodies, the scorpion’s head had a cluster of nerve cells, as a mammal does
(C) body, the scorpion has a cluster of nerve cells in its head, as a mammal does
(D) body, nerve cells are clustered in the scorpion’s head, like a mammal’s
(E) body, a cluster of nerve cells is in the scorpion’s head, like a mammal’s
897.Unlike Schoenberg’s twelve-tone system that dominated the music of the postwar period, Bartok founded no school and left behind only a handful of disciples.
(A) Schoenberg’s twelve-tone system that dominated
(B) Schoenberg and his twelve-tone system which dominated
(C) Schoenberg, whose twelve-tone system dominated
(D) the twelve-tone system of Schoenberg that has dominated
(E) Schoenberg and the twelve-tone system, dominating
898.Unlike that of human beings, who waste away when they go without food for long periods, hibernating bears exist for months on only their excess fat.
(A) Unlike that of human beings, who waste away when they go
(B) Unlike human beings, who waste away when they go
(C) Unlike human beings, wasting away when going
(D) Dissimilar to human beings, wasting away when
(E) Lacking similarity to human beings, who waste away when
899.Unlike that of the Native Americans of British Columbia, the Plains, and the Southwest, those of Puget Sound lived in relatively small, autonomous villages.
(A) Unlike that of
(B) Unlike those of
(C) Unlike
(D) In contrast to that of
(E) Dissimilar to
900.Unlike the acid smoke of cigarettes, pipe tobacco, cured by age-old methods, yields an alkaline smoke too irritating to be drawn into the lungs.
(A) Unlike the acid smoke of cigarettes, pipe tobacco, cured by age-old methods, yields an alkaline smoke
(B) Unlike the acid smoke of cigarettes, pipe tobacco is cured by age-old methods, yielding an alkaline smoke
(C) Unlike cigarette tobacco, which yields an acid smoke, pipe tobacco, cured by age-old methods, yields an alkaline smoke
(D) Differing from cigarettes’ acid smoke, pipe tobacco’s alkaline smoke, cured by age-old methods, is
(E) The alkaline smoke of pipe tobacco differs from cigarettes’ acid smoke in that it is cured by age-old methods and is
but also prevents tooth decay.

1.E 2.A 3.E 4.A 5.A
6.D 7.D 8.C 9.D 10.C
11.C 12.D 13.D 14.B 15.C
16.A 17.C 18.D 19.A 20.A

21.C 22.C 23.E 24.E 25.B
26.A 27.E 28.C 29.D 30.C
31.D 32.E 33.E 34.D 35.A
36.E 37.A 38.C 39.B 40.E

41.A 42.C 43.A 44.B 45.B
46.C 47.D 48.A 49.A 50.E
51.E 52.C 53.E 54.A 55.E
56.E 57.E 58.C 59.B 60.C

61.D 62.D 63.A 64.D 65.A
66.A 67.D 68.C 69.C 70.B
71.C 72.D 73.C 74.D 75.C
76.E 77.B 78.D 79.B 80.A

81.E 82.C 83.E 84.D 85.D
86.B 87.B 88.B 89.D 90.E
91.D 92.C 93.A 94.E 95.B
96.D 97.E 98.D 99.A 100.A

101.A 102.D 103.A 104.A 105.B
106.A 107.D 108.A 109.A 110.A
111.A 112.A 113.B 114.C 115.C
116.B 117.C 118.C 119.D 120.D

121.B 122.E 123.E 124.B 125.E
126.B 127.C 128.D 129.C 130.A
131.A 132.A 133.E 134.E 135.E
136.D 137.C 138.D 139.B 140.C

141.E 142.A 143.C 144.D 145.E
146.E 147.B 148.B 149.A 150.C
151.A 152.A 153.A 154.E 155.B
156.B 157.B 158.A 159.E 160.C

161.B 162.C 163.C 164.E 165.B
166.B 167.A 168.D 169.C 170.E
171.D 172.A 173.C 174.C 175.A
176.B 177.B 178.B 179.A 180.B

181.A 182.B 183.A 184.E 185.E
186.A 187.A 188.D 189.C 190.C
191.A 192.A 193.D 194.C 195.C
196.C 197.A 198.D 199.B 200.B
201.E 202.B 203.E 204.E 205.A
206.E 207.D 208.E 209.D 210.A
211.A 212.B 213.C 214.A 215.E
216.B 217.C 218.C 219.D 220.A

221.D 222.B 223.A 224.E 225.B
226.A 227.C 228.D 229.A 230.D
231.A 232.E 233.E 234.E 235.B
236.E 237.C 238.E 239.A 240.E

241.E 242.C 243.A 244.B 245.E
246.E 247.D 248.A 249.E 250.D
251.E 252.C 253.A 254.B 255.E
256.B 257.E 258.E 259.C 260.D

261.B 262.C 263.A 264.B 265.B
266.B 267.E 268.E 269.A 270.C
271.C 272.C 273.D 274.B 275.D
276.A 277.D 278.A 279.A 280.D

281.C 282.A 283.B 284.C 285.E
286.B 287.D 288.A 289.B 290.A
291.C 292.C 293.B 294.A 295.D
296.D 297.C 298.A 299.D 300.E

301.E 302.A 303.E 304.C 305.A
306.C 307.C 308.A 309.B 310.C
311.C 312.D 313.D 314.C 315.C
316.A 317.A 318.C 319.A 320.E

321.A 322.B 323.B 324.C 325.A
326.D 327.E 328.E 329.A 330.E
331.A 332.B 333.A 334.A 335.B
336.C 337.E 338.E 339.D 340.D

341.D 342.B 343.B 344.E 345.C
346.D 347.C 348.D 349.A 350.D
351.C 352.D 353.B 354.C 355.D
356.C 357.E 358.C 359.A 360.A

361.D 362.C 363.E 364.D 365.B
366.C 367.B 368.A 369.D 370.A
371.C 372.D 373.E 374.D 375.C
376.C 377.B 378.B 379.A 380.A

381.B 382.C 383.C 384.E 385.D
386.E 387.C 388.C 389.A 390.A
391.E 392.D 393.A 394.D 395.C
396.E 397.E 398.A 399.D 400.A
401.D 402.B 403.A 404.C 405.A
406.A 407.A 408.E 409.C 410.D
411.A 412.B 413.B 414.B 415.A
416.D 417.D 418.A 419.B 420.B

421.B 422.A 423.E 424.B 425.A
426.C 427.D 428.B 429.B 430.C
431.B 432.B 433.E 434.E 435.D
436.C 437.B 438.C 439.D 440.C

441.E 442.C 443.E 444.D 445.E
446.A 447.B 448.A 449.E 450.D
451.D 452.D 453.E 454.C 455.C
456.D 457.B 458.E 459.E 460.B

461.A 462.B 463.B 464.C 465.A
466.B 467.A 468.B 469.C 470.D
471.D 472.D 473.E 474.C 475.A
476.E 477.C 478.A 479.E 480.A

481.C 482.A 483.A 484.B 485.A
486.B 487.D 488.D 489.B 490.E
491.B 492.C 493.B 494.C 495.E
496.E 497.C 498.E 499.C 500.E

501.A 502.C 503.A 504.C 505.B
506.E 507.A 508.C 509.E 510.A
511.D 512.D 513.B 514.E 515.D
516.C 517.A 518.D 519.B 520.A

521.E 522.A 523.D 524.E 525.A
526.B 527.D 528.C 529.E 530.D
531.C 532.B 533.D 534.A 535.B
536.D 537.D 538.C 539.D 540.C

541.E 542.D 543.B 544.A 545.E
546.B 547.D 548.D 549.A 550.A
551.D 552.C 553.E 554.B 555.D
556.B 557.B 558.D 559.C 560.D

561.B 562.D 563.B 564.C 565.A
566.C 567.A 568.A 569.B 570.B
571.C 572.A 573.D 574.D 575.E
576.E 577.A 578.C 579.E 580.B

581.B 582.D 583.B 584.D 585.E
586.C 587.C 588.C 589.C 590.C
591.A 592.B 593.B 594.D 595.E
596.D 597.D 598.D 599.C 600.B
601.C 602.B 603.A 604.C 605.A
606.A 607.C 608.C 609.C 610.E
611.B 612.C 613.E 614.A 615.E
616.B 617.A 618.A 619.D 620.D

621.B 622.D 623.A 624.A 625.D
626.B 627.E 628.C 629.E 630.E
631.A 632.E 633.E 634.E 635.B
636.A 637.E 638.D 639.C 640.B

641.B 642.E 643.B 644.D 645.C
646.D 647.D 648.A 649.D 650.B
651.B 652.B 653.E 654.A 655.C
656.C 657.A 658.D 659.B 660.A

661.B 662.C 663.A 664.E 665.E
666.B 667.B 668.A 669.B 670.E
671.C 672.E 673.E 674.C 675.B
676.A 677.C 678.E 679.E 680.C

681.E 682.C 683.D 684.E 685.D
686.A 687.D 688.C 689.C 690.D
691.A 692.C 693.E 694.D 695.E
696.A 697.E 698.C 699.E 700.E

701.B 702.D 703.A 704.B 705.B
706.B 707.E 708.A 709.A 710.E
711.B 712.D 713.D 714.B 715.A
716.B 717.D 718.C 719.B 720.E

721.C 722.C 723.A 724.C 725.E
726.B 727.C 728.C 729.D 730.E
731.D 732.B 733.B 734.D 735.D
736.E 737.C 738.A 739.C 740.A

741.B 742.A 743.D 744.C 745.E
746.A 747.D 748.B 749.B 750.A
751.B 752.C 753.A 754.D 755.E
756.E 757.C 758.C 759.C 760.E

761.D 762.E 763.D 764.A 765.D
766.D 767.D 768.A 769.D 770.C
771.B 772.D 773.B 774.D 775.D
776.D 777.E 778.D 779.A 780.C

781.E 782.E 783.B 784.A 785.C
786.E 787.E 788.A 789.C 790.D
791.C 792.B 793.D 794.C 795.E
796.E 797.C 798.D 799.A 800.A
801.B 802.C 803.B 804.A 805.C
806.B 807.B 808.B 809.B 810.B
811.C 812.D 813.B 814.E 815.E
816.D 817.E 818.B 819.B 820.E

821.C 822.D 823.C 824.E 825.E
826.D 827.D 828.C 829.C 830.B
831.C 832.E 833.B 834.B 835.E
836.B 837.D 838.C 839.B 840.E

841.E 842.E 843.E 844.B 845.D
846.E 847.C 848.D 849.D 850.D
851.A 852.B 853.B 854.E 855.A
856.E 857.E 858.D 859.B 860.E

861.C 862.E 863.B 864.A 865.E
866.B 867.A 868.D 869.C 870.A
871.A 872.C 873.E 874.B 875.C
876.E 877.C 878.A 879.D 880.D

881.D 882.E 883.D 884.B 885.A
886.E 887.E 888.A 889.A 890.E
891.D 892.D 893.A 894.D 895.E
896.C 897.C 898.B 899.C 900.C








READING COMPREHENSION (RC)
Passage 1 (1/63)
(This passage was written in 1978.)
Recent years have brought minority-owned businesses in the United States unprecedented

opportunities—as well as new and significant risks. Civil rights activists have long argued

that one of the principal reasons why Blacks, Hispanics, and other minority groups have

difficulty establishing themselves in business is that they lack access to the sizable

orders and subcontracts that are generated by large companies. Now Congress, in apparent

agreement, has required by law that businesses awarded federal contracts of more than

$500,000 do their best to find minority subcontractors and record their efforts to do so on

forms filed with the government. Indeed, some federal and local agencies have gone so far as

to set specific percentage goals for apportioning parts of public works contracts to

minority enterprises.
Corporate response appears to have been substantial. According to figures collected in 1977,

the total of corporate contracts with minority businesses rose from $77 million in 1972 to

$1.1 billion in 1977. The projected total of corporate contracts with minority businesses

for the early 1980’s is estimated to be over 53 billion per year with no letup anticipated

in the next decade. Promising as it is for minority businesses, this increased patronage

poses dangers for them, too. First, minority firms risk expanding too fast and overextending

themselves financially, since most are small concerns and, unlike large businesses, they

often need to make substantial investments in new plants, staff, equipment, and the like in

order to perform work subcontracted to them. If, thereafter, their subcontracts are for some

reason reduced, such firms can face potentially crippling fixed expenses. The world of

corporate purchasing can be frustrating for small entrepreneurs who get requests for

elaborate formal estimates and bids. Both consume valuable and resources, and a small

company’s efforts must soon result in orders, or both the morale and the financial health of

the business will suffer.
A second risk is that White-owned companies may seek to cash in on the increasing

apportionments through formation of joint ventures with minority-owned concerns. Of course,

in many instances there are legitimate reasons for joint ventures; clearly, White and

minority enterprises can team up to acquire business that neither could acquire alone. But

civil rights groups and minority business owners have complained to Congress about

minorities being set up as “fronts” with White backing, rather than being accepted as full

partners in legitimate joint ventures.
Third, a minority enterprise that secures the business of one large corporate customer often

runs the danger of becoming—and remaining—dependent. Even in the best of circumstances,

fierce competition from larger, more established companies makes it difficult for small

concerns to broaden their customer bases: when such firms have nearly guaranteed orders from

a single corporate benefactor, they may truly have to struggle against complacency arising

from their current success.
1.The primary purpose of the passage is to
(A) present a commonplace idea and its inaccuracies
(B) describe a situation and its potential drawbacks
(C) propose a temporary solution to a problem
(D) analyze a frequent source of disagreement
(E) explore the implications of a finding
2.The passage supplies information that would answer which of the following ?
(A) What federal agencies have set percentage goals for the use of minority-owned businesses

in public works contracts?
(B) To which government agencies must businesses awarded federal contracts report their

efforts to find minority subcontractors?
(C) How widespread is the use of minority-owned concerns as “fronts” by White backers

seeking to obtain subcontracts?
(D) How many more minority-owned businesses were there in 1977 than in 1972?
(E) What is one set of conditions under which a small business might find itself financially

overextended?
3.According to the passage, civil rights activists maintain that one disadvantage under

which minority-owned businesses have traditionally had to labor is that they have
(A) been especially vulnerable to governmental mismanagement of the economy
(B) been denied bank loans at rates comparable to those afforded larger competitors
(C) not had sufficient opportunity to secure business created by large corporations
(D) not been able to advertise in those media that reach large numbers of potential

customers
(E) not had adequate representation in the centers of government power
4.The passage suggests that the failure of a large business to have its bids for

subcontracts result quickly in orders might cause it to
(A) experience frustration but not serious financial harm
(B) face potentially crippling fixed expenses
(C) have to record its efforts on forms filed with the government
(D) increase its spending with minority subcontractors
(E) revise its procedure for making bids for federal contracts and subcontracts
5.The author implies that a minority-owned concern that does the greater part of its

business with one large corporate customer should
(A) avoid competition with larger, more established concerns by not expanding
(B) concentrate on securing even more business from that corporation
(C) try to expand its customer base to avoid becoming dependent on the corporation
(D) pass on some of the work to be done for the corporation to other minority-owned concerns
(E) use its influence with the corporation to promote subcontracting with other minority

concerns
6.It can be inferred from the passage that, compared with the requirements of law, the

percentage goals set by “some federal and local agencies” (lines 14-15) are
(A) more popular with large corporations
(B) more specific
(C) less controversial
(D) less expensive to enforce
(E) easier to comply with
7.Which of the following, if true, would most weaken the author’s assertion that, in the

1970’s, corporate response to federal requirements (lines 18-19) was substantial
(A) Corporate contracts with minority-owned businesses totaled $2 billion in 1979.
(B) Between 1970 and 1972, corporate contracts with minority-owned businesses declined by 25

percent.
(C) The figures collected in 1977 underrepresented the extent of corporate contracts with

minority-owned businesses.
(D) The estimate of corporate spending with minority-owned businesses in 1980 is

approximately $10 million too high.
(E) The $1.1 billion represented the same percentage of total corporate spending in 1977 as

did $77 million in 1972.
8.The author would most likely agree with which of the following statements about corporate

response to working with minority subcontractors?
(A) Annoyed by the proliferation of “front” organizations, corporations are likely to reduce

their efforts to work with minority-owned subcontractors in the near future.
(B) Although corporations showed considerable interest in working with minority businesses

in the 1970’s, their aversion to government paperwork made them reluctant to pursue many

government contracts.
(C) The significant response of corporations in the 1970’s is likely to be sustained and

conceivably be increased throughout the 1980’s.
(D) Although corporations are eager to cooperate with minority-owned businesses, a shortage

of capital in the 1970’s made substantial response impossible.
(E) The enormous corporate response has all but eliminated the dangers of over-expansion

that used to plague small minority-owned businesses.
Passage 2 (2/63)
Woodrow Wilson was referring to the liberal idea of the economic market when he said that

the free enterprise system is the most efficient economic system. Maximum freedom means

maximum productiveness; our “openness” is to be the measure of our stability. Fascination

with this ideal has made Americans defy the “Old World” categories of settled possessiveness

versus unsettling deprivation, the cupidity of retention versus the cupidity of seizure, a

“status quo” defended or attacked. The United States, it was believed, had no status quo

ante. Our only “station” was the turning of a stationary wheel, spinning faster and faster.

We did not base our system on property but opportunity—which meant we based it not on

stability but on mobility. The more things changed, that is, the more rapidly the wheel

turned, the steadier we would be. The conventional picture of class politics is composed of

the Haves, who want a stability to keep what they have, and the Have-Nots, who want a touch

of instability and change in which to scramble for the things they have not. But Americans

imagined a condition in which speculators, self-makers, runners are always using the new

opportunities given by our land. These economic leaders (front-runners) would thus be mainly

agents of change. The nonstarters were considered the ones who wanted stability, a strong

referee to give them some position in the race, a regulative hand to calm manic speculation;

an authority that can call things to a halt, begin things again from compensatorily

staggered “starting lines.”
“Reform” in America has been sterile because it can imagine no change except through the

extension of this metaphor of a race, wider inclusion of competitors, “a piece of the

action,” as it were, for the disenfranchised. There is no attempt to call off the race.

Since our only stability is change, America seems not to honor the quiet work that achieves

social interdependence and stability. There is, in our legends, no heroism of the office

clerk, no stable industrial work force of the people who actually make the system work.

There is no pride in being an employee (Wilson asked for a return to the when everyone was

an employer). There has been no boasting about our social workers—they are merely signs of

the system’s failure, of opportunity denied or not taken, of things to be eliminated. We

have no pride in our growing interdependence, in the fact that our system can serve others,

that we are able to help those in need; empty boasts from the past make us ashamed of our

present achievements, make us try to forget or deny them, move away from them. There is no

honor but in the Wonderland race we must all run, all trying to win, none winning in the end

(for there is no end).
1.The primary purpose of the passage is to
(A) criticize the inflexibility of American economic mythology
(B) contrast “Old World” and “New World” economic ideologies
(C) challenge the integrity of traditional political leaders
(D) champion those Americans whom the author deems to be neglected
(E) suggest a substitute for the traditional metaphor of a race
2.According to the passage, “Old World” values were based on
(A) ability
(B) property
(C) family connections
(D) guild hierarchies
(E) education
3.In the context of the author’s discussion of regulating change, which of the following

could be most probably regarded as a “strong referee” (line 30) in the United States?
(A) A school principal
(B) A political theorist
(C) A federal court judge
(D) A social worker
(E) A government inspector
4.The author sets off the word “Reform” (line 35) with quotation marks in order to
(A) emphasize its departure from the concept of settled possessiveness
(B) show his support for a systematic program of change
(C) underscore the flexibility and even amorphousness of United States society
(D) indicate that the term was one of Wilson’s favorites
(E) assert that reform in the United States has not been fundamental
5.It can be inferred from the passage that the author most probably thinks that giving the

disenfranchised “a piece of the action” (line 38) is
(A) a compassionate, if misdirected, legislative measure
(B) an example of Americans’ resistance to profound social change
(C) an innovative program for genuine social reform
(D) a monument to the efforts of industrial reformers
(E) a surprisingly “Old World” remedy for social ills
6.Which of the following metaphors could the author most appropriately use to summarize his

own assessment of the American economic system (lines 35-60)?
(A) A windmill
(B) A waterfall
(C) A treadmill
(D) A gyroscope
(E) A bellows
7.It can be inferred from the passage that Woodrow Wilson’s ideas about the economic market
(A) encouraged those who “make the system work” (lines 45-46)
(B) perpetuated traditional legends about America
(C) revealed the prejudices of a man born wealthy
(D) foreshadowed the stock market crash of 1929
(E) began a tradition of presidential proclamations on economics
8.The passage contains information that would answer which of the following ?
I. What techniques have industrialists used to manipulate a free market?
II.In what ways are “New World” and “Old World” economic policies similar?
III.Has economic policy in the United States tended to reward independent action?
(A) I only
(B) II only
(C) III only
(D) I and II only
(E) II and III only
9.Which of the following best expresses the author’s main point?
(A) Americans’ pride in their jobs continues to give them stamina today.
(B) The absence of a status quo ante has undermined United States economic structure.
(C) The free enterprise system has been only a useless concept in the United States.
(D) The myth of the American free enterprise system is seriously flawed.
(E) Fascination with the ideal of “openness” has made Americans a progressive people.
Passage 3 (3/63)
No very satisfactory account of the mechanism that caused the formation of the ocean basins

has yet been given. The traditional view supposes that the upper mantle of the earth behaves

as a liquid when it is subjected to small forces for long periods and that differences in

temperature under oceans and continents are sufficient to produce convection in the mantle

of the earth with rising convection currents under the mid-ocean ridges and sinking currents

under the continents. Theoretically, this convection would carry the continental plates

along as though they were on a conveyor belt and would provide the forces needed to produce

the split that occurs along the ridge. This view may be correct: it has the advantage that

the currents are driven by temperature differences that themselves depend on the position of

the continents. Such a back-coupling, in which the position of the moving plate has an

impact on the forces that move it, could produce complicated and varying motions.
On the other hand, the theory is implausible because convection does not normally occur

along lines, and it certainly does not occur along lines broken by frequent offsets or

changes in direction, as the ridge is. Also it is difficult to see how the theory applies to

the plate between the Mid-Atlantic Ridge and the ridge in the Indian Ocean. This plate is

growing on both sides, and since there is no intermediate trench, the two ridges must be

moving apart. It would be odd if the rising convection currents kept exact pace with them.

An alternative theory is that the sinking part of the plate, which is denser than the hotter

surrounding mantle, pulls the rest of the plate after it. Again it is difficult to see how

this applies to the ridge in the South Atlantic, where neither the African nor the American

plate has a sinking part.
Another possibility is that the sinking plate cools the neighboring mantle and produces

convection currents that move the plates. This last theory is attractive because it gives

some hope of explaining the enclosed seas, such as the Sea of Japan. These seas have a

typical oceanic floor, except that the floor is overlaid by several kilometers of sediment.

Their floors have probably been sinking for long periods. It seems possible that a sinking

current of cooled mantle material on the upper side of the plate might be the cause of such

deep basins. The enclosed seas are an important feature of the earth’s surface, and

seriously require explanation because, in addition to the enclosed seas that are developing

at present behind island arcs, there are a number of older ones of possibly similar origin,

such as the Gulf of Mexico, the Black Sea, and perhaps the North Sea.
1.According to the traditional view of the origin of the ocean basins, which of the

following is sufficient to move the continental plates?
(A) Increases in sedimentation on ocean floors
(B) Spreading of ocean trenches
(C) Movement of mid-ocean ridges
(D) Sinking of ocean basins
(E) Differences in temperature under oceans and continents
2.It can be inferred from the passage that, of the following, the deepest sediments would be

found in the
(A) Indian Ocean
(B) Black Sea
(C) Mid-Atlantic
(D) South Atlantic
(E) Pacific
3.The author refers to a “conveyor belt” in line 13 in order to
(A) illustrate the effects of convection in the mantle
(B) show how temperature differences depend on the positions of the continents
(C) demonstrate the linear nature of the Mid-Atlantic Ridge
(D) describe the complicated motions made possible by back-coupling
(E) account for the rising currents under certain mid-ocean ridges
3.The author regards the traditional view of the origin of the oceans with
(A) slight apprehension
(B) absolute indifference
(C) indignant anger
(D) complete disbelief
(E) guarded skepticism
4.According to the passage, which of the following are separated by a plate that is growing

on both sides?
(A) The Pacific Ocean and the Sea of Japan
(B) The South Atlantic Ridge and the North Sea Ridge
(C) The Gulf of Mexico and the South Atlantic Ridge
(D) The Mid-Atlantic Ridge and the Indian Ocean Ridge
(E) The Black Sea and the Sea of Japan
5.Which of the following, if it could be demonstrated, would most support the traditional

view of ocean formation?
(A) Convection usually occurs along lines.
(B) The upper mantle behaves as a dense solid.
(C) Sedimentation occurs at a constant rate.
(D) Sinking plates cool the mantle.
(E) Island arcs surround enclosed seas.
6.According to the passage, the floor of the Black Sea can best be compared to a
(A) rapidly moving conveyor belt
(B) slowly settling foundation
(C) rapidly expanding balloon
(D) violently erupting volcano
(E) slowly eroding mountain
7.Which of the following titles would best describe the content of the passage?
(A) A Description of the Oceans of the World
(B) Several Theories of Ocean Basin Formation
(C) The Traditional View of the Oceans
(D) Convection and Ocean Currents
(E) Temperature Differences among the Oceans of the World
Passage 4 (4/63)
The fossil remains of the first flying vertebrates, the pterosaurs, have intrigued

paleontologists for more than two centuries. How such large creatures, which weighed in some

cases as much as a piloted hang-glider and had wingspans from 8 to 12 meters, solved the

problems of powered flight, and exactly what these creatures were—reptiles or birds—are

among the scientists have puzzled over.
Perhaps the least controversial assertion about the pterosaurs is that they were reptiles.

Their skulls, pelvises, and hind feet are reptilian. The anatomy of their wings suggests

that they did not evolve into the class of birds. In pterosaurs a greatly elongated fourth

finger of each forelimb supported a wing-like membrane. The other fingers were short and

reptilian, with sharp claws. In birds the second finger is the principal strut of the wing,

which consists primarily of feathers. If the pterosaurs walked on all fours, the three short

fingers may have been employed for grasping. When a pterosaur walked or remained stationary,

the fourth finger, and with it the wing, could only turn upward in an extended inverted

V-shape along each side of the animal’s body.
The pterosaurs resembled both birds and bats in their overall structure and proportions.

This is not surprising because the design of any flying vertebrate is subject to aerodynamic

constraints. Both the pterosaurs and the birds have hollow bones, a feature that represents

a savings in weight. In the birds, however, these bones are reinforced more massively by

internal struts.
Although scales typically cover reptiles, the pterosaurs probably had hairy coats. T. H.

Huxley reasoned that flying vertebrates must have been warm-blooded because flying implies a

high rate of metabolism, which in turn implies a high internal temperature. Huxley

speculated that a coat of hair would insulate against loss of body heat and might streamline

the body to reduce drag in flight. The recent discovery of a pterosaur specimen covered in

long, dense, and relatively thick hairlike fossil material was the first clear evidence that

his reasoning was correct.
Efforts to explain how the pterosaurs became airborne have led to suggestions that they

launched themselves by jumping from cliffs, by dropping from trees, or even by rising into

light winds from the crests of waves. Each hypothesis has its difficulties. The first

wrongly assumes that the pterosaurs’ hind feet resembled a bat’s and could serve as hooks by

which the animal could hang in preparation for flight. The second hypothesis seems unlikely

because large pterosaurs could not have landed in trees without damaging their wings. The

third calls for high waves to channel updrafts. The wind that made such waves however, might

have been too strong for the pterosaurs to control their flight once airborne.
1.It can be inferred from the passage that scientists now generally agree that the
(A) enormous wingspan of the pterosaurs enabled them to fly great distances
(B) structure of the skeleton of the pterosaurs suggests a close evolutionary relationship

to bats
(C) fossil remains of the pterosaurs reveal how they solved the problem of powered flight
(D) pterosaurs were reptiles
(E) pterosaurs walked on all fours
2.The author views the idea that the pterosaurs became airborne by rising into light winds

created by waves as
(A) revolutionary
(B) unlikely
(C) unassailable
(D) probable
(E) outdated
3.According to the passage, the skeleton of a pterosaur can be distinguished from that of a

bird by the
(A) size of its wingspan
(B) presence of hollow spaces in its bones
(C) anatomic origin of its wing strut
(D) presence of hooklike projections on its hind feet
(E) location of the shoulder joint joining the wing to its body
4.The ideas attributed to T. H. Huxley in the passage suggest that he would most likely

agree with which of the following statements?
(A) An animal’s brain size has little bearing on its ability to master complex behaviors.
(B) An animal’s appearance is often influenced by environmental requirements and physical

capabilities.
(C) Animals within a given family group are unlikely to change their appearance dramatically

over a period of .
(D) The origin of flight in vertebrates was an accidental development rather than the

outcome of specialization or adaptation.
(E) The pterosaurs should be classified as birds, not reptiles.
5.It can be inferred from the passage that which of the following is characteristic of the

pterosaurs?
(A) They were unable to fold their wings when not in use.
(B) They hung upside down from branches as bats do before flight.
(C) They flew in order to capture prey.
(D) They were an early stage in the evolution of the birds.
(E) They lived primarily in a forest-like habitat.
6.Which of the following best describes the organization of the last paragraph of the

passage?
(A) New evidence is introduced to support a traditional point of view.
(B) Three explanations for a phenomenon are presented, and each is disputed by means of

specific information.
(C) Three hypotheses are outlined, and evidence supporting each is given.
(D) Recent discoveries are described, and their implications for future study are projected.
(E) A summary of the material in the preceding paragraphs is presented, and conclusions are

drawn.
7.It can be inferred from the passage that some scientists believe that pterosaurs
(A) lived near large bodies of water
(B) had sharp teeth for tearing food
(C) were attacked and eaten by larger reptiles
(D) had longer tails than many birds
(E) consumed twice their weight daily to maintain their body temperature
Passage 5 (5/63)
How many really suffer as a result of labor market problems? This is one of the most

critical yet contentious social policy . In many ways, our social statistics exaggerate the

degree of hardship. Unemployment does not have the same dire consequences today as it did in

the 1930’s when most of the unemployed were primary breadwinners, when income and earnings

were usually much closer to the margin of subsistence, and when there were no countervailing

social programs for those failing in the labor market. Increasing affluence, the rise of

families with more than one wage earner, the growing predominance of secondary earners among

the unemployed, and improved social welfare protection have unquestionably mitigated the

consequences of joblessness. Earnings and income data also overstate the dimensions of

hardship. Among the millions with hourly earnings at or below the minimum wage level, the

overwhelming majority are from multiple-earner, relatively affluent families. Most of those

counted by the poverty statistics are elderly or handicapped or have family responsibilities

which keep them out of the labor force, so the poverty statistics are by no means an

accurate indicator of labor market pathologies.
Yet there are also many ways our social statistics underestimate the degree of

labor-market-related hardship. The unemployment counts exclude the millions of fully

employed workers whose wages are so low that their families remain in poverty. Low wages and

repeated or prolonged unemployment frequently interact to undermine the capacity for

self-support. Since the number experiencing joblessness at some during the year is several

s the number unemployed in any month, those who suffer as a result of forced idleness can

equal or exceed average annual unemployment, even though only a minority of the jobless in

any month really suffer. For every person counted in the monthly unemployment tallies, there

is another working part- because of the inability to find full- work, or else outside the

labor force but wanting a job. Finally, income transfers in our country have always focused

on the elderly, disabled, and dependent, neglecting the needs of the working poor, so that

the dramatic expansion of cash and in-kind transfers does not necessarily mean that those

failing in the labor market are adequately protected.
As a result of such contradictory evidence, it is uncertain whether those suffering

seriously as a result of labor market problems number in the hundreds of thousands or the

tens of millions, and, hence, whether high levels of joblessness can be tolerated or must be

countered by job creation and economic stimulus. There is only one area of agreement in this

debate—that the existing poverty, employment, and earnings statistics are inadequate for one

their primary applications, measuring the consequences of labor market problems.
1.Which of the following is the principal topic of the passage?
(A) What causes labor market pathologies that result in suffering
(B) Why income measures are imprecise in measuring degrees of poverty
(C) Which of the currently used statistical procedures are the best for estimating the

incidence of hardship that is due to unemployment
(D) Where the areas of agreement are among poverty, employment, and earnings figures
(E) How social statistics give an unclear picture of the degree of hardship caused by low

wages and insufficient employment opportunities
2.The author uses “labor market problems” in lines 1-2 to refer to which of the following?
(A) The overall causes of poverty
(B) Deficiencies in the training of the work force
(C) Trade relationships among producers of goods
(D) Shortages of jobs providing adequate income
(E) Strikes and inadequate supplies of labor
3.The author contrasts the 1930’s with the present in order to show that
(A) more people were unemployed in the 1930’s
(B) unemployment now has less severe effects
(C) social programs are more needed now
(D) there now is a greater proportion of elderly and handicapped people among those in

poverty
(E) poverty has increased since the 1930’s
4.Which of the following proposals best responds to the issues raised by the author?
(A) Innovative programs using multiple approaches should be set up to reduce the level of

unemployment.
(B) A compromise should be found between the positions of those who view joblessness as an

evil greater than economic control and those who hold the opposite view.
(C) New statistical indices should be developed to measure the degree to which unemployment

and inadequately paid employment cause suffering.
(D) Consideration should be given to the ways in which statistics can act as partial causes

of the phenomena that they purport to measure.
(E) The labor force should be restructured so that it corresponds to the range of job

vacancies.
5.The author’s purpose in citing those who are repeatedly unemployed during a twelve-month

period is most probably to show that
(A) there are several factors that cause the payment of low wages to some members of the

labor force
(B) unemployment statistics can underestimate the hardship resulting from joblessness
(C) recurrent inadequacies in the labor market can exist and can cause hardships for

individual workers
(D) a majority of those who are jobless at any one to not suffer severe hardship
(E) there are fewer individuals who are without jobs at some during a year than would be

expected on the basis of monthly unemployment figures
6.The author states that the mitigating effect of social programs involving income transfers

on the income level of low-income people is often not felt by
(A) the employed poor
(B) dependent children in single-earner families
(C) workers who become disabled
(D) retired workers
(E) full- workers who become unemployed
7.According to the passage, one factor that causes unemployment and earnings figures to

overpredict the amount of economic hardship is the
(A) recurrence of periods of unemployment for a group of low-wage workers
(B) possibility that earnings may be received from more than one job per worker
(C) fact that unemployment counts do not include those who work for low wages and remain

poor
(D) establishment of a system of record-keeping that makes it possible to compile poverty

statistics
(E) prevalence, among low-wage workers and the unemployed, of members of families in which

others are employed
8.The conclusion stated in lines 33-39 about the number of people who suffer as a result of

forced idleness depends primarily on the point that
(A) in s of high unemployment, there are some people who do not remain unemployed for long
(B) the capacity for self-support depends on receiving moderate-to-high wages
(C) those in forced idleness include, besides the unemployed, both underemployed part-

workers and those not actively seeking work
(D) at different s during the year, different people are unemployed
(E) many of those who are affected by unemployment are dependents of unemployed workers
9.Which of the following, if true, is the best criticism of the author’s argument concerning

why poverty statistics cannot properly be used to show the effects of problems in the labor

market?
(A) A short-term increase in the number of those in poverty can indicate a shortage of jobs

because the basic number of those unable to accept employment remains approximately

constant.
(B) For those who are in poverty as a result of joblessness, there are social programs

available that provide a minimum standard of living.
(C) Poverty statistics do not consistently agree with earnings statistics, when each is

taken as a measure of hardship resulting from unemployment.
(D) The elderly and handicapped categories include many who previously were employed in the

labor market.
(E) Since the labor market is global in nature, poor workers in one country are competing

with poor workers in another with respect to the level of wages and the existence of jobs.
Passage 6 (6/63)
In the eighteenth century, Japan’s feudal overlords, from the shogun to the humblest

samurai, found themselves under financial stress. In part, this stress can be attributed to

the overlords’ failure to adjust to a rapidly expanding economy, but the stress was also due

to factors beyond the overlords’ control. Concentration of the samurai in castle-towns had

acted as a stimulus to trade. Commercial efficiency, in turn, had put temptations in the way

of buyers. Since most samurai had been reduced to idleness by years of peace, encouraged to

engage in scholarship and martial exercises or to perform administrative tasks that took

little , it is not surprising that their tastes and habits grew expensive. Overlords’

income, despite the increase in rice production among their tenant farmers, failed to keep

pace with their expenses. Although shortfalls in overlords’ income resulted almost as much

from laxity among their tax collectors (the nearly inevitable outcome of hereditary

office-holding) as from their higher standards of living, a misfortune like a fire or flood,

bringing an increase in expenses or a drop in revenue, could put a domain in debt to the

city rice-brokers who handled its finances. Once in debt, neither the individual samurai nor

the shogun himself found it easy to recover.
It was difficult for individual samurai overlords to increase their income because the

amount of rice that farmers could be made to pay in taxes was not unlimited, and since the

income of Japan’s central government consisted in part of taxes collected by the shogun from

his huge domain, the government too was constrained. Therefore, the Tokugawa shoguns began

to look to other sources for revenue. Cash profits from government-owned mines were already

on the decline because the most easily worked deposits of silver and gold had been

exhausted, although debasement of the coinage had compensated for the loss. Opening up new

farmland was a possibility, but most of what was suitable had already been exploited and

further reclamation was technically unfeasible. Direct taxation of the samurai themselves

would be politically dangerous. This left the shoguns only commerce as a potential source of

government income.
Most of the country’s wealth, or so it seemed, was finding its way into the hands of city

merchants. It appeared reasonable that they should contribute part of that revenue to ease

the shogun’s burden of financing the state. A means of obtaining such revenue was soon found

by levying forced loans, known as goyo-kin; although these were not taxes in the strict

sense, since they were irregular in timing and arbitrary in amount, they were high in yield.

Unfortunately, they pushed up prices. Thus, regrettably, the Tokugawa shoguns’ search for

solvency for the government made it increasingly difficult for individual Japanese who lived

on fixed stipends to make ends meet.
1.The passage is most probably an excerpt from
(A) an economic history of Japan
(B) the memoirs of a samurai warrior
(C) a modern novel about eighteenth-century Japan
(D) an essay contrasting Japanese feudalism with its Western counterpart
(E) an introduction to a collection of Japanese folktales
2.Which of the following financial situations is most analogous to the financial situation

in which Japan’s Tokugawa shoguns found themselves in the eighteenth century?
(A) A small business borrows heavily to invest in new equipment, but is able to pay off its

debt early when it is awarded a lucrative government contract.
(B) Fire destroys a small business, but insurance covers the cost of rebuilding.
(C) A small business is turned down for a loan at a local bank because the owners have no

credit history.
(D) A small business has to struggle to meet operating expenses when its profits decrease.
(E) A small business is able to cut back sharply on spending through greater commercial

efficiency and thereby compensate for a loss of revenue.
3.Which of the following best describes the attitude of the author toward the samurai

discussed in lines 11-16?
(A) Warmly approving
(B) Mildly sympathetic
(C) Bitterly disappointed
(D) Harshly disdainful
(E) Profoundly shocked
4.According to the passage, the major reason for the financial problems experienced by

Japan’s feudal overlords in the eighteenth century was that
(A) spending had outdistanced income
(B) trade had fallen off
(C) profits from mining had declined
(D) the coinage had been sharply debased
(E) the samurai had concentrated in castle-towns
5.The passage implies that individual samurai did not find it easy to recover from debt for

which of the following reasons?
(A) Agricultural production had increased.
(B) Taxes were irregular in timing and arbitrary in amount.
(C) The Japanese government had failed to adjust to the needs of a changing economy.
(D) The domains of samurai overlords were becoming smaller and poorer as government revenues

increased.
(E) There was a limit to the amount in taxes that farmers could be made to pay.
6.The passage suggests that, in eighteenth-century Japan, the office of tax collector
(A) was a source of personal profit to the officeholder
(B) was regarded with derision by many Japanese
(C) remained within families
(D) existed only in castle-towns
(E) took up most of the officeholder’s
7.Which of the following could best be substituted for the word “This” in line 47 without

changing the meaning of the passage?
(A) The search of Japan’s Tokugawa shoguns for solvency
(B) The importance of commerce in feudal Japan
(C) The unfairness of the tax structure in eighteenth century Japan
(D) The difficulty of increasing government income by other means
(E) The difficulty experienced by both individual samurai and the shogun himself in

extricating themselves from debt
8.The passage implies that which of the following was the primary reason why the Tokugawa

shoguns turned to city merchants for help in financing the state?
(A) A series of costly wars had depleted the national treasury.
(B) Most of the country’s wealth appeared to be in city merchants’ hands.
(C) Japan had suffered a series of economic reversals due to natural disasters such as

floods.
(D) The merchants were already heavily indebted to the shoguns.
(E) Further reclamation of land would not have been economically advantageous.
9.According to the passage, the actions of the Tokugawa shoguns in their search for solvency

for the government were regrettable because those actions
(A) raised the cost of living by pushing up prices
(B) resulted in the exhaustion of the most easily worked deposits of silver and gold
(C) were far lower in yield than had originally been anticipated
(D) did not succeed in reducing government spending
(E) acted as a deterrent to trade
Passage 7 (7/63)
Between the eighth and eleventh centuries A. D., the Byzantine Empire staged an almost

unparalleled economic and cultural revival, a recovery that is all the more striking because

it followed a long period of severe internal decline. By the early eighth century, the

empire had lost roughly two-thirds of the territory it had possessed in the year 600, and

its remaining area was being raided by Arabs and Bulgarians, who at s threatened to take

Constantinople and extinguish the empire altogether. The wealth of the state and its

subjects was greatly diminished, and artistic and literary production had virtually ceased.

By the early eleventh century, however, the empire had regained almost half of its lost

possessions, its new frontiers were secure, and its influence extended far beyond its

borders. The economy had recovered, the treasury was full, and art and scholarship had

advanced.
To consider the Byzantine military, cultural, and economic advances as differentiated

aspects of a single phenomenon is reasonable. After all, these three forms of progress have

gone together in a number of states and civilizations. Rome under Augustus and fifth-century

Athens provide the most obvious examples in antiquity. Moreover, an examination of the

apparent sequential connections among military, economic, and cultural forms of progress

might help explain the dynamics of historical change.
The common explanation of these apparent connections in the case of Byzantium would run like

this: when the empire had turned back enemy raids on its own territory and had begun to raid

and conquer enemy territory, Byzantine resources naturally expanded and more money became

available to patronize art and literature. Therefore, Byzantine military achievements led to

economic advances, which in turn led to cultural revival.
No doubt this hypothetical pattern did apply at s during the course of the recovery. Yet it

is not clear that military advances invariably came first, economic advances second, and

intellectual advances third. In the 860’s the Byzantine Empire began to recover from Arab

incursions so that by 872 the military balance with the Abbasid Caliphate had been

permanently altered in the empire’s favor. The beginning of the empire’s economic revival,

however, can be placed between 810 and 830. Finally, the Byzantine revival of learning

appears to have begun even earlier. A number of notable scholars and writers appeared by 788

and, by the last decade of the eighth century, a cultural revival was in full bloom, a

revival that lasted until the fall of Constantinople in 1453. Thus the commonly expected

order of military revival followed by economic and then by cultural recovery was reversed in

Byzantium. In fact, the revival of Byzantine learning may itself have influenced the

subsequent economic and military expansion.
1.Which of the following best states the central idea of the passage?
(A) The Byzantine Empire was a unique case in which the usual order of military and economic

revival preceding cultural revival was reversed.
(B) The economic, cultural, and military revival in the Byzantine Empire between the eighth

and eleventh centuries was similar in its order to the sequence of revivals in Augustan Rome

and fifth century Athens.
(C) After 810 Byzantine economic recovery spurred a military and, later, cultural expansion

that lasted until 1453.
(D) The eighth-century revival of Byzantine learning is an inexplicable phenomenon, and its

economic and military precursors have yet to be discovered.
(E) The revival of the Byzantine Empire between the eighth and eleventh centuries shows

cultural rebirth preceding economic and military revival, the reverse of the commonly

accepted order of progress.
2.The primary purpose of the second paragraph is which of the following?
(A) To establish the uniqueness of the Byzantine revival
(B) To show that Augustan Rome and fifth-century Athens are examples of cultural, economic,

and military expansion against which all subsequent cases must be measured
(C) To suggest that cultural, economic, and military advances have tended to be closely

interrelated in different societies
(D) To argue that, while the revivals of Augustan Rome and fifth-century Athens were

similar, they are unrelated to other historical examples
(E) To indicate that, wherever possible, historians should seek to make comparisons with the

earliest chronological examples of revival
3.It can be inferred from the passage that by the eleventh century the Byzantine military

forces
(A) had reached their peak and begun to decline
(B) had eliminated the Bulgarian army
(C) were comparable in size to the army of Rome under Augustus
(D) were strong enough to withstand the Abbasid Caliphate’s military forces
(E) had achieved control of Byzantine governmental structures
4.It can be inferred from the passage that the Byzantine Empire sustained significant

territorial losses
(A) in 600
(B) during the seventh century
(C) a century after the cultural achievements of the Byzantine Empire had been lost
(D) soon after the revival of Byzantine learning
(E) in the century after 873
5.In the third paragraph, the author most probably provides an explanation of the apparent

connections among economic, military, and cultural development in order to
(A) suggest that the process of revival in Byzantium accords with this model
(B) set up an order of events that is then shown to be not generally applicable to the case

of Byzantium
(C) cast aspersions on traditional historical scholarship about Byzantium
(D) suggest that Byzantium represents a case for which no historical precedent exists
(E) argue that military conquest is the paramount element in the growth of empires
6.Which of the following does the author mention as crucial evidence concerning the manner

in which the Byzantine revival began?
(A) The Byzantine military revival of the 860’s led to economic and cultural advances.
(B) The Byzantine cultural revival lasted until 1453.
(C) The Byzantine economic recovery began in the 900’s.
(D) The revival of Byzantine learning began toward the end of the eighth century.
(E) By the early eleventh century the Byzantine Empire had regained much of its lost

territory.
7.According to the author, “The common explanation” (line 28) of connections between

economic, military, and cultural development is
(A) revolutionary and too new to have been applied to the history of the Byzantine Empire
(B) reasonable, but an antiquated theory of the nature of progress
(C) not applicable to the Byzantine revival as a whole, but does perhaps accurately describe

limited periods during the revival
(D) equally applicable to the Byzantine case as a whole and to the history of military,

economic, and cultural advances in ancient Greece and Rome
(E) essentially not helpful, because military, economic, and cultural advances are part of a

single phenomenon
Passage 8 (8/63)
Virtually everything astronomers known about objects outside the solar system is based on

the detection of photons—quanta of electromagnetic radiation. Yet there is another form of

radiation that permeates the universe: neutrinos. With (as its name implies) no electric

charge, and negligible mass, the neutrino interacts with other particles so rarely that a

neutrino can cross the entire universe, even traversing substantial aggregations of matter,

without being absorbed or even deflected. Neutrinos can thus escape from regions of space

where light and other kinds of electromagnetic radiation are blocked by matter. Furthermore,

neutrinos carry with them information about the site and circumstances of their production:

therefore, the detection of cosmic neutrinos could provide new information about a wide

variety of cosmic phenomena and about the history of the universe.
But how can scientists detect a particle that interacts so infrequently with other matter?

Twenty-five years passed between Pauli’s hypothesis that the neutrino existed and its actual

detection: since then virtually all research with neutrinos has been with neutrinos created

artificially in large particle accelerators and studied under neutrino microscopes. But a

neutrino telescope, capable of detecting cosmic neutrinos, is difficult to construct. No

apparatus can detect neutrinos unless it is extremely massive, because great mass is

synonymous with huge numbers of nucleons (neutrons and protons), and the more massive the

detector, the greater the probability of one of its nucleon’s reacting with a neutrino. In

addition, the apparatus must be sufficiently shielded from the interfering effects of other

particles.
Fortunately, a group of astrophysicists has proposed a means of detecting cosmic neutrinos

by harnessing the mass of the ocean. Named DUMAND, for Deep Underwater Muon and Neutrino

Detector, the project calls for placing an array of light sensors at a depth of five

kilometers under the ocean surface. The detecting medium is the seawater itself: when a

neutrino interacts with a particle in an atom of seawater, the result is a cascade of

electrically charged particles and a flash of light that can be detected by the sensors. The

five kilometers of seawater above the sensors will shield them from the interfering effects

of other high-energy particles raining down through the atmosphere.
The strongest motivation for the DUMAND project is that it will exploit an important source

of information about the universe. The extension of astronomy from visible light to radio

waves to x-rays and gamma rays never failed to lead to the discovery of unusual objects such

as radio galaxies, quasars, and pulsars. Each of these discoveries came as a surprise.

Neutrino astronomy will doubtless bring its own share of surprises.
1.Which of the following titles best summarizes the passage as a whole?
(A) At the Threshold of Neutrino Astronomy
(B) Neutrinos and the History of the Universe
(C) The Creation and Study of Neutrinos
(D) The DUMAND System and How It Works
(E) The Properties of the Neutrino
2.With which of the following statements regarding neutrino astronomy would the author be

most likely to agree?
(A) Neutrino astronomy will supersede all present forms of astronomy.
(B) Neutrino astronomy will be abandoned if the DUMAND project fails.
(C) Neutrino astronomy can be expected to lead to major breakthroughs in astronomy.
(D) Neutrino astronomy will disclose phenomena that will be more surprising than past

discoveries.
(E) Neutrino astronomy will always be characterized by a large lag between hypothesis and

experimental confirmation.
3.In the last paragraph, the author describes the development of astronomy in order to
(A) suggest that the potential findings of neutrino astronomy can be seen as part of a

series of astronomical successes
(B) illustrate the role of surprise in scientific discovery
(C) demonstrate the effectiveness of the DUMAND apparatus in detecting neutrinos
(D) name some cosmic phenomena that neutrino astronomy will illuminate
(E) contrast the motivation of earlier astronomers with that of the astrophysicists working

on the DUMAND project
4.According to the passage, one advantage that neutrinos have for studies in astronomy is

that they
(A) have been detected for the last twenty-five years
(B) possess a variable electric charge
(C) are usually extremely massive
(D) carry information about their history with them
(E) are very similar to other electromagnetic particles
5.According to the passage, the primary use of the apparatus mentioned in lines 24-32 would

be to
(A) increase the mass of a neutrino
(B) interpret the information neutrinos carry with them
(C) study the internal structure of a neutrino
(D) see neutrinos in distant regions of space
(E) detect the presence of cosmic neutrinos
6.The passage states that interactions between neutrinos and other matter are
(A) rare
(B) artificial
(C) undetectable
(D) unpredictable
(E) hazardous
7.The passage mentions which of the following as a reason that neutrinos are hard to detect?
(A) Their pervasiveness in the universe
(B) Their ability to escape from different regions of space
(C) Their inability to penetrate dense matter
(D) The similarity of their structure to that of nucleons
(E) The infrequency of their interaction with other matter
8.According to the passage, the interaction of a neutrino with other matter can produce
(A) particles that are neutral and massive
(B) a form of radiation that permeates the universe
(C) inaccurate information about the site and circumstances of the neutrino’s production
(D) charged particles and light
(E) a situation in which light and other forms of electromagnetic radiation are blocked
9.According to the passage, one of the methods used to establish the properties of neutrinos

was
(A) detection of photons
(B) observation of the interaction of neutrinos with gamma rays
(C) observation of neutrinos that were artificially created
(D) measurement of neutrinos that interacted with particles of seawater
(E) experiments with electromagnetic radiation
Passage 9 (9/63)
Most economists in the United States seem captivated by the spell of the free market.

Consequently, nothing seems good or normal that does not accord with the requirements of the

free market. A price that is determined by the seller or, for that matter, established by

anyone other than the aggregate of consumers seems pernicious. Accordingly, it requires a

major act of will to think of price-fixing (the determination of prices by the seller) as

both “normal” and having a valuable economic function. In fact, price-fixing is normal in

all industrialized societies because the industrial system itself provides, as an effortless

consequence of its own development, the price-fixing that it requires. Modern industrial

planning requires and rewards great size. Hence, a comparatively small number of large firms

will be competing for the same group of consumers. That each large firm will act with

consideration of its own needs and thus avoid selling its products for more than its

competitors charge is commonly recognized by advocates of free-market economic theories. But

each large firm will also act with full consideration of the needs that it has in common

with the other large firms competing for the same customers. Each large firm will thus avoid

significant price-cutting, because price-cutting would be prejudicial to the common interest

in a stable demand for products. Most economists do not see price-fixing when it occurs

because they expect it to be brought about by a number of explicit agreements among large

firms; it is not.
Moreover, those economists who argue that allowing the free market to operate without

interference is the most efficient method of establishing prices have not considered the

economies of non-socialist countries other than the United states. These economies employ

intentional price-fixing, usually in an overt fashion. Formal price-fixing by cartel and

informal price-fixing by agreements covering the members of an industry are commonplace.

Were there something peculiarly efficient about the free market and inefficient about

price-fixing, the countries that have avoided the first and used the second would have

suffered drastically in their economic development. There is no indication that they have.
Socialist industry also works within a framework of controlled prices. In the early 1970’s,

the Soviet Union began to give firms and industries some of the flexibility in adjusting

prices that a more informal evolution has accorded the capitalist system. Economists in the

United States have hailed the change as a return to the free market. But Soviet firms are no

more subject to prices established by a free market over which they exercise little

influence than are capitalist firms; rather, Soviet firms have been given the power to fix

prices.
1.The primary purpose of the passage is to
(A) refute the theory that the free market plays a useful role in the development of

industrialized societies
(B) suggest methods by which economists and members of the government of the United States

can recognize and combat price-fixing by large firms
(C) show that in industrialized societies price-fixing and the operation of the free market

are not only compatible but also mutually beneficial
(D) explain the various ways in which industrialized societies can fix prices in order to

stabilize the free market
(E) argue that price-fixing, in one form or another, is an inevitable part of and benefit to

the economy of any industrialized society
2.The passage provides information that would answer which of the following about

price-fixing?
I. What are some of the ways in which prices can be fixed?
II. For what products is price-fixing likely to be more profitable that the operation of the

free market?
III.Is price-fixing more common in socialist industrialized societies or in non-socialist

industrialized societies?
(A) I only
(B) III only
(C) I and II only
(D) II and III only
(E) I, II, and III
3.The author’s attitude toward “Most economists in the United States”(line 1) can best be

described as
(A) spiteful and envious
(B) scornful and denunciatory
(C) critical and condescending
(D) ambivalent but deferential
(E) uncertain but interested
4.It can be inferred from the author’s argument that a price fixed by the seller “seems

pernicious” (line 7) because
(A) people do not have confidence in large firms
(B) people do not expect the government to regulate prices
(C) most economists believe that consumers as a group should determine prices
(D) most economists associate fixed prices with communist and socialist economies
(E) most economists believe that no one group should determine prices
5.The suggestion in the passage that price-fixing in industrialized societies is normal

arises from the author’s statement that price-fixing is
(A) a profitable result of economic development
(B) an inevitable result of the industrial system
(C) the result of a number of carefully organized decisions
(D) a phenomenon common to industrialized and non-industrialized societies
(E) a phenomenon best achieved cooperatively by government and industry
6.According to the author, price-fixing in non-socialist countries is often
(A) accidental but productive
(B) illegal but useful
(C) legal and innovative
(D) traditional and rigid
(E) intentional and widespread
7.According to the author, what is the result of the Soviet Union’s change in economic

policy in the 1970’s?
(A) Soviet firms show greater profit.
(B) Soviet firms have less control over the free market.
(C) Soviet firms are able to adjust to technological advances.
(D) Soviet firms have some authority to fix prices.
(E) Soviet firms are more responsive to the free market.
8.With which of the following statements regarding the behavior of large firms in

industrialized societies would the author be most likely to agree?
(A) The directors of large firms will continue to anticipate the demand for products.
(B) The directors of large firms are less interested in achieving a predictable level of

profit than in achieving a large profit.
(C) The directors of large firms will strive to reduce the costs of their products.
(D) Many directors of large firms believe that the government should establish the prices

that will be charged for products.
(E) Many directors of large firms believe that the price charged for products is likely to

increase annually.
9.In the passage, the author is primarily concerned with
(A) predicting the consequences of a practice
(B) criticizing a point of view
(C) calling attention to recent discoveries
(D) proposing a topic for research
(E) summarizing conflicting opinions
Passage 10 (10/63)
Caffeine, the stimulant in coffee, has been called “the most widely used psychoactive

substance on Earth.” Snyder, Daly and Bruns have recently proposed that caffeine affect

behavior by countering the activity in the human brain of a naturally occurring chemical

called adenosine. Adenosine normally depresses neuron firing in many areas of the brain. It

apparently does this by inhibiting the release of neurotransmitters, chemicals that carry

nerve impulses from one neuron to the next. Like many other agents that affect neuron

firing, adenosine must first bind to specific receptors on neuronal membranes. There are at

least two classes of these receptors, which have been designated A1 and A2. Snyder et al

propose that caffeine, which is structurally similar to adenosine, is able to bind to both

types of receptors, which prevents adenosine from attaching there and allows the neurons to

fire more readily than they otherwise would.
For many years, caffeine’s effects have been attributed to its inhibition of the production

of phosphodiesterase, an enzyme that breaks down the chemical called cyclic AMP. A number of

neurotransmitters exert their effects by first increasing cyclic AMP concentrations in

target neurons. Therefore, prolonged periods at the elevated concentrations, as might be

brought about by a phosphodiesterase inhibitor, could lead to a greater amount of neuron

firing and, consequently, to behavioral stimulation. But Snyder et al point out that the

caffeine concentrations needed to inhibit the production of phosphodiesterase in the brain

are much higher than those that produce stimulation. Moreover, other compounds that block

phosphodiesterase’s activity are not stimulants.
To buttress their case that caffeine acts instead by preventing adenosine binding, Snyder et

al compared the stimulatory effects of a series of caffeine derivatives with their ability

to dislodge adenosine from its receptors in the brains of mice. “In general,” they reported,

“the ability of the compounds to compete at the receptors correlates with their ability to

stimulate locomotion in the mouse; i.e., the higher their capacity to bind at the receptors,

the higher their ability to stimulate locomotion.” Theophylline, a close structural relative

of caffeine and the major stimulant in tea, was one of the most effective compounds in both

regards.
There were some apparent exceptions to the general correlation observed between

adenosine-receptor binding and stimulation. One of these was a compound called

3-isobutyl-1-methylxanthine (IBMX), which bound very well but actually depressed mouse

locomotion. Snyder et al suggests that this is not a major stumbling block to their

hypothesis. The problem is that the compound has mixed effects in the brain, a not unusual

occurrence with psychoactive drugs. Even caffeine, which is generally known only for its

stimulatory effects, displays this property, depressing mouse locomotion at very low

concentrations and stimulating it at higher ones.
1.The primary purpose of the passage is to
(A) discuss a plan for investigation of a phenomenon that is not yet fully understood
(B) present two explanations of a phenomenon and reconcile the differences between them
(C) summarize two theories and suggest a third theory that overcomes the problems

encountered in the first two
(D) describe an alternative hypothesis and provide evidence and arguments that support it
(E) challenge the validity of a theory by exposing the inconsistencies and contradictions in

it
2.Which of the following, if true, would most weaken the theory proposed by Snyder et al?
(A) At very low concentrations in the human brain, both caffeine and theophylline tend to

have depressive rather than stimulatory effects on human behavior.
(B) The ability of caffeine derivatives at very low concentrations to dislodge adenosine

from its receptors in mouse brains correlates well with their ability to stimulate mouse

locomotion at these low concentrations.
(C) The concentration of cyclic AMP in target neurons in the human brain that leads to

increased neuron firing can be produced by several different phosphodiesterase inhibitors in

addition to caffeine.
(D) The concentration of caffeine required to dislodge adenosine from its receptors in the

human brain is much greater than the concentration that produces behavioral stimulation in

humans.
(E) The concentration of IBMX required to dislodge adenosine from its receptors in mouse

brains is much smaller than the concentration that stimulates locomotion in the mouse.
3.According so Snyder et al, caffeine differs from adenosine in that caffeine
(A) stimulates behavior in the mouse and in humans, whereas adenosine stimulates behavior in

humans only
(B) has mixed effects in the brain, whereas adenosine has only a stimulatory effect
(C) increases cyclic AMP concentrations in target neurons, whereas adenosine decreases such

concentrations
(D) permits release of neurotransmitters when it is bound to adenosine receptors, whereas

adenosine inhibits such release
(E) inhibits both neuron firing and the production of phosphodiesterase when there is a

sufficient concentration in the brain, whereas adenosine inhibits only neuron firing
4.In response to experimental results concerning IBMX, Snyder et al contended that it is not

uncommon for psychoactive drugs to have
(A) mixed effects in the brain
(B) inhibitory effects on enzymes in the brain
(C) close structural relationships with caffeine
(D) depressive effects on mouse locomotion
(E) the ability to dislodge caffeine from receptors in the brain
5.The passage suggests that Snyder et al believe that if the older theory concerning

caffeine’s effects were correct, which of the following would have to be the case?
I. All neurotransmitters would increase the short-term concentration of cyclic AMP in target

neurons.
II. Substances other than caffeine that inhibit the production of phosphodiesterase would be

stimulants.
III. All concentration levels of caffeine that are high enough to produce stimulation would

also inhibit the production of phosphodiesterase.
(A) I only
(B) I and II only
(C) I and III only
(D) II and III only
(E) I, II, and III
6.According to Snyder et al, all of the following compounds can bind to specific receptors

in the brain EXCEPT
(A) IBMX
(B) caffeine
(C) adenosine
(D) theophylline
(E) phosphodiesterase
7.Snyder et al suggest that caffeine’s ability to bind to A1 and A2 receptors can be at

least partially attributed to which of the following?
(A) The chemical relationship between caffeine and phosphodiesterase
(B) The structural relationship between caffeine and adenosine
(C) The structural similarity between caffeine and neurotransmitters
(D) The ability of caffeine to stimulate behavior
(E) The natural occurrence of caffeine and adenosine in the brain
8.The author quotes Snyder et al in lines 38-43 most probably in order to
(A) reveal some of the assumptions underlying their theory
(B) summarize a major finding of their experiments
(C) point out that their experiments were limited to the mouse
(D) indicate that their experiments resulted only in general correlations
(E) refute the objections made by supporters of the older theory
9.The last paragraph of the passage performs which of the following functions?
(A) Describes a disconfirming experimental result and reports the explanation given by

Snyder et al in an attempt to reconcile this result with their theory.
(B) Specifies the basis for the correlation observed by Snyder et al and presents an

explanation in an attempt to make the correlation consistent with the operation of

psychoactive drugs other than caffeine.
(C) Elaborates the description of the correlation observed by Snyder et al and suggests an

additional explanation in an attempt to make the correlation consistent with the older

theory.
(D) Reports inconsistent experimental data and describes the method Snyder et al will use to

reanalyze this data.
(E) Provides an example of the hypothesis proposed by Snyder et al and relates this example

to caffeine’s properties.
Passage 11 (11/63)
Archaeology as a profession faces two major problems. First, it is the poorest of the poor.

Only paltry sums are available for excavating and even less is available for publishing the

results and preserving the sites once excavated. Yet archaeologists deal with priceless

objects every day. Second, there is the problem of illegal excavation, resulting in

museum-quality pieces being sold to the highest bidder.
I would like to make an outrageous suggestion that would at one stroke provide funds for

archaeology and reduce the amount of illegal digging. I would propose that scientific

archeological expeditions and governmental authorities sell excavated artifacts on the open

market. Such sales would provide substantial funds for the excavation and preservation of

archaeological sites and the publication of results. At the same , they would break the

illegal excavator’s grip on the market, thereby decreasing the inducement to engage in

illegal activities.
You might object that professionals excavate to acquire knowledge, not money. Moreover,

ancient artifacts are part of our global cultural heritage, which should be available for

all to appreciate, not sold to the highest bidder. I agree. Sell nothing that has unique

artistic merit or scientific value. But, you might reply everything that comes out of the

ground has scientific value. Here we part company. Theoretically, you may be correct in

claiming that every artifact has potential scientific value. Practically, you are wrong.
I refer to the thousands of pottery vessels and ancient lamps that are essentially

duplicates of one another. In one small excavation in Cyprus, archaeologists recently

uncovered 2,000 virtually indistinguishable small jugs in a single courtyard, Even precious

royal seal impressions known as l’melekh handles have been found in abundance—more than

4,000 examples so far.
The basements of museums are simply not large enough to store the artifacts that are likely

to be discovered in the future. There is not enough money even to catalogue the finds; as a

result, they cannot be found again and become as inaccessible as if they had never been

discovered. Indeed, with the help of a computer, sold artifacts could be more accessible

than are the pieces stored in bulging museum basements. Prior to sale, each could be

photographed and the list of the purchasers could be maintained on the computer. A purchaser

could even be required to agree to return the piece if it should become needed for

scientific purposes.
It would be unrealistic to suggest that illegal digging would stop if artifacts were sold on

the open market. But the demand for the clandestine product would be substantially reduced.

Who would want an unmarked pot when another was available whose provenance was known, and

that was dated stratigraphically by the professional archaeologist who excavated it?
1.The primary purpose of the passage is to propose
(A) an alternative to museum display of artifacts
(B) a way to curb illegal digging while benefiting the archaeological profession
(C) a way to distinguish artifacts with scientific value from those that have no such value
(D) the governmental regulation of archaeological sites
(E) a new system for cataloguing duplicate artifacts
2.The author implies that all of the following statements about duplicate artifacts are true

EXCEPT:
(A) A market for such artifacts already exists.
(B) Such artifacts seldom have scientific value.
(C) There is likely to be a continuing supply of such artifacts.
(D) Museums are well supplied with examples of such artifacts.
(E) Such artifacts frequently exceed in quality those already catalogued in museum

collections.
3.Which of the following is mentioned in the passage as a disadvantage of storing artifacts

in museum basements?
(A) Museum officials rarely allow scholars access to such artifacts.
(B) Space that could be better used for display is taken up for storage.
(C) Artifacts discovered in one excavation often become separated from each other.
(D) Such artifacts are often damaged by variations in temperature and humidity.
(E) Such artifacts’ often remain uncatalogued and thus cannot be located once they are put

in storage.
4.The author mentions the excavation in Cyprus (lines 31-34) to emphasize which of the

following points?
(A) Ancient lamps and pottery vessels are less valuable, although more rare, than royal seal

impressions.
(B) Artifacts that are very similar to each other present cataloguing difficulties to

archaeologists.
(C) Artifacts that are not uniquely valuable, and therefore could be sold, are available in

large quantities.
(D) Cyprus is the most important location for unearthing large quantities of salable

artifacts.
(E) Illegal sales of duplicate artifacts are wide-spread, particularly on the island of

Cyprus.
5.The author’s argument concerning the effect of the official sale of duplicate artifacts on

illegal excavation is based on which of the following assumptions?
(A) Prospective purchasers would prefer to buy authenticated artifacts.
(B) The price of illegally excavated artifacts would rise.
(C) Computers could be used to trace sold artifacts.
(D) Illegal excavators would be forced to sell only duplicate artifacts.
(E) Money gained from selling authenticated artifacts could be used to investigate and

prosecute illegal excavators.
6.The author anticipates which of the following initial objections to the adoption of his

proposal?
(A) Museum officials will become unwilling to store artifacts.
(B) An oversupply of salable artifacts will result and the demand for them will fall.
(C) Artifacts that would have been displayed in public places will be sold to private

collectors.
(D) Illegal excavators will have an even larger supply of artifacts for resale.
(E) Counterfeiting of artifacts will become more commonplace.
7.The author implies that which of the following would occur if duplicate artifacts were

sold on the open market?
I. Illegal excavation would eventually cease completely.
II. Cyprus would become the primary source of marketable duplicate artifacts.
III.Archaeologists would be able to publish the results of their excavations more frequently

than they currently do.
(A) I only
(B) III only
(C) I and II only
(D) II and III only
(E) I, II, and III
Passage 12 (12/63)
(This passage is excerpted from material published in 1980.)
Federal efforts to aid minority businesses began in the 1960’s when the Small Business

Administration (SBA) began making federally guaranteed loans and government-sponsored

management and technical assistance available to minority business enterprises. While this

program enabled many minority entrepreneurs to form new businesses, the results were

disappointing, since managerial inexperience, unfavorable locations, and capital shortages

led to high failure rates. Even years after the program was implemented, minority business

receipts were not quite two percent of the national economy’s total receipts.
Recently federal policymakers have adopted an approach intended to accelerate development of

the minority business sector by moving away from directly aiding small minority enterprises

and toward supporting larger, growth-oriented minority firms through intermediary companies.

In this approach, large corporations participate in the development of successful and stable

minority businesses by making use of government-sponsored venture capital. The capital is

used by a participating company to establish a Minority Enterprise Small Business Investment

Company or MESBIC. The MESBIC then provides capital and guidance to minority businesses that

have potential to become future suppliers or customers of the sponsoring company.
MESBIC’s are the result of the belief that providing established firms with easier access to

relevant management techniques and more job-specific experience, as well as substantial

amounts of capital, gives those firms a greater opportunity to develop sound business

foundations than does simply making general management experience and small amounts of

capital available. Further, since potential markets for the minority businesses already

exist through the sponsoring companies, the minority businesses face considerably less risk

in terms of location and market fluctuation. Following early financial and operating

problems, sponsoring corporations began to capitalize MESBIC’s far above the legal minimum

of $500,000 in order to generate sufficient income and to sustain the quality of management

needed. MESBIC’s are now emerging as increasingly important financing sources for minority

enterprises.
Ironically, MESBIC staffs, which usually consist of Hispanic and Black professionals, tend

to approach investments in minority firms more practically than do many MESBIC directors,

who are usually senior managers from sponsoring corporations. The latter often still think

mainly in terms of the “social responsibility approach” and thus seem to prefer deals that

are riskier and less attractive than normal investment criteria would warrant. Such

differences in viewpoint have produced uneasiness among many minority staff members, who

feel that minority entrepreneurs and businesses should be judged by established business

considerations. These staff members believe their point of view is closer to the original

philosophy of MESBIC’s and they are concerned that, unless a more prudent course is

followed, MESBIC directors may revert to policies likely to re-create the disappointing

results of the original SBA approach.
1.Which of the following best states the central idea of the passage?
(A) The use of MESBIC’s for aiding minority entrepreneurs seems to have greater potential

for success than does the original SBA approach.
(B) There is a crucial difference in point of view between the staff and directors of some

MESBIC’s.
(C) After initial problems with management and marketing, minority businesses have begun to

expand at a steady rate.
(D) Minority entrepreneurs wishing to form new businesses now have several equally

successful federal programs on which to rely.
(E) For the first since 1960, large corporations are making significant contributions to

the development of minority businesses.
2.According to the passage, the MESBIC approach differs from the SBA approach in that

MESBIC’s
(A) seek federal contracts to provide markets for minority businesses
(B) encourage minority businesses to provide markets for other minority businesses
(C) attempt to maintain a specified rate of growth in the minority business sector
(D) rely on the participation of large corporations to finance minority businesses
(E) select minority businesses on the basis of their location
3.Which of the following does the author cite to support the conclusion that the results of

the SBA program were disappointing?
(A) The small number of new minority enterprises formed as a result of the program
(B) The small number of minority enterprises that took advantage of the management and

technical assistance offered under the program
(C) The small percentage of the nation’s business receipts earned by minority enterprises

following the programs, implementation
(D) The small percentage of recipient minority enterprises that were able to repay federally

guaranteed loans made under the program
(E) The small number of minority enterprises that chose to participate in the program
4.Which of the following statements about the SBA program can be inferred from the passage?
(A) The maximum term for loans made to recipient businesses was 15 years.
(B) Business loans were considered to be more useful to recipient businesses than was

management and technical assistance.
(C) The anticipated failure rate for recipient businesses was significantly lower than the

rate that actually resulted.
(D) Recipient businesses were encouraged to relocate to areas more favorable for business

development.
(E) The capitalization needs of recipient businesses were assessed and then provided for

adequately.
5.Based on information in the passage, which of the following would be indicative of the

prisms of MESBIC staff members?
I.A reluctance to invest in minority businesses that show marginal expectations of return on

the investments
II.A desire to invest in minority businesses that produce goods and services likely to be of

use to the sponsoring company
III.A belief that the minority business sector is best served by investing primarily in

newly established businesses
(A) I only
(B) III only
(C) I and II only
(D) II and III only
(E) I, II and III
6.The author refers to the “financial and operating problems” (line 38) encountered by

MESBIC’s primarily in order to
(A) broaden the scope of the discussion to include the legal considerations of funding

MESBIC’S through sponsoring companies
(B) call attention to the fact that MESBIC’s must receive adequate funding in order to

function effectively
(C) show that sponsoring companies were willing to invest only $500,000 of

government-sponsored venture capital in the original MESBIC’s
(D) compare SBA and MESBIC limits on minimum funding
(E) refute suggestions that MESBIC’s have been only marginally successful
7.The author’s primary objective in the passage is to
(A) disprove the view that federal efforts to aid minority businesses have been ineffective
(B) explain how federal efforts to aid minority businesses have changed since the 1960’s
(C) establish a direct link between the federal efforts to aid minority businesses made

before the 1960’s and those made in the 1980’s
(D) analyze the basis for the belief that job-specific experience is more useful to minority

businesses than is general management experience
(E) argue that the “social responsibility approach” to aiding minority businesses is

superior to any other approach
8.It can be inferred from the passage that the attitude of some MESBIC staff members toward

the investments preferred by some MESBIC directors can best be described as
(A) defensive
(B) resigned
(C) indifferent
(D) shocked
(E) disapproving
9.The passage provides information that would answer which of the following ?
(A) What was the average annual amount, in dollars, of minority business receipts before the

SBA strategy was implemented?
(B) What locations are considered to be unfavorable for minority businesses?
(C) What is the current success rate for minority businesses that are capitalized by

MESBIC’s?
(D) How has the use of federal funding for minority businesses changed since the 1960’s?
(E) How do minority businesses apply to participate in a MESBIC program?
Passage 13 (13/63)
The majority of successful senior managers do not closely follow the classical rational

model of first clarifying goals, assessing the problem, formulating options, estimating

likelihoods of success, making a decision, and only then taking action to implement the

decision. Rather, in their day-by-day tactical maneuvers, these senior executives rely on

what is vaguely termed “intuition” to manage a network of interrelated problems that require

them to deal with ambiguity, inconsistency, novelty, and surprise; and to integrate action

into the process of thinking.
Generations of writers on management have recognized that some practicing managers rely

heavily on intuition. In general, however, such writers display a poor grasp of what

intuition is. Some see it as the opposite of rationality; others view it as an excuse for

capriciousness.
Isenberg’s recent research on the cognitive processes of senior managers reveals that

managers’ intuition is neither of these. Rather, senior managers use intuition in at least

five distinct ways. First, they intuitively sense when a problem exists. Second, managers

rely on intuition to perform well-learned behavior patterns rapidly. This intuition is not

arbitrary or irrational, but is based on years of painstaking practice and hands-on

experience that build skills. A third function of intuition is to synthesize isolated bits

of data and practice into an integrated picture, often in an “Aha!” experience. Fourth, some

managers use intuition as a check on the results of more rational analysis. Most senior

executives are familiar with the formal decision analysis models and tools, and those who

use such systematic methods for reaching decisions are occasionally leery of solutions

suggested by these methods which run counter to their sense of the correct course of action.

Finally, managers can use intuition to bypass in-depth analysis and move rapidly to engender

a plausible solution. Used in this way, intuition is an almost instantaneous cognitive

process in which a manager recognizes familiar patterns. One of the implications of the

intuitive style of executive management is that “thinking” is inseparable from acting. Since

managers often “know” what is right before they can analyze and explain it, they frequently

act first and explain later. Analysis is inextricably tied to action in thinking/acting

cycles, in which managers develop thoughts about their companies and organizations not by

analyzing a problematic situation and then acting, but by acting and analyzing in close

concert.
Given the great uncertainty of many of the management issues that they face, senior managers

often instigate a course of action simply to learn more about an issue. They then use the

results of the action to develop a more complete understanding of the issue. One implication

of thinking/acting cycles is that action is often part of defining the problem, not just of

implementing the solution.
1.According to the passage, senior managers use intuition in all of the following ways

EXCEPT to
(A) speed up of the creation of a solution to a problem
(B) identify a problem
(C) bring together disparate facts
(D) stipulate clear goals
(E) evaluate possible solutions to a problem
2.The passage suggests which of the following about the “writers on management” mentioned in

line 12?
(A) They have criticized managers for not following the classical rational model of decision

analysis.
(B) They have not based their analyses on a sufficiently large sample of actual managers.
(C) They have relied in drawing their conclusions on what managers say rather than on what

managers do.
(D) They have misunderstood how managers use intuition in making business decisions.
(E) They have not acknowledged the role of intuition in managerial practice.
3.Which of the following best exemplifies “an ‘Aha!’ experience” (line 28) as it is

presented in the passage?
(A) A manager risks taking an action whose outcome is unpredictable to discover whether the

action changes the problem at hand.
(B) A manager performs well-learned and familiar behavior patterns in creative and

uncharacteristic ways to solve a problem.
(C) A manager suddenly connects seemingly unrelated facts and experiences to create a

pattern relevant to the problem at hand.
(D) A manager rapidly identifies the methodology used to compile data yielded by systematic

analysis.
(E) A manager swiftly decides which of several sets of tactics to implement in order to deal

with the contingencies suggested by a problem.
4.According to the passage, the classical model of decision analysis includes all of the

following EXCEPT
(A) evaluation of a problem
(B) creation of possible solutions to a problem
(C) establishment of clear goals to be reached by the decision
(D) action undertaken in order to discover more information about a problem
(E) comparison of the probable effects of different solutions to a problem
5.It can be inferred from the passage that which of the following would most probably be one

major difference in behavior between Manager X, who uses intuition to reach decisions, and

Manager Y, who uses only formal decision analysis?
(A) Manager X analyzes first and then acts; Manager Y does not.
(B) Manager X checks possible solutions to a problem by systematic analysis; Manager Y does

not.
(C) Manager X takes action in order to arrive at the solution to a problem; Manager Y does

not.
(D) Manager Y draws on years of hands-on experience in creating a solution to a problem;

Manager X does not.
(E) Manger Y depends on day-to-day tactical maneuvering; manager X does not.
6.It can be inferred from the passage that “thinking/acting cycles” (line 45) in managerial

practice would be likely to result in which of the following?
I.A manager analyzes a network of problems and then acts on the basis of that analysis.
II.A manager gathers data by acting and observing the effects of action.
III.A manager takes action without being able to articulate reasons for that particular

action.
(A) I only
(B) II only
(C) I and II only
(D) II and III only
(E) I, II, and III
7.The passage provides support for which of the following statements?
(A) Managers who rely on intuition are more successful than those who rely on formal

decision analysis.
(B) Managers cannot justify their intuitive decisions.
(C) Managers’ intuition works contrary to their rational and analytical skills.
(D) Logical analysis of a problem increases the number of possible solutions.
(E) Intuition enables managers to employ their practical experience more efficiently.
8.Which of the following best describes the organization of the first paragraph of the

passage?
(A) An assertion is made and a specific supporting example is given.
(B) A conventional model is dismissed and an alternative introduced.
(C) The results of recent research are introduced and summarized.
(D) Two opposing points of view are presented and evaluated.
(E) A widely accepted definition is presented and qualified.
Passage 14 (14/63)
Nearly a century ago, biologists found that if they separated an invertebrate animal embryo

into two parts at an early stage of its life, it would survive and develop as two normal

embryos. This led them to believe that the cells in the early embryo are undetermined in the

sense that each cell has the potential to develop in a variety of different ways. Later

biologists found that the situation was not so simple. It matters in which plane the embryo

is cut. If it is cut in a plane different from the one used by the early investigators, it

will not form two whole embryos.
A debate arose over what exactly was happening. Which embryo cells are determined, just when

do they become irreversibly committed to their fates, and what are the “morphogenetic

determinants” that tell a cell what to become? But the debate could not be resolved because

no one was able to ask the crucial in a form in which they could be pursued productively.

Recent discoveries in molecular biology, however, have opened up prospects for a resolution

of the debate. Now investigators think they know at least some of the molecules that act as

morphogenetic determinants in early development. They have been able to show that, in a

sense, cell determination begins even before an egg is fertilized.
Studying sea urchins, biologist Paul Gross found that an unfertilized egg contains

substances that function as morphogenetic determinants. They are located in the cytoplasm of

the egg cell; i.e., in that part of the cell’s protoplasm that lies outside of the nucleus.

In the unfertilized egg, the substances are inactive and are not distributed homogeneously.

When the egg is fertilized, the substances become active and, presumably, govern the

behavior of the genes they interact with. Since the substances are unevenly distributed in

the egg, when the fertilized egg divides, the resulting cells are different from the start

and so can be qualitatively different in their own gene activity.
The substances that Gross studied are maternal messenger RNA’s—products of certain of the

maternal genes. He and other biologists studying a wide variety of organisms have found that

these particular RNA’s direct, in large part, the synthesis of histones, a class of proteins

that bind to DNA. Once synthesized, the histones move into the cell nucleus, where of DNA

wrap around them to form a structure that resembles beads, or knots, on a string. The beads

are DNA segments wrapped around the histones; the string is the intervening DNA. And it is

the structure of these beaded DNA strings that guide the fate of the cells in which they are

located.
1.The passage is most probably directed at which kind of audience?
(A) State legislators deciding about funding levels for a state-funded biological laboratory
(B) Scientists specializing in molecular genetics
(C) Readers of an alumni newsletter published by the college that Paul Gross attended
(D) Marine biologists studying the processes that give rise to new species
(E) Undergraduate biology majors in a molecular biology course
2.It can be inferred from the passage that the morphogenetic determinants present in the

early embryo are
(A) located in the nucleus of the embryo cells
(B) evenly distributed unless the embryo is not developing normally
(C) inactive until the embryo cells become irreversibly committed to their final function
(D) identical to those that were already present in the unfertilized egg
(E) present in larger quantities than is necessary for the development of a single

individual
3.The main topic of the passage is
(A) the early development of embryos of lower marine organisms
(B) the main contribution of modern embryology to molecular biology
(C) the role of molecular biology in disproving older theories of embryonic development
(D) cell determination as an issue in the study of embryonic development
(E) scientific dogma as a factor in the recent debate over the value of molecular biology
4.According to the passage, when biologists believed that the cells in the early embryo were

undetermined, they made which of the following mistakes?
(A) They did not attempt to replicate the original experiment of separating an embryo into

two parts.
(B) They did not realize that there was a connection between the issue of cell determination

and the outcome of the separation experiment.
(C) They assumed that the results of experiments on embryos did not depend on the particular

animal species used for such experiments.
(D) They assumed that it was crucial to perform the separation experiment at an early stage

in the embryo’s life.
(E) They assumed that different ways of separating an embryo into two parts would be

equivalent as far as the fate of the two parts was concerned.
5.It can be inferred from the passage that the initial production of histones after an egg

is fertilized takes place
(A) in the cytoplasm
(B) in the maternal genes
(C) throughout the protoplasm
(D) in the beaded portions of the DNA strings
(E) in certain s of the cell nucleus
6.It can be inferred from the passage that which of the following is dependent on the

fertilization of an egg?
(A) Copying of maternal genes to produce maternal messenger RNA’s
(B) Synthesis of proteins called histones
(C) Division of a cell into its nucleus and the cytoplasm
(D) Determination of the egg cell’s potential for division
(E) Generation of all of a cell’s morphogenetic determinants
7.According to the passage, the morphogenetic determinants present in the unfertilized egg

cell are which of the following?
(A) Proteins bound to the nucleus
(B) Histones
(C) Maternal messenger RNA’s
(D) Cytoplasm
(E) Nonbeaded intervening DNA
8.The passage suggests that which of the following plays a role in determining whether an

embryo separated into two parts will develop as two normal embryos?
I. The stage in the embryo’s life at which the separation occurs
Lithe instrument with which the separations is accomplished
III.The plane in which the cut is made that separates the embryo
(A) I only
(B) II only
(C) I and II only
(D) I and III only
(E) I, II, and III
9.Which of the following circumstances is most comparable to the impasse biologists

encountered in trying to resolve the debate about cell determination (lines 12-18)?
(A) The problems faced by a literary scholar who wishes to use original source materials

that are written in an unfamiliar foreign language
(B) The situation of a mathematician who in preparing a proof of a theorem for publication

detects a reasoning error in the proof
(C) The difficulties of a space engineer who has to design equipment to function in an

environment in which it cannot first be tested
(D) The predicament of a linguist trying to develop a theory of language acquisition when

knowledge of the structure of language itself is rudimentary at best
(E) The dilemma confronting a foundation when the funds available to it are sufficient to

support one of two equally deserving scientific projects but not both
Passage 15 (15/63)
In the two decades between 1910 and 1930, over ten percent of the Black population of the

United States left the South, where the preponderance of the Black population had been

located, and migrated to northern states, with the largest number moving, it is claimed,

between 1916 and 1918. It has been frequently assumed, but not proved, that the majority of

the migrants in what has come to be called the Great Migration came from rural areas and

were motivated by two concurrent factors: the collapse of the cotton industry following the

boll weevil infestation, which began in 1898, and increased demand in the North for labor

following the cessation of European immigration caused by the outbreak of the First World

War in 1914. This assumption has led to the conclusion that the migrants’ subsequent lack of

economic mobility in the North is tied to rural background, a background that implies

unfamiliarity with urban living and a lack of industrial skills.
But the question of who actually left the South has never been rigorously investigated.

Although numerous investigations document an exodus from rural southern areas to southern

cities prior to the Great Migration, no one has considered whether the same migrants then

moved on to northern cities. In 1910 over 600,000 Black workers, or ten percent of the Black

work force, reported themselves to be engaged in “manufacturing and mechanical pursuits,”

the federal census category roughly encompassing the entire industrial sector. The Great

Migration could easily have been made up entirely of this group and their families. It is

perhaps surprising to argue that an employed population could be enticed to move, but an

explanation lies in the labor conditions then prevalent in the South.
About thirty-five percent of the urban Black population in the South was engaged in skilled

trades. Some were from the old artisan class of slavery—blacksmiths, masons,

carpenters—which had had a monopoly of certain trades, but they were gradually being pushed

out by competition, mechanization, and obsolescence. The remaining sixty-five percent, more

recently urbanized, worked in newly developed industries—tobacco, lumber, coal and iron

manufacture, and railroads. Wages in the South, however, were low, and Black workers were

aware, through labor recruiters and the Black press, that they could earn more even as

unskilled workers in the North than they could as artisans in the South. After the boll

weevil infestation, urban Black workers faced competition from the continuing influx of both

Black and White rural workers, who were driven to undercut the wages formerly paid for

industrial jobs. Thus, a move north would be seen as advantageous to a group that was

already urbanized and steadily employed, and the easy conclusion tying their subsequent

economic problems in the North to their rural background comes into question.
1.The author indicates explicitly that which of the following records has been a source of

information in her investigation?
(A) United States Immigration Service reports from 1914 to 1930
(B) Payrolls of southern manufacturing firms between 1910 and 1930
(C) The volume of cotton exports between 1898 and 1910
(D) The federal census of 1910
(E) Advertisements of labor recruiters appearing in southern newspapers after 1910
2.In the passage, the author anticipates which of the following as a possible objection to

her argument?
(A) It is uncertain how many people actually migrated during the Great Migration.
(B) The eventual economic status of the Great Migration migrants has not been adequately

traced.
(C) It is not likely that people with steady jobs would have reason to move to another area

of the country.
(D) It is not true that the term “manufacturing and mechanical pursuits” actually

encompasses the entire industrial sector.
(E) Of the Black workers living in southern cities, only those in a small number of trades

were threatened by obsolescence.
3.According to the passage, which of the following is true of wages in southern cities in

1910?
(A) They were being pushed lower as a result of increased competition.
(B) They had begun t to rise so that southern industry could attract rural workers.
(C) They had increased for skilled workers but decreased for unskilled workers.
(D) They had increased in large southern cities but decreased in small southern cities.
(E) They had increased in newly developed industries but decreased in the older trades.
4.The author cites each of the following as possible influences in a Black worker’s decision

to migrate north in the Great Migration EXCEPT
(A) wage levels in northern cities
(B) labor recruiters
(C) competition from rural workers
(D) voting rights in northern states
(E) the Black press
5.It can be inferred from the passage that the “easy conclusion” mentioned in line 53 is

based on which of the following assumptions?
(A) People who migrate from rural areas to large cities usually do so for economic reasons.
(B) Most people who leave rural areas to take jobs in cities return to rural areas as soon

as it is financially possible for them to do so.
(C) People with rural backgrounds are less likely to succeed economically in cities than are

those with urban backgrounds.
(D) Most people who were once skilled workers are not willing to work as unskilled workers.
(E) People who migrate from their birthplaces to other regions of country seldom undertake a

second migration.
6.The primary purpose of the passage is to
(A) support an alternative to an accepted methodology
(B) present evidence that resolves a contradiction
(C) introduce a recently discovered source of information
(D) challenge a widely accepted explanation
(E) argue that a discarded theory deserves new attention
7.According to information in the passage, which of the following is a correct sequence of

groups of workers, from highest paid to lowest paid, in the period between 1910 and 1930?
(A) Artisans in the North; artisans in the South; unskilled workers in the North; unskilled

workers in the South
(B) Artisans in the North and South; unskilled workers in the North; unskilled workers in

the South
(C) Artisans in the North; unskilled workers in the North; artisans in the South
(D) Artisans in the North and South; unskilled urban workers in the North; unskilled rural

workers in the South
(E) Artisans in the North and South, unskilled rural workers in the North and South;

unskilled urban workers in the North and South
8.The material in the passage would be most relevant to a long discussion of which of the

following topics?
(A) The reasons for the subsequent economic difficulties of those who participated in the

Great Migration
(B) The effect of migration on the regional economies of the United States following the

First World War
(C) The transition from a rural to an urban existence for those who migrated in the Great

Migration
(D) The transformation of the agricultural South following the boll weevil infestation
(E) The disappearance of the artisan class in the United States as a consequence of

mechanization in the early twentieth century
Passage 16 (16/63)
In 1896 a Georgia couple suing for damages in the accidental death of their two year old was

told that since the child had made no real economic contribution to the family, there was no

liability for damages. In contrast, less than a century later, in 1979, the parents of a

three-year-old sued in New York for accidental-death damages and won an award of $750,000.
The transformation in social values implicit in juxtaposing these two incidents is the

subject of Viviana Zelizer’s excellent book, Pricing the Priceless Child. During the

nineteenth century, she argues, the concept of the “useful” child who contributed to the

family economy gave way gradually to the present-day notion of the “useless” child who,

though producing no income for, and indeed extremely costly to, its parents, is yet

considered emotionally “priceless.” Well established among segments of the middle and upper

classes by the mid-1800’s, this new view of childhood spread throughout society in the

late-nineteenth and early-twentieth centuries as reformers introduced child-labor

regulations and compulsory education laws predicated in part on the assumption that a

child’s emotional value made child labor taboo.
For Zelizer the origins of this transformation were many and complex. The gradual erosion of

children’s productive value in a maturing industrial economy, the decline in birth and death

rates, especially in child mortality, and the development of the companionate family (a

family in which members were united by explicit bonds of love rather than duty) were all

factors critical in changing the assessment of children’s worth. Yet “expulsion of children

from the ‘cash nexus,’ although clearly shaped by profound changes in the economic,

occupational, and family structures,” Zelizer maintains, “was also part of a cultural

process ‘of sacrelization’ of children’s lives.” Protecting children from the crass business

world became enormously important for late-nineteenth-century middle-class Americans, she

suggests; this sacralization was a way of resisting what they perceived as the relentless

corruption of human values by the marketplace.
In stressing the cultural determinants of a child’s worth, Zelizer takes issue with

practitioners of the new “sociological economics,” who have analyzed such traditionally

sociological topics as crime, marriage, education, and health solely in terms of their

economic determinants. Allowing only a small role for cultural forces in the form of

individual “preferences,” these sociologists tend to view all human behaviors as directed

primarily by the principle of maximizing economic gain. Zelizer is highly critical of this

approach, and emphasizes instead the opposite phenomenon: the power of social values to

transform price. As children became more valuable in emotional terms, she argues, their

“exchange” or “surrender” value on the market, that is, the conversion of their intangible

worth into cash terms, became much greater.
1.It can be inferred from the passage that accidental-death damage awards in America during

the nineteenth century tended to be based principally on the
(A) earnings of the person at of death
(B) wealth of the party causing the death
(C) degree of culpability of the party causing the death
(D) amount of money that had been spent on the person killed
(E) amount of suffering endured by the family of the person killed
2.It can be inferred from the passage that in the early 1800’s children were generally

regarded by their families as individuals who
(A) needed enormous amounts of security and affection
(B) required constant supervision while working
(C) were important to the economic well-being of a family
(D) were unsuited to spending long hours in school
(E) were financial burdens assumed for the good of society
3.Which of the following alternative explanations of the change in the cash value of

children would be most likely to be put forward by sociological economists as they are

described in the passage?
(A) The cash value of children rose during the nineteenth century because parents began to

increase their emotional investment in the upbringing of their children.
(B) The cash value of children rose during the nineteenth century because their expected

earnings over the course of a life increased greatly.
(C) The cash value of children rose during the nineteenth century because the spread of

humanitarian ideals resulted in a wholesale reappraisal of the worth of an individual.
(D) The cash value of children rose during the nineteenth century because compulsory

education laws reduced the supply, and thus raised the costs, of available child labor.
(E) The cash value of children rose during the nineteenth century because of changes in the

way negligence law assessed damages in accidental death cases.
4.The primary purpose of the passage is to
(A) review the literature in a new academic sub-field
(B) present the central thesis of a recent book
(C) contrast two approaches to analyzing historical change
(D) refute a traditional explanation of a social phenomenon
(E) encourage further work on a neglected historical topic
5.It can be inferred from the passage that which of the following statements was true of

American families over the course of the nineteenth century?
(A) The average size of families grew considerably.
(B) The percentage of families involved in industrial work declined dramatically.
(C) Family members became more emotionally bonded to one another.
(D) Family members spent an increasing amount of working with each other.
(E) Family members became more economically dependent on each other.
6.Zelizer refers to all of the following as important influences in changing the assessment

of children’s worth EXCEPT changes in
(A) the mortality rate
(B) the nature of industry
(C) the nature of the family
(D) attitudes toward reform movements
(E) attitudes toward the marketplace
7.Which of the following would be most consistent with the practices of sociological

economics as these practices are described in the passage?
(A) Arguing that most health-care professionals enter the field because they believe it to

be the most socially useful of any occupation
(B) Arguing that most college students choose majors that they believe will lead to the most

highly paid jobs available to them
(C) Arguing that most decisions about marriage and divorce are based on rational assessments

of the likelihood that each partner will remain committed to the relationship
(D) Analyzing changes in the number of people enrolled in colleges and universities as a

function of changes in the economic health of these institutions
(E) Analyzing changes in the ages at which people get married as a function of a change in

the average number of years that young people have lived away from their parents
Passage 17 (17/63)
Prior to 1975, union efforts to organize public-sector clerical workers, most of whom are

women, were somewhat limited. The factors favoring unionization drives seem to have been

either the presence of large numbers of workers, as in New York City, to make it worth the

effort, or the concentration of small numbers in one or two locations, such as a hospital,

to make it relatively easy. Receptivity to unionization on the workers’ part was also a

consideration, but when there were large numbers involved or the clerical workers were the

only unorganized group in a jurisdiction, the multi-occupational unions would often try to

organize them regardless of the workers’ initial receptivity. The strategic reasoning was

based, first, on the concern that politicians and administrators might play off unionized

against non-unionized workers, and, second, on the conviction that a fully unionized public

work force meant power, both at the bargaining table and in the legislature. In localities

where clerical workers were few in number, were scattered in several workplaces, and

expressed no interest in being organized, unions more often than not ignored them in the

pre-1975 period.
But since the mid-1970’s, a different strategy has emerged. In 1977, 34 percent of

government clerical workers were represented by a labor organization, compared with 46

percent of government professionals, 44 percent of government blue-collar workers, and 41

percent of government service workers. Since then, however, the biggest increases in

public-sector unionization have been among clerical workers. Between 1977 and 1980, the

number of unionized government workers in blue-collar and service occupations increased only

about 1.5 percent, while in the white-collar occupations the increase was 20 percent and

among clerical workers in particular, the increase was 22 percent.
What accounts for this upsurge in unionization among clerical workers? First, more women

have entered the work force in the past few years, and more of them plan to remain working

until retirement age. Consequently, they are probably more concerned than their predecessors

were about job security and economic benefits. Also, the women’s movement has succeeded in

legitimizing the economic and political activism of women on their own behalf, thereby

producing a more positive attitude toward unions. The absence of any comparable increase in

unionization among private-sector clerical workers, however, identifies the primary

catalyst—the structural change in the multi-occupational public-sector unions themselves.

Over the past twenty years, the occupational distribution in these unions has been steadily

shifting from predominantly blue-collar to predominantly white-collar. Because there are far

more women in white-collar jobs, an increase in the proportion of female members has

accompanied the occupational shift and has altered union policy-making in favor of

organizing women and addressing women’s issues.
1.According to the passage, the public-sector workers who were most likely to belong to

unions in 1977 were
(A) professionals
(B) managers
(C) clerical workers
(D) service workers
(E) blue-collar workers
2.The author cites union efforts to achieve a fully unionized work force (line 13-19) in

order to account for why
(A) politicians might try to oppose public-sector union organizing
(B) public-sector unions have recently focused on organizing women
(C) early organizing efforts often focused on areas where there were large numbers of

workers
(D) union efforts with regard to public-sector clerical workers increased dramatically after

1975
(E) unions sometimes tried to organize workers regardless of the workers’ initial interest

in unionization
3.The author’s claim that, since the mid-1970’s, a new strategy has emerged in the

unionization of public-sector clerical workers (line 23) would be strengthened if the author
(A) described more fully the attitudes of clerical workers toward labor unions
(B) compared the organizing strategies employed by private-sector unions with those of

public-sector unions
(C) explained why politicians and administrators sometimes oppose unionization of clerical

workers
(D) indicated that the number of unionized public-sector clerical workers was increasing

even before the mid-1970’s
(E) showed that the factors that favored unionization drives among these workers prior to

1975 have decreased in importance
4.According to the passage, in the period prior to 1975, each of the following

considerations helped determine whether a union would attempt to organize a certain group of

clerical workers EXCEPT
(A) the number of clerical workers in that group
(B) the number of women among the clerical workers in that group
(C) whether the clerical workers in that area were concentrated in one workplace or

scattered over several workplaces
(D) the degree to which the clerical workers in that group were interested in unionization
(E) whether all the other workers in the same jurisdiction as that group of clerical workers

were unionized
5.The author states that which of the following is a consequence of the women’s movement of

recent years?
(A) An increase in the number of women entering the work force
(B) A structural change in multi-occupational public-sector unions
(C) A more positive attitude on the part of women toward unions
(D) An increase in the proportion of clerical workers that are women
(E) An increase in the number of women in administrative positions
6.The main concern of the passage is to
(A) advocate particular strategies for future efforts to organize certain workers into labor

unions
(B) explain differences in the unionized proportions of various groups of public-sector

workers
(C) evaluate the effectiveness of certain kinds of labor unions that represent public-sector

workers
(D) analyzed and explain an increase in unionization among a certain category of workers
(E) describe and distinguish strategies appropriate to organizing different categories of

workers
7.The author implies that if the increase in the number of women in the work force and the

impact of the women’s movement were the main causes of the rise in unionization of

public-sector clerical workers, then
(A) more women would hold administrative positions in unions
(B) more women who hold political offices would have positive attitudes toward labor unions
(C) there would be an equivalent rise in unionization of private-sector clerical workers
(D) unions would have shown more interest than they have in organizing women
(E) the increase in the number of unionized public-sector clerical workers would have been

greater than it has been
8.The author suggests that it would be disadvantageous to a union if
(A) many workers in the locality were not unionized
(B) the union contributed to political campaigns
(C) the union included only public-sector workers
(D) the union included workers from several jurisdictions
(E) the union included members from only a few occupations
9.The author implies that, in comparison with working women today, women working in the

years prior to the mid-1970’s showed a greater tendency to
(A) prefer smaller workplaces
(B) express a positive attitude toward labor unions
(C) maximize job security and economic benefits
(D) side with administrators in labor disputes
(E) quit working prior of retirement age
Passage 18 (18/63)
Milankovitch proposed in the early twentieth century that the ice ages were caused by

variations in the Earth’s orbit around the Sun. For some this theory was considered

untestable, largely because there was no sufficiently precise chronology of the ice ages

with which the orbital variations could be matched.
To establish such a chronology it is necessary to determine the relative amounts of land ice

that existed at various s in the Earth’s past. A recent discovery makes such a determination

possible: relative land-ice volume for a given period can be deduced from the ratio of two

oxygen isotopes, 16 and 18, found in ocean sediments. Almost all the oxygen in water is

oxygen 16, but a few molecules out of every thousand incorporate the heavier isotope 18.

When an ice age begins, the continental ice sheets grow, steadily reducing the amount of

water evaporated from the ocean that will eventually return to it. Because heavier isotopes

tend to be left behind when water evaporates from the ocean surfaces, the remaining ocean

water becomes progressively enriched in oxygen 18. The degree of enrichment can be

determined by analyzing ocean sediments of the period, because these sediments are composed

of calcium carbonate shells of marine organisms, shells that were constructed with oxygen

atoms drawn from the surrounding ocean. The higher the ratio of oxygen 18 to oxygen 16 in a

sedimentary specimen, the more land ice there was when the sediment was laid down.
As an indicator of shifts in the Earth’s climate, the isotope record has two advantages.

First, it is a global record: there is remarkably little variation in isotope ratios in

sedimentary specimens taken from different continental locations. Second, it is a more

continuous record than that taken from rocks on land. Because of these advantages,

sedimentary evidence can be dated with sufficient accuracy by radiometric methods to

establish a precise chronology of the ice ages. The dated isotope record shows that the

fluctuations in global ice volume over the past several hundred thousand years have a

pattern: an ice age occurs roughly once every 100,000 years. These data have established a

strong connection between variations in the Earth’s orbit and the periodicity of the ice

ages.
However, it is important to note that other factors, such as volcanic particulates or

variations in the amount of sunlight received by the Earth, could potentially have affected

the climate. The advantage of the Milankovitch theory is that it is testable: changes in the

Earth’s orbit can be calculated and dated by applying Newton’s laws of gravity to

progressively earlier configurations of the bodies in the solar system. Yet the lack of

information about other possible factors affecting global climate does not make them

unimportant.
1.In the passage, the author is primarily interested in
(A) suggesting an alternative to an outdated research method
(B) introducing a new research method that calls an accepted theory into question
(C) emphasizing the instability of data gathered from the application of a new scientific

method
(D) presenting a theory and describing a new method to test that theory
(E) initiating a debate about a widely accepted theory
2.The author of the passage would be most likely to agree with which of the following

statements about the Milankovitch theory?
(A) It is the only possible explanation for the ice ages.
(B) It is too limited to provide a plausible explanation for the ice ages, despite recent

research findings.
(C) It cannot be tested and confirmed until further research on volcanic activity is done.
(D) It is one plausible explanation, though not the only one, for the ice ages.
(E) It is not a plausible explanation for the ice ages, although it has opened up promising

possibilities for future research.
3.It can be inferred from the passage that the isotope record taken from ocean sediments

would be less useful to researchers if which of the following were true?
(A) It indicated that lighter isotopes of oxygen predominated at certain s.
(B) It had far more gaps in its sequence than the record taken from rocks on land.
(C) It indicated that climate shifts did not occur every 100,000 years.
(D) It indicated that the ratios of oxygen 16 and oxygen 18 in ocean water were not

consistent with those found in fresh water.
(E) It stretched back for only a million years.
4.According to the passage, which of the following is true of the ratios of oxygen isotopes

in ocean sediments?
(A) They indicate that sediments found during an ice age contain more calcium carbonate than

sediments formed at other s.
(B) They are less reliable than the evidence from rocks on land in determining the volume of

land ice.
(C) They can be used to deduce the relative volume of land ice that was present when the

sediment was laid down.
(D) They are more unpredictable during an ice age than in other climatic conditions.
(E) They can be used to determine atmospheric conditions at various s in the past.
5.It can be inferred from the passage that precipitation formed from evaporated ocean water

has
(A) the same isotopic ratio as ocean water
(B) less oxygen 18 than does ocean water
(C) less oxygen 18 than has the ice contained in continental ice sheets
(D) a different isotopic composition than has precipitation formed from water on land
(E) more oxygen 16 than has precipitation formed from fresh water
6.According to the passage, which of the following is (are) true of the ice ages?
I. The last ice age occurred about 25,000 years ago.
II.Ice ages have lasted about 10,000 years for at least the last several hundred thousand

years.
III.Ice ages have occurred about every 100,000 years for at least the last several hundred

thousand years.
(A) I only
(B) II only
(C) III only
(D) I and only
(E) I, II and III
7.It can be inferred from the passage that calcium carbonate shells
(A) are not as susceptible to deterioration as rocks
(B) are less common in sediments formed during an ice age
(C) are found only in areas that were once covered by land ice
(D) contain radioactive material that can be used to determine a sediment’s isotopic

composition
(E) reflect the isotopic composition of the water at the shells were formed
8.The purpose of the last paragraph of the passage is to
(A) offer a note of caution
(B) introduce new evidence
(C) present two recent discoveries
(D) summarize material in the preceding paragraphs
(E) offer two explanations for a phenomenon
9.According to the passage, one advantage of studying the isotope record of ocean sediments

is that it
(A) corresponds with the record of ice volume taken from rocks on land
(B) shows little variation in isotope ratios when samples are taken from different

continental locations
(C) corresponds with predictions already made by climatologists and experts in other fields
(D) confirms the record of ice volume initially established by analyzing variations in

volcanic emissions
(E) provides data that can be used to substantiate records concerning variations in the

amount of sunlight received by the Earth
Passage 19 (19/63)
In contrast to traditional analyses of minority business, the sociological analysis contends

that minority business ownership is a group-level phenomenon, in that it is largely

dependent upon social-group resources for its development. Specifically, this analysis

indicates that support networks play a critical role in starting and maintaining minority

business enterprises by providing owners with a range of assistance, from the informal

encouragement of family members and friends to dependable sources of labor and clientele

from the owner’s ethnic group. Such self-help networks, which encourage and support ethnic

minority entrepreneurs, consist of “primary” institutions, those closest to the individual

in shaping his or her behavior and beliefs. They are characterized by the face-to-face

association and cooperation of persons united by ties of mutual concern. They form an

intermediate social level between the individual and larger “secondary” institutions based

on impersonal relationships. Primary institutions comprising the support network include

kinship, peer, and neighborhood or community subgroups.
A major function of self-help networks is financial support. Most scholars agree that

minority business owners have depended primarily on family funds and ethnic community

resources for investment capital. Personal savings have been accumulated, often through

frugal living habits that require sacrifices by the entire family and are thus a product of

long-term family financial behavior. Additional loans and gifts from relatives, forthcoming

because of group obligation rather than narrow investment calculation, have supplemented

personal savings. Individual entrepreneurs do not necessarily rely on their kin because they

cannot obtain financial backing from commercial resources. They may actually avoid banks

because they assume that commercial institutions either cannot comprehend the special needs

of minority enterprise or charge unreasonably high interest rates.
Within the larger ethnic community, rotating credit associations have been used to raise

capital. These associations are informal clubs of friends and other trusted members of the

ethnic group who make regular contributions to a fund that is given to each contributor in

rotation. One author estimates that 40 percent of New York Chinatown firms established

during 1900-1950 utilized such associations as their initial source of capital. However,

recent immigrants and third or fourth generations of older groups now employ rotating credit

associations only occasionally to raise investment funds. Some groups, like Black Americans,

found other means of financial support for their entrepreneurial efforts. The first

Black-operated banks were created in the late nineteenth century as depositories for dues

collected from fraternal or lodge groups, which themselves had sprung from Black churches.

Black banks made limited investments in other Black enterprises. Irish immigrants in

American cities organized many building and loan associations to provide capital for home

construction and purchase. They, in turn, provided work for many Irish home-building

contractor firms. Other ethnic and minority groups followed similar practices in founding

ethnic-directed financial institutions.
1.Based on the information in the passage, it would be LEAST likely for which of the

following persons to be part of a self-help network?
(A) The entrepreneur’s childhood friend
(B) The entrepreneur’s aunt
(C) The entrepreneur’s religious leader
(D) The entrepreneur’s neighbor
(E) The entrepreneur’s banker
2.Which of the following illustrates the working of a self-help support network, as such

networks are described in the passage?
(A) A public high school offers courses in book-keeping and accounting as part of its

open-enrollment adult education program.
(B) The local government in a small city sets up a program that helps teen-agers find summer

jobs.
(C) A major commercial bank offers low-interest loans to experienced individuals who hope to

establish their own businesses.
(D) A neighborhood-based fraternal organization develops a program of on-the-job training

for its members and their friends.
(E) A community college offers country residents training programs that can lead to

certification in a variety of technical trades.
3.Which of the following can be inferred from the passage about rotating credit

associations?
(A) They were developed exclusively by Chinese immigrants.
(B) They accounted for a significant portion of the investment capital used by Chinese

immigrants in New York in the early twentieth century.
(C) Third-generation members of an immigrant group who started businesses in the 1920’s

would have been unlikely to rely on them.
(D) They were frequently joint endeavors by members of two or three different ethnic groups.
(E) Recent immigrants still frequently turn to rotating credit associations instead of banks

for investment capital.
4.The passage best supports which of the following statements?
(A) A minority entrepreneur who had no assistance from family members would not be able to

start a business.
(B) Self-help networks have been effective in helping entrepreneurs primarily in the last 50

years.
(C) Minority groups have developed a range of alternatives to standard financing of business

ventures.
(D) The financial institutions founded by various ethnic groups owe their success to their

unique formal organization.
(E) Successful minority-owned businesses succeed primarily because of the personal strengths

of their founders.
5.Which of the following best describes the organization of the second paragraph?
(A) An argument is delineated, followed by a counter-argument.
(B) An assertion is made and several examples are provided to illustrate it.
(C) A situation is described and its historical background is then outlined.
(D) An example of a phenomenon is given and is then used as a basis for general conclusions.
(E) A group of parallel incidents is described and the distinctions among the incidents are

then clarified.
6.According to the passage, once a minority-owned business is established, self-help

networks contribute which of the following to that business?
(A) Information regarding possible expansion of the business into nearby communities
(B) Encouragement of a business climate that is nearly free of direct competition
(C) Opportunities for the business owner to reinvest profits in other minority-owned

businesses
(D) Contact with people who are likely to be customers of the new business
(E) Contact with minority entrepreneurs who are members of other ethnic groups
7.It can be inferred from the passage that traditional analyses of minority business would

be LEAST likely to do which of the following?
(A) Examine businesses primarily in their social contexts
(B) Focus on current, rather than historical, examples of business enterprises
(C) Stress common experiences of individual entrepreneurs in starting businesses
(D) Focus on the maintenance of businesses, rather than means of starting them
(E) Focus on the role of individual entrepreneurs in starting a business
8.Which of the following can be inferred from the passage about the Irish building and loan

associations mentioned in the last paragraph?
(A) They were started by third- or fourth-generation immigrants.
(B) They originated as offshoots of church-related groups.
(C) They frequently helped Irish entrepreneurs to finance business not connected with

construction.
(D) They contributed to the employment of many Irish construction workers.
(E) They provided assistance for construction businesses owned by members of other ethnic

groups.
Passage 20 (20/63)
Species interdependence in nature confers many benefits on the species involved, but it can

also become a point of weakness when one species involved in the relationship is affected by

a catastrophe. Thus, flowering plant species dependent on insect pollination, as opposed to

self-pollination or wind pollination, could be endangered when the population of

insect-pollinators is depleted by the use of pesticides.
In the forests of New Brunswick, for example, various pesticides have been sprayed in the

past 25 years in efforts to control the spruce budworm, an economically significant pest.

Scientists have now investigated the effects of the spraying of Matacil, one of the

anti-budworm agents that is least toxic to insect-pollinators. They studied Matacil’s

effects on insect mortality in a wide variety of wild insect species and on plant fecundity,

expressed as the percentage of the total flowers on an individual plant that actually

developed fruit and bore seeds. They found that the most pronounced mortality after the

spraying of Matacil occurred among the smaller bees and one family of flies, insects that

were all important pollinators of numerous species of plants growing beneath the tree canopy

of forests. The fecundity of plants in one common indigenous species, the red-osier dogwood,

was significantly reduced in the sprayed areas as compared to that of plants in control

plots where Matacil was not sprayed. This species is highly dependent on the

insect-pollinators most vulnerable to Matacil. The creeping dogwood, a species similar to

the red-osier dogwood, but which is pollinated by large bees, such as bumblebees, showed no

significant decline in fecundity. Since large bees are not affected by the spraying of

Matacil, these results add weight to the argument that spraying where the pollinators are

sensitive to the pesticide used decreases plant fecundity.
The question of whether the decrease in plant fecundity caused by the spraying of pesticides

actually causes a decline in the overall population of flowering plant species still remains

unanswered. Plant species dependent solely on seeds for survival or dispersal are obviously

more vulnerable to any decrease in plant fecundity that occurs, whatever its cause. If, on

the other hand, vegetative growth and dispersal (by means of shoots or runners) are

available as alternative reproductive strategies for a species, then decreases in plant

fecundity may be of little consequence. The fecundity effects described here are likely to

have the most profound impact on plant species with all four of the following

characteristics: a short life span, a narrow geographic range, an incapacity for vegetative

propagation, and a dependence on a small number of insect-pollinator species. Perhaps we

should give special attention to the conservation of such plant species since they lack key

factors in their defenses against the environmental disruption caused by pesticide use.
1.Which of the following best summarizes the main point of the passage?
(A) Species interdependence is a point of weakness for some plants, but is generally

beneficial to insects involved in pollination.
(B) Efforts to control the spruce budworm have had deleterious effects on the red-osier

dogwood.
(C) The use of pesticides may be endangering certain plant species dependent on insects for

pollination.
(D) The spraying of pesticides can reduce the fecundity of a plant species, but probably

does not affect its overall population stability.
(E) Plant species lacking key factors in their defenses against human environmental

disruption will probably become extinct.
2.According to the author, a flowering plant species whose fecundity has declined due to

pesticide spraying may not experience an overall population decline if the plant species can

do which of the following?
(A) Reproduce itself by means of shoots and runners.
(B) Survive to the end of the growing season.
(C) Survive in harsh climates.
(D) Respond to the fecundity decline by producing more flowers.
(E) Attract large insects as pollinators.
3.The passage suggests that the lack of an observed decline in the fecundity of the creeping

dogwood strengthens the researchers conclusions regarding pesticide use because the
(A) creeping dogwood is a species that does not resemble other forest plants
(B) creeping dogwood is a species pollinated by a broader range of insect species than are

most dogwood species
(C) creeping dogwood grows primarily in regions that were not sprayed with pesticide, and so

served as a control for the experiment
(D) creeping dogwood is similar to the red-osier dogwood, but its insect pollinators are

known to be insensitive to the pesticide used in the study
(E) geographical range of the creeping dogwood is similar to that of the red-osier dogwood,

but the latter species relies less on seeds for reproduction
4.The passage suggests that which of the following is true of the forest regions in New

Brunswick sprayed with most anti-budworm pesticides other than Matacil?
(A) The fecundity of some flowering plants in those regions may have decreased to an even

greater degree than in the regions where Matacil is used.
(B) Insect mortality in those regions occurs mostly among the larger species of insects,

such as bumblebees.
(C) The number of seeds produced by common plant species in those regions is probably

comparable to the number produced where Matacil is sprayed.
(D) Many more plant species have become extinct in those regions than in the regions where

Matacil is used.
(E) The spruce budworm is under better control in those regions than in the regions where

Matacil is sprayed.
5.It can be inferred that which of the following is true of plant fecundity as it is defined

in the passage?
(A) A plant’s fecundity decreases as the percentage of unpollinated flowers on the plant

increases.
(B) A plant’s fecundity decreases as the number of flowers produced by the plant decreases.
(C) A plant’s fecundity increases as the number of flowers produced by the plant increases.
(D) A plant’s fecundity is usually low if the plant relies on a small number of insect

species for pollination.
(E) A plant’s fecundity is high if the plant can reproduce quickly by means of vegetative

growth as well as by the production of seeds.
6.It can be inferred from the passage that which of the following plant species would be

LEAST likely to experience a decrease in fecundity as a result of the spraying of a

pesticide not directly toxic to plants?
(A) A flowering tree pollinated by only a few insect species
(B) A kind of insect-pollinated vine producing few flowers
(C) A wind-pollinated flowering tree that is short-lived
(D) A flowering shrub pollinated by a large number of insect species
(E) A type of wildflower typically pollinated by larger insects
7.Which of the following assumptions most probably underlies the author’s tentative

recommendation in lines 51-54?
(A) Human activities that result in environmental disruption should be abandoned.
(B) The use of pesticides is likely to continue into the future.
(C) It is economically beneficial to preserve endangered plant species.
(D) Preventing the endangerment of a species is less costly than trying to save an already

endangered one.
(E) Conservation efforts aimed at preserving a few well-chosen species are more

cost-effective than are broader-based efforts to improve the environment.
Passage 21 (21/63)
Bernard Bailyn has recently reinterpreted the early history of the United States by applying

new social research findings on the experiences of European migrants. In his

reinterpretation, migration becomes the organizing principle for rewriting the history of

preindustrial North America. His approach rests on four separate propositions.
The first of these asserts that residents of early modern England moved regularly about

their countryside; migrating to the New World was simply a “natural spillover.” Although at

first the colonies held little positive attraction for the English—they would rather have

stayed home—by the eighteenth century people increasingly migrated to America because they

regarded it as the land of opportunity. Secondly, Bailyn holds that, contrary to the notion

that used to flourish in America history textbooks, there was never a typical New World

community. For example, the economic and demographic character of early New England towns

varied considerably.
Bailyn’s third proposition suggests two general patterns prevailing among the many thousands

of migrants: one group came as indentured servants, another came to acquire land.

Surprisingly, Bailyn suggests that those who recruited indentured servants were the driving

forces of transatlantic migration. These colonial entrepreneurs helped determine the social

character of people who came to preindustrial North America. At first, thousands of

unskilled laborers were recruited; by the 1730’s, however, American employers demanded

skilled artisans.
Finally, Bailyn argues that the colonies were a half-civilized hinterland of the European

culture system. He is undoubtedly correct to insist that the colonies were part of an

Anglo-American empire. But to divide the empire into English core and colonial periphery, as

Bailyn does, devalues the achievements of colonial culture. It is true, as Bailyn claims,

that high culture in the colonies never matched that in England. But what of

seventeenth-century New England, where the settlers created effective laws, built a

distinguished university, and published books? Bailyn might respond that New England was

exceptional. However, the ideas and institutions developed by New England Puritans had

powerful effects on North American culture.
Although Bailyn goes on to apply his approach to some thousands of indentured servants who

migrated just prior to the revolution, he fails to link their experience with the political

development of the United States. Evidence presented in his work suggests how we might make

such a connection. These indentured servants were treated as slaves for the period during

which they had sold their to American employers. It is not surprising that as soon as they

served their they passed up good wages in the cities and headed west to ensure their

personal independence by acquiring land. Thus, it is in the west that a peculiarly American

political culture began, among colonists who were suspicious of authority and intensely

anti-aristocratic.
1.Which of the following statements about migrants to colonial North America is supported by

information in the passage?
(A) A larger percentage of migrants to colonial North America came as indentured servants

than as free agents interested in acquiring land.
(B) Migrants who came to the colonies as indentured servants were more successful at making

a livelihood than were farmers and artisans.
(C) Migrants to colonial North America were more successful at acquiring their own land

during the eighteenth century than during the seven-tenth century.
(D) By the 1730’s, migrants already skilled in a trade were in more demand by American

employers than were unskilled laborers.
(E) A significant percentage of migrants who came to the colonies to acquire land were

forced to work as field hands for prosperous American farmers.
2.The author of the passage states that Bailyn failed to
(A) give sufficient emphasis to the cultural and political interdependence of the colonies

and England
(B) describe carefully how migrants of different ethnic backgrounds preserved their culture

in the united States
(C) take advantage of social research on the experiences of colonists who migrated to

colonial North America specifically to acquire land
(D) relate the experience of the migrants to the political values that eventually shaped the

character of the United States
(E) investigate the lives of Europeans before they came to colonial North America to

determine more adequately their motivations for migrating
3.Which of the following best summarizes the author’s evaluation of Bailyn’s fourth

proposition?
(A) It is totally implausible.
(B) It is partially correct.
(C) It is highly admirable.
(D) It is controversial though persuasive.
(E) It is intriguing though unsubstantiated.
4.According to the passage, Bailyn and the author agree on which of the following statements

about the culture of colonial New England?
(A) High culture in New England never equaled the high culture of England.
(B) The cultural achievements of colonial New England have generally been unrecognized by

historians.
(C) The colonists imitated the high culture of England, and did not develop a culture that

was uniquely their own.
(D) The southern colonies were greatly influenced by the high culture of New England.
(E) New England communities were able to create laws and build a university, but unable to

create anything innovative in the arts.
5.According to the passage, which of the following is true of English migrants to the

colonies during the eighteenth century?
(A) Most of them were farmers rather than trades people or artisans.
(B) Most of them came because they were unable to find work in England.
(C) They differed from other English people in that they were willing to travel.
(D) They expected that the colonies would offer them increased opportunity.
(E) They were generally not as educated as the people who remained in England.
6.The author of the passage is primarily concerned with
(A) comparing several current interpretations of early American history
(B) suggesting that new social research on migration should lead to revisions in current

interpretations of early American history
(C) providing the theoretical framework that is used by most historians in understanding

early American history
(D) refuting an argument about early American history that has been proposed by social

historians
(E) discussing a reinterpretation of early American history that is based on new social

research on migration
7.It can be inferred from the passage that American history textbooks used to assert that
(A) many migrants to colonial North America were not successful financially
(B) more migrants came to America out of religious or political conviction that came in the

hope of acquiring land
(C) New England communities were much alike in terms of their economics and demographics
(D) many migrants to colonial North America failed to maintain ties with their European

relations
(E) the level of literacy in New England communities was very high
8.The author of the passage would be most likely to agree with which of the following

statements about Bailyn’s work?
(A) Bailyn underestimates the effects of Puritan thought on North American culture.
(B) Bailyn overemphasizes the economic dependence of the colonies on Great Britain.
(C) Bailyn’s description of the colonies as part of an Anglo-American empire is misleading

and incorrect.
(D) Bailyn failed to test his propositions on a specific group of migrants to colonial North

America.
(E) Bailyn overemphasizes the experiences of migrants to the New England colonies, and

neglects the southern and the western parts of the New World.
Passage 22 (22/63)
Many United States companies have, unfortunately, made the search for legal protection from

import competition into a major line of work. Since 1980 the United States International

Trade Commission (ITC) has received about 280 complaints alleging damage from imports that

benefit from subsidies by foreign governments. Another 340 charge that foreign companies

“dumped” their products in the United States at “less than fair value.” Even when no unfair

practices are alleged, the simple claim that an industry has been injured by imports is

sufficient grounds to seek relief.
Contrary to the general impression, this quest for import relief has hurt more companies

than it has helped. As corporations begin to function globally, they develop an intricate

web of marketing, production, and research relationships. The complexity of these

relationships makes it unlikely that a system of import relief laws will meet the strategic

needs of all the units under the same parent company.
Internationalization increases the danger that foreign companies will use import relief laws

against the very companies the laws were designed to protect. Suppose a United States-owned

company establishes an overseas plant to manufacture a product while its competitor makes

the same product in the United States. If the competitor can prove injury from the

imports—and that the United States company received a subsidy from a foreign government to

build its plant abroad—the United States company’s products will be uncompetitive in the

United States, since they would be subject to duties.
Perhaps the most brazen case occurred when the ITC investigated allegations that Canadian

companies were injuring the United States salt industry by dumping rock salt, used to de-ice

roads. The bizarre aspect of the complaint was that a foreign conglomerate with United

States operations was crying for help against a United States company with foreign

operations. The “United States” company claiming injury was a subsidiary of a Dutch

conglomerate, while the “Canadian” companies included a subsidiary of a Chicago firm that

was the second-largest domestic producer of rock salt.
1.The passage is chiefly concerned with
(A) arguing against the increased internationalization of United States corporations
(B) warning that the application of laws affecting trade frequently has unintended

consequences
(C) demonstrating that foreign-based firms receive more subsidies from their governments

than United States firms receive from the United States government
(D) advocating the use of trade restrictions for “dumped” products but not for other imports
(E) recommending a uniform method for handling claims of unfair trade practices
2.It can be inferred from the passage that the minimal basis for a complaint to the

International Trade Commission is which of the following?
(A) A foreign competitor has received a subsidy from a foreign government.
(B) A foreign competitor has substantially increased the volume of products shipped to the

United States.
(C) A foreign competitor is selling products in the United States at less than fair market

value.
(D) The company requesting import relief has been injured by the sale of imports in the

United States.
(E) The company requesting import relief has been barred from exporting products to the

country of its foreign competitor.
3.The last paragraph performs which of the following functions in the passage?
(A) It summarizes the discussion thus far and suggests additional areas of research.
(B) It presents a recommendation based on the evidence presented earlier.
(C) It discusses an exceptional case in which the results expected by the author of the

passage were not obtained.
(D) It introduces an additional area of concern not mentioned earlier.
(E) It cites a specific case that illustrates a problem presented more generally in the

previous paragraph.
4.The passage warns of which of the following dangers?
(A) Companies in the United States may receive no protection from imports unless they

actively seek protection from import competition.
(B) Companies that seek legal protection from import competition may incur legal costs that

far exceed any possible gain.
(C) Companies that are United States-owned but operate internationally may not be eligible

for protection from import competition under the laws of the countries in which their plants

operate.
(D) Companies that are not United States-owned may seek legal protection from import

competition under United States import relief laws.
(E) Companies in the United States that import raw materials may have to pay duties on those

materials.
5.The passage suggests that which of the following is most likely to be true of United

States trade laws?
(A) They will eliminate the practice of “dumping” products in the United States.
(B) They will enable manufacturers in the United States to compete more profitably outside

the United States.
(C) They will affect United States trade with Canada more negatively than trade with other

nations.
(D) Those that help one unit within a parent company will not necessarily help other units

in the company.
(E) Those that are applied to international companies will accomplish their intended result.
6.It can be inferred from the passage that the author believes which of the following about

the complaint mentioned in the last paragraph?
(A) The ITC acted unfairly toward the complainant in its investigation.
(B) The complaint violated the intent of import relief laws.
(C) The response of the ITC to the complaint provided suitable relief from unfair trade

practices to the complainant.
(D) The ITC did not have access to appropriate information concerning the case.
(E) Each of the companies involved in the complaint acted in its own best interest.
7.According to the passage, companies have the general impression that International Trade

Commission import relief practices have
(A) caused unpredictable fluctuations in volumes of imports and exports
(B) achieved their desired effect only under unusual circumstances
(C) actually helped companies that have requested import relief
(D) been opposed by the business community
(E) had less impact on international companies than the business community expected
8.According to the passage, the International Trade Commission is involved in which of the

following?
(A) Investigating allegations of unfair import competition
(B) Granting subsidies to companies in the United States that have been injured by import

competition
(C) Recommending legislation to ensure fair
(D) Identifying international corporations that wish to build plants in the United States
(E) Assisting corporations in the United States that wish to compete globally
Passage 23 (23/63)
At the end of the nineteenth century, a rising interest in Native American customs and an

increasing desire to understand Native American culture prompted ethnologists to begin

recording the life stories of Native American. Ethnologists had a distinct reason for

wanting to hear the stories: they were after linguistic or anthropological data that would

supplement their own field observations, and they believed that the personal stories, even

of a single individual, could increase their understanding of the cultures that they had

been observing from without. In addition many ethnologists at the turn of the century

believed that Native American manners and customs were rapidly disappearing, and that it was

important to preserve for posterity as much information as could be adequately recorded

before the cultures disappeared forever.
There were, however, arguments against this method as a way of acquiring accurate and

complete information. Franz Boas, for example, described autobiographies as being “of

limited value, and useful chiefly for the study of the perversion of truth by memory,” while

Paul Radin contended that investigators rarely spent enough with the tribes they were

observing, and inevitably derived results too tinged by the investigator’s own emotional

tone to be reliable.
Even more importantly, as these life stories moved from the traditional oral mode to

recorded written form, much was inevitably lost. Editors often decided what elements were

significant to the field research on a given tribe. Native Americans recognized that the

essence of their lives could not be communicated in English and that events that they

thought significant were often deemed unimportant by their interviewers. Indeed, the very

act of telling their stories could force Native American narrators to distort their

cultures, as taboos had to be broken to speak the names of dead relatives crucial to their

family stories.
Despite all of this, autobiography remains a useful tool for ethnological research: such

personal reminiscences and impressions, incomplete as they may be, are likely to throw more

light on the working of the mind and emotions than any amount of speculation from an

ethnologist or ethnological theorist from another culture.
1.Which of the following best describes the organization of the passage?
(A) The historical backgrounds of two currently used research methods are chronicled.
(B) The validity of the data collected by using two different research methods is compared.
(C) The usefulness of a research method is questioned and then a new method is proposed.
(D) The use of a research method is described and the limitations of the results obtained

are discussed.
(E) A research method is evaluated and the changes necessary for its adaptation to other

subject areas are discussed.
2.Which of the following is most similar to the actions of nineteenth-century ethnologists

in their editing of the life stories of Native Americans?
(A) A witness in a jury trial invokes the Fifth Amendment in order to avoid relating

personally incriminating evidence.
(B) A stockbroker refuses to divulge the source of her information on the possible future

increase in a stock’s value.
(C) A sports announcer describes the action in a team sport with which he is unfamiliar.
(D) A chef purposely excludes the special ingredient from the recipe of his prizewinning

dessert.
(E) A politician fails to mention in a campaign speech the similarities in the positions

held by her opponent for political office and by herself.
3.According to the passage, collecting life stories can be a useful methodology because
(A) life stories provide deeper insights into a culture than the hypothesizing of academics

who are not members of that culture
(B) life stories can be collected easily and they are not subject to invalid interpretations
(C) ethnologists have a limited number of research methods from which to choose
(D) life stories make it easy to distinguish between the important and unimportant features

of a culture
(E) the collection of life stories does not require a culturally knowledgeable investigator
4.Information in the passage suggests that which of the following may be a possible way to

eliminate bias in the editing of life stories?
(A) Basing all inferences made about the culture on an ethnological theory
(B) Eliminating all of the emotion-laden information reported by the informant
(C) Translating the informant’s words into the researcher’s language
(D) Reducing the number of and carefully specifying the content of the that the

investigator can ask the informant
(E) Reporting all of the information that the informant provides regardless of the

investigator’s personal opinion about its intrinsic value
5.The primary purpose of the passage as a whole is to
(A) question an explanation
(B) correct a misconception
(C) critique a methodology
(D) discredit an idea
(E) clarify an ambiguity
6.It can be inferred from the passage that a characteristic of the ethnological research on

Native Americans conducted during the nineteenth century was the use of which of the

following?
(A) Investigators familiar with the culture under study
(B) A language other than the informant’s for recording life stories
(C) Life stories as the ethnologist’s primary source of information
(D) Complete transcriptions of informants’ descriptions of tribal beliefs
(E) Stringent guidelines for the preservation of cultural data
7.The passage mentions which of the following as a factor that can affect the accuracy of

ethnologists’ transcriptions of life stories?
(A) The informants’ social standing within the culture
(B) The inclusiveness of the theory that provided the basis for the research
(C) The length of the researchers spent in the culture under study
(D) The number of life stories collected by the researchers
(E) The verifiability of the information provided by the research informants
8.It can be inferred from the passage that the author would be most likely to agree with

which of the following statements about the usefulness of life stories as a source of

ethnographic information?
(A) They can be a source of information about how people in a culture view the world.
(B) They are most useful as a source of linguistic information.
(C) They require editing and interpretation before they can be useful.
(D) They are most useful as a source of information about ancestry.
(E) They provide incidental information rather than significant insights into a way of life.
Passage 24 (24/63)
All of the cells in a particular plant start out with the same complement of genes. How then

can these cells differentiate and form structures as different as roots, stems, leaves, and

fruits? The answer is that only a small subset of the genes in a particular kind of cell are

expressed, or turned on, at a given . This is accomplished by a complex system of chemical

messengers that in plants include hormones and other regulatory molecules. Five major

hormones have been identified: auxin, abscisic acid, cytokinin, ethylene, and gibberellin.

Studies of plants have now identified a new class of regulatory molecules called

oligosaccharins.
Unlike the oligosaccharins, the five well-known plant hormones are pleiotropic rather than

specific; that is, each has more than one effect on the growth and development of plants.

The five has so many simultaneous effects that they are not very useful in artificially

controlling the growth of crops. Auxin, for instance, stimulates the rate of cell

elongation, causes shoots to grow up and roots to grow down, and inhibits the growth of

lateral shoots. Auxin also causes the plant to develop a vascular system, to form lateral

roots, and to produce ethylene.
The pleiotropy of the five well-studied plant hormones is somewhat analogous to that of

certain hormones in animal. For example, hormones from the hypothalamus in the brain

stimulate the anterior lobe of the pituitary gland to synthesize and release many different

hormones, one of which stimulates the release of hormones from the adrenal cortex. These

hormones have specific effects on target organs all over the body. One hormone stimulates

the thyroid gland, for example, another the ovarian follicle cells, and so forth. In other

words, there is a hierarchy of hormones. Such a hierarchy may also exist in plants.

Oligosaccharins are fragments of the cell wall released by enzymes: different enzymes

release different oligosaccharins. There are indications that pleiotropic plant hormones may

actually function by activating the enzymes that release these other, more specific chemical

messengers from the cell wall.
1.According to the passage, the five well-known plant hormones are not useful in controlling

the growth of crops because
(A) it is not known exactly what functions the hormones perform
(B) each hormone has various effects on plants
(C) none of the hormones can function without the others
(D) each hormone has different effects on different kinds of plants
(E) each hormone works on only a small subset of a cell’s genes at any particular
2.The passage suggests that the place of hypothalamic hormones in the hormonal hierarchies

of animals is similar to the place of which of the following in plants?
(A) Plant cell walls
(B) The complement of genes in each plant cell
(C) A subset of a plant cell’s gene complement
(D) The five major hormones
(E) The oligosaccharins
3.The passage suggests that which of the following is a function likely to be performed by

an oligosaccharin?
(A) To stimulate a particular plant cell to become part of a plant’s root system
(B) To stimulate the walls of a particular cell to produce other oligosaccharins
(C) To activate enzymes that release specific chemical messengers from plant cell walls
(D) To duplicate the gene complement in a particular plant cell
(E) To produce multiple effects on a particular subsystem of plant cells
4.The author mentions specific effects that auxin has on plant development in order to

illustrate the
(A) point that some of the effects of plant hormones can be harmful
(B) way in which hormones are produced by plants
(C) hierarchical nature of the functioning of plant hormones
(D) differences among the best-known plant hormones
(E) concept of pleiotropy as it is exhibited by plant hormones
5.According to the passage, which of the following best describes a function performed by

oligosaccharins?
(A) Regulating the daily functioning of a plant’s cells
(B) Interacting with one another to produce different chemicals
(C) Releasing specific chemical messengers from a plant’s cell walls
(D) Producing the hormones that cause plant cells to differentiate to perform different

functions
(E) Influencing the development of a plant’s cells by controlling the expression of the

cells’ genes
6.The passage suggests that, unlike the pleiotropic hormones, oligosaccharins could be used

effectively to
(A) trace the passage of chemicals through the walls of cells
(B) pinpoint functions of other plant hormones
(C) artificially control specific aspects of the development of crops
(D) alter the complement of genes in the cells of plants
(E) alter the effects of the five major hormones on plant development
7.The author discusses animal hormones primarily in order to
(A) introduce the idea of a hierarchy of hormones
(B) explain the effects that auxin has on plant cells
(C) contrast the functioning of plant hormones and animals hormones
(D) illustrate the way in which particular hormones affect animals
(E) explain the distinction between hormones and regulatory molecules
Passage 25 (25/63)
In 1977 the prestigious Ewha Women’s University in Seoul, Korea, announced the opening of

the first women’s studies program in Asia. Few academic programs have ever received such

public attention. In broadcast debates, critics dismissed the program as a betrayal of

national identity, an imitation of Western ideas, and a distraction from the real task of

national unification and economic development. Even supporters underestimated the program;

they thought it would be merely another of the many Western ideas that had already proved

useful in Asian culture, akin to airlines, electricity, and the assembly line. The founders

of the program, however, realized that neither view was correct. They had some reservations

about the applicability of Western feminist theories to the role of women in Asia and felt

that such theories should be closely examined. Their approach has thus far yielded important

critiques of Western theory, informed by the special experience of Asian women.
For instance, like the Western feminist critique of the Freudian model of the human psyche,

the Korean critique finds Freudian theory culture-bound, but in ways different from those

cited by Western theorists. The Korean theorists claim that Freudian theory assumes the

universality of the Western nuclear, male-headed family and focuses on the personality

formation of the individual, independent of society. An analysis based on such assumptions

could be valid for a highly competitive, individualistic society. In the Freudian family

drama, family members are assumed to be engaged in a Darwinian struggle against each

other—father against son and sibling against sibling. Such a concept projects the

competitive model of Western society onto human personalities. But in the Asian concept of

personality there is no ideal attached to individualism or to the independent self. The

Western model of personality development does not explain major characteristics of the

Korean personality, which is social and group-centered. The “self” is a social being defined

by and acting in a group, and the well-being of both men and women is determined by the

equilibrium of the group, not by individual self-assertion. The ideal is one of

interdependency.
In such a context, what is recognized as “dependency” in Western psychiatric terms is not,

in Korean terms, an admission of weakness or failure. All this bears directly on the Asian

perception of men’s and women’s psychology because men are also “dependent.” In Korean

culture, men cry and otherwise easily show their emotions, something that might be

considered a betrayal of masculinity in Western culture. In the kinship-based society of

Korea, four generations may live in the same house, which means that people can be sons and

daughters all their lives, whereas in Western culture, the roles of husband and son, wife

and daughter, are often incompatible.
1.Which of the following best summarizes the content of the passage?
(A) A critique of a particular women’s studies program
(B) A report of work in social theory done by a particular women’s studies program
(C) An assessment of the strengths and weaknesses of a particular women’s studies program
(D) An analysis of the philosophy underlying women’s studies programs
(E) An abbreviated history of Korean women’s studies programs
2.It can be inferred from the passage that Korean scholars in the field of women’s studies

undertook an analysis of Freudian theory as a response to which of the following?
(A) Attacks by critics of the Ewha women’s studies program
(B) The superficiality of earlier critiques of Freudian theory
(C) The popularity of Freud in Korean psychiatric circles
(D) Their desire to encourage Korean scholars to adopt the Freudian model
(E) Their assessment of the relevance and limitations of Western feminist theory with

respect to Korean culture
3.Which of the following conclusions about the introduction of Western ideas to Korean

society can be supported by information contained in the passage?
(A) Except for technological innovations, few Western ideas have been successfully

transplanted into Korean society.
(B) The introduction of Western ideas to Korean society is viewed by some Koreans as a

challenge to Korean identity.
(C) The development of the Korean economy depends heavily on the development of new academic

programs modeled after Western programs.
(D) The extent to which Western ideas must be adapted for acceptance by Korean society is

minimal.
(E) The introduction of Western ideas to Korean society accelerated after 1977.
4.It can be inferred from the passage that the broadcast media in Korea considered the

establishment of the Ewha women’s studies program
(A) praiseworthy
(B) insignificant
(C) newsworthy
(D) imitative
(E) incomprehensible
5.It can be inferred from the passage that the position taken by some of the supporters of

the Ewha women’s studies program was problematic to the founders of the program because

those supporters
(A) assumed that the program would be based on the uncritical adoption of Western theory
(B) failed to show concern for the issues of national unification and economic development
(C) were unfamiliar with Western feminist theory
(D) were not themselves scholars in the field of women’s studies
(E) accepted the universality of Freudian theory
6.Which of the following statements is most consistent with the view of personality

development held by the Ewha women’s studies group?
(A) Personality development occurs in identifiable stages, beginning with dependency in

childhood and ending with independence in adulthood.
(B) Any theory of personality development, in order to be valid, must be universal.
(C) Personality development is influenced by the characteristics of the society in which a

person lives.
(D) Personality development is hindered if a person is not permitted to be independent.
(E) No theory of personality development can account for the differences between Korean and

Western culture.
7.Which of the following statements about the Western feminist critique of Freudian theory

can be supported by information contained in the passage?
(A) It recognizes the influence of Western culture on Freudian theory.
(B) It was written after 1977.
(C) It acknowledges the universality of the nuclear, male-headed family.
(D) It challenges Freud’s analysis of the role of daughters in Western society.
(E) It fails to address the issue of competitiveness in Western society.
8.According to the passage, critics of the Ewha women’s studies program cited the program as

a threat to which of the following?
I.National identity
II.National unification
III.Economic development
IV.Family integrity
(A) I only
(B) I and II only
(C) I, II, and III only
(D) II, III, and IV only
(E) I, II, III, and IV
Passage 26 (26/63)
In choosing a method for determining climatic conditions that existed in the past,

paleoclimatologists invoke four principal criteria. First, the material—rocks, lakes,

vegetation, etc.—on which the method relies must be widespread enough to provide plenty of

information, since analysis of material that is rarely encountered will not permit

correlation with other regions or with other periods of geological history. Second, in the

process of formation, the material must have received an environmental signal that reflects

a change in climate and that can be deciphered by modern physical or chemical means. Third,

at least some of the material must have retained the signal unaffected by subsequent changes

in the environment. Fourth, it must be possible to determine the at which the inferred

climatic conditions held. This last criterion is more easily met in dating marine sediments,

because dating of only a small number of layers in a marine sequence allows the age of other

layers to be estimated fairly reliably by extrapolation and interpolation. By contrast,

because sedimentation is much less continuous in continental regions, estimating the age of

a continental bed from the known ages of beds above and below is more risky.
One very old method used in the investigation of past climatic conditions involves the

measurement of water levels in ancient lakes. In temperate regions, there are enough lakes

for correlations between them to give us a reliable picture. In arid and semiarid regions,

on the other hand, the small number of lakes and the great distances between them reduce the

possibilities for correlation. Moreover, since lake levels are controlled by rates of

evaporation as well as by precipitation, the interpretation of such levels is ambiguous. For

instance, the fact that lake levels in the semiarid southwestern United States appear to

have been higher during the last ice age than they are now was at one attributed to

increased precipitation. On the basis of snow-line elevations, however, it has been

concluded that the climate then was not necessarily wetter than it is now, but rather that

both summers and winters were cooler, resulting in reduced evaporation.
Another problematic method is to reconstruct former climates on the basis of pollen

profiles. The type of vegetation in a specific region is determined by identifying and

counting the various pollen grains found there. Although the relationship between vegetation

and climate is not as direct as the relationship between climate and lake levels, the method

often works well in the temperate zones. In arid and semiarid regions in which there is not

much vegetation, however, small changes in one or a few plant types can change the picture

dramatically, making accurate correlations between neighboring areas difficult to obtain.
1.Which of the following statements about the difference between marine and continental

sedimentation is supported by information in the passage?
(A) Data provided by dating marine sedimentation is more consistent with researchers’

findings in other disciplines than is data provided by dating continental sedimentation.
(B) It is easier to estimate the age of a layer in a sequence of continental sedimentation

than it is to estimate the age of a layer in a sequence of marine sedimentation.
(C) Marine sedimentation is much less widespread than continental sedimentation.
(D) Researchers are more often forced to rely on extrapolation when dating a layer of marine

sedimentation than when dating a layer of continental sedimentation.
(E) Marine sedimentation is much more continuous than is continental sedimentation.
2.Which of the following statements best describes the organization of the passage as a

whole?
(A) The author describes a method for determining past climatic conditions and then offers

specific examples of situations in which it has been used.
(B) The author discusses the method of dating marine and continental sequences and then

explains how dating is more difficult with lake levels than with pollen profiles.
(C) The author describes the common requirements of methods for determining past climatic

conditions and then discusses examples of such methods.
(D) The author describes various ways of choosing a material for determining past climatic

conditions and then discusses how two such methods have yielded contradictory data.
(E) The author describes how methods for determining past climatic conditions were first

developed and then describes two of the earliest known methods.
3.It can be inferred from the passage that paleoclimatologists have concluded which of the

following on the basis of their study of snow-line elevations in the southwestern United

States?
(A) There is usually more precipitation during an ice age because of increased amounts of

evaporation.
(B) There was less precipitation during the last ice age than there is today.
(C) Lake levels in the semiarid southwestern United States were lower during the last ice

age than they are today.
(D) During the last ice age, cooler weather led to lower lake levels than

paleoclimatologists had previously assumed.
(E) The high lake levels during the last ice age may have been a result of less evaporation

rather than more precipitation.
4.Which of the following would be the most likely topic for a paragraph that logically

continues the passage?
(A) The kinds of plants normally found in arid regions
(B) The effect of variation in lake levels on pollen distribution
(C) The material best suited to preserving signals of climatic changes
(D) Other criteria invoked by paleoclimatologists when choosing a method to determine past

climatic conditions
(E) A third method for investigating past climatic conditions
5.The author discusses lake levels in the southwestern United States in order to
(A) illustrate the mechanics of the relationship between lake level, evaporation, and

precipitation
(B) provide an example of the uncertainty involved in interpreting lake levels
(C) prove that there are not enough ancient lakes with which to make accurate correlations
(D) explain the effects of increased rates of evaporation on levels of precipitation
(E) suggest that snow-line elevations are invariably more accurate than lake levels in

determining rates of precipitation at various points in the past
6.It can be inferred from the passage that an environmental signal found in geological

material would not be useful to paleoclimatologists if it
(A) had to be interpreted by modern chemical means
(B) reflected a change in climate rather than a long-term climatic condition
(C) was incorporated into a material as the material was forming
(D) also reflected subsequent environmental changes
(E) was contained in a continental rather than a marine sequence
7.According to the passage, the material used to determine past climatic conditions must be

widespread for which of the following reasons?
I.Paleoclimatologists need to make comparisons between periods of geological history.
II.Paleoclimatologists need to compare materials that have supported a wide variety of

vegetation.
III.Paleoclimatologists need to make comparisons with data collected in other regions.
(A) I only
(B) II only
(C) I and II only
(D) I and III only
(E) II and III only
8.Which of the following can be inferred from the passage about the study of past climates

in arid and semiarid regions?
(A) It is sometimes more difficult to determine past climatic conditions in arid and

semiarid regions than in temperate regions.
(B) Although in the past more research has been done on temperate regions,

paleoclimatologists have recently turned their attention to arid and semiarid regions.
(C) Although more information about past climates can be gathered in arid and semiarid than

in temperate regions, dating this information is more difficult.
(D) It is difficult to study the climatic history of arid and semiarid regions because their

climates have tended to vary more than those of temperate regions.
(E) The study of past climates in arid and semiarid regions has been neglected because

temperate regions support a greater variety of plant and animal life.
Passage 27 (27/63)
Since the late 1970’s, in the face of a severe loss of market share in dozens of industries,

manufacturers in the United States have been trying to improve productivity—and therefore

enhance their international competitiveness—through cost-cutting programs. (Cost-cutting

here is defined as raising labor output while holding the amount of labor constant.)

However, from 1978 through 1982, productivity—the value of goods manufactured divided by the

amount of labor input—did not improve; and while the results were better in the business

upturn of the three years following, they ran 25 percent lower than productivity

improvements during earlier, post-1945 upturns. At the same , it became clear that the

harder manufactures worked to implement cost-cutting, the more they lost their competitive

edge.
With this paradox in mind, I recently visited 25 companies; it became clear to me that the

cost-cutting approach to increasing productivity is fundamentally flawed. Manufacturing

regularly observes a “40, 40, 20” rule. Roughly 40 percent of any manufacturing-based

competitive advantage derives from long-term changes in manufacturing structure (decisions

about the number, size, location, and capacity of facilities) and in approaches to

materials. Another 40 percent comes from major changes in equipment and process technology.

The final 20 percent rests on implementing conventional cost-cutting. This rule does not

imply that cost-cutting should not be tried. The well-known tools of this approach—including

simplifying jobs and retraining employees to work smarter, not harder—do produce results.

But the tools quickly reach the limits of what they can contribute.
Another problem is that the cost-cutting approach hinders innovation and discourages

creative people. As Abernathy’s study of automobile manufacturers has shown, an industry can

easily become prisoner of its own investments in cost-cutting techniques, reducing its

ability to develop new products. And managers under pressure to maximize cost-cutting will

resist innovation because they know that more fundamental changes in processes or systems

will wreak havoc with the results on which they are measured. Production managers have

always seen their job as one of minimizing costs and maximizing output. This dimension of

performance has until recently sufficed as a basis of evaluation, but it has created a

penny-pinching, mechanistic culture in most factories that has kept away creative managers.
Every company I know that has freed itself from the paradox has done so, in part, by

developing and implementing a manufacturing strategy. Such a strategy focuses on the

manufacturing structure and on equipment and process technology. In one company a

manufacturing strategy that allowed different areas of the factory to specialize in

different markets replaced the conventional cost-cutting approach; within three years the

company regained its competitive advantage. Together with such strategies, successful

companies are also encouraging managers to focus on a wider set of objectives besides

cutting costs. There is hope for manufacturing, but it clearly rests on a different way of

managing.
1.The author of the passage is primarily concerned with
(A) summarizing a thesis
(B) recommending a different approach
(C) comparing points of view
(D) making a series of predictions
(E) describing a number of paradoxes
2.It can be inferred from the passage that the manufacturers mentioned in line 2 expected

that the measures they implemented would
(A) encourage innovation
(B) keep labor output constant
(C) increase their competitive advantage
(D) permit business upturns to be more easily predicted
(E) cause managers to focus on a wider set of objectives
3.The primary function of the first paragraph of the passage is to
(A) outline in brief the author’s argument
(B) anticipate challenges to the prescriptions that follow
(C) clarify some disputed definitions of economic terms
(D) summarize a number of long-accepted explanations
(E) present a historical context for the author’s observations
4.The author refers to Abernathy’s study (line 36) most probably in order to
(A) qualify an observation about one rule governing manufacturing
(B) address possible objections to a recommendation about improving manufacturing

competitiveness
(C) support an earlier assertion about one method of increasing productivity
(D) suggest the centrality in the United States economy of a particular manufacturing

industry
(E) given an example of research that has questioned the wisdom of revising a manufacturing

strategy
5.The author’s attitude toward the culture in most factories is best described as
(A) cautious
(B) critical
(C) disinterested
(D) respectful
(E) adulatory
6.In the passage, the author includes all of the following EXCEPT
(A) personal observation
(B) a business principle
(C) a definition of productivity
(D) an example of a successful company
(E) an illustration of a process technology
7.The author suggests that implementing conventional cost-cutting as a way of increasing

manufacturing competitiveness is a strategy that is
(A) flawed and ruinous
(B) shortsighted and difficult to sustain
(C) popular and easily accomplished
(D) useful but inadequate
(E) misunderstood but promising
Passage 28 (28/63)
The settlement of the United States has occupied traditional historians since 1893 when

Frederick Jackson Turner developed his Frontier Thesis, a thesis that explained American

development in terms of westward expansion. From the perspective of women’s history,

Turner’s exclusively masculine assumptions constitute a major drawback: his defenders and

critics alike have reconstructed men’s, not women’s, lives on the frontier. However,

precisely because of this masculine orientation, revising the Frontier Thesis by focusing on

women’s experience introduces new themes into women’s history—woman as lawmaker and

entrepreneur—and, consequently, new interpretations of women’s relationship to capital,

labor, and statute.
Turner claimed that the frontier produced the individualism that is the hallmark of American

culture, and that this individualism in turn promoted democratic institutions and economic

equality. He argued for the frontier as an agent of social change. Most novelists and

historians writing in the early to midtwentieth century who considered women in the West,

when they considered women at all, fell under Turner’s spell. In their works these authors

tended to glorify women’s contributions to frontier life. Western women, in Turnerian

tradition, were a fiercely independent, capable, and durable lot, free from the constraints

binding their eastern sisters. This interpretation implied that the West provided a

congenial environment where women could aspire to their own goals, free from constrictive

stereotypes and sexist attitudes. In Turnerian terminology, the frontier had furnished “a

gate of escape from the bondage of the past.”
By the middle of the twentieth century, the Frontier Thesis fell into disfavor among

historians. Later, Reactionist writers took the view that frontier women were lonely,

displaced persons in a hostile milieu that intensified the worst aspects of gender

relations. The renaissance of the feminist movement during the 1970’s led to the Stasist

school, which sidestepped the good bad dichotomy and argued that frontier women lived lives

similar to the live of women in the East. In one now-standard text, Faragher demonstrated

the persistence of the “cult of true womanhood” and the illusionary quality of change on the

westward journey. Recently the Stasist position has been revised but not entirely discounted

by new research.
1.The primary purpose of the passage is to
(A) provide a framework within which the history of women in nineteenth-century America can

be organized
(B) discuss divergent interpretations of women’s experience on the western frontier
(C) introduce a new hypothesis about women’s experience in nineteenth-century America
(D) advocate an empirical approach to women’s experience on the western frontier
(E) resolve ambiguities in several theories about women’s experience on the western frontier
2.Which of the following can be inferred about the novelists and historians mentioned in

lines 19-20?
(A) They misunderstood the powerful influence of constrictive stereotypes on women in the

East.
(B) They assumed that the frontier had offered more opportunities to women than had the

East.
(C) They included accurate information about women’s experiences on the frontier.
(D) They underestimated the endurance and fortitude of frontier women.
(E) They agreed with some of Turner’s assumptions about frontier women, but disagreed with

other assumptions that he made.
3.Which of the following, if true, would provide additional evidence for the Stasists’

argument as it is described in the passage?
(A) Frontier women relied on smaller support groups of relatives and friends in the West

than they had in the East.
(B) The urban frontier in the West offered more occupational opportunity than the

agricultural frontier offered.
(C) Women participated more fully in the economic decisions of the family group in the West

than they had in the East.
(D) Western women received financial compensation for labor that was comparable to what

women received in the East.
(E) Western women did not have an effect on divorce laws, but lawmakers in the West were

more responsive to women’s concerns than lawmakers in the East were.
4.According to the passage, Turner makes which of the following connections in his Frontier

Thesis?
I.A connection between American individualism and economic equality
II.A connection between geographical expansion and social change
III.A connection between social change and financial prosperity
(A) I only
(B) II only
(C) III only
(D) I and II only
(E) I, II and III
5.It can be inferred that which of the following statements is consistent with the

Reactionist position as it is described in the passage?
(A) Continuity, not change, marked women’s lives as they moved from East to West.
(B) Women’s experience on the North American frontier has not received enough attention from

modern historians.
(C) Despite its rigors, the frontier offered women opportunities that had not been available

in the East.
(D) Gender relations were more difficult for women in the West than they were in the East.
(E) Women on the North American frontier adopted new roles while at the same reaffirming

traditional roles.
6.Which of the following best describes the organization of the passage?
(A) A current interpretation of a phenomenon is described and then ways in which it was

developed are discussed.
(B) Three theories are presented and then a new hypothesis that discounts those theories is

described.
(C) An important theory and its effects are discussed and then ways in which it has been

revised are described.
(D) A controversial theory is discussed and then viewpoints both for and against it are

described.
(E) A phenomenon is described and then theories concerning its correctness are discussed.
7.Which of the following is true of the Stasist School as it is described in the passage?
(A) It provides new interpretations of women’s relationship to work and the law.
(B) It resolves some of the ambiguities inherent in Turnerian and Reactionist thought.
(C) It has recently been discounted by new research gathered on women’s experience.
(D) It avoids extreme positions taken by other writers on women’s history.
(E) It was the first school of thought to suggest substantial revisions to the Frontier

Thesis.
Passage 29 (29/63)
Studies of the Weddell seal in the laboratory have described the physiological mechanisms

that allow the seal to cope with the extreme oxygen deprivation that occurs during its

longest dives, which can extend 500 meters below the ocean’s surface and last for over 70 .

Recent field studies, however, suggest that during more typical dives in the wild, this

seal’s physiological behavior is different.
In the laboratory, when the seal dives below the surface of the water and stops breathing,

its heart beats more slowly, requiring less oxygen, and its arteries become constricted,

ensuring that the seal’s blood remains concentrated near those organs most crucial to its

ability to navigate underwater. The seal essentially shuts off the flow of blood to other

organs, which either stop functioning until the seal surfaces or switch to an anaerobic

(oxygen-independent) metabolism. The latter results in the production of large amounts of

lactic acid which can adversely affect the pH of the seal’s blood, but since the anaerobic

metabolism occurs only in those tissues which have been isolated from the seal’s blood

supply, the lactic acid is released into the seal’s blood only after the seal surfaces, when

the lungs, liver, and other organs quickly clear the acid from the seal’s bloodstream.
Recent field studies, however, reveal that on dives in the wild, the seal usually heads

directly for its prey and returns to the surface in less than twenty . The absence of high

levels of lactic acid in the seal’s blood after such dives suggests that during them, the

seal’s organs do not resort to the anaerobic metabolism observed in the laboratory, but are

supplied with oxygen from the blood. The seal’s longer excursions underwater, during which

it appears to be either exploring distant routes or evading a predator, do evoke the diving

response seen in the laboratory. But why do the seal’s laboratory dives always evoke this

response, regardless of their length or depth? Some biologists speculate that because in

laboratory dives the seal is forcibly submerged, it does not know how long it will remain

underwater and so prepares for the worst.
1.The passage provides information to support which of the following generalizations?
(A) Observations of animals’ physiological behavior in the wild are not reliable unless

verified by laboratory studies.
(B) It is generally less difficult to observe the physiological behavior of an animal in the

wild than in the laboratory.
(C) The level of lactic acid in an animal’s blood is likely to be higher when it is

searching for prey than when it is evading predators.
(D) The level of lactic acid in an animal’s blood is likely to be lowest during those

periods in which it experiences oxygen deprivation.
(E) The physiological behavior of animals in a laboratory setting is not always consistent

with their physiological behavior in the wild.
2.It can be inferred from the passage that by describing the Weddell seal as preparing “for

the worst” (line 41), biologists mean that it
(A) prepares to remain underwater for no longer than twenty
(B) exhibits physiological behavior similar to that which characterizes dives in which it

heads directly for its prey
(C) exhibits physiological behavior similar to that which characterizes its longest dives in

the wild
(D) begins to exhibit predatory behavior
(E) clears the lactic acid from its blood before attempting to dive
3.The passage suggests that during laboratory dives, the pH of the Weddell seal’s blood is

not adversely affected by the production of lactic acid because
(A) only those organs that are essential to the seal’s ability to navigate underwater revert

to an anaerobic mechanism
(B) the seal typically reverts to an anaerobic metabolism only at the very end of the dive
(C) organs that revert to an anaerobic metabolism are temporarily isolated from the seal’s

bloodstream
(D) oxygen continues to be supplied to organs that clear lactic acid from the seal’s

bloodstream
(E) the seal remains submerged for only short periods of
4.Which of the following best summarizes the main point of the passage?
(A) Recent field studies have indicated that descriptions of the physiological behavior of

the Weddell seal during laboratory dives are not applicable to its most typical dives in the

wild.
(B) The Weddell seal has developed a number of unique mechanisms that enable it to remain

submerged at depths of up to 500 meters for up to 70 .
(C) The results of recent field studies have made it necessary for biologists to revise

previous perceptions of how the Weddell seal behaves physiologically during its longest

dives in the wild.
(D) Biologists speculate that laboratory studies of the physiological behavior of seals

during dives lasting more than twenty would be more accurate if the seals were not forcibly

submerged.
(E) How the Weddell seal responds to oxygen deprivation during its longest dives appears to

depend on whether the seal is searching for prey or avoiding predators during such dives.
5.According to the author, which of the following is true of the laboratory studies

mentioned in line 1?
(A) They fail to explain how the seal is able to tolerate the increased production of lactic

acid by organs that revert to an anaerobic metabolism during its longest dives in the wild.
(B) They present an oversimplified account of mechanisms that the Weddell seal relies on

during its longest dives in the wild.
(C) They provide evidence that undermines the view that the Weddell seal relies on an

anaerobic metabolism during its most typical dives in the wild.
(D) They are based on the assumption that Weddell seals rarely spend more than twenty

underwater on a typical dive in the wild.
(E) They provide an accurate account of the physiological behavior of Weddell seals during

those dives in the wild in which they are either evading predators or exploring distant

routes.
6.The author cites which of the following as characteristic of the Weddell seal’s

physiological behavior during dives observed in the laboratory?
I.A decrease in the rate at which the seal’s heart beats
II.A constriction of the seal’s arteries
III.A decrease in the levels of lactic acid in the seal’s blood
IV.A temporary halt in the functioning of certain organs
(A) I and III only
(B) II and IV only
(C) II and III only
(D) I, II, and IV only
(E) I, III, and IV only
7.The passage suggests that because Weddell seals are forcibly submerged during laboratory

dives, they do which of the following?
(A) Exhibit the physiological responses that are characteristic of dives in the wild that

last less than twenty .
(B) Exhibit the physiological responses that are characteristic of the longer dives they

undertake in the wild.
(C) Cope with oxygen deprivation less effectively than they do on typical dives in the wild.
(D) Produce smaller amounts of lactic acid than they do on typical dives in the wild.
(E) Navigate less effectively than they do on typical dives in the wild.
Passage 30 (30/63)
Since the early 1970’s, historians have begun to devote serious attention to the working

class in the United States. Yet while we now have studies of working-class communities and

culture, we know remarkably little of worklessness. When historians have paid any attention

at all to unemployment, they have focused on the Great Depression of the 1930’s. The

narrowness of this perspective ignores the pervasive recessions and joblessness of the

previous decades, as Alexander Keyssar shows in his recent book. Examining the period

1870-1920, Keyssar concentrates on Massachusetts, where the historical materials are

particularly rich, and the findings applicable to other industrial areas.
The unemployment rates that Keyssar calculates appear to be relatively modest, at least by

Great Depression standards: during the worst years, in the 1870’s and 1890’s, unemployment

was around 15 percent. Yet Keyssar rightly understands that a better way to measure the

impact of unemployment is to calculate unemployment frequencies—measuring the percentage of

workers who experience any unemployment in the course of a year. Given this perspective,

joblessness looms much larger.
Keyssar also scrutinizes unemployment patterns according to skill level, ethnicity, race,

age, class, and gender. He finds that rates of joblessness differed primarily according to

class: those in middle-class and white-collar occupations were far less likely to be

unemployed. Yet the impact of unemployment on a specific class was not always the same. Even

when dependent on the same trade, adjoining communities could have dramatically different

unemployment rates. Keyssar uses these differential rates to help explain a phenomenon that

has puzzled historians—the startlingly high rate of geographical mobility in the

nineteenth-century United States. But mobility was not the dominant working-class strategy

for coping with unemployment, nor was assistance from private charities or state agencies.

Self-help and the help of kin got most workers through jobless spells.
While Keyssar might have spent more developing the implications of his findings on

joblessness for contemporary public policy, his study, in its thorough research and creative

use of quantitative and qualitative evidence, is a model of historical analysis.
1.The passage is primarily concerned with
(A) recommending a new course of investigation
(B) summarizing and assessing a study
(C) making distinctions among categories
(D) criticizing the current state of a field
(E) comparing and contrasting two methods for calculating data
2.The passage suggests that before the early 1970’s, which of the following was true of the

study by historians of the working class in the United States?
(A) The study was infrequent or superficial, or both.
(B) The study was repeatedly criticized for its allegedly narrow focus.
(C) The study relied more on qualitative than quantitative evidence.
(D) The study focused more on the working-class community than on working-class culture.
(E) The study ignored working-class joblessness during the Great Depression.
3.According to the passage, which of the following is true of Keyssar’s findings concerning

unemployment in Massachusetts?
(A) They tend to contradict earlier findings about such unemployment.
(B) They are possible because Massachusetts has the most easily accessible historical

records.
(C) They are the first to mention the existence of high rates of geographical mobility in

the nineteenth century.
(D) They are relevant to a historical understanding of the nature of unemployment in other

states.
(E) They have caused historians to reconsider the role of the working class during the Great

Depression.
4.According to the passage, which of the following is true of the unemployment rates

mentioned in line 15?
(A) They hovered, on average, around 15 percent during the period 1870-1920.
(B) They give less than a full sense of the impact of unemployment on working-class people.
(C) They overestimate the importance of middle class and white-collar unemployment.
(D) They have been considered by many historians to underestimate the extent of

working-class unemployment.
(E) They are more open to question when calculated for years other than those of peak

recession.
5.Which of the following statements about the unemployment rate during the Great Depression

can be inferred from the passage?
(A) It was sometimes higher than 15 percent.
(B) It has been analyzed seriously only since the early 1970’s.
(C) It can be calculated more easily than can unemployment frequency.
(D) It was never as high as the rate during the 1870’s.
(E) It has been shown by Keyssar to be lower than previously thought.
6.According to the passage, Keyssar considers which of the following to be among the

important predictors of the likelihood that a particular person would be unemployed in late

nineteenth-century Massachusetts?
I. The person’s class
II.Where the person lived or worked
III.The person’s age
(A) I only
(B) II only
(C) I and II only
(D) I and III only
(E) I, II, and III
7.The author views Keyssar’s study with
(A) impatient disapproval
(B) wary concern
(C) polite skepticism
(D) scrupulous neutrality
(E) qualified admiration
8.Which of the following, if true, would most strongly support Keyssar’s findings as they

are described by the author?
(A) Boston, Massachusetts, and Quincy, Massachusetts, adjoining communities, had a higher

rate of unemployment for working-class people in 1870 than in 1890.
(B) White-collar professionals such as attorneys had as much trouble as day laborers in

maintaining a steady level of employment throughout the period 1870-1920.
(C) Working-class women living in Cambridge, Massachusetts, were more likely than

working-class men living in Cambridge to be unemployed for some period of during the year

1873.
(D) In the 1890’s, shoe-factory workers moved away in large numbers from Chelmsford,

Massachusetts, where shoe factories were being replaced by other industries, to adjoining

West Chelmsford, where the shoe industry flourished.
(E) In the late nineteenth century, workers of all classes in Massachusetts were more likely

than workers of all classes in other states to move their place of residence from one

location to another within the state.
Passage 31 (31/63)
The number of women directors appointed to corporate boards in the United States has

increased dramatically, but the ratio of female to male directors remains low. Although

pressure to recruit women directors, unlike that to employ women in the general work force,

does not derive from legislation, it is nevertheless real.
Although small companies were the first to have women directors, large corporations

currently have a higher percentage of women on their boards. When the chairs of these large

corporations began recruiting women to serve on boards, they initially sought women who were

chief executive officers (CEO’s) of large corporations. However, such women CEO’s are still

rare. In addition, the ideal of six CEO’s (female or male) serving on the board of each of

the largest corporations is realizable only if every CEO serves on six boards. This raises

the specter of director over-commitment and the resultant dilution of contribution.

Consequently, the chairs next sought women in business who had the equivalent of CEO

experience. However, since it is only recently that large numbers of women have begun to

rise in management, the chairs began to recruit women of high achievement outside the

business world. Many such women are well known for their contributions in government,

education, and the nonprofit sector. The fact that the women from these sectors who were

appointed were often acquaintances of the boards’ chairs seems quite reasonable: chairs have

always considered it important for directors to interact comfortably in the boardroom.
Although many successful women from outside the business world are unknown to corporate

leaders, these women are particularly qualified to serve on boards because of the changing

nature of corporations. Today a company’s ability to be responsive to the concerns of the

community and the environment can influence that company’s growth and survival. Women are

uniquely positioned to be responsive to some of these concerns. Although conditions have

changed, it should be remembered that most directors of both sexes are over fifty years old.

Women of that generation were often encouraged to direct their attention toward efforts to

improve the community. This fact is reflected in the career development of most of the

outstandingly successful women of the generation now in their fifties, who currently serve

on corporate boards: 25 percent are in education and 22 percent are in government, law, and

the nonprofit sector.
One organization of women directors is helping business become more responsive to the

changing needs of society by raising the level of corporate awareness about social issues,

such as problems with the economy, government regulation, the aging population, and the

environment. This organization also serves as a resource center of information on

accomplished women who are potential candidates for corporate boards.
1.The author of the passage would be most likely to agree with which of the following

statements about achievement of the “ideal” mentioned in line 14?
(A) It has only recently become a possibility.
(B) It would be easier to meet if more CEO’s were women.
(C) It is very close to being a reality for most corporate boards.
(D) It might affect the quality of directors’ service to corporations.
(E) It would be more realizable if CEO’s had a more extensive range of business experience.
2.According to the passage, the pressure to appoint women to corporate boards differs from

the pressure to employ women in the work force in which of the following ways?
(A) Corporate boards are under less pressure because they have such a small number of

openings.
(B) Corporate boards have received less pressure from stockholders, consumers, and workers

within companies to include women on their boards.
(C) Corporate boards have received less pressure from the media and the public to include

women on their boards.
(D) Corporations have only recently been pressured to include women on their boards.
(E) Corporations are not subject to statutory penalty for failing to include women on their

boards.
3.All of the following are examples of issues that the organization described in the last

paragraph would be likely to advise corporations on EXCEPT
(A) long-term inflation
(B) health and safety regulations
(C) retirement and pension programs
(D) the energy shortage
(E) how to develop new markets
4.It can be inferred from the passage that, when seeking to appoint new members to a

corporation’s board, the chair traditionally looked for candidates who
(A) had legal and governmental experience
(B) had experience dealing with community affairs
(C) could work easily with other members of the board
(D) were already involved in establishing policy for that corporation
(E) had influential connections outside the business world
5.According to the passage, which of the following is true about women outside the business

world who are currently serving on corporate boards?
(A) Most do not serve on more than one board.
(B) A large percentage will eventually work on the staff of corporations.
(C) Most were already known to the chairs of the board to which they were appointed.
(D) A larger percentage are from government and law than are from the nonprofit sector.
(E) Most are less than fifty years old.
6.The passage suggests that corporations of the past differ from modern corporations in

which of the following ways?
(A) Corporations had greater input on government policies affecting the business community.
(B) Corporations were less responsive to the financial needs of their employees.
(C) The ability of a corporation to keep up with changing markets was not a crucial factor

in its success.
(D) A corporation’s effectiveness in coping with community needs was less likely to affect

its growth and prosperity.
(E) Corporations were subject to more stringent government regulations.
7.Which of the following best describes the organization of the passage?
(A) A problem is described, and then reasons why various proposed solutions succeeded or

failed are discussed.
(B) A problem is described, and then an advantage of resolving it is offered.
(C) A problem is described, and then reasons for its continuing existence are summarized.
(D) The historical origins of a problem are described, and then various measures that have

successfully resolved it are discussed.
(E) The causes of a problem are described, and then its effects are discussed.
8.It can be inferred from the passage that factors making women uniquely valuable members of

modern corporate boards would include which of the following?
I. The nature of modern corporations
Lithe increased number of women CEO’s
III.The careers pursued by women currently available to serve on corporate boards
(A) I only
(B) II only
(C) III only
(D) I and III only
(E) I, II, and III
Passage 32 (32/63)
Increasingly, historians are blaming diseases imported from the Old World for the staggering

disparity between the indigenous population of America in 1492—new estimates of which soar

as high as 100 million, or approximately one-sixth of the human race at that —and the few

million full-blooded Native Americans alive at the end of the nineteenth century. There is

no doubt that chronic disease was an important factor in the precipitous decline, and it is

highly probable that the greatest killer was epidemic disease, especially as manifested in

virgin-soil epidemics.
Virgin-soil epidemics are those in which the populations at risk have had no previous

contact with the diseases that strike them and are therefore immunologically almost

defenseless. That virgin-soil epidemics were important in American history is strongly

indicated by evidence that a number of dangerous maladies—smallpox, measles, malaria, yellow

fever, and undoubtedly several more—were unknown in the pre-Columbian New World. The effects

of their sudden introduction are demonstrated in the early chronicles of America, which

contain reports of horrendous epidemics and steep population declines, confirmed in many

cases by recent quantitative analyses of Spanish tribute records and other sources. The

evidence provided by the documents of British and French colonies is not as definitive

because the conquerors of those areas did not establish permanent settlements and begin to

keep continuous records until the seventeenth century, by which the worst epidemics had

probably already taken place. Furthermore, the British tended to drive the native

populations away, rather than enslaving them as the Spaniards did, so that the epidemics of

British America occurred beyond the range of colonists’ direct observation.
Even so, the surviving records of North America do contain references to deadly epidemics

among the indigenous population. In 1616-1619 an epidemic, possibly of bubonic or pneumonic

plague, swept coastal New England, killing as many as nine out of ten. During the 1630’s

smallpox, the disease most fatal to the Native American people, eliminated half the

population of the Huron and Iroquois confederations. In the 1820’s fever devastated the

people of the Columbia River area, killing eight out of ten of them.
Unfortunately, the documentation of these and other epidemics is slight and frequently

unreliable, and it is necessary to supplement what little we do know with evidence from

recent epidemics among Native Americans. For example, in 1952 an outbreak of measles among

the Native American inhabitants of Ungava Bay, Quebec, affected 99 percent of the population

and killed 7 percent, even though some had the benefit of modern medicine. Cases such as

this demonstrate that even diseases that are not normally fatal can have devastating

consequences when they strike an immunologically defenseless community.
1.The primary purpose of the passage is to
(A) refute a common misconception
(B) provide support for a hypothesis
(C) analyze an argument
(D) suggest a solution to a dilemma
(E) reconcile opposing viewpoints
2.According to the passage, virgin-soil epidemics can be distinguished from other

catastrophic outbreaks of disease in that virgin-soil epidemics
(A) recur more frequently than other chronic diseases
(B) affect a minimum of one-half of a given population
(C) involve populations with no prior exposure to a disease
(D) usually involve a number of interacting diseases
(E) are less responsive to medical treatment than are other diseases
3.According to the passage, the British colonists were unlike the Spanish colonists in that

the British colonists
(A) collected tribute from the native population
(B) kept records from a very early date
(C) drove Native Americans off the land
(D) were unable to provide medical care against epidemic disease
(E) enslaved the native populations in America
4.Which of the following can be inferred from the passage concerning Spanish tribute

records?
(A) They mention only epidemics of smallpox.
(B) They were instituted in 1492.
(C) They were being kept prior to the seventeenth century.
(D) They provide quantitative and qualitative evidence about Native American populations.
(E) They prove that certain diseases were unknown in the pre-Columbian New World.
5.The author implies which of the following about measles?
(A) It is not usually a fatal disease.
(B) It ceased to be a problem by the seventeenth century.
(C) It is the disease most commonly involved in virgin-soil epidemics.
(D) It was not a significant problem in Spanish colonies.
(E) It affects only those who are immunologically defenseless against it.
6.Which of the following can be inferred from the passage about the Native American

inhabitants of Ungava Bay?
(A) They were almost all killed by the 1952 epidemic.
(B) They were immunologically defenseless against measles.
(C) They were the last native people to be struck by a virgin-soil epidemic.
(D) They did not come into frequent contact with white Americans until the twentieth

century.
(E) They had been inoculated against measles.
7.The author mentions the 1952 measles outbreak most probably in order to
(A) demonstrate the impact of modern medicine on epidemic disease
(B) corroborate the documentary evidence of epidemic disease in colonial America
(C) refute allegations of unreliability made against the historical record of colonial

America
(D) advocate new research into the continuing problem of epidemic disease
(E) challenge assumptions about how the statistical evidence of epidemics should be

interpreted
8.Which of the following, if newly discovered, would most seriously weaken the author’s

argument concerning the importance of virgin-soil epidemics in the depopulation of Native

Americans?
(A) Evidence setting the pre-Columbian population of the New World at only 80 million
(B) Spanish tribute records showing periodic population fluctuations
(C) Documents detailing sophisticated Native American medical procedures
(D) Fossils indicating Native American contact with smallpox prior to 1492
(E) Remains of French settlements dating back to the sixteenth century
Passage 33 (33/63)
Until recently most astronomers believed that the space between the galaxies in our universe

was a near-perfect vacuum. This orthodox view of the universe is now being challenged by

astronomers who believe that a heavy “rain” of gas is falling into many galaxies from the

supposedly empty space around them. The gas apparently condenses into a collection of small

stars, each a little larger than the planet Jupiter. These stars vastly outnumber the other

stars in a given galaxy. The amount of “intergalactic rainfall” into some of these galaxies

has been enough to double their mass in the since they formed. Scientists have begun to

suspect that this intergalactic gas is probably a mixture of gases left over from the “big

bang” when the galaxies were formed and gas was forced out of galaxies by supernova

explosions.
It is well known that when gas is cooled at a constant pressure its volume decreases. Thus,

the physicist Fabian reasoned that as intergalactic gas cools, the cooler gas shrinks inward

toward the center of the galaxy. Meanwhile its place is taken by hotter intergalactic gas

from farther out on the edge of the galaxy, which cools as it is compressed and flows into

the galaxy. The net result is a continuous flow of gas, starting as hot gases in

intergalactic space and ending as a drizzle of cool gas called a “cooling flow,” falling

into the central galaxy.
A fairly heretical idea in the 1970’s, the cooling-flow theory gained support when Fabian

observed a cluster of galaxies in the constellation Perseus and found the central galaxy,

NGC 1275, to be a strange-looking object with irregular, thin strands of gas radiating from

it. According to previous speculation, these strands were gases that had been blown out by

an explosion in the galaxy. Fabian, however, disagreed. Because the strands of gas radiating

from NGC 1275 are visible in optical photographs, Fabian suggested that such strands

consisted not of gas blown out of the galaxy but of cooling flows of gas streaming inward.

He noted that the wavelengths of the radiation emitted by a gas would changes as the gas

cooled, so that as the gas flowed into the galaxy and became cooler, it would emit not

x-rays, but visible light, like that which was captured in the photographs. Fabian’s

hypothesis was supported by Canizares’ determination in 1982 that most of the gas in the

Perseus cluster was at a temperature of 80 million degrees Kelvin, whereas the gas

immediately surrounding NGC 1275 (the subject of the photographs) was at one-tenth this

temperature.
1.The primary purpose of the passage is to
(A) illustrate a hypothesis about the origin of galaxies
(B) provide evidence to dispute an accepted theory about the evolution of galaxies
(C) summarize the state of and prospects for research in intergalactic astronomy
(D) report new data on the origins of intergalactic gas
(E) reconcile opposing views on the formation of intergalactic gas
2.The author uses the phrase “orthodox view of the universe” (line 3) to refer to the belief

that
(A) the space between the galaxies is devoid of matter
(B) the space between galaxies is occupied by stars that cannot be detected by optical

photographs
(C) galaxies have decreased in mass by half since their formation
(D) galaxies contain stars, each the size of Jupiter, which form clusters
(E) galaxies are being penetrated by gas forced out of other galaxies by supernova

explosions
3.It can be inferred from the passage that, if Fabian is correct, gas in the peripheral

regions of a galaxy cluster
(A) streams outward into intergalactic space
(B) is hotter than gas in the central regions of the galaxy
(C) is composed primarily of gas left over from the big bang
(D) results in the creation of unusually large stars
(E) expands to increase the size of the galaxy
4.The author of the passage probably mentions Canizares’ determination in order to
(A) clarify an ambiguity in Fabian’s research findings
(B) illustrate a generalization about the temperature of gas in a galaxy cluster
(C) introduce a new argument in support of the orthodox view of galaxies
(D) provide support for Fabian’s assertions about the Perseus galaxies
(E) provide an alternate point of view concerning the movement of gas within a galaxy

cluster
5.According to the passage, Fabian believes that gas flowing into a central galaxy has which

of the following characteristics?
(A) It is one-tenth hotter than it was in the outer regions of the galaxy cluster.
(B) It emits radiation with wavelengths that change as the gas moves toward the center of

the galaxy.
(C) The total amount of radiation emitted diminishes as the gas cools.
(D) It loses 90 percent of its energy as it moves to the center of the galaxy.
(E) It condenses at a rate much slower than the rate of decrease in temperature as the gas

flows inward.
6.According to the passage, Fabian’s theory makes use of which of the following principles?
(A) Gas emanating from an explosion will be hotter the more distant it is from the origin.
(B) The wavelength of radiation emitted by a gas as it cools remains constant.
(C) If pressure remains constant, the volume of a gas will decrease as it is cooled.
(D) The volume of a gas will increase as the pressure increases.
(E) As gas cools, its density decreases.
7.It can be inferred from the passage that which of the following is true of Fabian’s

theory?
(A) It did not receive approval until Canizares’ work was published.
(B) It was not widely accepted in the 1970’s.
(C) It did not receive support initially because technology was not available to confirm its

tenets.
(D) It supports earlier speculation that intergalactic gas was largely the result of

explosions outside the galaxy.
(E) It was widely challenged until x-ray evidence of gas temperatures in NGC 1275 had been

presented.
Passage 34 (34/63)
Kazuko Nakane’s history of the early Japanese immigrants to central California’s Pajaro

Valley focuses on the development of farming communities there from 1890 to 1940. The Issei

(first-generation immigrants) were brought into the Pajaro Valley to raise sugar beets. Like

Issei laborers in American cities, Japanese men in rural areas sought employment via the

“boss” system. The system comprised three elements: immigrant wage laborers; Issei

boardinghouses where laborers stayed; and labor contractors, who gathered workers for a

particular job and then negotiated a contract between workers and employer. This same system

was originally utilized by the Chinese laborers who had preceded the Japanese. A related

institution was the “labor club,” which provided job information and negotiated employment

contracts and other legal matters, such as the rental of land, for Issei who chose to belong

and paid an annual fee to the cooperative for membership.
When the local sugar beet industry collapsed in 1902, the Issei began to lease land from the

valley’s strawberry farmers. The Japanese provided the labor and the crop was divided

between laborers and landowners. The Issei thus moved quickly from wage-labor employment to

sharecropping agreements. A limited amount of economic progress was made as some Issei were

able to rent or buy farmland directly, while others joined together to form farming

corporations. As the Issei began to operate farms, they began to marry and start families,

forming an established Japanese American community. Unfortunately, the Issei’s efforts to

attain agricultural independence were hampered by government restrictions, such as the Alien

Land Law of 1913. But immigrants could circumvent such exclusionary laws by leasing or

purchasing land in their American-born children’s names.
Nakane’s case study of one rural Japanese American community provides valuable information

about the lives and experiences of the Issei. It is, however, too particularistic. This

limitation derives from Nakane’s methodology—that of oral history—which cannot substitute

for a broader theoretical or comparative perspective. Future research might well consider

two issues raised by her study: were the Issei of the Pajaro Valley similar to or different

from Issei in urban settings, and what variations existed between rural Japanese American

communities?
1.The primary purpose of the passage is to
(A) defend a controversial hypothesis presented in a history of early Japanese immigrants to

California
(B) dismiss a history of an early Japanese settlement in California as narrow and ill

constructed
(C) summarize and critique a history of an early Japanese settlement in California
(D) compare a history of one Japanese American community with studies of Japanese

settlements throughout California
(E) examine the differences between Japanese and Chinese immigrants to central California in

the 1890’s
2.Which of the following best describes a “labor club,” as defined in the passage?
(A) An organization to which Issei were compelled to belong if they sought employment in the

Pajaro Valley
(B) An association whose members included labor contractors and landowning “bosses”
(C) A type of farming corporation set up by Issei who had resided in the Pajaro Valley for

some
(D) A cooperative association whose members were dues-paying Japanese laborers
(E) A social organization to which Japanese laborers and their families belonged
3.Based on information in the passage, which of the following statements concerning the

Alien Land Law of 1913 is most accurate?
(A) It excluded American-born citizens of Japanese ancestry from landownership.
(B) It sought to restrict the number of foreign immigrants to California.
(C) It successfully prevented Issei from ever purchasing farmland.
(D) It was applicable to first-generation immigrants but not to their American-born

children.
(E) It was passed under pressure from the Pajaro Valley’s strawberry farmers.
4.Several Issei families join together to purchase a strawberry field and the necessary

farming equipment. Such a situation best exemplifies which of the following, as it is

described in the passage?
(A) A typical sharecropping agreement
(B) A farming corporation
(C) A “labor club”
(D) The “boss” system
(E) Circumvention of the Alien Land Law
5.The passage suggests that which of the following was an indirect consequence of the

collapse of the sugar beet industry in the Pajaro Valley?
(A) The Issei formed a permanent, family-based community.
(B) Boardinghouses were built to accommodate the Issei.
(C) The Issei began to lease land in their children’s names.
(D) The Issei adopted a labor contract system similar to that used by Chinese immigrants.
(E) The Issei suffered a massive dislocation caused by unemployment.
6.The author of the passage would most likely agree that which of the following, if it had

been included in Nakane’s study, would best remedy the particularistic nature of that study?
(A) A statistical table comparing per capita income of Issei wage laborers and sharecroppers

in the Pajaro Valley
(B) A statistical table showing per capita income of Issei in the Pajaro Valley from 1890 to

1940
(C) A statistical table showing rates of farm ownership by Japanese Americans in four

central California counties from 1890 to 1940
(D) A discussion of original company documents dealing with the Pajaro Valley sugar beet

industry at the turn of the century
(E) Transcripts of interviews conducted with members of the Pajaro Valley Japanese American

community who were born in the 1920’s and 1930’s
7.It can be inferred from the passage that, when the Issei began to lease land from the

Valley’s strawberry farmers, the Issei most probably did which of the following?
(A) They used profits made from selling the strawberry crop to hire other Issei.
(B) They negotiated such agricultural contracts using the “boss” system.
(C) They paid for the use of the land with a share of the strawberry crop.
(D) They earned higher wages than when they raised sugar beets.
(E) They violated the Alien Land Law.
Passage 35 (35/63)
It can be argued that much consumer dissatisfaction with marketing strategies arises from an

inability to aim advertising at only the likely buyers of a given product.
There are three groups of consumers who are affected by the marketing process. First, there

is the market segment—people who need the commodity in question. Second, there is the

program target—people in the market segment with the “best fit” characteristics for a

specific product. Lots of people may need trousers, but only a few qualify as likely buyers

of very expensive designer trousers. Finally, there is the program audience?all people who

are actually exposed to the marketing program without regard to whether they need or want

the product.
These three groups are rarely identical. An exception occurs occasionally in cases where

customers for a particular industrial product may be few and easily identifiable. Such

customers, all sharing a particular need, are likely to form a meaningful target, for

example, all companies with a particular application of the product in question, such as

high-speed fillers of bottles at breweries. In such circumstances, direct selling (marketing

that reaches only the program target) is likely to be economically justified, and highly

specialized trade media exist to expose members of the program target—and only members of

the program target—to the marketing program.
Most consumer-goods markets are significantly different. Typically, there are many rather

than few potential customers. Each represents a relatively small percentage of potential

sales. Rarely do members of a particular market segment group themselves neatly into a

meaningful program target. There are substantial differences among consumers with similar

demographic characteristics. Even with all the past decade’s advances in information

technology, direct selling of consumer goods is rare, and mass marketing—a marketing

approach that aims at a wide audience—remains the only economically feasible mode.

Unfortunately, there are few media that allow the marketer to direct a marketing program

exclusively to the program target. Inevitably, people get exposed to a great deal of

marketing for products in which they have no interest and so they become annoyed.
1.The passage suggests which of the following about highly specialized trade media?
(A) They should be used only when direct selling is not economically feasible.
(B) They can be used to exclude from the program audience people who are not part of the

program target.
(C) They are used only for very expensive products.
(D) They are rarely used in the implementation of marketing programs for industrial

products.
(E) They are used only when direct selling has not reached the appropriate market segment.
2.According to the passage, most consumer-goods markets share which of the following

characteristics?
I.Customers who differ significantly from each other
II.Large numbers of potential customers
III.Customers who each represent a small percentage of potential sales
(A) I only
(B) II only
(C) I and II only
(D) II and III only
(E) I, II, and III
3.The passage suggests which of the following about direct selling?
(A) It is used in the marketing of most industrial products.
(B) It is often used in cases where there is a large program target.
(C) It is not economically feasible for most marketing programs.
(D) It is used only for products for which there are many potential customers.
(E) It is less successful at directing a marketing program to the target audience than are

other marketing approaches.
4.The author mentions “trousers” (lines 9 and 11) most likely in order to
(A) make a comparison between the program target and the program audience
(B) emphasize the similarities between the market segment and the program target
(C) provide an example of the way three groups of consumers are affected by a marketing

program
(D) clarify the distinction between the market segment and the program target
(E) introduce the concept of the program audience
5.Which of the following best exemplifies the situation described in the last two sentences

of the passage?
(A) A product suitable for women age 21-30 is marketed at meetings attended only by

potential customers.
(B) A company develops a new product and must develop an advertising campaign to create a

market for it.
(C) An idea for a specialized product remains unexplored because media exposure of the

product to its few potential customers would be too expensive.
(D) A new product is developed and marketers collect demographic data on potential consumers

before developing a specific advertising campaign.
(E) A product suitable for men age 60 and over is advertised in a magazine read by adults of

all ages.
6.The passage suggests that which of the following is true about the marketing of industrial

products like those discussed in the third paragraph?
(A) The market segment and program target are identical.
(B) Mass marketing is the only feasible way of advertising such products.
(C) The marketing program cannot be directed specifically to the program target.
(D) More customers would be needed to justify the expense of direct selling.
(E) The program audience would necessarily be made up of potential customers, regardless of

the marketing approach that was used.
7.The passage supports which of the following statements about demographic characteristics

and marketing?
(A) Demographic research is of no use in determining how successful a product will be with a

particular group of consumers.
(B) A program audience is usually composed of people with similar demographic

characteristics.
(C) Psychological factors are more important than demographic factors in defining a market

segments.
(D) Consumers with similar demographic characteristics do not necessarily form a meaningful

program target.
(E) Collecting demographic data is the first step that marketers take in designing a

marketing program.
8.It can be inferred from the passage that which of the following is true for most

consumer-goods markets?
(A) The program audience is smaller than the market segment.
(B) The program audience and the market segment are usually identical.
(C) The market segment and the program target are usually identical.
(D) The program target is larger than the market segment.
(E) The program target and the program audience are not usually identical.
Passage 36 (36/63)
Protein synthesis begins when the gene encoding a protein is activated. The gene’s sequence

of nucleotides is transcribed into a molecule of messenger RNA (mRNA), which reproduces the

information contained in that sequence. Transported outside the nucleus to the cytoplasm,

the mRNA is translated into the protein it encodes by an organelle known as a ribosome,

which strings together amino acids in the order specified by the sequence of elements in the

mRNA molecule. Since the amount of mRNA in a cell determines the amount of the corresponding

protein, factors affecting the abundance of mRNA’s play a major part in the normal

functioning of a cell by appropriately regulating protein synthesis. For example, an excess

of certain proteins can cause cells to proliferate abnormally and become cancerous; a lack

of the protein insulin results in diabetes.
Biologists once assumed that the variable rates at which cells synthesize different mRNA’s

determine the quantities of mRNA’s and their corresponding proteins in a cell. However,

recent investigations have shown that the concentrations of most mRNA’s correlate best, not

with their synthesis rate, but rather with the equally variable rates at which cells degrade

the different mRNA’s in their cytoplasm. If a cell degrades both a rapidly and a slowly

synthesized mRNA slowly, both mRNA’s will accumulate to high levels.
An important example of this phenomenon is the development of red blood cells from their

unspecialized parent cells in bone marrow. For red blood cells to accumulate sufficient

concentrations of hemoglobin (which transports oxygen) to carry out their main function, the

cells’ parent cells must simultaneously produce more of the constituent proteins of

hemoglobin and less of most other proteins. To do this, the parent cells halt synthesis of

non-hemoglobin mRNA’s in the nucleus and rapidly degrade copies of the non-hemoglobin mRNA’s

remaining in the cytoplasm. Halting synthesis of mRNA alone would not affect the quantities

of proteins synthesized by the mRNA’s still existing in the cytoplasm. Biologists now

believe that most cells can regulate protein production most efficiently by varying both

mRNA synthesis and degradation, as developing red cells do, rather than by just varying one

or the other.
1.The passage is primarily concerned with discussing the
(A) influence of mRNA concentrations on the development of red blood cells
(B) role of the synthesis and degradation of mRNA in cell functioning
(C) mechanism by which genes are transcribed into mRNA
(D) differences in mRNA concentrations in cell nuclei and in the cytoplasm
(E) way in which mRNA synthesis contributes to the onset of diabetes
2.The passage suggests that a biologist who held the view described in the first sentence of

the second paragraph would most probably also have believed which of the following?
(A) The rate of degradation of specific mRNA’s has little effect on protein concentrations.
(B) The rate of degradation of specific mRNA’s should be studied intensively.
(C) The rates of synthesis and degradation for any given mRNA are normally equal.
(D) Different mRNA’s undergo degradation at widely varying rates.
(E) Most mRNA’s degrade very rapidly.
3.Which of the following best describes the relationship between the second and third

paragraphs of the passage?
(A) The second paragraph presents arguments in support of a new theory and the third

paragraph presents arguments against that same theory.
(B) The second paragraph describes a traditional view and the third paragraph describes the

view that has replaced it on the basis of recent investigations.
(C) The third paragraph describes a specific case of a phenomenon that is described

generally in the second paragraph.
(D) The third paragraph describes an investigation that was undertaken to resolve problems

raised by phenomena described in the second paragraph.
(E) Both paragraphs describe in detail specific examples of the phenomenon that is

introduced in the first paragraph.
4.The accumulation of concentrations of hemoglobin in red blood cells is mentioned in the

passage as an example of which of the following?
(A) The effectiveness of simultaneous variation of the rates of synthesis and degradation of

mRNA
(B) The role of the ribosome in enabling a parent cell to develop properly into a more

specialized form
(C) The importance of activating the genes for particular proteins at the correct moment
(D) The abnormal proliferation of a protein that threatens to make the cell cancerous
(E) The kind of evidence that biologists relied on for support of a view of mRNA synthesis

that is now considered obsolete
5.To begin to control a disease caused by a protein deficiency, the passage suggests that a

promising experimental treatment would be to administer a drug that would reduce
(A) only the degradation rate for the mRNA of the protein involved
(B) only the synthesis rate for the mRNA of the protein involved
(C) both the synthesis and degradation rates for the mRNA of the protein involved
(D) the incidence of errors in the transcription of mRNA’s from genetic nucleotide sequences
(E) the rate of activity of ribosomes in the cytoplasm of most cells
6.According to the passage, which of the following best describes the current view on the

relationship between the synthesis and the degradation of mRNA in regulating protein

synthesis?
(A) Biologists have recently become convinced that the ribosome controls the rates of

synthesis and degradation of mRNA.
(B) There is no consensus among biologists as to the significance of mRNA degradation in

regulating protein synthesis.
(C) The concept of mRNA degradation is so new that most biologists still believe that the

vital role in protein regulation belongs to mRNA synthesis.
(D) Degradation of mRNA is now considered to be the key process and mRNA synthesis is no

longer believed to play a significant role.
(E) Degradation of mRNA is now considered to be as important as mRNA synthesis has been, and

still is, believed to be.
7.According to the passage, which of the following can happen when protein synthesis is not

appropriately regulated?
(A) Diabetes can result from errors that occur when the ribosomes translate mRNA into

protein.
(B) Cancer can result from an excess of certain proteins and diabetes can result from an

insulin deficiency.
(C) A deficiency of red blood cells can occur if bone marrow cells produce too much

hemoglobin.
(D) Cancer can be caused by excessively rapid degradation of certain amino acids in the

cytoplasm of cells.
(E) Excessive synthesis of one protein can trigger increased degradation of mRNA’s for other

proteins and create severe protein imbalances.
8.The passage suggests that a biologist who detected high levels of two proteins in a

certain type of cell would be likely to consider which of the following as a possible

explanation?
(A) The rate of mRNA degradation for one of the proteins increases as this type of cell

develops a more specialized function.
(B) The two proteins are most likely constituents of a complex substance supporting the

cells’ specialized function.
(C) The cells are likely to proliferate abnormally and possibly become cancerous due to the

levels of these proteins.
(D) The mRNA’s for both proteins are being degraded at a low rate in that type of cell.
(E) The mRNA’s for the two proteins are being synthesized at identical rates in that type of

cell.
Passage 37 (37/63)
(This passage was excerpted from material written in 1990.)
As the economic role of multinational, global corporations expands, the international

economic environment will be shaped increasingly not by governments or international

institutions, but by the interaction between governments and global corporations, especially

in the United States, Europe, and Japan. A significant factor in this shifting world economy

is the trend toward regional trading blocs of nations, which has a potentially large effect

on the evolution of the world trading system. Two examples of this trend are the United

States-Canada Free Trade Agreement (FTA) and Europe 1992, the move by the European Community

(EC) to dismantle impediments to the free flow of goods, services, capital, and labor among

member states by the end of 1992. However, although numerous political and economic factors

were operative in launching the move to integrate the EC’s markets, concern about

protectionism within the EC does not appear to have been a major consideration. This is in

sharp contrast to the FTA; the overwhelming reason for that bilateral initiative was fear of

increasing United States protectionism. Nonetheless, although markedly different in origin

and nature, both regional developments are highly significant in that they will foster

integration in the two largest and richest markets of the world, as well as provoke about

the future direction of the world trading system.
1.The primary purpose of the passage as a whole is to
(A) describe an initiative and propose its continuance
(B) chronicle a development and illustrate its inconsistencies
(C) identify a trend and suggest its importance
(D) summarize a process and question its significance
(E) report a phenomenon and outline its probable future
2.According to the passage, all of the following are elements of the shifting world economy

EXCEPT
(A) an alteration in the role played by governments
(B) an increase in interaction between national governments and international regulatory

institutions
(C) an increase in the formation of multinational trading alliances
(D) an increase in integration in the two richest markets of the world
(E) a fear of increasing United States protectionism
3.The passage suggests which of the following about global corporations?
(A) Their continued growth depends on the existence of a fully integrated international

market.
(B) Their potential effect on the world market is a matter of ongoing concern to

international institutions.
(C) They will have to assume quasi-governmental functions if current economic trends

continue.
(D) They have provided a model of economic success for regional trading blocs.
(E) Their influence on world economics will continue to increase.
4.According to the passage, one similarity between the FTA and Europe 1992 is that they both
(A) overcame concerns about the role of politics in the shifting world economy
(B) originated out of concern over unfair trade practices by other nations
(C) exemplify a trend toward regionalization of commercial markets
(D) place the economic needs of the trading bloc ahead of those of the member nations
(E) help to ensure the continued economic viability of the world community
5.Which of the following can be inferred from the passage about the European Community prior

to the adoption of the Europe 1992 program?
(A) There were restrictions on commerce between the member nations.
(B) The economic policies of the member nations focused on global trading issues.
(C) There were few impediments to trade between the member nations and the United States.
(D) The flow of goods between the member nations and Canada was insignificant.
(E) Relations between multinational corporations and the governments of the member nations

were strained.
6.The author discusses the FTA and Europe 1992 most likely in order to
(A) point out the similarities between two seemingly disparate trading alliances
(B) illustrate how different economic motivations produce different types of trading blocs
(C) provide contrasting examples of a trend that is influencing the world economy
(D) identify the most important characteristics of successful economic integration
(E) trace the history of regional trading blocs
7.Which of the following best describes the organization of the passage?
(A) An argument is put forth and evidence for and against it given.
(B) An assertion is made and opposing evidence presented.
(C) Two hypotheses are described and shown to inconsistent with one another.
(D) A phenomenon is identified and illustrations of this phenomenon offered.
(E) A specific case of a phenomenon is discussed a generalization drawn.
Passage 38 (38/63)
In Forces of Production, David Noble examines the transformation of the machine-tool

industry as the industry moved from reliance on skilled artisans to automation. Noble writes

from a Marxist perspective, and his central argument is that management, in its decisions to

automate, conspired against labor: the power that the skilled machinists wielded in the

industry was intolerable to management. Noble fails to substantiate this claim, although his

argument is impressive when he applies the Marxist concept of “de-skilling”—the use of

technology to replace skilled labor—to the automation of the machine-tool industry. In

automating, the industry moved to computer-based, digitized “numerical-control” (N/C)

technology, rather than to artisan-generated “record-playback” (R/P) technology.
Although both systems reduced reliance on skilled labor, Noble clearly prefers R/P, with its

inherent acknowledgment of workers’ skills: unlike N/C, its programs were produced not by

engineers at their computers, but by skilled machinists, who recorded their own movements to

“teach” machines to duplicate those movements. However, Noble’s only evidence of conspiracy

is that, although the two approaches were roughly equal in technical merit, management chose

N/C. From this he concludes that automation is undertaken not because efficiency demands it

or scientific advances allow it, but because it is a tool in the ceaseless war of

capitalists against labor.
1.The author of the passage is primarily concerned with
(A) reexamining a political position and defending its validity
(B) examining a management decision and defending its necessity
(C) analyzing a scholarly study and pointing out a central weakness
(D) explaining a trend in automation and warning about its dangers
(E) chronicling the history of an industry and criticizing its development
2.According to information in the passage, the term “de-skilling” refers to the
(A) loss of skills to industry when skilled workers are replaced by unskilled laborers
(B) substitution of mechanized processes for labor formerly performed by skilled workers
(C) labor theory that automation is technologically comparable to skilled labor
(D) process by which skilled machinists “teach” machines to perform certain tasks
(E) exclusion of skilled workers from participation in the development of automated

technology
3.Which of the following best characterizes the function of the second paragraph of the

passage?
(A) It develops a topic introduced in the first paragraph.
(B) It provides evidence to refute a claim presented in the first paragraph.
(C) It gives examples of a phenomenon mentioned in the first paragraph.
(D) It presents a generalization about examples given in the first paragraph.
(E) It suggests two possible solutions to a problem presented in the first paragraph.
4.The passage suggests which of the following about N/C automation in the machine-tool

industry?
(A) It displaced fewer skilled workers than R/P automation did.
(B) It could have been implemented either by experienced machinists or by computer

engineers.
(C) It was designed without the active involvement skilled machinists.
(D) It was more difficult to design than R/P automation was.
(E) It was technically superior to R/P automation.
5.Which of the following phrases most clearly reveals the attitude of the author of the

passage toward Noble’s central argument?
(A) “conspired against” (line 6)
(B) “intolerable to management” (line 7)
(C) “impressive when he applies the Marxist concept” (line 9)
(D) “clearly prefers” (line 16)
(E) “only evidence of conspiracy” (line 21)
6.The author of the passage commends Noble’s book for which of the following?
(A) Concentrating on skilled as opposed to unskilled workers in its discussion of the

machine-tool industry
(B) Offering a generalization about the motives behind the machine-tool industry’s decision

to automate
(C) Making an essential distinction between two kinds of technology employed in the

machine-tool industry
(D) Calling into question the notion that managers conspired against labor in the automation

of the machine-tool industry
(E) Applying the concept of de-skilling to the machine tool industry
7.Which of the following best characterizes Forces of Production as it is described in the

passage?
(A) A comparison of two interpretations of how a particular industry evolved
(B) An examination of the origin of a particular concept in industrial economics
(C) A study that points out the weakness of a particular interpretation of an industrial

phenomenon
(D) A history of a particular industry from an ideological point of view
(E) An attempt to relate an industrial phenomenon in one industry to a similar phenomenon in

another industry
Passage 39 (39/63)
The sensation of pain cannot accurately be described as “located” at the point of an injury,

or, for that matter, in any one place in the nerves or brain. Rather, pain signals—and pain

relief—are delivered through a highly complex interacting circuitry.
When a cell is injured, a rush of prostaglandins sensitizes nerve endings at the injury.

Prostaglandins are chemicals produced in and released from virtually all mammalian cells

when they are injured: these are the only pain signals that do not originate in the nervous

system. Aspirin and other similar drugs (such as indomethacin and ibuprofen) keep

prostaglandins from being made by interfering with an enzyme known as prostaglandin

synthetase, or cyclooxygenase. The drugs’ effectiveness against pain is proportional to

their success in blocking this enzyme at the site of injury.
From nerve endings at the injury, pain signals move to nerves feeding into the spinal cord.

The long, tubular membranes of nerve cells carry electrical impulses. When electrical

impulses get to the spinal cord, a pain-signaling chemical known as substance P is released

there. Substance P then excites nearby neurons to send impulses to the brain. Local

anesthetics such as novocaine and xylocaine work by blocking the electrical transmission

along nerves in a particular area. They inhibit the flow of sodium ions through the

membranes, making the nerves electrically quiescent; thus no pain signals are sent to the

spinal cord or to the brain.
Recent discoveries in the study of pain have involved the brain itself—the supervising organ

that notices pain signals and that sends messages down to the spinal cord to regulate

incoming pain traffic. Endorphins—the brain’s own morphine—are a class of small peptides

that help to block pain signals within the brain itself. The presence of endorphins may also

help to explain differences in response to pain signals, since individuals seem to differ in

their ability to produce endorphins. It now appears that a number of techniques for blocking

chronic pain—such as acupuncture and electrical stimulation of the central brain

stem—involve the release of endorphins in the brain and spinal cord.
1.The passage is primarily concerned with
(A) analyzing ways that enzymes and other chemicals influence how the body feels pain
(B) describing the presence of endorphins in the brain and discussing ways the body blocks

pain within the brain itself
(C) describing how pain signals are conveyed in the body and discussing ways in which the

pain signals can be blocked
(D) demonstrating that pain can be influenced by acupuncture and electrical stimulation of

the central brain stem
(E) differentiating the kinds of pain that occur at different points in the body’s nervous

system
2.According to the passage, which of the following is one of the first things to occur when

cells are injured?
(A) The flow of electrical impulses through nerve cells at the site of the injury is broken.
(B) The production of substance P traveling through nerve cells to the brain increases.
(C) Endorphins begin to speed up the response of nerve cells at the site of the injury.
(D) A flood of prostaglandins sensitizes nerve endings at the site of the injury.
(E) Nerve cells connected to the spinal cord become electrically quiescent.
3.Of the following, which is most likely attributable to the effect of endorphins as

described in the passage?
(A) After an injection of novocaine, a patient has no feeling in the area where the

injection was given.
(B) After taking ibuprofen, a person with a headache gets quick relief.
(C) After receiving a local anesthetic, an injured person reports relief in the anesthetized

area.
(D) After being given aspirin, a child with a badly scraped elbow feels better.
(E) After acupuncture, a patient with chronic back pain reports that the pain is much less

severe.
4.It can be inferred from the passage that if the prostaglandin synthetase is only partially

blocked, which of the following is likely to be true?
(A) Some endorphins will be produced, and some pain signals will be intensified.
(B) Some substance P is likely to be produced, so some pain signals will reach the brain.
(C) Some sodium ions will be blocked, so some pain signals will not reach the brain.
(D) Some prostaglandins will be produced, but production of substance P will be prevented.
(E) Some peptides in the brain will receive pain signals and begin to regulate incoming pain

traffic.
Passage 40 (40/63)
Japanese firms have achieved the highest levels of manufacturing efficiency in the world

automobile industry. Some observers of Japan have assumed that Japanese firms use the same

manufacturing equipment and techniques as United States firms but have benefited from the

unique characteristics of Japanese employees and the Japanese culture. However, if this were

true, then one would expect Japanese auto plants in the United States to perform no better

than factories run by United States companies. This is not the case; Japanese-run automobile

plants located in the United States and staffed by local workers have demonstrated higher

levels of productivity when compared with factories owned by United States companies.
Other observers link high Japanese productivity to higher levels of capital investment per

worker. But a historical perspective leads to a different conclusion. When the two top

Japanese automobile makers matched and then doubled United States productivity levels in the

mid-sixties, capital investment per employee was comparable to that of United States firms.

Furthermore, by the late seventies, the amount of fixed assets required to produce one

vehicle was roughly equivalent in Japan and in the United States.
Since capital investment was not higher in Japan, it had to be other factors that led to

higher productivity. A more fruitful explanation may lie with Japanese production

techniques. Japanese automobile producers did not simply implement conventional processes

more effectively: they made critical changes in United States procedures. For instance, the

mass-production philosophy of United States automakers encouraged the production of huge

lots of cars in order to utilize fully expensive, component-specific equipment and to occupy

fully workers who have been trained to execute one operation efficiently. Japanese

automakers chose to make small-lot production feasible by introducing several departures

from United States practices, including the use of flexible equipment that could be altered

easily to do several different production tasks and the training of workers in multiple

jobs. Automakers could schedule the production of different components or models on single

machines, thereby eliminating the need to store the buffer stocks of extra components that

result when specialized equipment and workers are kept constantly active.
1.The primary purpose of the passage is to
(A) present the major steps of a process
(B) clarify an ambiguity
(C) chronicle a dispute
(D) correct misconceptions
(E) defend an accepted approach
2.The author suggests that if the observers of Japan mentioned in line 3 were correct, which

of the following would be the case?
(A) The equipment used in Japanese automobile plants would be different from the equipment

used in United States plants.
(B) Japanese workers would be trained to do several different production jobs.
(C) Culture would not have an influence on the productivity levels of workers.
(D) The workers in Japanese-run plants would have higher productivity levels regardless of

where they were located.
(E) The production levels of Japanese-run plants located in the United States would be equal

to those of plants run by United States companies.
3.Which of the following statements concerning the productivity levels of automakers can be

inferred from the passage?
(A) Prior to the 1960’s, the productivity levels of the top Japanese automakers were

exceeded by those of United States automakers.
(B) The culture of a country has a large effect on the productivity levels of its

automakers.
(C) During the late 1970’s and early 1980’s, productivity levels were comparable in Japan

and the United States.
(D) The greater the number of cars that are produced in a single lot, the higher a plant’s

productivity level.
(E) The amount of capital investment made by automobile manufacturers in their factories

determines the level of productivity.
4.According to the passage, which of the following statements is true of Japanese automobile

workers?
(A) Their productivity levels did not equal those of United States automobile workers until

the late seventies.
(B) Their high efficiency levels are a direct result of cultural influences.
(C) They operate component-specific machinery.
(D) They are trained to do more than one job.
(E) They produce larger lots of cars than do workers in United States factories.
5.Which of the following best describes the organization of the first paragraph?
(A) A thesis is presented and supporting examples are provided.
(B) Opposing views are presented, classified, and then reconciled.
(C) A fact is stated, and an explanation is advanced and then refuted.
(D) A theory is proposed, considered, and then amended.
(E) An opinion is presented, qualified, and then reaffirmed.
6.It can be inferred from the passage that one problem associated with the production of

huge lots of cars is which of the following?
(A) The need to manufacture flexible machinery and equipment
(B) The need to store extra components not required for immediate use
(C) The need for expensive training programs for workers, which emphasize the development of

facility in several production jobs
(D) The need to alter conventional mass-production processes
(E) The need to increase the investment per vehicle in order to achieve high productivity

levels
7.Which of the following statements is supported by information stated in the passage?
(A) Japanese and United States automakers differ in their approach to production processes.
(B) Japanese automakers have perfected the use of single-function equipment.
(C) Japanese automakers invest more capital per employee than do United States automakers.
(D) United States-owned factories abroad have higher production levels than do Japanese

owned plants in the United States.
(E) Japanese automakers have benefited from the cultural heritage of their workers.
8.With which of the following predictive statement regarding Japanese automakers would the

author most likely agree?
(A) The efficiency levels of the Japanese automakers will decline if they become less

flexible in their approach to production.
(B) Japanese automakers productivity levels double during the late 1990’s.
(C) United States automakers will originate new production processes before Japanese

automakers do.
(D) Japanese automakers will hire fewer workers than will United States automakers because

each worker is required to perform several jobs.
(E) Japanese automakers will spend less on equipment repairs than will United States

automakers because Japanese equipment can be easily altered.
Passage 41 (41/63)
It was once believed that the brain was independent of metabolic processes occurring

elsewhere in the body. In recent studies, however, we have discovered that the production

and release in brain neurons of the neurotransmitter serotonin (neurotransmitters are

compounds that neurons use to transmit signals to other cells) depend directly on the food

that the body processes.
Our first studies sought to determine whether the increase in serotonin observed in rats

given a large injection of the amino acid tryptophan might also occur after rats ate meals

that change tryptophan levels in the blood. We found that, immediately after the rats began

to eat, parallel elevations occurred in blood tryptophan, brain tryptophan, and brain

serotonin levels. These findings suggested that the production and release of serotonin in

brain neurons were normally coupled with blood-tryptophan increases. In later studies we

found that injecting insulin into a rat’s bloodstream also caused parallel elevations in

blood and brain tryptophan levels and in serotonin levels. We then decided to see whether

the secretion of the animal’s own insulin similarly affected serotonin production. We gave

the rats a carbohydrate-containing meal that we knew would elicit insulin secretion. As we

had hypothesized, the blood tryptophan level and the concentrations of tryptophan serotonin

in the brain increased after the meal.
Surprisingly, however, when we added a large amount of protein to the meal, brain tryptophan

and serotonin levels fell. Since protein contains tryptophan, why should it depress brain

tryptophan levels? The answer lies in the mechanism that provides blood tryptophan to the

brain cells. This same mechanism also provides the brain cells with other amino acids found

in protein, such as tyrosine and Leucine. The consumption of protein increases blood

concentration of the other amino acids much more, proportionately, than it does that of

tryptophan. The more protein in the meal, the lower is the ratio of the resulting

blood-tryptophan concentration to the concentration of competing amino acids, and the more

slowly is tryptophan provided to the brain. Thus the more protein in a meal, the less

serotonin subsequently produced and released.
1.Which of the following titles best summarizes the contents of the passage?
(A) Neurotransmitters: Their Crucial Function in Cellular Communication
(B) Diet and Survival: An Old Relationship Reexamined
(C) The Blood Supply and the Brain: A Reciprocal Dependence
(D) Amino Acids and Neurotransmitters: The Connection Between Serotonin Levels and Tyrosine
(E) The Effects of Food Intake on the Production and Release of Serotonin: Some Recent

Findings
2.According to the passage, the speed with which tryptophan is provided to the brain cells

of a rat varies with the
(A) amount of protein present in a meal
(B) concentration of serotonin in the brain before a meal
(C) concentration of leucine in the blood rather than on the concentration of tyrosine in

the blood after a meal
(D) concentration of tryptophan in the brain before a meal
(E) number of serotonin-containing neurons present in the brain before a meal
3.According to the passage, when the authors began their first studies, they were aware that
(A) they would eventually need to design experiments that involved feeding rats high

concentrations of protein
(B) tryptophan levels in the blood were difficult to monitor with accuracy
(C) serotonin levels increased after rats were fed meals rich in tryptophan
(D) there were many neurotransmitters whose production was dependent on metabolic processes

elsewhere in the body
(E) serotonin levels increased after rats were injected with a large amount of tryptophan
4.According to the passage, one reason that the authors gave rats carbohydrates was to
(A) depress the rats’ tryptophan levels
(B) prevent the rats from contracting diseases
(C) cause the rats to produce insulin
(D) demonstrate that insulin is the most important substance secreted by the body
(E) compare the effect of carbohydrates with the effect of proteins
5.According to the passage, the more protein a rat consumes, the lower will be the
(A) ratio of the rat’s blood-tryptophan concentration to the amount of serotonin produced

and released in the rat’s brain
(B) ratio of the rat’s blood-tryptophan concentration to the concentration in its blood of

the other amino acids contained in the protein
(C) ratio of the rat’s blood-tyrosine concentration to its blood-leucine concentration
(D) number of neurotransmitters of any kind that the rat will produce and release
(E) number of amino acids the rat’s blood will contain
6.The authors’ discussion of the “mechanism that provides blood tryptophan to the brain

cells” (lines 31-32) is meant to
(A) stimulate further research studies
(B) summarize an area of scientific investigation
(C) help explain why a particular research finding was obtained
(D) provide supporting evidence for a controversial scientific theory
(E) refute the conclusions of a previously mentioned research study
7.According to the passage, an injection of insulin was most similar in its effect on rats

to an injection of
(A) tyrosine
(B) leucine
(C) blood
(D) tryptophan
(E) protein
8.It can be inferred from the passage that which of the following would be LEAST likely to

be a potential source of aid to a patient who was not adequately producing and releasing

serotonin?
(A) Meals consisting almost exclusively of protein
(B) Meals consisting almost exclusively of carbohydrates
(C) Meals that would elicit insulin secretion
(D) Meals that had very low concentrations of tyrosine
(E) Meals that had very low concentrations of leucine
9.It can be inferred from the passage that the authors initially held which of the following

hypotheses about what would happen when they fed large amounts of protein to rats?
(A) The rats’ brain serotonin levels would not decrease.
(B) The rats’ brain tryptophan levels would decrease.
(C) The rats’ tyrosine levels would increase less quickly than would their leucine levels.
(D) The rats would produce more insulin.
(E) The rats would produce neurotransmitters other than serotonin.
Passage 42 (42/63)

Historians sometimes forget that history is continually being made and experienced before it

is studied, interpreted, and read. These latter activities have their own history, of

course, which may impinge in unexpected ways on public events. It is difficult to predict

when “new pasts” will overturn established historical interpretations and change the course

of history.
In the fall of 1954, for example, C. Vann Woodward delivered a lecture series at the

University of Virginia which challenged the prevailing dogma concerning the history,

continuity, and uniformity of racial segregation in the South. He argued that the Jim Crow

laws of the late nineteenth and early twentieth centuries not only codified traditional

practice but also were a determined effort to erase the considerable progress made by Black

people during and after Reconstruction in the 1870’s. This revisionist view of Jim Crow

legislation grew in part from the research that Woodward had done for the NAACP legal

campaign during its preparation for Brown v. Board of Education. The Supreme Court had

issued its ruling in this epochal desegregation case a few months before Woodward’s

lectures.
The lectures were soon published as a book, The Strange Career of Jim Crow. Ten years later,

in a preface to the second revised edition, Woodward confessed with ironic modesty that the

first edition “had begun to suffer under some of the handicaps that might be expected in a

history of the American Revolution published in 1776.” That was a bit like hearing Thomas

Paine apologize for the timing of his pamphlet Common Sense, which had a comparable impact.

Although Common Sense also had a mass readership, Paine had intended to reach and inspire:

he was not a historian, and thus not concerned with accuracy or the dangers of historical

anachronism. Yet, like Paine, Woodward had an unerring sense of the revolutionary moment,

and of how historical evidence could undermine the mythological tradition that was crushing

the dreams of new social possibilities. Martin Luther King, Jr., testified to the profound

effect of The Strange Career of Jim Crow on the civil rights movement by praising the book

and quoting it frequently.
1.The “new pasts” mentioned in line 6 can best be described as the
(A) occurrence of events extremely similar to past events
(B) history of the activities of studying, interpreting, and reading new historical writing
(C) change in people’s understanding of the past due to more recent historical writing
(D) overturning of established historical interpretations by politically motivated

politicians
(E) difficulty of predicting when a given historical interpretation will be overturned
2.It can be inferred from the passage that the “prevailing dogma” (line 10) held that
(A) Jim Crow laws were passed to give legal status to well-established discriminatory

practices in the South
(B) Jim Crow laws were passed to establish order and uniformity in the discriminatory

practices of different southern states
(C) Jim Crow laws were passed to erase the social gains that Black people had achieved since

Reconstruction
(D) the continuity of racial segregation in the South was disrupted by passage of Jim Crow

laws
(E) the Jim Crow laws of the late nineteenth and early twentieth centuries were passed to

reverse the effect of earlier Jim Crow laws
3.Which of the following is the best example of writing that is likely to be subject to the

kinds of “handicaps” referred to in line 27?
(A) A history of an auto manufacturing plant written by an employee during an auto-buying

boom
(B) A critique of a statewide school-desegregation plan written by an elementary school

teacher in that state
(C) A newspaper article assessing the historical importance of a United States President

written shortly after the President has taken office
(D) A scientific paper describing the benefits of a certain surgical technique written by

the surgeon who developed the technique
(E) Diary entries narrating the events of a battle written by a soldier who participated in

the battle
4.The passage suggests that C. Vann Woodward and Thomas Paine were similar in all of the

following ways EXCEPT:
(A) Both had works published in the midst of important historical events.
(B) Both wrote works that enjoyed widespread popularity.
(C) Both exhibited an understanding of the relevance of historical evidence to contemporary

issues.
(D) The works of both had a significant effect on events following their publication.
(E) Both were able to set aside worries about historical anachronism in order to reach and

inspire.
5.The attitude of the author of the passage toward the work of C. Vann Woodward is best

described as one of
(A) respectful regard
(B) qualified approbation
(C) implied skepticism
(D) pointed criticism
(E) fervent advocacy
6.Which of the following best describes the new idea expressed by C. Vann Woodward in his

University of Virginia lectures in 1954?
(A) Southern racial segregation was continuous and uniform.
(B) Black people made considerable progress only after Reconstruction.
(C) Jim Crow legislation was conventional in nature.
(D) Jim Crow laws did not go as far in codifying traditional practice as they might have.
(E) Jim Crow laws did much more than merely reinforce a tradition of segregation.
Passage 43 (43/63)
Joseph Glatthaar’s Forged in Battle is not the first excellent study of Black soldiers and

their White officers in the Civil War, but it uses more soldiers’ letters and

diaries—including rare material from Black soldiers—and concentrates more intensely on

Black-White relations in Black regiments than do any of its predecessors. Glatthaar’s title

expresses his thesis: loyalty, friendship, and respect among White officers and Black

soldiers were fostered by the mutual dangers they faced in combat.
Glatthaar accurately describes the government’s discriminatory treatment of Black soldiers

in pay, promotion, medical care, and job assignments, appropriately emphasizing the campaign

by Black soldiers and their officers to get the opportunity to fight. That chance remained

limited throughout the war by army policies that kept most Black units serving in

rear-echelon assignments and working in labor battalions. Thus, while their combat death

rate was only one-third that of White units, their mortality rate from disease, a major

killer in his war, was twice as great. Despite these obstacles, the courage and

effectiveness of several Black units in combat won increasing respect from initially

skeptical or hostile White soldiers. As one White officer put it, “they have fought their

way into the respect of all the army.”
In trying to demonstrate the magnitude of this attitudinal change, however, Glatthaar seems

to exaggerate the prewar racism of the White men who became officers in Black regiments.

“Prior to the war,” he writes of these men, “virtually all of them held powerful racial

prejudices.” While perhaps true of those officers who joined Black units for promotion or

other self-serving motives, this statement misrepresents the attitudes of the many

abolitionists who became officers in Black regiments. Having spent years fighting against

the race prejudice endemic in American society, they participated eagerly in this military

experiment, which they hoped would help African Americans achieve freedom and postwar civil

equality. By current standards of racial egalitarianism, these men’s paternalism toward

African Americans was racist. But to call their feelings “powerful racial prejudices” is to

indulge in generational chauvinism—to judge past eras by present standards.
1.The passage as a whole can best be characterized as which of the following?
(A) An evaluation of a scholarly study
(B) A description of an attitudinal change
(C) A discussion of an analytical defect
(D) An analysis of the causes of a phenomenon
(E) An argument in favor of revising a view
2.According to the author, which of the following is true of Glatthaar’s Forged in Battle

compared with previous studies on the same topic?
(A) It is more reliable and presents a more complete picture of the historical events on

which it concentrates than do previous studies.
(B) It uses more of a particular kind of source material and focuses more closely on a

particular aspect of the topic than do previous studies.
(C) It contains some unsupported generalizations, but it rightly emphasizes a theme ignored

by most previous studies.
(D) It surpasses previous studies on the same topic in that it accurately describes

conditions often neglected by those studies.
(E) It makes skillful use of supporting evidence to illustrate a subtle trend that previous

studies have failed to detect.
3.The author implies that the title of Glatthaar’s book refers specifically to which of the

following?
(A) The sense of pride and accomplishment that Black soldiers increasingly felt as a result

of their Civil War experiences
(B) The civil equality that African Americans achieved after the Civil War, partly as a

result of their use of organizational skills honed by combat
(C) The changes in discriminatory army policies that were made as a direct result of the

performance of Black combat units during the Civil War
(D) The improved interracial relations that were formed by the races’ facing of common

dangers and their waging of a common fight during the Civil War
(E) The standards of racial egalitarianism that came to be adopted as a result of White

Civil War veterans’ repudiation of the previous racism
4.The passage mentions which of the following as an important theme that receives special

emphasis in Glatthaar’s book?
(A) The attitudes of abolitionist officers in Black units
(B) The struggle of Black units to get combat assignments
(C) The consequences of the poor medical care received by Black soldiers
(D) The motives of officers serving in Black units
(E) The discrimination that Black soldiers faced when trying for promotions
5.The passage suggests that which of the following was true of Black units’ disease

mortality rates in the Civil War?
(A) They were almost as high as the combat mortality rates of White units.
(B) They resulted in part from the relative inexperience of these units when in combat.
(C) They were especially high because of the nature of these units’ usual duty assignments.
(D) They resulted in extremely high overall casualty rates in Black combat units.
(E) They exacerbated the morale problems that were caused by the army’s discriminatory

policies.
6.The author of the passage quotes the White officer in lines 23-24 primarily in order to

provide evidence to support the contention that
(A) virtually all White officers initially had hostile attitudes toward Black soldiers
(B) Black soldiers were often forced to defend themselves from physical attacks initiated by

soldiers from White units
(C) the combat performance of Black units changed the attitudes of White soldiers toward

Black soldiers
(D) White units paid especially careful attention to the performance of Black units in

battle
(E) respect in the army as a whole was accorded only to those units, whether Black or White,

that performed well in battle
7.Which of the following best describes the kind of error attributed to Glatthaar in lines

25-28?
(A) Insisting on an unwarranted distinction between two groups of individuals in order to

render an argument concerning them internally consistent
(B) Supporting an argument in favor of a given interpretation of a situation with evidence

that is not particularly relevant to the situation
(C) Presenting a distorted view of the motives of certain individuals in order to provide

grounds for a negative evaluation of their actions
(D) Describing the conditions prevailing before a given event in such a way that the

contrast with those prevailing after the event appears more striking than it actually is
(E) Asserting that a given event is caused by another event merely because the other event

occurred before the given event occurred
8.Which of the following actions can best be described as indulging in “generational

chauvinism” (lines 40-41) as that practice is defined in the passage?
(A) Condemning a present-day monarch merely because many monarchs have been tyrannical in

the past.
(B) Clinging to the formal standards of politeness common in one’s youth to such a degree

that any relaxation of those standards is intolerable.
(C) Questioning the accuracy of a report written by an employee merely because of the

employee’s gender.
(D) Deriding the superstitions accepted as “science” in past eras without acknowledging the

prevalence of irrational beliefs today.
(E) Labeling a nineteenth-century politician as “corrupt” for engaging in once-acceptable

practices considered intolerable today.
Passage 44 (44/63)
It was once assumed that all living things could be divided into two fundamental and

exhaustive categories. Multicellular plants and animals, as well as many unicellular

organisms, are eukaryotic—their large, complex cells have a well-formed nucleus and many

organelles. On the other hand, the true bacteria are prokaryotic cell, which are simple and

lack a nucleus. The distinction between eukaryotes and bacteria, initially defined in terms

of subcellular structures visible with a microscope, was ultimately carried to the molecular

level. Here prokaryotic and eukaryotic cells have many features in common. For instance,

they translate genetic information into proteins according to the same type of genetic

coding. But even where the molecular processes are the same, the details in the two forms

are different and characteristic of the respective forms. For example, the amino acid

sequences of various enzymes tend to be typically prokaryotic or eukaryotic. The differences

between the groups and the similarities within each group made it seem certain to most

biologists that the tree of life had only two stems. Moreover, arguments pointing out the

extent of both structural and functional differences between eukaryotes and true bacteria

convinced many biologists that the precursors of the eukaryotes must have diverged from the

common ancestor before the bacteria arose.
Although much of this picture has been sustained by more recent research, it seems

fundamentally wrong in one respect. Among the bacteria, there are organisms that are

significantly different both from the cells of eukaryotes and from the true bacteria, and it

now appears that there are three stems in the tree of life. New techniques for determining

the molecular sequence of the RNA of organisms have produced evolutionary information about

the degree to which organisms are related, the since they diverged from a common ancestor,

and the reconstruction of ancestral versions of genes. These techniques have strongly

suggested that although the true bacteria indeed form a large coherent group, certain other

bacteria, the archaebacteria, which are also prokaryotes and which resemble true bacteria,

represent a distinct evolutionary branch that far antedates the common ancestor of all true

bacteria.
1.The passage is primarily concerned with
(A) detailing the evidence that has led most biologists to replace the trichotomous picture

of living organisms with a dichotomous one
(B) outlining the factors that have contributed to the current hypothesis concerning the

number of basic categories of living organisms
(C) evaluating experiments that have resulted in proof that the prokaryotes are more ancient

than had been expected
(D) summarizing the differences in structure and function found among true bacteria,

archaebacteria, and eukaryotes
(E) formulating a hypothesis about the mechanisms of evolution that resulted in the

ancestors of the prokaryotes
2.According to the passage, investigations of eukaryotic and prokaryotic cells at the

molecular level supported the conclusion that
(A) most eukaryotic organisms are unicellular
(B) complex cells have well-formed nuclei
(C) prokaryotes and eukaryotes form two fundamental categories
(D) subcellular structures are visible with a microscope
(E) prokaryotic and eukaryotic cells have similar enzymes
3.According to the passage, which of the following statements about the two-category

hypothesis is likely to be true?
(A) It is promising because it explains the presence of true bacteria-like organisms such as

organelles in eukaryotic cells.
(B) It is promising because it explains why eukaryotic cells, unlike prokaryotic cells, tend

to form multicellular organisms.
(C) It is flawed because it fails to account for the great variety among eukaryotic

organisms.
(D) It is flawed because it fails to account for the similarity between prokaryotes and

eukaryotes.
(E) It is flawed because it fails to recognize an important distinction among prokaryotes.
4.It can be inferred from the passage that which of the following have recently been

compared in order to clarify the fundamental classifications of living things?
(A) The genetic coding in true bacteria and that in other prokaryotes
(B) The organelle structures of archaebacteria, true bacteria, and eukaryotes
(C) The cellular structures of multicellular organisms and unicellular organisms
(D) The molecular sequences in eukaryotic RNA, true bacterial RNA, and archaebacterial RNA
(E) The amino acid sequences in enzymes of various eukaryotic species and those of enzymes

in archaebacterial species
5.If the “new techniques” mentioned in line 31 were applied in studies of biological

classifications other than bacteria, which of the following is most likely?
(A) Some of those classifications will have to be reevaluated.
(B) Many species of bacteria will be reclassified.
(C) It will be determined that there are four main categories of living things rather than

three.
(D) It will be found that true bacteria are much older than eukaryotes.
(E) It will be found that there is a common ancestor of the eukaryotes, archaebacteria, and

true bacteria.
6.According to the passage, researchers working under the two-category hypothesis were

correct in thinking that
(A) prokaryotes form a coherent group
(B) the common ancestor of all living things had complex properties
(C) eukaryotes are fundamentally different from true bacteria
(D) true bacteria are just as complex as eukaryotes
(E) ancestral versions of eukaryotic genes functioned differently from their modern

counterparts
7.All of the following statements are supported by the passage EXCEPT:
(A) True bacteria form a distinct evolutionary group.
(B) Archaebacteria are prokaryotes that resemble true bacteria.
(C) True bacteria and eukaryotes employ similar types of genetic coding.
(D) True bacteria and eukaryotes are distinguishable at the subcellular level.
(E) Amino acid sequences of enzymes are uniform for eukaryotic and prokaryotic organisms.
8.The author’s attitude toward the view that living things are divided into three categories

is best described as one of
(A) tentative acceptance
(B) mild skepticism
(C) limited denial
(D) studious criticism
(E) whole hearted endorsement
Passage 45 (45/63)
Excess inventory, a massive problem for many businesses, has several causes, some of which

are unavoidable. Overstocks may accumulate through production overruns or errors. Certain

styles and colors prove unpopular. With some products—computers and software, toys, and

books—last year’s models are difficult to move even at huge discounts. Occasionally the

competition introduces a better product. But in many cases the public’s buying tastes simply

change, leaving a manufacturer or distributor with thousands (or millions) of items that the

fickle public no longer wants.
One common way to dispose of this merchandise is to sell it to a liquidator, who buys as

cheaply as possible and then resells the merchandise through catalogs, discount stores, and

other outlets. However, liquidators may pay less for the merchandise than it cost to make

it. Another way to dispose of excess inventory is to dump it. The corporation takes a

straight cost write-off on its taxes and hauls the merchandise to a landfill. Although it is

hard to believe, there is a sort of convoluted logic to this approach. It is perfectly

legal, requires little or preparation on the company’s part, and solves the problem

quickly. The drawback is the remote possibility of getting caught by the news media. Dumping

perfectly useful products can turn into a public relations nightmare. Children living in

poverty are freezing and XYZ Company has just sent 500 new snowsuits to the local dump.

Parents of young children are barely getting by and QRS Company dumps 1,000 cases of

disposable diapers because they have slight imperfections.
The managers of these companies are not deliberately wasteful; they are simply unaware of

all their alternatives. In 1976 the Internal Revenue Service provided a tangible incentive

for businesses to contribute their products to charity. The new tax law allowed corporations

to deduct the cost of the product donated plus half the difference between cost and fair

market selling price, with the proviso that deductions cannot exceed twice cost. Thus, the

federal government sanctions—indeed, encourages—an above-cost federal tax deduction for

companies that donate inventory to charity.
1.The author mentions each of the following as a cause of excess inventory EXCEPT
(A) production of too much merchandise
(B) inaccurate forecasting of buyers’ preferences
(C) unrealistic pricing policies
(D) products’ rapid obsolescence
(E) availability of a better product
2.The passage suggests that which of the following is a kind of product that a liquidator

who sells to discount stores would be unlikely to wish to acquire?
(A) Furniture
(B) Computers
(C) Kitchen equipment
(D) Baby-care products
(E) Children’s clothing
3.The passage provides information that supports which of the following statements?
(A) Excess inventory results most often from insufficient market analysis by the

manufacturer.
(B) Products with slight manufacturing defects may contribute to excess inventory.
(C) Few manufacturers have taken advantage of the changes in the federal tax laws.
(D) Manufacturers who dump their excess inventory are often caught and exposed by the news

media.
(E) Most products available in discount stores have come from manufacturers’

excess-inventory stock.
4.The author cites the examples in lines 25-29 most probably in order to illustrate
(A) the fiscal irresponsibility of dumping as a policy for dealing with excess inventory
(B) the waste-management problems that dumping new products creates
(C) the advantages to the manufacturer of dumping as a policy
(D) alternatives to dumping explored by different companies
(E) how the news media could portray dumping to the detriment of the manufacturer’s

reputation
5.By asserting that manufacturers “are simply unaware” (line 31), the author suggests which

of the following?
(A) Manufacturers might donate excess inventory to charity rather than dump it if they knew

about the provision in the federal tax code.
(B) The federal government has failed to provide sufficient encouragement to manufacturers

to make use of advantageous tax policies.
(C) Manufacturers who choose to dump excess inventory are not aware of the possible effects

on their reputation of media coverage of such dumping.
(D) The manufacturers of products disposed of by dumping are unaware of the needs of those

people who would find the products useful.
(E) The manufacturers who dump their excess inventory are not familiar with the employment

of liquidators to dispose of overstock.
6.The information in the passage suggests that which of the following, if true, would make

donating excess inventory to charity less attractive to manufacturers than dumping?
(A) The costs of getting the inventory to the charitable destination are greater than the

above-cost tax deduction.
(B) The news media give manufacturers’ charitable contributions the same amount of coverage

that they give dumping.
(C) No straight-cost tax benefit can be claimed for items that are dumped.
(D) The fair-market value of an item in excess inventory is 5 s its cost.
(E) Items end up as excess inventory because of a change in the public’s preferences.
7.Information in the passage suggests that one reason manufacturers might take advantage of

the tax provision mentioned in the last paragraph is that
(A) there are many kinds of products that cannot be legally dumped in a landfill
(B) liquidators often refuse to handle products with slight imperfections
(C) the law allows a deduction in excess of the cost of manufacturing the product
(D) media coverage of contributions of excess-inventory products to charity is widespread

and favorable
(E) no tax deduction is available for products dumped or sold to a liquidator